Review of Microbiology & Immunology Apurba Sankar Sastry, Sandhya Bhat K
INDEX
×
Chapter Notes

Save Clear


fm1Review of Microbiology and Immunology
4th Edition
Apurba Sankar Sastry MD (JIPMER) DNB MNAMS PDCR Hospital Infection Control Officer Assistant Professor Department of Microbiology Jawaharlal Institute of Postgraduate Medical Education & Research (JIPMER), Pondicherry, India Sandhya Bhat K MD DNB MNAMS PDCR Assistant Professor Department of Clinical Microbiology Pondicherry Institute of Medical Sciences (PIMS), Pondicherry, India
fm2
Jaypee Brothers Medical Publishers (P) Ltd
Headquarters
Jaypee Brothers Medical Publishers (P) Ltd
4838/24, Ansari Road, Daryaganj
New Delhi 110 002, India
Phone: +91-11-43574357
Fax: +91-11-43574314
Overseas Offices
J.P. Medical Ltd
83, Victoria Street, London
SW1H 0HW (UK)
Phone: +44-20 3170 8910
Fax: +44 (0)20 3008 6180
Jaypee Medical Inc.
The Bourse
111, South Independence Mall East
Suite 835, Philadelphia, PA 19106, USA
Phone: +1 267-519-9789
Jaypee-Highlights Medical Publishers Inc
City of Knowledge, Bld. 237, Clayton
Panama City, Panama
Phone: +1 507-301-0496
Fax: +1 507-301-0499
Jaypee Brothers Medical Publishers (P) Ltd
17/1-B, Babar Road, Block-B, Shaymali
Mohammadpur, Dhaka-1207
Bangladesh
Mobile: +08801912003485
Jaypee Brothers Medical Publishers (P) Ltd
Bhotahity, Kathmandu, Nepal
Phone: +977-9741283608
© 2015, Jaypee Brothers Medical Publishers
The views and opinions expressed in this book are solely those of the original contributor(s)/author(s) and do not necessarily represent those of editor(s) of the book.
All rights reserved. No part of this publication may be reproduced, stored or transmitted in any form or by any means, electronic, mechanical, photocopying, recording or otherwise, without the prior permission in writing of the publishers.
All brand names and product names used in this book are trade names, service marks, trademarks or registered trademarks of their respective owners. The publisher is not associated with any product or vendor mentioned in this book.
Medical knowledge and practice change constantly. This book is designed to provide accurate, authoritative information about the subject matter in question. However, readers are advised to check the most current information available on procedures included and check information from the manufacturer of each product to be administered, to verify the recommended dose, formula, method and duration of administration, adverse effects and contraindications. It is the responsibility of the practitioner to take all appropriate safety precautions. Neither the publisher nor the author(s)/editor(s) assume any liability for any injury and/or damage to persons or property arising from or related to use of material in this book.
This book is sold on the understanding that the publisher is not engaged in providing professional medical services. If such advice or services are required, the services of a competent medical professional should be sought.
Every effort has been made where necessary to contact holders of copyright to obtain permission to reproduce copyright material. If any have been inadvertently overlooked, the publisher will be pleased to make the necessary arrangements at the first opportunity.
Inquiries for bulk sales may be solicited at: jaypee@jaypeebrothers.com
Review of Microbiology and Immunology
First Edition: 2011
Second Edition: 2012
Third Edition: 2014
Fourth Edition: 2015
9789351527336
Printed at
fm3Dedicated to
Our Beloved Parents
Brother and Sister-in-law
Sister and Brother-in-law
Our Beloved Son
And above all, Lord Ganesha who gave us the knowledge and inspiration
fm4
fm5Preface to the Fourth Edition
Importance of Microbiology for PG Entrances
Study methodology – Antegrade vs Retrograde
Each chapter contains
Apurba Sankar Sastry
Sandhya Bhat K
fm6Preface to the First Edition
Tips for your preparation
Wish You “ALL THE BEST FOR THE SUCCESS AND THE BRIGHT FUTURE AHEAD”.
Dr Apurba Sankar Sastry
Dr Sandhya Bhat K
fm8Acknowledgments
Our efforts bore fruit with the successful completion of this project ‘Review of Microbiology and Immunology’ 4th edition. However, there are many others who share the reward of this effort simply because it would never have been this good without their help. It gives us great pleasure to acknowledge the contribution of those who guided, supported and stood by us through the arduous journey of completing this tedious work.
We express our deep sense of reverence and gratitude to our teachers for giving us the wonderful opportunity to work under their guidance during our postgraduation and residency time and enlightened us by their knowledge about the subject without which we would not ever thought of initiating this project.
We sincerely appreciate the help provided by the following people for their constant support:
We also offer our gratitude to some of the students who constantly gave their inputs and supports during the correction and edition of the book.
We are also thankful to Murali Dharan and Suvithra S staff of Jaypee Brothers Medical Publishers, Pondicherry.
We are grateful to Shri Jitendar P Vij (Group Chairman), Ms Chetna Malhotra Vohra (Associate Director), Ms Saima (Project Manager), Dr Sakshi Arora, Mr Subhash Chandra, Mr Mukund Kumar and all the members of Jaypee Brothers Medical Publishers (P) Ltd, New Delhi, India for giving us this wonderful opportunity of writing this book and their excellent support throughout the journey especially during the editing work.
fm9Annexure
RECENT UPDATE IN INFECTIOUS DISEASE 2015
 
1. EBOLA VIRUS UPDATE 2015
Ebola virus has become a global threat, because of its recent outbreak in 2014; which was declared by WHO as a public health emergency of international concern.
 
2. POLIO ERADICATION UPDATE 2015
Poliomyelitis is now at the verge of eradication. This is attributed to the extensive immunization programmes on going globally. (Please remember……it is still not eradicated.)
 
Endgame Strategic Plan (2013–2018) - It has four objectives
 
VAPP and VDPV
Vaccine-associated paralytic poliomyelitis (VAPP) denotes all the cases of paralytic poliomyelitis occur following OPV administration.
 
Vaccine-Derived Polioviruses (VDPVS)
VDPVs isolates exhibit a higher level of genetic divergence from their parental OPV strains at VP1 sequence, which helps in their prolonged replication, and transmission.
 
VDPVs can be categorized as-
 
3. MERS-COV (MIDDLE EAST RESPIRATORY SYNDROME CORONAVIRUS) UPDATE 2015
 
Human coronavirus
There are six recognized coronaviruses that are known to cause human infections; most of them belong to Betacoronavirus except the first two which belong to Alphacoronavirus.
 
Morphology
Corona viruses are enveloped; carrying petal- or club-shaped or crown like peplomer spikes giving rise to solar corona appearance
 
Manifestations
Most human coronaviruses are widespread affecting people of most part of the world and produce mild respiratory tract infection. Two exceptions are SARS-CoV and MERS-CoV which are geographically restricted, transmitted from man to man and had produced outbreaks of severe respiratory disease with higher mortality.
 
fm12SARS-CoV (Severe Acute Respiratory Syndrome coronavirus)
 
MERS-COV (Middle East respiratory syndrome coronavirus)
 
4. DENGUE VACCINE UPDATE 2015
Live-attenuated tetravalent vaccine based on chimeric yellow fever-dengue virus (CYD-TDV) has been developed by Sanofi Pasteur Company.
It was found to be safe and effective in Phase III clinical trial done in Latin America and is expected to be marketed by 2015.
 
5. IMPORTANT NATIONAL REFERENCE CENTRES FOR INFECTIOUS DISEASES IN INDIA:
 
fm136. INCUBATION PERIOD OF IMPORTANT ORGANISMS
Incubation Period of Important Organisms
MMR + 2Pox
Hepatitis viruses
Malaria
STD
Mumps
19d
HAV
15-45d
P.vivax
14d
Syphilis
9-90d
Measles
10d
HBV
30-180d
P.falciparum
12d
Chancroid
1–14d
Rubella
14d
HCV
15-160d
P.malariae
28d
Herpes
2–7d
Small pox
12d
HDV
30-180d
P.ovale
17d
Donovaniasis
1–4 wks
Chicken pox
15d
HEV
14-60d
Filariasis
8-16month
LGV
3 d-6wk
Arboviruses
Other viruses
Leishmaniasis
1-4month
Gas gangrene
Dengue
5-6d
Rabies
1-3m
DPT
Cl.perfringens
10-48hr
Chikugunya
5-6d
Polio
7-14d
Diphtheria
2-6d
Cl.septicum
2-3d
KFD
3-8d
HIV
10yrs (Average)
Pertussis
7-14d
Cl.novyi
5-6d
Japanese encephalitis
5-15d
Infectious mononucleosis
4-8wks
Tetanus
6-10d
Plague
Yellow fever
3-6d
Influenza
18-72hr
Leprosy
3-5yrs
Bubonic & Septicaemic
2-7d
Cholera
1-2d
Pneumonic
1-3d
Typhoid
10—14d
 
7. ARTHROPOD BORNE DISEASES
Mosquito
Female Anopheles- Malaria,
Aedes- Dengue, Chikungunya, Yellow fever
Culex- JE, Filaria(Lymphatic)
Louse
Trench fever (Bartonella quintana),
Epidemic Relapsing fever (Borrelia recurrentis),
Epidemic Typhus,
Pediculosis
Flea
Bubonic Plague,
Endemic Typhus,
Hymenolepis dimunata
Hard Tick
Tick Typhus,
Arbo (KFD, RSSE, Crimerian Congo hemorrhagic fever, Colorado tick fever),
Babesia,
Tularemia
Soft Tick
Endemic Relapsing fever (B.duttoni),
Q fever (in animal)
Mite
Scrub Typhus,
Rickettsial Pox,
Scabies
Cyclops
Guinea worm disease,
Fish Tapeworm (D.latum)
Black fly
Onchocerciasis
Reduviid bug
Chaga's disease
 
fm148. EXAMPLES OF SPECIAL MEDIA (E-ENRICHED, EN-ENRICHMENT, S-SELECTIVE, D-DIFFERENTIAL MEDIA)
Organism
Medium
Enteric pathogens – for Salmonella, Shigella
Hektoen enteric agar (S)
Deoxycholate citrate agar (S)
MacConkey (D and S)
Wilson blair for Salmonella (S)
Xylose-lysin-deoxycholate agar (S)
Eosin Methylene blue agar (S)
Salmonella Shigella agar (S)
Selenite F borth (En), Tetrathionate broth (En)
Blood culture- for blood borne pathogens
Castaneda's biphasic media (E)- Brain heart infusion agar slope & broth
Vibrio cholerae(likes alkaline growth medium)
TCBS (Thiosulfate Citrate Bile Sucrose agar) (S)
Mansour's Gelatin Taurocholate Trypticase agar (S)
Alkaline Bile salt agar (S)
APW- Alkaline peptone water (En)
S.aureus
Mannitol salt agar (S)
Milk salt agar (S)
Ludlam's medium (S)
Streptococcus
Crystal violet blood agar (S)
Neisseria
Chocolate agar (E), Thayer-Martin media (S),
Modified New York medium (S)
Corynebacterium
Loffler's serum medium (E)
Potassium Tellurite agar (S)
Bacillus anthracis
PLET – Polymyxin Lithium EDTA Thallous acetate- (S)
Bacillus cereus
MYPA- Mannitol egg yolk phenol red polymyxin agar- (S)
Anaerobes
Thioglycollate (En)
Robertson cooked meat broth(En)
Listeria
PALCAM agar (S)
Pseudomonas
Cetrimide agar (S), King's media (for pigment)
Burkholderia
Ashdown's medium
Haemophilus
Blood agar with staph streak (E)
Chocolate agar (E)
Levinthal's medium (E), Fildes agar (E)
Bordetella
Regan Low media(E)
Bordet Gengou Glycerin potato blood agar (E)
Lacey's DFP media (S)
Mycobacterium
Lowenstein Jensen, Dorset egg (S)
Leptospira
EMJH (E), Fletcher's (E), Korthof's (E)
Campylobacter
Skirrow's, Butzler, Campy BAP (S)
Legionella
BCYE (Buffered charcoal yeast extract) (E)
Reiter's Treponema
Smith Noguchi media
Urinary pathogen
Mac Conkey agar(D and S)
Cystein Lactose Electrolyte Deficient agar (CLED agar) (D and S)
Organism
Transport media
Streptococcus
Pike's media
Neisseria
Amies, Stuart's media
Vibrio
VR, Autoclaved sea water, Carry Blair
Enteric pathogen
Carry Blair medium
Shigella, Salmonella
Buffered glycerol saline
Bordetella
Modified Stuart's (with casamino acid)
Mischulow's charcoal agar
Dacron or calcium alginate swab used
1General Microbiology
  • 1.1 History, Morphology, Growth & Nutrition of Bacteria
  • 1.2 Sterilization and Disinfection, Hospital Waste Management, Bacteriology of Water, Milk and Air
  • 1.3 Culture Media and Culture Methods
  • 1.4 Bacterial Genetics2

History, Morphology, Growth & Nutrition of BacteriaCHAPTER 1.1

 
HISTORY
 
Louis Pasteur
  • Known as Father of Microbiology
  • Coined the term “Microbiology” and “vaccine”
  • Proposed fermentation principle
  • Described pasteurization of milk
  • Introduced sterilization techniques and developed steam sterilizer, hot air oven and autoclave
  • Prepared the Vaccines for – Anthrax, Rabies, Cholera (CAR)
  • Disproved spontaneous generation of life and postulated germ theory of life
  • Used nutrient fluid to grow microorganisms.
 
Robert Koch
  • Known as Father of Medical Microbiology
  • Proposed Koch Postulates
  • Discovered – Anthrax bacilli, Tubercle bacilli, Cholera bacilli
  • Started Aniline dye staining
  • Proposed Solid culture media concept
  • Described hanging drop method for motility
 
Koch's Postulates
  • Four criteria-
    • The microorganism must be present in every case of the disease but absent from healthy organisms.
    • The suspected microorganism must be isolated and grown in a pure culture.
    • The same disease must result when the isolated microorganism is inoculated into a healthy host.
    • The same microorganism must be isolated again from the diseased host.
  • Additional 5th criteria- Antibody to the causative organism should be demonstrable in the patient's serum.
  • Molecular Koch's postulate- formulated by the microbiologist Stanley Fallow.
  • Gene (coding for virulence) of a pathogenic microorganism that contributes to the disease should satisfy all the criteria rather the microorganism parse.
  • Organism that doesn't follow the Koch's postulates-
    • Mycobacterium leprae & Treponema pallidum- Cannot be grown in vitro, how- ever maintained in animals.
    • Neisseria gonorrhea- There is no animal model; however bacteria can be grown in vitro.
  • Koch's phenomenon - Hypersensitivity reaction against tubercle bacilli can be demonstrated in guinea pigs.
 
Paul Ehrlich
  • Proposed Ehrlich phenomena
  • Detected- Ehrlichia(Bacteria)
  • Founder of- Acid fast stain
  • Standardized toxin and antitoxin
  • Proposed Side chain theory for Antibody production
 
OTHER IMPORTANT CONTRIBUTORS
  • Joseph Lister – Antiseptic measures to prevent surgical sepsis
  • Antony van Leeuwenhoek –Founder of Microscopy4
  • Ernst Ruska - Founder of Electron Microscope
  • Karry B Mullis – Discovered Polymerase Chain Reaction (PCR)
  • d'Herelle- Founder of the principles of modern virology
  • Goodpasteur- Viral culture in chick embryo
  • Lady Mary Workley Montague- Introduced variolation for immunizing against small pox
  • Edward Jenner- Used Cowpox for the vaccination against small pox
  • Elie Metchnikoff- Described phagocytosis and termed phagocytes.
  • Panborg-Described VDRL test
  • Klinberger-Described L forms
  • Barbara McClintock-Described about transposons
  • Jerne-Natural selection theory of antibody formation
 
Bacteria Named After the Discoverers
  • Kleb-Loeffler bacilli– Corynebacterium diphtheriae
  • Preisz Nocard bacilli – Corynebacterium pseudotuberculosis
  • Koch Week bacilli – Haemophilus aegypticus
  • Pfeiffer's bacilli – Haemophilus influenzae
  • Whitmore bacilli – Burkholderia pseudomallei
  • Battey bacillus- Mycobacterium intracellulare
  • Johne's bacillus- Mycobacterium paratuberculosis
  • Eaton's agent- Mycoplasma pneumonia
  • Gaffky-Eberth bacillus- SalmonellaTyphi
Discovery of important bacterial agents of human diseases
Discoverer
Organism
  • Ogston
  • Staphylococcus aureus
  • Neisser
  • Neisseria gonorrhoeae
  • Weichselbaum
  • Neisseria meningitidis
  • Loeffler
  • Corynebacterium diphtheria
  • Fraenkel
  • Streptococcus pneumoniae
  • Bruce
  • Brucella melitensis
  • Kitasato
  • Clostridium tetani
  • Hansen
  • Mycobacterium leprae
  • Yersin & Kitasato
  • Yersinia pestis
  • Schaudinn & Hoffman
  • Treponema pallidum
  • Daniel Carrion
  • Bartonella bacilliformis
  • d'Herelle
  • Bacteriophages
  • Nicolaier
  • Clostridium tetani
  • W.H. Welch
  • Clostridium perfringens
List of Nobel Winners
Scientists
Organism
  • Emil von Behring
  • Development of antitoxin against diphtheria toxin
  • Sir Ronal Ross
  • Malarial parasite life cycle in mosquitoes
  • Robert Koch
  • Discovery of the causative agent of tuberculosis
  • C. L. A. Laveran
  • Discovery of malarial parasite in unstained preparation of blood
  • Paul Ehrlich and Elie Metchnikoff
  • Selective theory of antibody formation
  • Charles Richet
  • Discovered anaphylaxis
  • Jules Bordet
  • Discovered complement & developed complementfixation test5
  • Karl Landsteiner
  • Described ABO blood group
  • Sir Alexander Fleming
  • Discovery of penicillin
  • Enders, Robbins and Weller
  • Cultivation of Polio viruses
  • Lederberg and Tatum
  • First described Conjugation in E.coli
  • Sir Burnett & Medawar
  • Postulated Clonal selection theory of antibody formation
  • Watson and Crick
  • Double helix structure of DNA
  • Peyton Roux
  • Viral oncogenesis
  • Holley, Khurana and Nirenberg
  • Genetic code
  • Charles Nicolle
  • Typhus fever
  • Blumberg
  • Australian antigen (HBSAg)
  • Rosalyn Yallow
  • Developed Radioimmunoassay
  • Georges Kohler
  • Developed hybridoma technology for monoclonal antibodies
  • S Tonegawa
  • Elucidated the nature of antibody diversity
  • Kary B Mulis
  • Polymerase Chain Reaction
  • Stanley B Prusiner
  • Described Prion & PK Reaction
 
MORPHOLOGY
  • Microorganisms- classified under the kingdom Protista
  • Kingdom Protista- Divided into following groups:
    • Prokaryotes –Include bacteria and blue green algae
    • Eukaryotes– Include fungi, algae (other than blue green), protozoa and slime moulds
Characteristics
Prokaryote
Eukaryote
Nuclear membrane, nucleolus, Ribo Nucleo-protein
Absent
Present
Cell division
Binary fission
Mitosis, Meiosis
Chromosome
One, Circular
Many, liner
Cell wall
Peptidoglycan
Sterols
Cellular Organelles
Absent
Present
Respiration
Mesosome
Mitochondria
Cytoskeleton
Absent
Present
Ribosome
70s
80s
Pinocytosis
Absent
Present
mRNA
Only exons
Both Introns & Exons
 
MICROSCOPY
 
Principle of Microscope Depends on
  • Magnification-takes place at Objective lens and Eye piece (increased by using convex lens)
  • Contrast-increased by staining
  • Resolution–ability to distinguish two points separate
 
6Resolution Power of
  • Human eye - 0.2 mm
  • Light microscope - 0.2 µm
  • Electron Microscope - 0.2 nm
 
Type of Light Used in Microscopes
  • Reflected light- used in dark field
  • Polarized light- used in differential interference contrast microscope
  • Transmitted light- used in light microscope
 
Microscopes Used in Microbiology
  • Light microscope
  • Dark field microscope
  • Phase contrast microscope
  • Fluorescence microscope
  • Electron microscope
  • Confocal microscope
 
DARK FIELD MICROSCOPY
  • The specimen is illuminated by rays of light obliquely so that, the light which is scattered by the object enters the objective and reaches the eye. Un reflected light don't enter objective
  • Hence object appears bright against dark background
  • Use- For visualization of live thin organism (Spirochaete) and flagella
 
PHASE CONTRAST MICROSCOPE
  • Principle: Converts slight differences in refractive index and cell density into easily detectable variations in light intensity
  • Use- Same as dark ground microscope + demonstration of endospores and inclusion bodies
 
FLUORESCENCE MICROSCOPE
  • Principle: When fluorescent dyes are exposed to UV rays, they convert this invisible, short wavelength rays into rays of longer wavelengths (visible light) and so become luminous and are said to fluoresce.
  • Flourochrome dyes used as stains: Auramine O, Acridine orange, and Fluorescent Isothiocyanate
Uses
  • Auto fluorescence – Cyclospora
  • Microbes coated with Flourochrome dyes
    • Acridine orange dyes (QBC) - détection of malarial parasites
    • Auramine phenol- detection of Tubercle bacilli
  • Immunofluroscence- Used in detection of antigen or antibody.
 
ELECTRON MICROSCOPE
  • Magnification > 1 lakh
  • Resolution power - 0.2 nm
  • Radiation source used – electron beam
  • Medium – vacuum
  • Electromagnetic lens used for focusing image
  • Steps
    • Specimen is mounted on – metal slide (copper)
    • Fixation of specimen using -Glutaraldehyde/Osmium tetroxide
    • Specimen is embedded in plastic polymer and cut to thin slices by ultra microtome knife
  • 7Types of Electron microscopes used
    • Transmission Electron microscope
    • Scanning Electron microscope
 
STAINING METHODS
Used to demonstrate structural details of the bacteria by producing colour contrast.
 
Negative Staining
  • Stains used- India ink and Nigrosin
  • Uses- Demonstration of capsule and thin bacteria such as Spirochetes.
  • Both background and organisms are stained while the structure to be demonstrated (i.e. capsule surrounding the organism) is not stained.
 
Silver Impregnation Methods
  • To visualize too thin bacteria (Treponema pallidum, Leptosptra, Borellia) by impregnation of silver on their surface.
  • E.g- Fontana and Levaditi stain
 
Differential Stains
Gram stain
  • It differentiates bacteria into two groups
  • Gram positive bacteria takes primary stain color and appear violet
  • Gram-negative bacteria takes counter stain, appearing pink.
 
Classification of Bacteria According to Gram Staining
  • Gram-positive cocci – Staphylococcus, Streptococcus, Enterococcus, Pneumococcus
  • Gram-negative cocci – Meningococcus, Gonococci, Veillonella, Moraxella
  • Gram-positive bacilli – Corynebacterium, Clostridium, Bacillus, Listeria, Rodococcus, Actinomyces/Nocardia, Mycobacteria, Erysopilothrixetc.
  • Gram-negative bacilli- Enterobacteriaceae, Vibrio, Pseudomonasetc.
 
Acid-Fast Stain
  • Acid-fast bacilli (AFB) – Appears bright red against blue background
  • List of acid fast micro-organisms:
    • M. tuberculosis(20% sulfuric acid)- Ziehl-Nielsen (ZN) technique
    • M. leprae (5% sulfuric acid) and Atypical Mycobacteria
    • Nocardia(1% sulfuric acid),
    • Spores(0.25% sulfuric acid)
    • Rhodococcus
    • Legionella micdadei
    • Acid fast parasites:
      • ✓ Coccidian parasites such as Cryptosporidium, Cyclosporaand Isospora,
      • Tinea saginatascolex & eggs and hooklet of hydatid cyst.
Acid fastness is due to
  • The high content of mycolic acid and alcohols found in the cell wall
  • Depends on integrity of the cell wall.
Procedure:
  • Primary stain as strong carbol fuchsin with intermittent heating followed by
  • Decolorizer − 20% sulfuric acid for 5-10 min followed by
  • Counter stain as methylene blue or malachite green for 1 min.
Modification of acid fast stain:
  • Kinyon's method of cold acid fast stain- Heating is not required. Used to stain acid fast parasites.
  • Gabbet's method- Sulfuric acid and methylene blue added together
  • Acid alcohol is used as decolorizer – For renal tuberculosis to differentiate M.tuberculosis (acid fast & alcohol fast) from M.smegmatis (commensal, acid fast but not alcohol fast)
 
8Other Staining Methods
  • Volutin granules(metachromatic granules) of Corynebacterium diphtheriae
    • Stained by Albert's stain, Neisser's stain or Ponder's stain
  • Spore staining:
    • Phase contrast microscope of unstained wet film (Best method)
    • Acid fast stain(using 0.25% sulfuric acid)
    • Malachite green stain (Schaeffer & fulton method modified by Ashby)
  • Lipids stained by- Sudan Black stain
  • Carbohydrate (Glycogen) stained by- Iodine stain
  • Flagellar stain- Tannic acid staining (Leifson method)
 
SHAPE OF BACTERIA
Classification of bacteria depending on their shape
  1. Cocci: oval or spherical cells. Arranged in
    • Clusters –e.g.-Staphylococci
    • Pairs - e.g.
      • Pneumococci(lanceolate shaped),
      • Meningococcus (Lens shaped)
      • Gonococci(kidney shaped)
      • Enterococcus(spectacle eye shaped)
    • Tetrads-e.g.- Micrococci
    • Chains-e.g-Streptococci
    • Octate-e.g.-Sarcina
  2. Bacilli: Rod shaped: Arranged in
    • Bacilli in chains. E.g. B.anthracis (Bamboo stick appearance), Streptobacillus, Klebsiella pneumoniae (diplobacilli)
    • Cuneiform pattern or Chinese letter arrangement. E.g. Corynebacterium diphtheriae.
    • Comma shaped: E.g. Vibrio (fish in stream appearance)
    • Branched and filamentous bacilli- Actinomyces and Nocardia
    • Spirally coiled, flexible- Spirochetes
    • Rigid spiral forms E.g. Spirillum
    • Curved gram negative bacilli- Campylobacter(Gull-wing shaped) and Helicobacter
    • Pleomorphic e.g- Haemophilus, Proteus, Brucella
    • Thumb print appearance- Bordetella pertussis
 
BACTERIAL CELL STRUCTURE
 
Cell Wall
Differences between Gram Positive and Gram Negative bacterial cell wall
Characters
Gram Positive cell wall
Gram Negative cell wall
Peptidoglycan layer
Thicker (15-80nm)
Thinner (2nm)
Lipid content
Nil or scanty (2-5%)
Present (15-20%)
Lipopolysaccharide (endotoxin)
Teichoic acid
Present
Absent
Variety of amino acid
Few
Several
Aromatic amino acid
Absent
Present
Peptidoglycan layer Composed of alternate molecule of-NAM (N-acetyl muramic acid) & NAG (N- acetyl glucosamine)
  • Each NAM molecule is attached with a pentapeptide side chain which are cross linked through a pentaglycine cross bridge
  • L- Lysine present at 3rd position of pentapeptide side chain
  • Pentapeptide side chains are directly liked to each other
  • No pentaglycine bridge
  • Meso-diaminopimelic acid present at 3rd position of pentapeptide side chain
9
Fig. 1.1.1: Gram positive peptidoglycan layer
Fig. 1.1.2: Gram negative peptidoglycan layer
 
Cell Membrane
  • Semi permeable membrane acting as an osmotic barrier, Composed of phospholipid and proteins
  • They differ from eukaryotic membranes in lacking sterolssuch as cholesterol (except in Mycoplasma which contains sterols)
 
Cytoplasm
Consists of:
  • Ribosomes- site of protein synthesis (70s)
  • Mesosomes- respiratory organelle for bacteria
  • Intracytoplasmic inclusions –
    • Storage of nutrients/ energy
    • Reduce osmotic pressure
    • 2 types:
      • ✓ Organic inclusion bodies (glycogen or poly-hydroxyl butyrate)
      • ✓ Inorganic inclusion bodies (polyphosphate granules or sulphur granules)
 
Nucleus
  • Single super coiled circular double stranded DNA.
  • Stained- with the Feulgen stain
  • Bacteria posses extra-chromosomal DNA -plasmids
 
Capsule and Slime Layer
The capsule has various functions:
  • It contributes to invasiveness of bacteria by protecting the bacteria from phagocytosis.
  • It facilitates adherence of bacteria to surfaces.
  • It plays a role in formation of biofilms
  • When capsule is loosely formed- called as slime layer- e.g. Pseudomonas
  • Demonstration of capsule by:Negative staining – India Ink and Nigrosin stain
    • M'Faydean capsule stain-used for demonstration of capsule of Bacillus anthracis (polychrome methylene blue stain)
    • Quellung's reaction –Capsular serotypes of Streptococcus pneumoniaecan be detected by adding antisera mixed with methylene blue. Capsule becomes swollen, refractile and delineated.
  • 10Capsulated bacteria:
    • Pneumococcus
    • Meningococcus
    • Haemophilius influenzae
    • Klebsiella pneumoniae
    • Bacillus anthracis –Made of Polypeptide (glutamate)
    • Streptococcus pyogenes (Hyaluronic acid)
    • Presence of both capsule and Slime layer -in Streptococcus salivarius
  • Capsulated fungus- Cryptococcus neoformans
 
Flagella
  • Thread-like appendages, made of protein-flagellin
  • Size- 3-20µm long, 0.2 µm thickness, Confers motility to the bacteria.
  • Classification of bacteria depending on the arrangement of flagella:
    • Monotrichous (single polar flagellum) - e.g. Vibrio cholerae
    • Lophotrichous (multiple polar flagella) - e.g. Spirilla
    • Peritrichous (flagella distributed over the entire cell surface)- e.g. Salmonella Typhi, Escherichia coli
    • Amphitrichous (single flagellum at both the ends)- e.g. Spirillum minus
  • Demonstration by:
    • Tannic acid staining (Leifson method & Ryu's method)
    • Dark ground/phase contrast/electron microscope
    • By demonstration of motility:
      • ✓ Craige tube,
      • ✓ Hanging drop,
      • ✓ Semisolid medium- Mannitol motility medium
      • ✓ Wet mount
  • Various types of motility
    • Tumbling motility – Listeria,
    • Gliding – Mycoplasma,
    • Stately – Clostridium,
    • Darting –Vibrio cholerae/Campylobacter
    • Swarming- Proteus, Clostridium tetani
    • Corkscrew, lashing, flexion extension motility - Spirochete
    • Falling leaf motility – Giardia trophozoite
    • Twitching motility- Trichomonas trophozoite
 
Fimbriae or Pili
  • Hair-like, Organ of adhesion, made up protein-pilin
  • Size- 0.5µm long, 10 nm thickness
  • Found in motile as well as in nonmotile organisms
  • They are antigenic
  • Form surface pellicle in liquid culture
  • Sex pili helps in conjugation (in Gonococcus)
  • Fimbria of E.colihas a property to bind to RBCs (agglutination).
 
Atypical Forms of Bacteria
  • Cell wall deficient forms - Mycoplasma, spheroplast, protoplast, or L- forms.
  • Pleomorphic bacteria –Proteus and Yersinia pestis
  • Involution forms- Swollen and aberrant forms of bacteria (Gonococci and Yersinia pestis) in ageing culture in high salt concentration
 
Cell Wall Deficient forms (L form)
  • Discovered by Kleineberger while studying Streptobacillus moniliformis
  • Named L form after Lister Institute, London.
  • Bacteria loose cell wall and become spherical which may occur spontaneous- ly or after exposure to penicillin or lysozyme.
  • Play a role in the persistence of pyelonephritis and other chronic infections
  • Types of cell wall deficient forms - Protoplast, Spheroplast and Mycoplasma
  • They may be unstable form, maintained only in presence of penicillin
  • Mycoplasma represents stable L form where the peptidoglycan layer is replaced by sterol.
 
11GROWTH AND MULTIPLICATION OF BACTERIA
  • Generation time - Time required for a bacterium to give rise to two daughter cells under optimum condition.
  • Generation time for different pathogenic bacteria-
    • Escherichia coli-20 minutes;
    • Mycobacterium tuberculosis− 20 hours
    • Mycobacterium leprae− 20 days.
 
Phases of Bacterial Growth Curve
When a bacterium is inoculated in a liquid media, it divides in four phases.
  • Lag phase:
    • Time required to enzyme and metabolites builds up,
    • Bacteria increase size but don't multiply
    • Size reaches maximum at end of lag phase
  • Log phase:
    • Bacteria divide exponentially,
    • Biochemically active
    • Smaller size
    • Uniformly stained
  • Stationary phase:
    • Bacteria starts dying which nullifying its multiplication
    • Gram variable
    • More storage granules,
    • Sporulation occurs in this phase,
    • Produce exotoxin, antibiotics and bacteriocin
  • Decline phase:
    • Bactria only dies without multiplication
    • Involution forms seen
Fig. 1.1.3: Bacterial growth curve
 
Factors Affecting Growth of Bacteria
 
Oxygen
On basis of their oxygen requirements bacteria are classified as:
  • Obligate aerobes: Can grow only in the presence of oxygen (e.g. Pseudomonas, M.tuberculosis, Bacillus, Brucella and Nocardia)
  • Facultative anaerobes: Ordinarily aerobes but can also grow without oxygen (e.g. most of the pathogenic bacteria.
  • Microaerophilic bacteria: Can grow in the presence of low oxygen (e.g. Campylobacter, Helicobacter, M.bovis).
  • Obligate anaerobes: Are those bacteria which can grow only in the absence of oxygen. (e.g. Clostridium)
  • Aerotolerant anaerobe: Can tolerate oxygen for some time, but don't use it. (Clostridium histolyticum)
 
12Carbon Dioxide
  • Organisms that require higher amounts of carbon dioxide growth - capnophilic bacteria
  • Examples: Haemophilus influenzae, Brucella abortusetc.
 
Temperature
  • Optimum temperature for most of the pathogenic bacteria is 37°C.
  • Classification of bacteria depending on their temperature range:
    • Psychrophiles: grow within a temperature range of 0-20°C. E.g Saprophytes
    • Mesophiles:Grow within a temperature range 25°C and 40°C. E.g Most of pathogenic bacteria
    • Thermophiles:grow at a high temperature range of 55°C - 80°C. E.g Bacillus stearothermophilus.
 
pH
  • Most pathogenic bacteria grow between pH 7.2 - pH 7.6
  • Very few bacteria (lactobacilli) - Grow at acidic pH below pH 4
  • Grow at alkaline pH (8.2-8.9) - Vibrio cholerae
 
Light
  • Depending on the source of energy bacteria may be classified as-
  • Phototrophs: Bacteria deriving energy from sunlight. E.g blue green algae
  • Chemotrophs: Bacteria deriving energy from chemical sources. E.g Most of pathogenic bacteria
 
INFECTION
 
Sources of Infection
  • Humans
    • Case: Harbours pathogen with its ill effect
    • Carrier: Harbours pathogen without any ill effect
      • ✓ Healthy carrier: Harbours pathogen but never suffered
      • ✓ Convalescent carrier: Harbours pathogen but recovered
      • ✓ Contact carrier: A carrier who acquires the pathogen from a patient
      • ✓ Paradoxical carrier: A carrier who acquires the pathogen from another carrier
  • Animals
    • Reservoir host: A host in which infectious agents stays without any ill effect and acts as source of infection.
    • Amplifier host: A host in which infectious agents multiplies exponentially. E.g. pigs for Japanese encephalitis virus.
  • Insects
  • Soil, Water and Food
  • Methods of transmission- Direct contact, inhalation, ingestion, inoculation, insects, congenital and iatrogenic and laboratory associated infection
 
Factors Predisposing to Microbial Pathogenicity
  • Virulence
    • Attenuation – Reduction in virulence
    • Exaltation – Enhancement of Virulence
  • Adhesion
    • Glycocalyx
    • Adherence proteins
    • Lipoteichoic acids
    • Fimbriae
  • Invasiveness: Ability to spread to host tissue after establishing infection by producing enzymes
    • hyaluronidase, collagenase, streptokinase, hemolysins, leukocidins
  • Toxigenicity: Exotoxin and Endotoxin13
Feature
Endotoxin
Exotoxin
Nature
Lipopolysaccharide
Protein (Polypeptide)
Source
Gram negative bacterial cell wall
Both Gram –ve & +ve bacteria
Location of genes
Chromosome
Plasmid or bacteriophage
Nature of secretion
Not secreted by the bacterial cell
Actively secreted by the bacteria
Release of toxin
Cell lysis
Filtration of bacterial cultures
Heat stability
Highly stable
Heat labile, destroyed at 60°C
Mode of action
By interleukins (IL-1) and TNF
Mostly enzyme like action
Effect
Non-specific (fever, shock, etc)
Specific pharmacological effect
Tissue affinity
No
Specific affinity for tissues
Fatal dose
Only large doses are fatal
Small doses - fatal
Antigenicity
Highly antigenic
Poorly antigenic
Neutralisation by antibodies
Ineffective
Neutralized by specific antibodies
Vaccine
No effective vaccine
Specific toxoids are available
14MULTIPLE CHOICE QUESTIONS
 
HISTORY
1. Corynebacterium diphtheriae is also called as: (NEET Pattern Based)
  1. Koch's bacillus
  2. Kleb-Loeffler bacillus
  3. Roux bacillus
  4. Yersin bacillus
2. Leeuwenhoek is associated with: (DNB DEC 2012)
  1. Telescope
  2. Microscope
  3. Stains
  4. Immunization
3. Theory of web of causation was given by? (DNB DEC 2012)
  1. Mc Mohan and Pugh
  2. Pettenkoffer
  3. John snow
  4. Louis Pasteur
4. Louis Pasteur is associated with all EXCEPT: (DNB DEC 2012)
  1. Vaccination of small pox
  2. Germ theory
  3. Pasteurization
  4. Vaccination of rabies
5. Credit of -Discovery of transmission of malaria by Anopheles mosquito goes to: (TN 2003)
  1. Ronald Ross
  2. Laveron
  3. Muller
  4. Pampana
6. Small pox vaccine was invented by: (MP 2003)
  1. Edward Jenner
  2. Ronald Ross
  3. Louis Pasteur
  4. Robert Koch
7. Match the following- Scientists vs contributions?
  1. Edward Jenner, I-Rabies, Anthrax (AIIMS May 2001)
  2. Louis Pasteur, II- Small pox
  3. Albert Calmette, Camille Guerin, III-Poliomyelitis
  4. Pierre Lepine, IV-Tuberculosis
  1. A-II, B-III, C-IV, D-I
  2. A-II, B-I, C-IV, D-III
  3. A-IV, B-I, C-III, D-II
  4. A-I, B-IV, C-II, D-III
8. Match the following- Scientists vs contributions: (AIIMS May 2003, 1993)
  1. Louis Pasteur I-Multifactorial causation of disease
  2. Pattenkoffer II- Germ theory of disease
  3. Aristotle III-Spontaneous generation of life
  1. A-II, B-I, C-III
  2. A-I, B-II, C-III
  3. A-III, B-I, C-II
 
BACTERIAL CELL BIOLOGY AND STRUCTURE
9. Non motile: (NEET Pattern Based)
  1. Shigella
  2. E.coli
  3. Proteus
  4. Vibrio
10. Not true about Flagella: (NEET Pattern Based)
  1. Organ of locomotion
  2. Antigenic
  3. Attachment
  4. Proteinaceous
11. Swarming is seen in: (NEET Pattern Based)
  1. Clostridium tetani
  2. Clostridium perfringens
  3. Clostridium botulinum
  4. Clostridium difficile
12. All are capsulated except: (DNB Dec 2011)
  1. Klebsiella pneumoniae
  2. Hemophilus influenzae
  3. Bacillus anthracis
  4. Escherichia coli
13. Which of the following is a bacteria taxonomically: (PGI Dec 2009)
  1. Chlamydia
  2. Rickettsia
  3. Mycoplasma
  4. Prion
  5. Bacteriophage
14. Which is a eukaryote: (PGI Dec 2008)
  1. Mycoplasma
  2. Bacteria
  3. Fungus
  4. Chlamydia
15. Eukaryote is: (NEET Pattern Based)
  1. Divide by binary fission
  2. Organized cell organelles
  3. Evolutionally ancient
  4. Ribosome 70S
16. Which are prokaryotes: (PGI May 10)
  1. Bacteria
  2. Fungi
  3. Protozoa
  4. Mycoplasma
  5. Virus
17. Capsulated organism: (PGI June 2003)
  1. Candida
  2. Klebsiella
  3. Proteus
  4. Cryptococcus
  5. Histoplasma
18. 15True about bacteria:(PGI Dec 2000)
  1. Mitochondria always absent
  2. Sterols always present in cell wall
  3. Divide by binary fission
  4. Can be seen only under electron microscope
19. Which is not present in Gram negative bacteria: (TNPG 2014)
  1. Peptidoglycan
  2. Teichoic acid
  3. LPS
  4. Porin channels
 
STAINING TECHNIQUES AND MICROSCOPY
20. Correct sequence in Gram staining? (NEET Pattern Based)
  1. Methyl violet → Iodine → Acetone → Carbol fuchsin
  2. Carbol fuchsin → Iodine → Acetone → Methyl violet
  3. Methyl violet → Acetone → Iodine → Carbol fuchsin
  4. Methyl violet → Carbol fuchsin → Acetone → Iodine
21. Not used in grams staining:(DNB DEC 2012)
  1. Methylene blue
  2. Crystal violet
  3. Iodine
  4. Safranin
22. Which of the following is negative staining technique? (DNB June 2010)
  1. Gram stain
  2. Fontana stain
  3. India ink
  4. ZN staining
23. All of the following are acid fast bacteria except: (SGPGI 2009)
  1. Cryptosporidium
  2. Mycoplasma
  3. Mycobacteria
  4. Nocardia
24. Acid fast organisms are: (PGI June 2008)
  1. Spores
  2. Nocardia
  3. Legionella
  4. Rodococcus
  5. Staphylococcus
25. Light microscopy resolution: (PGI Dec 2005)
  1. 200 nm
  2. 20 nm
  3. 0.2 nm
  4. 300 nm
26. Dye used in fluorescent microscopy: (PGI Dec 2004)
  1. Thioflavin T
  2. Congo red
  3. Brilliant blue
  4. Eosin
  5. Auramine
27. Dark ground microscopy is used to see: (AI 2001, AI 1995)
  1. Refractile organisms
  2. Flagella
  3. Capsule
  4. Fimbriae
28. Which of the following are AFB positive with 5% sulphuric acid: (PGI June 2001)
  1. M.avium
  2. M.leprae
  3. M.tuberculosis
  4. Nocardia
29. Reflected light used in: (TNPG 2014)
  1. Light microscope
  2. Electron microscope
  3. Phase contrast microscope
  4. Dark Field microscope
30. Indian ink staining is positive for which of the following cocci: (PGI Nov 2014, TNPG 2014)
  1. Pneumococcus
  2. Staphylococcus
  3. Meningococcus
  4. Gonococcus
  5. Enterococcus
31. Which of the following about Gram staining is most correct: (Latest Question 2013)
  1. Heating is done to promote entry of stain into cell wall of both Gram +ve and –ve bacteria
  2. If observed staining technique and microscopy before decolourization, both Gram +ve and –ve bacteria appear same
  3. Teichoic acid is present in the cell wall of Gram-ve bacteria
 
BACTERIAL GROWTH AND NUTRITION
32. Which of the following is microaerophilic: (AIIMS May 2009)
  1. Campylobacter
  2. Vibrio
  3. Bacteroides
  4. Pseudomonas
33. Growth on a cell- free solid medium is possible for following except: (TN 2009)
  1. Ureaplasma urealyticum
  2. Mycoplasma Pneumoniae
  3. C and L form of proteus vulgaris
  4. Chlamydia
34. The term viable not cultivable is used for: (PGI Dec 2007)
  1. M. leprae
  2. M. tuberculosis
  3. Treponema pallidum
  4. Salmonella
  5. Staphylococcus
35. Correct sequence of bacterial growth curve: (PGI Dec 2007)
  1. Log phase – Lag Phase – Stationary phase – Decline phase
  2. Lag Phase – Log phase – Stationary phase – Decline phase
  3. Stationary phase – Log phase – Lag Phase – Decline phase
  4. Lag phase – Exponential phase – Log phase – Death phase
  5. Exponential phase – Lag phase – Death phase – Stationary
36. Population doubling time in coliform bacilli is: (MHPG 2014)
  1. 20 seconds
  2. 20 minutes
  3. 20 hours
  4. 20 days
 
MICROBIAL PATHOGENESIS
37. Which organism grows in alkaline pH? (NEET Pattern Based)
  1. Klebsiella (NEET Pattern Based)
  2. Vibrio
  3. Pseudomonas
  4. E.coli
38. 16Obligate intracellular organism is: (NEET Pattern Based)
  1. Mycoplasma
  2. Chlamydia
  3. Cryptococcus
  4. Helicobacter
39. All of the following organisms are known to survive Intracellularly except: (AI 2005)
  1. Neisseria meningitides
  2. Salmonella typhi
  3. Streptococcus pyogenes
  4. Legionella pneumophila
40. Viable but not cultivable state is seen in which of the following organism: (PGI 2000)
  1. M. leprae
  2. M.tuberculosis
  3. V.cholerae
  4. Staph. aureus
  5. Srept. pyogenes
41. Which of the following are intracellular: (PGI 2000)
  1. Virus
  2. Chlamydia
  3. Mycoplasma
  4. Rickettsia
42. Antigen activity on bacterial surface occurs due to: (PGI May 2012)
  1. Capsular polysaccharide
  2. Flagella
  3. Nuclear membrane
  4. Cell wall
43. Microorganism adhere to host cell by: (PGI Dec 2008)
  1. Lipoteichoic acid
  2. Lectin
  3. Fimbriae
  4. Capsule
  5. Viral capsid
44. The endotoxin which leads to endotoxic shock is actually: (AIIMS 2000)
  1. Lipoprotein
  2. Lipopolysaccharide
  3. Polysaccharide
  4. Polyamide
45. Endotoxin from gram negative organism is: (AIIMS June 2000)
  1. Polysaccharide
  2. Glycoprotein
  3. Lipoprotein
  4. Lipopolysaccharide
46. True about exotoxin:
  1. LPS in nature
  2. Heat stable
  3. Not antigenic
  4. Toxoid can be prepared17
EXPLANATIONS
 
History
1. Ans. (b) (Kleb-Loeffler's bacillus) Ref: Ananthanarayan 9 e/p236
  • Corynebacterium diphtheriae is also called as Klebs-Loeffler bacillusw
2. Ans. (b) (Microscope), Ref: Ananthanarayan 9/e p3, 8/e p3, Baveja's Microbiology 5/e p3 Arora'Microbiology 4/e p3
  • Antony van Leeuwenhoekfrom Holland was the first person to observe and describe microorganisms accurately by using lenses and was considered as the founder of simple microscope (1694).
  • Zacharias Jansen & his father – described the first compound microscope in 1590
  • Ernst Ruska-Invented Electron Microscope in 1931
3. Ans. (a) (Mc Mohan and Pugh),Ref: Park 21/e p32
  • MacMahon, Pugh, and Ipsen (1960) recognized that chains of causation suffered from “the defect of oversimplification” and proposed a new model that they called a “web of causation.”
  • Concept of causation of disease.
  • Spontaneous generation of life- Life can be created by chemical reactions. It was disproved by Louis Pasteur.
  • Germ theory of life(proposed by Louis Pasteur) suffered the weakness of being a single cause theory i.e. one to one relationship between causative agent and disease, i.e.[Agent→ Man→ Disease]. Not everyone exposed to an agent develops disease.
  • The classic epidemiologic triadof host, agent, and environment provided a better model for understanding the complex realities of disease causation.
  • Multifactorial causation- Proposed by Pettenkofer and Munich.
  • Theory of web of causation- Suggested by MacMahon, Pugh, and Ipsen (1960) in their book “Epidemiologic principles and methods’ that disease is due to the result of complex interrelationship between all the predisposing factors with variable relative risk.
4. Ans. (a) (Vaccination of small pox)
Ref: Ananthanarayan 9/e p4, 8/e p4, Arora'Microbiology 4/e p4, Baveja's Microbiology 5/e p4
  • Vaccination of small pox by using cow pox was proposed by Edward Jenner.
  • Louis Pasteur prepared the vaccines for – Anthrax, Rabies, Cholera (CAR)
  • For other contributions of Louis Pasteur- Refer text.
5. Ans. (a) (Sir Ronal Ross)
Ref: Ananthanarayan 9/e p4,8/E p4, Arora'Microbiology 4/e p4, Baveja's Microbiology 5/e p4
6. Ans. (a) (Edward Jenner)
Ref: Ananthanarayan 9/e p4,8/e p4, Arora'Microbiology 4/e p4, Baveja's Microbiology 5/e p4
  • Small pox vaccine- discovered by Edward Jenner (He used cowpox)
7. Ans. (b) (A-II,B-I,C-IV,D-III)
Ref: Ananthanarayan 9/e p4, 8/e p4, Arora'Microbiology 4/e p4, Baveja's Microbiology 5/e p4
  • Edward Jenner- Small pox
  • Louis Pasteur- discovered vaccine for Rabies, Anthrax and Cholera
  • Albert Calmette, Camille Guerin- discovered vaccine for Tuberculosis
  • Dr. Pierre Lépine (1901-1989), a polio specialist at the Pasteur Institute
8. Ans. (a) (A-II,B-I,C-III)
Ref: Ananthanarayan 9/e p4, 8/e p4, Arora ‘Microbiology 4/e p4, Park 22/e p5,31
  • Refer the previous explanation18
 
Bacterial Cell Biology and Structure
9. Ans. (a) (Shigella)Ref: Ananthanarayan 9/e p284
  • Non Motile organism- Shigella, Klebsiella
10. Ans. (c) (Attachment)Ref: Ananthanarayan 9/e p20
  • Fimbria or pili helps in attachment
  • Flagella is the organ of locomotion, it is proteinaceous and antigenic
11. Ans. (a) (Clostridium tetani)Ref: Ananthanarayan 9/e p259
  • Swarming is exhibited by- Cl.tetani, Proteus, Vibrio parahemolyticus & Vibrio alginolyticus
12. Ans. (d) (Escherichia coli)Ref: Ananthanarayan 9/e p275, 8/e p273-74
  • Mostly Escherichia coli is not capsulated (except few strains of Enteropathogenic E.coli & uropathogenic E.coli that causes pyelonephritis).
  • List of capsulated bacteria- Refer text.
13. Ans. (a), (b) and (c) (Chlamydia, Rickettsia and Mycoplasma)Ref: B. Arora's Microbiology 3/e p42
  • Bacteria are free-living, microscopic, unicellular organisms capable of performing all essential functions of life.
    • ✓ They possess both DNA and RNA
    • ✓ They lack chlorophyll
    • ✓ They are prokaryotes.
  • Prions are the infectious agents, proteinaceous in nature and devoid of both DNA and RNA ,Ananthanarayan 8/e p552
  • Bacteriophage is a virus that infects bacteria. Ref: Ananthanarayan 9 e/p456, 8/e p455
14. Ans. (c) (fungus)Ref: Ananthanarayan 9 e/p427, 8/e p425
  • Kingdom Protista- divided into following groups:
    • ✓ Prokaryotes – include bacteria and blue green algae
    • ✓ Eukaryotes – include fungi, algae (other than blue green), protozoa, helminths and slime moulds
    • ✓ See the table from the text for detail.
15. Ans. (b) (Organized cell organelles) Ref: Ananthanarayan 9 e/p427-28
  • See the table from the text for detail.
16. Ans. (a) and (d) (Bacteria, Mycoplasma)Ref: Ananthanarayan 9 e/p9,427
  • ✓ See the table from the text for detail.
17. Ans. (b) and (d) (Klebsiella and Cryptococcus)Ref: Ananthanarayan 9 e/p280,613, 8/e p278, 610
  • Caspule of Klebsiella can be made out in Gram-stained smears as haloes around the bacilli.
  • Cryptococcus is only pathogenic fungi which is capsulated.
  • All capsulated microbes produce mucoid colonies in culture media.
  • List of capsulated bacteria-Refer text
18. Ans. (a) and (c) (Mitochondria always absent and divide by binary fission)
Ref: Ananthanarayan 9 e/p16,22, 8/e p13, 426
Properties of bacteria:
  • They are prokaryotes, multiply by binary fission
  • Cellular organelle- Absent like Mitochondria, Golgi apparatus, endoplasmic reticulum and lysosomes
  • Only ribosome present
  • Cell wall is made up peptidoglycan but doesn't conatin sterols (except in Mycoplasma).
  • They measure in micrometers, hence can be visualized in light microscopes, electron microscope is not essential.
19. Ans. (b) (Teichoic acid)Ref: Ananthnarayan 9th/p15
  • Refer chapter review.19
 
Staining Techniques and Microscopy
20. Ans. (a) (Methyl violet→ Iodine→ Acetone→ Carbol fuchsin)
Ref: Ananthanarayan 9/e p13,8/e p15, Baveja's Microbiology 5/e p12
Refer the explanation below (Answer 20)
21. Ans. (a) (Methylene blue) Ref: Ananthanarayan 9/e p13,8/e p15, Baveja's Microbiology 5/e p12
  • Gram stain has four steps:
    • ✓ Primary stain- Methyl violet, crystal violet or gentian violet for 1 min.
    • ✓ Mordant as Gram's iodine for 1 min
    • ✓ Decolorizer- Acetone (for 2 sec) or absolute alcohol for 20-30 sec
    • ✓ Counterstain- Safranin or dilute carbol fuchsin or neutral red for 1min
  • Note- Methylene blue is used a counter stain for acid fast stain
22. Ans. (c) (India ink)Ref: Ananthanarayan 9/e p12, 8/e p14, Baveja's Microbiology 5/e p11
  • Negative stain- Demonstration of capsule and thin bacteria such as Spirochetes. E.g include India ink and Nigrosin stain. Both background and organisms are stained while the structure to be demonstrated (i.e capsule surrounding the organism) is not stained.
23. Ans. (b) (Mycoplasma) Ref: B. Arora 3/e p18-19
  • List of acid-fast micro-organisms- Refer text
  • Mycoplasma are cell wall deficient bacteria, hence cannot be stained by routine stains, but are stained by Geimsa staining.
24. Ans. (a), (b), (c) and (d) (Spores, Nocardia, Legionella and Rodococcus)Ref: B. Arora 3/e p18-19
Journal - J Clin Microbiol. 1986 December; 24(6): 1102–1103 states:
Legionella micdadei is an Acid-fast bacillus found in sputum of patient suffering from pneumonia”.
Other options already explained.
25. Ans. (a) (200 nm)Ref: LM Prescott's Microbiology 5th /chapter 2.2/p21, LM Prescott 5th /chapter 2.2/p2 states………..
“At best, a bright-field microscope can distinguish between two dots around 0.2 µm (200nm) apart (the same size as a very small bacterium)”.
  • Resolution–Ability to distinguish two points separate. Resolution power of:
    • ✓ Human eye - 0.2 mm
    • ✓ Light microscope - 0.2 µm/200 nm
    • ✓ Electron microscope - 0.2 nm
  • Resolution power is directly proportional to refractive index, hence can be improved by using oil (refractive index 1.5) instead of air (refractive index 1).
  • Ananthanarayan 8/e p13 states “the limit of resolution of light microscope is 300nm”
  • You should not go with this option; as far as Microscopy is concerned LM Prescottis the standard reference.
26. Ans. (a) and (e) (Thioflavin T and Auramine)Ref: B. Arora 3/e p18-19, Journal-J Struct Biol. 2005 Sep; 151(3)
  • Thioflavin T is a benzothiazole dye (fluorescent dye) that exhibits enhanced fluorescence upon binding to amyloid fibrils and is commonly used to diagnose amyloid fibrils in Alzheimer's disease.
………….. Journal-J Struct Biol. 2005 Sep;151(3)
  • Auramine andrhodamine fluorescent dyes used to demonstrate Tubercle bacilliB. Arora 3/e p18-19
  • Other fluorescent dyes:
    • ✓ Calcofluor white- stains fungal cell wall
    • ✓ Acridine orange- stains malarial parasites
27. Ans. (b) (Flagella)Ref: Ananthanarayan 9 e/p11, 8/e p20, LM Prescott 5th /chapter 2.2/p24
  • Dark ground microscopy is used for:
    • ✓ Visualization of live thin organism (Spirochaete) and flagella
    • ✓ Demonstration of motility (Spirochaete and Vibrio) of bacteria
  • Refractile bacterial components such as endospores and inclusion bodies are better visualized by phase contrast microscopy as they have refractive indexes markedly different from that of water.
2028. Ans. (b) (M.leprae)Ref: Ananthanarayan 9 e/p364, 8/e p364
Other options already explained.
29. Ans. (d) (Dark Field microscope)Ref: Ananthnarayan 9th/p15
  • Refer chapter review.
30. Ans. (a) (c) (Pneumococcus, Meningococcus)Ref: Ananthnarayan 9th/p13
India ink stain is used to demonstrate the capsule. Among cocci, Pneumococcus, Meningococcus are capsulated.
31. Ans. (b) (If observed under microscopy before decolourization, both Gram +ve and –ve bacteria appear same) Ref: Ananthanarayan 9/e p13.
  • In Gram stain-
    • ✓ Under decolorization – Everything will look gram positive (violet)
    • ✓ Over decolorization- Everything will look gram negative (pink)
  • Heating is done in acid fast stain (not for gram stain)
 
Bacterial Growth and Nutrition
32. Ans. (a) (Campylobacter)Ref: Ananthanarayan 9 e/p24, 8/e p25, SC Parija 1/e p26
  • Microaerophilic bacteria: Require small amount of oxygen (5%). E.g. Campylobacter,Helicobacter and Mycobacterium bovis.
  • On basis of their oxygen requirements bacteria are classified as: Obligate aerobes, Facultative anaerobes, Microaerophilic, Obligate anaerobe and aerotolerant organism.
  • For detail explanation: Refer text
33. Ans. (d) (Chlamydia)Ref: Ananthanarayan 9 e/p415, 8/e p388, 415
  • Bacteria that cannot grow both on artificial media & in cell line -M.leprae, Treponema pallidum
  • Bacteria that cannot grow on artificial media but can grow in cell line - Chlamydia and Rickettsia
  • Mycoplasma and Ureaplasma can be cultivated in medium containing horse and human serum with yeast extract
  • Proteus grows easily in all culture media including basal medium like nutrient agar.
34. Ans. (a) and (c) (M.lepare and Treponema pallidum)Ref: Ananthanarayan 9 e/p371,364, 8/e p372, 364 Already explained
  • Bacteria that cannot grow on artificial media include:
    • ✓ M.leprae, Treponema pallidum, Chlamydia and Rickettsia:
35. Ans. (b) (Lag phase- Log phase- Stationary phase- Decline phase)Ref: Ananthanarayan 9 e/p22, 8/e p24
  • Bacterial growth curve include four phases that take place in the following sequence:
    • Lag phase→ Log phase →Stationary /plateau phase→ Decline phase
    • ✓ For detail explanation- refer text
36. Ans. (b) (20minutes)Ref: Ananthnarayan 9th /p22
Population doubling time or the generation time of- E.coli- 20min, M.tuberculosis-20hr and M.leprae-20days
 
Microbial Pathogenesis
37. Ans. (b) (Vibrio)Ref: Ananthanarayan 9 e/p25,8/e p24
  • Vibrio survives in alkaline pH of 8.6
38. Ans. (b) (Chlamydia)Ref: P.Chakraborty's Microbiology 2/e p204
  • Refer the explanation below.
39. Ans. (c) (Streptococcus pyogenes)Ref: P.Chakraborty's Microbiology 2/e p204
Intracellular survival may be obligatory or facultative.
21Examples of intracellular pathogens:
Bacteria
Fungus
Virus
Protozoa
M.tuberculosis
M.leprae
Neisseria
Listeria monocytogens
Legionella pneumophila
Yersinia pestis
Brucella spp.
Rickettsia
Chlamydia
Francisella
Shigella dysenteriae
Salmonella typhi and other spp.
Histoplasma capsulatum
All viruses
Toxoplasma gondii
Leishmania donovani
Trypanosoma cruzi
Plasmodium
Babesia
Cryptosporidium
Microsporidium
40. Ans. (a), (c) (M.lepare, V.cholerae)Ref: Ananthanarayan 9 e/p364, 8/e p364
  • M.lepare-Already explained.
  • Viable non-culturable state of V. cholerae is one of the mechanisms of survival of during inter-epidemic periods……..Journal: Appl Environ Microbiol. 2004 Dec;70 (12):7481-9
41. Ans. (a), (b), (d) (Virus, Chlamydia and Rickettsia)Ref: P. Chakraborty 2/e p204
Already explained
42. Ans. (a),(b),(d) (Capsular polysaccharide, Flagella, Cell wall),Ref: Ananthanarayan 9 e/p16,18, 19, 8/e p272
  • Cell wall and its appendages like pili, flagella and surrounding capsule- All are antigenic.
  • Many bacteria like E.coli can induce antibodies formation against flagella (H), LPS (O) and capsule (K) antigens.
  • Nuclear membrane is not present on bacterial surface.
43. Ans. (a), (b), (c), (d) and (e) (Lipoteichoic acid, Lectin, Fimbriae, Capsule and Viral capsid)
Ref: Ananthanarayan 9 e/p16,18, 19
  • First step of infection and disease development is microbial attachment to host cell
Adhesins are factors involved in microbial adhesion.
  • Lipoteichoic acid mediates microbial adhesion, present on the cell wall of Gram positive cocci like Streptococcus pyogenes, Staphylococcus aureus. …………..Bailey and Scott 12/e p799
  • Lectin antigens are the carbohydrate residues present on microbial surface of Entamoeba histolytica, Streptococcus pneumoniae and Pseudomonas aeruginosa which helps in adhesion to human cells.
    ……………. Journal-Eur J Clin Microbiol. 1987 Oct;6(5):591-3
  • Fimbriae/pilus is the organ of adhesion present on surface of many bacteria - Bailey and Scott 12/e p799
    • Gonococcus, Enterobacteriaceae, Pseudomonas spp., Bordetella pertussis, Vibrio cholerae
  • The capsule has various functions: Ananthanarayan 9 e/p18, 8/e p19
    • ✓ It protects the bacteria from phagocytosis, It helps in bacterial adhesion,helps in biofilm formation
  • Viral capsid- One of the major function of Viral capsid is to introduce viral genome into host cells by adhering to host cell surfaces. Ananthanarayan 9 e/p428, 8/e p426
44. Ans. (b) (Lipopolysaccharide)Ref: B. Arora 3/e p96, Ananthanarayan 9 e/p16, 8/e p78
  • Endotoxins are components of outer membrane of GNB.
  • They are lipopolysaccharide in nature.
  • Released from bacterial surface by natural lysis of bacteria.
  • Produced by Gram –ve Bacilli only
  • They are poor antigens
  • They exert wide spectrum of effects on host- fever, shock syndrome
  • Larger amounts of endotoxin may cause irreversible shock seen in association with a fulminating gram-negative bacteraemia.
45. Ans (d) (Lipopolysaccharide)Ref: B. Arora 3/e p96, Ananthanarayan 9 e/p16, 8/e p78
Already explained
46. Ans (d) (Toxoid can be prepared)Ref: Ananthanarayan 9 e/p75
Refer text

Sterilization and Disinfection, Hospital Waste Management, Bacteriology of Water, Milk and AirCHAPTER 1.2

STERILIZATION
  • Defined as a process by which an article, surface or medium is freed of all living microorganisms either in the vegetative or spore state.
  • It results in reduction of >106 log colony forming unit of spores.
DISINFECTION
  • Process of destruction or removal of all pathogenic organisms or organisms capable of giving rise to infection but not bacterial spores.
  • It results in reduction of >103 log colony forming unit of most of the microorganism but not spores
Asepsis: Practice that prevents the entry of infectious agents into sterile living tissues and thus prevents infection.
 
METHODS OF STERILIZATION
CLASSIFIED AS
  • Physical methods of sterilization – Example- Heat, Filtration, Radiation & Sound (sonic) waves.
  • Chemical methods of sterilization.
 
PHYSICAL METHODS OF STERILIZATION
 
Dry Heat
  • The dry heat kills microorganisms by (CODE)
    • Charring, Oxidative damage, Denaturation of protein, Elevated electrolyte
  • Types- Sunlight, Flaming, Incineration and Hot air oven
  • Hot air oven:
    • Temperature maintained- 160°C for 2 hr
    • Materials sterilized – glass ware, forceps, liquid paraffin, grease, fat, glycerol, dusting powder.
    • Sterilization Control: Clostridium tetani (non toxigenic strain), Browne's tube, Thermocouple
 
Moist Heat
Mechanism-Coagulation and Denaturation of Protein:
  1. Sterilization at a temperature below 100ºC- e.g
    • Pasteurization: Holder method (63ºC for 30 min), Flash method (72ºC for 20s) followed by rapid cooling to 13ºC. It is inaffective for Coxiella burnettii
    • Vaccine bath: 60ºC for 1 hr. Used for serum & body fluid sterilization
    • Inspissation (fractional sterilization): At 80ºC for 30 min for 3 consecutive days. Suitable for egg based media (LJ media & Dorset's egg media) and serum based media (Loeffler's serum slope).
  1. Sterilization at a Temperature of 100ºC:
    • Boiling, Koch's or Arnold's steam sterilizer
    • Tyndalisation (Intermittent sterilization)-1000c for 20 min for 3 days. Used for Sugar solution, gelatin
  2. 23Sterilization at a Temperature above 100ºC- Autoclaves
    • 121°C for 15min (at 15 psi pressure)
    • Sterilization Control – Bacillus stearothermophillus and Browne's tube and Chemical indicators
    • Materials to be sterilized in autoclave- Culture media, instruments, dressings and numerous other items that can withstand high temperatures and pressures.
 
Filtration
  • Excellent way to reduce the microbial population in solutions of heat-labile materials- like vaccine, antibiotics, toxin, serum and sugar.
  • Bacteriological filters:
    • Candle filters -Diatomaceous earth (e.g. Berkefeld filters) & Unglazed porcelain (e.g. Chamberland filters)
    • Asbestos filters (Seitz and Sterimat filters),
    • Sintered glass filters
  • Membrane filters:
    • Made up of cellulose acetate, cellulose nitrate, polycarbonate, polyvinylidene fluoride, or other synthetic materials.
    • Average pore diameter- 0.22µ, Used to study the size of viruses
  • HEPA & ULPA filters:
    • HEPA- (High-efficiency particulate air)- HEPA filter removes 99.97% of particles that have a size of 0.3µm. Used for biosafety cabinets & laminar airflow systems.
    • ULPA - (Ultra-low particulate/penetration air)- A ULPA filter can remove from the air at least 99.999% of dust, pollen, mold, bacteria and any airborne particles with a size of 120 nanometers (0.12 micron) or larger.
 
Radiation
Types
Used for Sterilization of
Ionizing radiation
Example- Cobalt 60, X-rays, gamma rays and cosmic rays
Mechanism- Breakage of DNA without temperature rise. (Cold sterilization)
Dose- 2.5 Rad
Disposable rubber or plastic syringes, infusion sets, catheters, catgut sutures, bone & tissue grafts & adhesive dressings.
Non-ionizing radiation -
Example- infra red and ultraviolet radiations Dose-250-300nm wavelength for 30min
Clear surfaces in OTs, laminar flow hoods
Sound (sonic) waves
  • High-frequency sound waves disrupt cells.
 
DISINFECTION (CHEMICAL METHOD OF STERILIZATION)
 
Types of Disinfectants
  • Phenolic compounds - (e.g phenol, lysol, cresol, dettol),
  • Diphenyl compounds - (e.g hexachlorophene and chlorheximide),
  • Halogens – e.g fluorine, bromine, chlorine, iodine, providone iodine (10%) and hypochorite(0.5-5%).
  • Alcohols -Ethyl alcohol and isopropyl alcohol
  • Aldehydes -Formaldehyde and glutaraldehyde
  • Gaseous agents -Ethylene oxide, formaldehyde gas and betapropiolactone
  • Surface active agents - detergents, wetting agents and emulsifiers.
  • Oxidising agents - halogens, hydrogen peroxide, potassium permanganate
  • Acridine dyes- (e.g. acriflavine, euflavine, proflavine and aminacrine)
  • Aniline dyes - (e.g. gentian violent, crystal violet and malachite green)
24Classification of Disinfectant Based on Efficacy:
Low level- Kill vegetative bacteria + enveloped /medium-large virus
Intermediate level- Low level + fungi
High level- Intermediate level + MTB + Non enveloped and small sized virus
Chemical sterilant – High level + spore + Cryptosporidium cyst
Decreasing order of resistance – Prion > Cryptosporidium cyst> spore> Non enveloped and small sized virus> MTB > fungi > vegetative bacteria + enveloped /medium-large virus
Prions are sterilized by: 0.5% Hypochlorite for 2 hr or 1N NaOH for 1 hour or Autoclave for 134°C for1-1.5 hour
 
Testing of Disinfectants:
  • Phenol coefficient (Rideal Walker) test
    • Phenol coefficient is determined by the dilution of the disinfectant in question which sterilizes the suspension of S. Typhi in a given time divided by the dilution of phenol which sterilizes the suspension in the same time.
  • Chick Martin test
    • It is a modification of Rideal and walker test
    • Disinfectants act in the presence of organic contaminants (e.g. dried yeast, faeces, etc.) to simulate the natural conditions.
  • Capacity (Kelsey-Sykes) test
    • Capacity of a disinfectant to retain its activity when repeatedly used microbiologically
  • In-use (Kelsey and Maurer) test
    • Which determines whether the chosen disinfectant is effective in actual use in Hospital practice
 
Sporicidal Agents
  • EFGH- Ethylene oxide, Formaldehyde, Glutaral dehyde, Hypochlorite, H2O2
  • O- Phthalic acid, Peracetic acid
  • Autoclave, Hot air oven, Plasma sterilization
 
Biological Sterilization Indicator
Hot air oven
Clostridium tetani non toxigenic strain, B.subtilis subsp niger
Autoclave
Bacillus stearothermophillus, Bacillus subtilis subsp niger
Filtration
Brivundimonas dimunata, Serratia
Ionizing radiation
Bacillus pumilus
Ethylne oxide
Bacillus globigi
Plasma sterilization
Bacillus stearothermophillus, Bacillus subtilis subsp niger
Various Methods of Sterilization
Material
Method of Sterilization
Clinical thermometer
Isopropyl alcohol
Paraffin, glass syringe, flask, slide, oil, grease, fat, glycerol
Hot air oven
OT, entryway, ward, lab fumigation, Preservation of anatomical specimen, woolen blanket
Formaldehyde gas > UV > BPL (beta propiolactone)
Cystoscope, bronchoscope
Orthophthaldehyde > glutaraldehyde 2% (cidex)
Heart lung machine, respirator, dental equipments
Ethylene oxide
Vaccine, sera, antibiotic, sugar, antibiotic & body fluid
Filtration
Sharp instrument
Cresol
Milk
Pasteurization
Plastic syringe, cadgut suture, swab, catheter, bone & tissue grafts, adhesive dressing
Ionizing radiation
Culture media, all instrument except sharp and glass, all suture materials except cadgut
Autoclave
Metallic inoculation wire
Red heat by Bunsen burner
Infective material like soiled dressing, bedding, animal carcasses
Burning (incineration)
Metallic surgical instruments
Autoclave, hot air oven, infra red radiation25
Water
Chlorine as hypochlorite 0.2%
Skin
Tinture iodine, spirit (70% ethanol), savlon
Contact lens
H2O2
Germicide & theirs Concentrations
Level of Disinfectant
Bacteria & Enveloped Viruses
Unenveloped Viruses
M. Tuberculosis
Fungi
Spore
Inactivated by Organic Matter
Glutaraldehyde (2%)
High/CS
+
+
+
+
+
-
Formaldehyde (3-8%)
High
+
+
+
+
+
H2O2 (3-25%)
High/CS
+
+
+
+
+
+/-
Chlorine (100-1000 ppm of free chlorine)
High
+
+
+
+/-
-
+
Isopropyl alcohol
Intermediate
+
+/-
+
+
-
+/-
Phenol (0.4-5%)
Intermediate
+
+/-
+
-
-
-
Halogens
Intermediate
+
+
+/-
+
-
+
Quaternary ammonium compunds
Low
+
-
-
-
-
+
Alcohol (70-95%)
Intermediate
+
+
+
+
-
+
CS- Chemical Sterilant, + effectively kills, - Unable to kill, +/- variably kills
 
Spaulding's criteria of devices
Clinical device
Definition
Example
Sterilization done by
Critical device
Enters a normally sterile site
Surgical Instruments, cardiac and urinary catheters, Implants, eye & dental instruments,
High level disinfectant
Semi-critical device
Comes in contact with mucous membranes or minor skin breaches
Respirators, anesthesia equipments, endoscopes, laryngoscope, rectal/vaginal /esophageal probes
High level disinfectant
Or intermediate level disinfectant
Non-critical devices
Comes in contact with intact skin
BP cuff, ECG electrodes, bedpans, crutches. Stethoscope, thermometer
Intermediate levelor low level disinfectant
Medical equipments
Does not come in direct contact with patient
Examination table, computers
Low level disinfectant
 
HOSPITAL WASTE MANAGEMENT
Biomedical waste
  • A vast amount of waste is generated in the process of health care, research, testing, or related procedures on human beings or animals conducted in hospitals, clinics, labs.
  • It can act as a source of infection to residents or visitors of hospitals
  • This form of waste requires special attention for its safe disposal.
    • Incineration
    • Deep Burial
    • Autoclave
    • Chemical treatment
    • Microwaving
Waste Category
Category No 1
Human anatomical waste (human tissues, organs and body parts).
Category No 2
Animal waste (animal tissues, organs, body parts carcasees, bleeding parts, fluid, blood and experimental animals used in research, waste generated by verterinary hospitals/ colleges, discharge from hospitals, animal houses).
Category No 3
Microbiology and biotechnology waste (wastes from laboratory cultures, stocks or specimens of micro-organisms live or attenuated vaccines, human and animal cell culture used in research and infectious agents from research and industrial laboratories, wastes from production of biologica, toxins, dishes and devices used for transfer of cultures).
Category No 4
Waste sharps (needles, syringes, scalpels blades, glass etc. that may cause puncture and cuts. This includes both used and unused sharps).26
Category No 5
Discarded medicines and cytotoxic drugs (wastes comprising of outdated contaminated and discarded medicines).
Category No 6
Solid waste (items contaminated with blood and body fluids including cotton, dressings, soiled plaster casts, line beddings, other material contaminated with blood).
Category No 7
Solid waste (disposable items other than the waste sharps such as tubing, catheters, intravenous sets etc.).
Category No 8
Liquid waste (waste generated from laboratory and washing, cleaning, house-keeping and disinfecting activities).
Category No 9
Incineration ash (ash from incineration of any bio-medical waste).
Category No 10
Chemical waste (chemicals used in production of biological, chemicals, used in disinfection, as insecticides, etc).
Colour Coding
Waste Category
Treatment Options
Yellow (plastic bag)
1,2,3,6
Incineration/deep burial
Red (plastic bag)
3,6,7
Autoclaving/Micro waving/ Chemical Treatment
Blue/White (puncture proof)
4,7
Autoclaving/Micro waving/ chemical treatment and destruction/shredding
Black (plastic bag)
5,9,10
Disposal in secured land fill
 
BACTERIOLOGY OF WATER, AIR AND MILK
 
Bacteriology of Water
Indicator of fecal contamination of water:
  • Thermo tolerant E.coli (recent contamination)- most definite
  • Coliform count
  • Fecal streptococci
  • Clostridium perfringens
Detection of coliform count:
  • Presumptive coliform count:
    • Multiple tube method is used
    • MacConkey fluid medium (broth) is used
    • Most probable number detected by McCrady table- probable no. of bacteria per 100 ml
  • Differential coliform count- Eijkman test-
    • Done to detect Thermo tolerant E.coli
    • By growth at 44°c with gas production & +ve indole test
  • Membrane filtration method
 
Bacteriology of Air
  • Bacteria count should not exceed:
    • 50/ foot3 in office, homes and factories
    • 10/ foot3 in operation theatres
    • 1/ foot3 in operation theatre for neurosurgery
  • Measurement of air contamination:
    • Settle plate method
    • Slit sampler method
Bacteriological examination of milk can be done by:
  • Viable count
  • Coliform count
  • Phosphatase test
  • Turbidity test
  • Methylene blue test
  • Examination of specific pathogen- Tubercle bacilli & Brucella
27MULTIPLE CHOICE QUESTIONS
 
STERILIZATION
1. Best method of sera sterilization is: (APPG 2014)
  1. Filtration
  2. Autoclaving
  3. Radiation
  4. Heating
2. Heating at 630C for 30 min doesn't kill: (PGI Nov2012)
  1. Prion
  2. Coxiella
  3. Tubercle bacilli
  4. Spores
3. Tyndallization is a type of: (NEET Pattern Based)
  1. Intermittent sterilization
  2. Boiling
  3. Pasteurization
  4. Autoclaving
4. Browne's tube is used for indicator of efficacy of: (NEET Pattern Based)
  1. Heat sterilization
  2. Chemical sterilization
  3. Filtration
  4. Ultraviolet rays
5. Which of the following is used to test the efficiency of sterilization in an autoclave? (AIIMS Nov 2011, AIIMS Nov 2010, MHPG 2014)
  1. Clostridium tetani
  2. Bacillus stearothermophilus
  3. Bacillus pumilus
  4. Bacillus cereus
6. Spore of which bacteria is used as sterilization control of Plasma sterilization (AIIMS Nov 2010)
  1. B.subtilis
  2. B. pumilis
  3. Cl. tetani
  4. B. stearothermophilus
7. Condition required for autoclave is ? (DNB June 2012)
  1. 121 C temperature for 20 min
  2. 121 C temperature for 15 min
  3. 100 C temperature for 60 min
  4. 100 C temperature for 90 min
8. Which of the following is most resistant to sterilization? (AI 2012, 2008)
  1. Cysts
  2. Prions
  3. Spores
  4. Viruses
9. Choose the correct ones for the decreasing order of resistance to sterilization: (PGI Dec 2007)
  1. Prions, Bacterial spores, Bacteria
  2. Bacterial spores, Bacteria, Prions
  3. Bacteria, Prions, Bacterial spores
  4. Bacterial spores, Prions, Bacteria
  5. Bacterial spores, Prions, Bacteria
10. Sterilizing agents include: (PGI June 2002)
  1. Dry heat
  2. Ethylene oxide
  3. Ether
  4. Alcohol
  5. Chlorhexidine
11. Which of the following is true about pasteurization: (AI 2000)
  1. It kills all bacteria and spores
  2. It kills all bacteria except thermoduric bacteria
  3. It kills 95% of microorganisms
  4. All bacteria are destroyed
 
DISINFECTION
12. Endoscope is sterilized by (NEET Pattern Based)
  1. Glutaraldehyde
  2. Formaldehyde
  3. Autoclaving
  4. Hot air oven
13. Prions are best killed by ? (NEET Pattern Based)
  1. Autoclaving at 121°C
  2. 5% formalin
  3. Sodium hydroxide for 1 hr
  4. Sodium hypochloride for 10min
14. About disinfectant –all true except: (AIIMS May 2011)
  1. Hypochlorite is virucidal,
  2. Glutaral dehyde is sporicidal,
  3. Phenol can act in presence of organic material
  4. Ethanol cannot kill Mycobacteria
15. Sterilization of fibreoptic is done by? (DNB DEC 2012, AIIMS Nov 2003)
  1. Glutaraldehyde
  2. Chlorine
  3. Autoclaved. Phenol
16. Which is false about spaulding classification? (AIIMS Nov 2010)
  1. Non critical items also included in classification
  2. Semicritical items- contact with mucus mebrane
  3. Semicritical items- needs low disinfectant
  4. Cardiac catheter- e.g of critical items
17. Which of the following statements regarding disinfectants is not true: (AI 2009)
  1. Hypochlorites are bactericidal and inactivated by organic matter
  2. Glutaraldehyde is sporicidal and not inactivated by organic matter
  3. Formaldehyde is bactericidal, sporicidal and virucidal
  4. Phenol is bactericidal and readily inactivated by organic matter
18. Phenol Coefficient indicates: (DNB DEC 2012, JIPMER 2009)
  1. Efficacy of a disinfectant
  2. Dilution of a disinfectant
  3. Quantity of a disinfectant
  4. Purity of a disinfectant
19. 28Which disinfectant works by plasma membrane damage? (DNB June 2011)
  1. Alcohol
  2. Metalic salt
  3. Phenol
  4. Halogen Compounds
20. Sputum can be disinfected by all except: (AIIMS May 2012)
  1. Auto claving
  2. Boiling
  3. Cresol
  4. Chlorhexidine
21. Surgical blade is sterilized by? (DNB June 2009)
  1. Autoclave
  2. Gamma radiation
  3. Hot air oven
  4. Steaming
22. Disinfection of sputum is done by: (PGI Dec 2008)
  1. Boiling
  2. Autoclaving
  3. Sunlight
  4. Burning
23. Hospital dressing is best disposed off by: (TN 2008)
  1. Incineration
  2. Dumping
  3. Autoclaving
  4. Burying
24. False about alcohol in disinfection is: (PGI Dec 2007)
  1. Ethanol is used
  2. Isopropyl alcohol is used
  3. Has sporicidal activity
  4. Has bactericidal activity
25. Phenolic disinfectant are: (PGI 2005)
  1. Dettol
  2. Cresol
  3. Lysol
  4. Carbolic acid
  5. Savlon
26. The operating temperature in an ethylene oxide sterilization during a warm cycle is: (AIIMS Nov 2004)
  1. 20-35C
  2. 49-63 C
  3. 68-88 C
  4. 92-110 C
27. Which is a form of cold sterilization: (JIPMER 2003)
  1. Gamma rays
  2. Beta rays
  3. Infrared rays
  4. Autoclave
28. Heat labile instruments for use in surgical procedures can be best sterilized by: (AI 2003)
  1. Absolute alcohol
  2. Ultraviolet rays
  3. Chlorine releasing compounds
  4. Ethylene oxide gas
29. Chest physician performs bronchoscopy in the procedure room of OPD. To make instrument safe for use in the next patient, most appropriate method to disinfect the endoscope is by:
  1. 70% alcohol for 5 min
  2. 2% glutaraldehyde for 20 min
  3. 2% formaldehyde for 10 min
  4. 1% sodium hypochlorite for 15 min
30. Which of following can be reliably used for hand washing: (PGI Dec 2000, June 2001)
  1. Chlorhexidine
  2. Isopropyl alcohol
  3. Lysol
  4. Cresol
  5. Glutaraldehyde
31. Which of these will kill spores? (JIPMER 2010)
  1. Alcohol
  2. Phenol
  3. Chlorine
  4. Aldehyde
32. Sporicidal agents are: (PG June 2009, June 2006)
  1. Glutaraldehyde
  2. Ethylene oxide
  3. Formaldehyde
  4. Benzalkonium chloride
  5. Chlorine
33. Chlorinated disinfectants are all except: (AIIMS May 2001)
  1. Bleaching powder
  2. Sodium hypochlorite
  3. Halozen tablets
  4. Cetrimide
34. Savlon contains: (AIIMS May 2010)
  1. Cetrimide + Chlorhexidine
  2. Cetrimide + + Butyl alcohol
  3. Cetrimide + Chlorhexidine + Butyl alcohol
  4. Cetrimide + Cetavlon
35. Disinfection of water by routine chlorination is classified under: (Tharnataka 2005, 07)
  1. Sterilization
  2. Disinfection
  3. Terminal Disinfection
  4. Pre-current Disinfection
36. Which of the following are used for sterilization of surgical instrument: (PGI MAY 2013)
  1. Ethylene oxide
  2. Gamma radiation
  3. Autoclaving
  4. Glutaraldehyde
  5. Hot air oven
37. All of the following are the tests to check the efficiency of disinfectant EXCEPT: (TNPG 2014)
  1. Chick martin test
  2. Riedel Walker test
  3. Hugh Leifson test
  4. Thelsey Sykes test
 
HOSPITAL WASTE MANAGEMENT
38. A known HIV positive patient is admitted in an isolation ward after an abdominal surgery following an accident. The resident doctor who changed his dressing the next day found it to be soaked in blood. Which of the following would be the right method of choice of discarding the dressing: (AIIMS Nov 2005)
  1. Pour 1% hypochlorite on the dressing material and send it for incineration in appropriate bag
  2. Pour 5% hypochlorite on the dressing material and send it for incineration in appropriate bag
  3. 29Put the dressing material directly in an appropriate bag and send for incineration
  4. Pour 2% hypochlorite on the dressing material and send it for incineration in appropriate bag
39. All of the following categories of biomedical waste can be disposed by Incineration EXCEPT: (MHPG 2014)
  1. Category 1
  2. Category 2
  3. Category 3
  4. Category 4
 
BACTERIOLOGY OF WATER, AIR & MILK
40. 40. Coliform count is done by: (NEET Pattern Based)
  1. VR medium
  2. McConkey's agar
  3. Multiple tube method
  4. None
41. Which of the following is not true regarding surveillance of water pollution? (AIIMS May 2011)
  1. Multiple tube method is used to calculate the number of bacteria
  2. E.coli can be tested by Indole test at 44 C
  3. Sodium thiosulfate is used to neutralize chlorine
  4. Presence of Corynebacterium indicates recent contamination
42. Pasteurized milk is most commonly tested by: (NEET Pattern Based)
  1. Phosphatase test
  2. Coliform test
  3. Catalase test
  4. Oxidase test
43. Yellow bag contains: (PGI Nov 2012, APPG 2012, MHPG 2014)
  1. Incineration ash
  2. Sharps
  3. Human anatomical waste
  4. Solid waste
44. The most suitable disinfectant for decontamination of HIV contaminated endoscope is: (MHPG 2014)
  1. 1% sodium hypochlorite
  2. 2% glutaraldehyde
  3. 5% phenol
  4. 70% ethanol
30EXPLANATIONS
 
STERILIZATION
1. Ans. (a) (Filtration) Ref: Ananthnarayan 9th /p33
Filtration is the best method of sterilization of heat labile liquids such as sera and solutions of sugars or antibiotics, toxins and vaccines. It helps to remove bacteria.
2. Ans (a) (b) (Prions, Coxiella) Ref: Ananthnarayan 9th /p30
Holder's method of Pasteurization(Heating at 63°C for 30 min) is ineffective for Coxiella burnettii, spores and prions.
3. Ans. (a) (Intermittent sterilization) Ref: Ananthanarayan 9/e p31, 8/e p34
  • Tyndalisation (Intermittent sterilization)-100°C for 20 min for 3 days. Used for Sugar solution, gelatin
4. Ans. (a) (Heat sterilization) Ref: Ananthanarayan 9/e p30, 8/e p34
Browne'sIndicatior-A small glass tube containing a red heat-sensitive dye used as a chemical indicator for sterilization. The dye changes colour to green after a defined period of time at a certain temperature. It was invented by Albert Browne Ltd in 1930.
For other sterilization Control- Refer text
5. Ans. (b) (Bacillus stearothermophilus), Ref: Ananthanarayan 9/e p32, 8/e p34
Biological Sterilization Indicator: (Detail list- Refer text)
  • Autoclave- B.stearothermophillus
  • Hot air oven -Clostridium tetani non toxigenic strain
6. Ans. (d) (B. stearothermophilus) Ref: Ananthanarayan 8/e p32, Mackie McCartney's practical microbiology 14/e p817
  • Biological Sterilization Indicator of plasma sterilization: B.stearothermophillus, B.subtilis subspecies niger
    (Detail list- Refer text)
7. Ans. (b) (121°C temperature for 15 min)
Ref: Ananthanarayan 9/e p31-32, 8/e p32, Mackie McCartney's practical microbiology 14/e p817
  • Autoclave – Recommended condition- 121-1240C temperature for 15 min at 1.1 bar pressure
  • Alternate: 134-138°C temperature for 3 min at 2.2 bar pressure
  • Hot air oven- 160°C temperature for 120 min or 1800C temperature for 30 min
8. Ans. (b) (Prions) Ref: Patrick Murray 9/e p69
  • Prions are the highest resistance structure
  • Refer text to know detail
9. Ans. (a) (Prions, Bacterial spores, Bacteria) Ref: Patrick Murray 9/e p69
  • Refer text to know detail
10. Ans. (a) and (b) (Dry heat and Ethylene oxide) Ref: Ananthanarayan 9/e p30,35, 8/e p32, 36
  • Sterilization is defined as a process by which an article, surface or medium is freed of all living microorganisms either in the vegetative or spore state.
  • A sterilizing agent therefore is a sporicidal agent, as it is lethal to even bacterial spores.
Sporicidal agents include:
  • EFGH- Ethylene oxide, Formaldehyde, Glutaraldehyde, Hypochlorite, H2 O2
  • O- Phthalic acid, Peracetic acid
  • Autoclave, Hot air oven (dry heat), Plasma sterilization
3111. Ans. (b) (It kills bacteria except thermoduric bacteria) Ref: SC Parija 1/e p30
  • Pasteurization is a method of moist heat sterilization at a temperature below 100°C
  • The method is extensively used for sterilization of milk and other liquid beverages.
  • Pasteurization inactivates most viruses and destroy the vegetative stages of 97–99% of bacteria and fungi, but they do not kill endospores or thermoduric species and Coxiella burneti
  • Milk is not sterile after regular pasteurization.
  • Newer techniques used to produce sterile milk, where milk is processed with ultrahigh temperature (UHT) of 134°C for 1 to 2 seconds.
 
DISINFECTION
12. Ans. (a) (Glutaraldehyde) Ref: Ananthanarayan 9/e p34, 8/e p36
  • Endoscopes are best sterilized by 2% Glutaraldehyde or ortho-ophthaldehyde.
13. Ans. (c) (NaOH for1 hour) Ref: Ananthanarayan 9/e p34, 8/e p36,37, Mackie McCartney's practical microbiology 14/e p827
  • Prions are sterilized by : 0.5% Hypochlorite for 2 hr or 1N NaOH for1 hour or Autoclave for 134°C for1-1.5 hour
14. Ans. (d) (Ethanol cannot kill Mycobacteria)
Ref: Ananthanarayan 9/e p34, 8/e p36,37, Mackie McCartney's practical microbiology 14/e p827
  • Ethanol acts well against bacteria and Mycobacteria but has poor action against viruses and spores
About Other Options
  • Phenolic disinfectants can act readily even in presence of organic material without getting inactivated
  • Glutaral dehyde is sporicidal (refer text for other sporicidal agents)
  • Hypochlorite is active agent gram positive & negative bacteria, viruses including HIV & HBV, however they are less active against Mycobacteria
15. Ans. (a) (Glutaraldehyde)
Ref: Ananthanarayan 9/e p34, 8/e p36, Guidelines for cleaning Fibreoptic Laryngoscopes, Oct 2005/ https://entuk.org/docs/prof/publications/full
  • A fibre optic is a flexible, transparent fiber made of high quality extruded glass (silica) or plastic used for laryngoscopes.
  • Fibreoptic laryngoscopes are best sterilized by Glutaraldehyde or ortho-ophthaldehyde.
16. Ans. (c) (Semicritical items- needs low disinfectant), Ref: Ananthanarayan 8/e p37
  • Semicritical items are those that come in contact with mucous membranes or minor skin breaches and they needs high to intermediate level disinfectant for sterilization.
  • For the detail of Spaulding's criteria of devices -Refer text
17. Ans. (d) (Phenol is bactericidal and readily inactivated by organic matter)
Ref: Ananthanarayan 9/e p35, 8/e p36-37, SC Parija 1/e p37
  • Option a- Correct- Chlorine and Hypochlorites are markedly bactericidal, virucidal
    • Disadvantages- They are ineffective if used at alkaline pH
    • ✓ Less effective in presence of organic matter
    • ✓ Relatively unstable if exposed to sunlight.
  • Option b- Correct- Gluteraldehyde is sporicidal and not inactivated by organic matter
    • ✓ Specially effective against tubercle bacilli, fungi and viruses
    • ✓ Less toxic and irritant to eyes skin than formaldehyde
  • Option c - Correct- Formaldehyde is bactericidal, sporicidal and virucidal
    • ✓ It is used for sterilizing instruments and heat sensitive catheter
    • ✓ Formaldehyde gas is used for operation theatre fumigation
  • Option d- Incorrect- Phenol is active against wide range of microorganisms
    • They are not inactivated by presence of organic matter. Ananthanarayan 8/e p27
18. Ans. (a) (Efficacy of a disinfectant) Ref: Ananthanarayan 9/e p36, 8/e p38, SC Parija 1/e p40-41
  • Testing efficacy of disinfectants by Phenol coefficient (Rideal Walker) test - Phenol coefficient is determined by the dilution of the disinfectant in question which sterilizes the suspension of S. Typhi in a given time divided by the dilution of phenol which sterilizes the suspension in the same time.
  • For detail of Testing efficacy of disinfectants - Refer text
3219. Ans. (c) (Phenol) Ref: Ananthanarayan 9/e p35, 8/e p36, Chakraworthy's Microbiology/p44
Mechanism of action of various disinfectants:
  • Alcohol- coagulation of bacterial protein
  • Metalic salt- - Protein coagulation & Removal of free sulhydryl group
  • Phenol- Disruption of cell membrane (high conc.), inactivate enzyme & precipitates protein (at low conc.)
  • Halogen Compounds- Oxidation & removal of free sulhydryl group
  • Ethyele oxide- Alkylating amino, carboxyl, hydroxyl and sulfhydril group in protein molecules
  • Quaternary ammonium compounds- Disruption of cell membrane
20. Ans. (d) (Chlorhexidine), Ref: Internet- Centers of disease control.gov, Park 21/e p118
  • The commonest pathogen suspected to be present in sputum- Tubercle bacilli. So it has to be disinfected by an appropriate technique.
  • Chorhexidine is a skin antiseptic, used for burns or hand disinfection. It is not mycobactericidal.
  • Sputum can be disinfected by:
    • ✓ Autoclaving – Ideal method
    • ✓ Other methods include- Burning of the material, Cresol 5% or Phenol or Boiling
  • Steam is the preferred method for sterilizing critical medical and surgical instruments that are not damaged by heat, steam, pressure, or moisture. ...........CDC Guideline,2008
  • Metal instruments and surgical blades are better sterilized by autoclave
22. Ans. (a), (b) and (d) (Boiling, Autoclaving and Burning) Ref: Internet- Centers of disease control.gov.
  • Already explained.
23. Ans. (c) (Autoclaving) Ref: B Arora 3/e p62-63
  • Infectious non sharp wastes like soiled bandages, dressings, soiled cotton, soiled plaster casts, bedding and other solid wastes first segregated in red bag and disposed by Autoclaving.
  • Autoclaving is the best method for treating infectious waste before disposal.
24. Ans. (c) (Has sporicidal activity) Ref: Ananthanarayan 9/e p34, 8/e p36, SC Parija 1/e p39
  • Alcohols - most widely used disinfectants and antiseptics.
  • They are bactericidal and fungicidal but not sporicidal and some lipid-containing viruses are also destroyed.
  • Popular alcohol germicides are ethanol and isopropanol - used in 70 % concentration.
  • They act by denaturing proteins and possibly by dissolving membrane lipids.
  • Used as skin antiseptics.
  • Isopropyl alcohol is used for disinfection of clinical thermometers.
25. Ans. (a), (b), (c), (d) and (e) (Dettol, Cresol, Lysol, Carbolic acid and Savlon)
Ref: Ananthanarayan 9/e p35, 8/e p 37 Wikipedia-Internet
Phenolic disinfectant are:
  • Phenol-1&
  • Carbolic acid
  • Chloroxylenol or Dettol- 4%
  • Chlorophenols
  • Chloroxyphenols
  • Lysol 2.5%
  • Cresol
  • Hexachlorophane-1%
  • Chlorhexidine or Hibitane
  • Savlon/ Cetavlon- (Chlorhexidine + Cetrimide)- 2%, 5%
26. Ans. (b) (49-63 °C) Ref: SC Parija 1/e p38
  • Ethylene oxide is used for gaseous sterilization.
  • It is active against all kind of bacteria, spores and viruses.
  • It kills all types of microorganisms by inhibiting proteins and nucleic acids.
  • Highly inflammable and carcinogenic.
  • Useful to sterilize disposable plastic Petri dishes, sutures, syringes, heart-lung machine, respirators, and dental equipments
  • 33The materials are sterilized if treated with ethylene oxide gas for 4 hours at 55°C, and relative humidity at 40 to 50%. Hence, the most appropriate answer is option-b -49-63 °C
27. Ans. (a) and (b) (Gamma rays and beta) Ref: Ananthanarayan 9/e p33, 8/e p35, Greenwood 16/e p73
  • Ionizing radiations like X-rays, gamma rays, Beta and cosmic rays are highly lethal to DNA and they have high penetrating power.
  • Since there is no appreciable increase in temperature in this method its called as cold sterilization
  • Infra red radiations are considered as hot air sterilization
28. Ans. (d) (Ethylene oxide gas) Ref: Ananthanarayan 9/e p35, 8/e p37, Greenwood 16/e p74
  • Ethylene oxide is mainly is sporicidal and low temperature (550c) is maintained, hence is the better option among given for sterilization of heat labile surgical instruments.
29. Ans. (b) (2% Glutaraldehyde for 20 minutes) Ref: Patrick Murray 9/e p73
  • 2% solution of Glutaraldehyde is bactericidal, tubercuocidal, sporicidal, fungicidal and virucidal
  • Bacterial spores are destroyed by 2% solution of Glutaraldehyde exposure for 3 hours
  • 2% Glutaraldehyde exposure for 20 minutes at 20°C kills Tubercle bacilli.
  • It has no deleterious effect on cement or lenses of instruments such as Cystoscopes and bronchoscopes
  • Most appropriate option- b
30. Ans. (a) and (b) (Chlorhexidine and Isopropyl alcohol) Ref : Greenwood 16/e p80, Ananthanarayan 9/e p34, 35, 8/e p36, 37
  • Antiseptics - chemical disinfectants which can be safely applied on skin or mucous membrane and are used to prevent infection by inhibiting the growth of bacteria and hence they can be safely used for hand washing.
  • Commonly used skin disinfectants
  • Alcohols- Ethyl alcohol and Isopropyl alcohol
  • Halogens- Iodophores (Povidone Iodine)
  • Chlorhexidine (hibitane)
Other given options-
  • ✓ Lysol and Cresols mainly used as disinfectants mopping the floor
  • ✓ Glutaraldehyde is used for sterilization of cystoscopes and bronchoscopes.
31. Ans. (d) (Aldehyde) Ref: Ananthanarayan 9/e p34, 8/e p36, SC Parija 1/e p40
Among the given option best sporicidal agent is Aldehydes
Sporicidal agents:
  • EFGH- Ethylene oxide, Formaldehyde, Glutaraldehyde, Hypochlorite, H2 O2
  • O- Phthalic acid, Peracetic acid
  • Autoclave, Hot air oven, Plasma sterilization
32. Ans. (a), (b), (c) (Glutaraldehyde, Ethylene oxide, Formaldehyde)
Ref: Ananthanarayan 9/e p34, 8/e p36, SC Parija 1/e p40
  • Already explained.
  • Benzalkonium chloride is a surface active agent, widely used as wetting agents, detergents and emulsifiers.
33. Ans. (d) (Cetrimide) Ref: Park 22nd/e p120-121
Halogens include:
  • Bleaching powder
  • Sodium hypochlorite
  • Iodine
  • Iodophore
  • Providone iodine
34. Ans. (a) (Cetrimide + Chlorhexidine) Ref: Park 22/e p119
  • Savlon contains- Cetrimide 3% (Cetavlon) + Chlorhexidine 0.3% (Hibitane)
3435. Ans. (d) (Pre-current Disinfection) Ref: Park 22/p119, 21/e p117, 20/e p93
  • Concurrent disinfection- They are applied as soon as the infective material is discharged from the patient. E.g. Disinfection of sputum, faces, vomit, urine & contaminated clothes etc.
  • Pre-current Disinfection- They are applied prior to occurrence of infection. E.g. chlorination of water, pasteurization of milk, hand washing
  • Terminal disinfection- They are applied after the patient is ceased to be a source of infection. E.g after death or discharge from the hospital.
36. Ans. (c) (d) (e) (Autoclaving, Glutaraldehyde, Hot air oven) Ref: Guideline for Disinfection and Sterilization in Healthcare Facilities, CDC,2008
For the sterilization of surgical instruments- Most medical and surgical devices are made of materials that are heat stable and therefore can be sterilized best by autoclave (moist heat).
  • Heat labile instruments (e.g., plastics) can be sterilized by Ethylene oxide gas or other new, low-temperature sterilization systems (e.g., hydrogen peroxide gas plasma, peracetic acid immersion, ozone)
  • Dry-heat sterilizers such as hot air oven should be used only for materials that might be damaged by moist heat or that are impenetrable to moist heat (e.g., powders, petroleum products, sharp instruments).
  • Ionizing Radiation- There are no FDA-approved ionizing radiation sterilization processes for use in healthcare facilities.
37. Ans. (c) (Hugh Leifson test) Ref: Ananthnarayan 9th/36
  • Hugh Leifson test is used to differentiate micrococci from staphylococci
 
HOSPITAL WASTE MANAGEMENT
38. Ans. (c) (Put the dressing material directly in an appropriate bag and send for incineration in appropriate bag).
Ref: PSM park 20/e p 696-699
  • Dressing material soaked with HIV contaminated blood belongs to Waste Category No. 6
  • Methods of disposal of Waste Category No. 6- Incineration, Autoclaving , Microwaving or Chemical treatment
  • Incineration is the commonly used method of disposal of contaminated wastes in hospital other health care facilities providers
39. Ans. (d) (Category 4) Ref: Park 20th/p696
  • Refer chapter review.
 
BACTERIOLOGY OF WATER, AIR & MILK
40. Ans. (c) (Multiple tube method) Ref: Ananthanarayan 9/e p626-27, 8/e p591
  • Presumptive coliform count- is calculated by Multiple tube method is used
  • Differential coliform count- is done by Eijkman test
41. Ans. (d) (Presence of Corynebacterium indicates recent contamination) Ref: Ananthanarayan 9/e p626-27, 8/e p591
  • Corynebacterium is not an indicator for water contamination
  • Indicator of feacal contamination of water:
    • Thermotolerant E.coli (recent contamination)- most definite
    • ✓ Coliform count
    • ✓ Fecal streptococci
    • ✓ Clostridium perfringens
42. Ans. (a) (Phosphatase test) Ref: Ananthanarayan 9/e p 631, 9/e p596
  • Phosphatase test- Used to check adequacy of pasteurization of milk
    • ✓ Enzyme phosphatase is normally present in raw milk and will be inactivated by pasteurization
    • Its residual presence following pasteurization indicates, pasteurization has not been adequate.
  • Other tests used for bacteriological examination of milk- Refer text
43. Ans. (c), (d) (Human anatomical waste, Solid waste) Ref: Park 22nd 739/, 21st/p734-35
  • Yellow bag contains item of category −1, 2, 3&6. Refer chapter review for details.
44. Ans. (b) (2% glutaraldehyde) Ref: Ananthnarayan 9th /p34
  • All Scopes are sterilized best by glutaraldehyde(CIDEX 2%).

Culture Media and Culture MethodsCHAPTER 1.3

 
CULTURE METHODS
 
Streak Culture
  • Useful method for obtaining discrete colonies and for pure cultures.
  • It is done on the surface of a dry agar plate using a platinum or nichrome loop of 2-4 mm diameter with intermittent heating
  • Single isolated colonies are very useful to study various properties of the bacteria.
 
Lawn Culture
  • It provides uniform surface of growth of the bacterium on the solid medium.
  • Useful for carrying out antibiotic sensitivity testing by disc diffusion method
  • For bacteriophage typing
  • Producing large amount of bacterial growth required for preparation of bacterial antigens and vaccines
 
Pour-plate Culture
  • Used to quantitate bacteria in urine cultures
  • To estimate viable bacterial count in a suspension.
 
Stroke Culture
  • This provides a pure growth of bacteria for carrying out diagnostic tests.
  • Example where stroke culture is used- citrate test and urease test.
 
Stab Culture
  • Stab culture is used for maintaining stock cultures.
  • For demonstration of oxygen requirement of the bacteria
  • Example where stab culture is used- mannitol motility medium, Triple sugar iron test.
 
Liquid Culture
  • Used for blood culture and for sterility tests, where, the concentration of bacteria is small.
  • Disadvantage - It does not provide a pure culture of the bacteria
 
Aerobic Culture
  • For aerobic bacteria inoculated culture plates are incubated at 37 ºC for overnight
  • Brucella abortus and capnophilic Streptococci require additional carbon dioxide (CO2).
  • Pneumococci and gonococci require −5-10% CO2 for luxuriant growth
  • Campylobacter, Helicobacter and Actinomyces israellimicroaerophilic, require- 5% O2, 85% N2 and 10% CO2.
 
Anaerobic Culture Methods
  • Obligate anaerobes are bacteria that grow only in the absence of oxygen.
  • Commonly used media-
    • Robertson cooked meat broth (RCM),
    • Thioglycollate broth,
    • Neomycin blood agar.
    • Egg yolk agar
  • Cultures are carried out in an environment that is free of oxygen, followed by incubation at 35°C for at least 48 hours.
  • McIntosh-Fildes anaerobic jar,
  • Gas pack system
  • Anaerobic glove box - Most widely used methods for anaerobic culture.
 
36CULTURE MEDIA
 
Basic Ingradients
  • Agar or Agar agar:
    • Made up of long chain polysaccharide, obtained from seaweed.
    • It has no nutritive value
    • Usual conc. used in solid media – 1-2% (Japanese agar 2% or New Zealand agar 1.2%)
    • It melts at 98°c & solidifies at 42°c
    • New Zealand agar has more jellifying property than Japanese agar.
  • Peptone
  • Meat extract
 
Simple/ Basal Media
  • Peptone water- Peptone + NaCl
  • Nutrient broth- Peptone + meat extract + NaCl
  • Nutrient agar- Nutrient broth + 2% agar
 
Complex Media
  • Added ingredients for growth or bringing out certain property
 
Defined Media
  • Exact quantity of each chemical is known
 
Enriched Media
  • Contains- blood, serum, egg
    • Eg- Blood agar, Chocolate agar, Loeffler's serum slope, Dorset egg media
 
Enrichment Broth (Liquid Media)
  • Selectively allows certain organism to grow and inhibit others
    • Eg- Tetrathionate broth (S. Typhi), Alkaline peptone water broth (APW)- Vibrio, Selenite F broth- Shigella
 
Selective Media (Solid Media)
  • Selectively allows certain organism to grow and inhibit others
    • Lowestein Jensen media-Mycobacterium tuberculosis
    • Potassium tellurite agar (PTA)- Corynebacterium diphtheriae
    • Wilson Blair bismuth sulfite medium– Salmonella Typhi
    • Thiosulphate Citrate Bile salt Sucrose agar- Vibrio
 
Transport Media
  • Bacteria doesn't multiply, only maintains viability of desired pathogenic bacteria
Organism
Transport Media
Streptococcus
Pike's media
Neisseria
Amies, Stuart's media
Vibrio
Venkatraman Ramakrishnan (VR) fluid, Autoclaved sea water, Carry Blair media
Enteric pathogen
Carry Blair medium
Shigella, Salmonella
Buffered glycerol saline
Bordetella
Modified Stuart's (with casamino acid)
Mischulow's charcoal agar
Dacron or calcium alginate swab used
 
Differential Media
  • Mac Conkey agar
  • Cystein Lactose Electrolyte Deficient agar (CLED)
37Examples of Special Media (E-enriched, En-enrichment, S-selective, D-differential media)
Organism
Medium
Enteric pathogens – for Salmonella, Shigella
Hektoen enteric agar (S)
Xylose-lysin-deoxycholate agar (S)
Deoxycholate citrate agar (S)
Eosin Methylene blue agar (S)
MacConkeys (D and S)
Salmonella Shigella agar (S)
Wilson Blair Bismuth Sufite Media for Salmonella (S)
Selenite F broth (En), Tetrathionate broth (En)
Blood culture- for blood borne pathogens
Castaneda's biphasic media (E)- Brain heart infusion agar slope & broth
Vibrio cholerae (likes alkaline growth medium)
TCBS (Thiosulfate Citrate Bile salt Sucrose agar) (S)
Mansour's Gelatin Taurocholate Trypticase agar (S)
Alkaline Bile salt agar (S)
APW- Alkaline peptone water (En)
S.aureus
Mannitol salt agar (S)
Milk salt agar (S)
Ludlam's medium (S)
Streptococcus
Crystal violet blood agar (S)
Neisseria
Chocolate agar (E), Thayer-Martin media (S),
Modified New York medium (S)
Corynebacterium
Loffler's serum medium (E)
Potassium Tellurite agar (S)
Bacillus anthracis
PLET –
Polymyxin Lithium EDTA Thallous acetate- (S)
Bacillus cereus
MYPA-
Mannitol egg yolk phenol red polymyxin agar- (S)
Anaerobes
Thioglycollate broth
Robertson cooked meat broth
Listeria
PALCAM agar (S)
Pseudomonas
Cetrimide agar (S), King's media (for pigment)
Burkholderia
Ashdown's medium (S)
Haemophilus
Blood agar with S.aureus streak (E)
Chocolate agar (E)
Levinthal's medium (E), Fildes agar (E)
Bordetella
Regan Low media(E)
Bordet Gengou Glycerin potato blood agar (E)
Lacey's DFP media (S)
Mycobacterium
Lowenstein Jensen, Dorset egg (S)
Leptospira
EMJH (E), Fletcher's (E), Korthoff's (E)
Campylobacter
Skirrow's, Butzler, Campy BAP (S)
Legionella
BCYE (Buffered charcoal yeast extract) (E)
Reiter's Treponema
Smith Noguchi media
Miscellaneous Growth Requirements
Cholesterol and purines and pyrimidines
Mycoplasma
Cysteine
Francisella, Brucella, Legionella, Pasteurella
X factor (haemin & porphrin) and V factor (NADH)
Haemophillus influenzae
Pyridoxal
Streptococcus abiotrophia
38MULTIPLE CHOICE QUESTIONS
1. Recommended transport medium for stool specimen suspected to contain enteric pathogens is: (NEET Pattern Based)
  1. Amie's medium
  2. Buffered glycerol saline medium
  3. MacConkey medium
  4. Blood agar
2. Which is an enrichment media? (NEET Pattern Based, PGI2001)
  1. Selenite F broth
  2. Meat extract broth
  3. Dorset egg emdia
  4. Chocolate agar
3. Agar conc. In nutrient agar is: (NEET Pattern Based)
  1. 2%
  2. 4%
  3. 1%
  4. 3%
4. Robert Koch's assistant had advised him to use agar instead of gelatin because: (NEET Pattern based)
  1. Agar is more nutritious
  2. Gelatin melts at 250C
  3. Gelatin is not easily available
  4. Agar is cheaper
5. Robertson cooked meat broth is an example: (NEET Pattern Based)
  1. Enriched media
  2. Enrichment media
  3. Nutrient media
  4. Anaerobic media
6. Blood agar is an example: (DNB June 2012)
  1. Enriched media
  2. Enrichment media
  3. Nutrient media
  4. Special media
7. Which one of the following is true: (AIIMS Nov 2006, AI 2007)
  1. Agar has nutrient properties
  2. Chocolate medium is selective medium
  3. Addition of selective substances in a solid medium is called enrichment media
  4. Nutrient broth is basal medium
8. In blood culture the ratio of blood to reagent is: (JIPMER 2003)
  1. 1:5
  2. 1:20
  3. 1:10
  4. 1:100
9. Smith Noguchi's media is used for:(DNB 2007)
  1. Salmonella
  2. Thlebsiella
  3. Spirochetes
  4. Bacillus
10. In a patient with UTI, CLED (Cystine, Lactose Electrolyte Deficient) Media is preferred over Mac Conkeys media because: (AIIMS Nov 2001, AI 2001)
  1. It is a differential medium
  2. In inhibits swarming of Proteus
  3. Promotes growth of Pseudomonas
  4. Promotes growth of staph aureus and Candida
11. To prevent swarming the % agar in Nutrient agar has to be increased at least to ___ % ? (TNPG 2014)
  1. 2%
  2. 4%
  3. 6%
  4. 8%
12. Which of the following is not a selective media: (TNPG 2014)
  1. Blood Agar
  2. Thayer Martin media
  3. Wilson Blair Media
  4. LJ medium
39EXPLANATIONS
1. Ans. (b) (Buffered glycerol saline medium) Ref: Ananthanarayan 9/e p42, 8/e p39
Recommended transport medium for stool specimen is-
  • Buffered glycerol saline medium (when Salmonella or Shigella is suspected)
  • VR medium-(when Vibrio cholerae is suspected)
2. Ans. (a) (Selenite F broth) Ref: Ananthanarayan 9/e p42,8/e p39
  • Selenite F broth is an enrichment broth used for Shigella and Salmonella
3. Ans. (a) (2%) Ref: Ananthanarayan 9/e p39, 8/e p39
  • 2% of agar is routinely used for making solid media.
4. Ans. (b) (Gelatin melts at 25°C) Ref: Ananthanarayan 9/e p39, 8/e p39
  • Gelatin melts at 25°C, hence at room temperature it remains as liquid form. Whereas agar melts at 420C, it stays in solid form at room temperature. This reason why agar is preferred over gelatin. The use of agar to use as solid media was suggested by Frau Hesse, the wife of one of the Robert Koch’ assistant.
  • Agar has no nutritious value
  • Gelatin is easily available
  • Both gelatin and agar is cheaper
5. Ans. (d) (Anaerobic media) Ref: Ananthanarayan 9/e p43,8/e p39
  • Roberson cooked meat broth is used for culture of anaerobic organisms.
6. Ans. (a) (Enriched media) Ref: Ananthanarayan 9/e p40, 8/e p39
  • Enriched media contains extra nutritional factors like blood, serum, egg so that it supports the fastidious organ- isms. Eg- Blood agar, Chocolate agar, Loeffler's serum slope
7. Ans. (d) (Nutrient broth is basal medium) Ref: Ananthanarayan 9/e p40, 8/e p39
  • Option a- Incorrect- Agar is long chain polysaccharide
    • ✓ Used to prepare solid media, but i t has no nutritive value
    • ✓ It melts at 98°C and sets at 42°C depending on agar concentration
  • Option b- Incorrect- Chocolate medium is enriched medium
  • Option c- Incorrect- Addition of selective substances in a solid medium is called as selective medium
  • Option d- Correct- Nutrient broth, Peptone water and Nutrient Agar are the basal media
8. Ans. (a) (1: 5) Ref: Bailey and Scott diagnostic microbiology 12/e p786, Mackie and McCartney's Practical Microbiology 14/e p106, Ananthanarayan 9/e p298, 8/e p295
  • Blood culture broth- Blood contains substances that inhibit the growth of the bacilli and hence it is essential that broth should be taken in sufficient quantity to provide at least 4 fold dilution of blood.
  • Blood to blood culture medium ratio in blood agar:
    • ✓ 5-10% (Ananthanarayan 8/e p295)
    • 5% dilution is adequate (Bailey and Scott diagnostic microbiology 12/e p786….. Standard reference fol- lowed worldwide for blood culture)
  • Also remember-
    • Blood agar- contains 10% of blood and it is added to the molten agar at 45-50°C
    • Chocolate agar (heated blood agar)- Blood is added to the molten agar at 75°C, so that RBC gets lysed providing extra nutrient to the medium
9. Ans. (c) (Spirochetes) Ref: Ananthanarayan 9/e p42, 371, 8/e p47, 372
  • Smith Noguchi's medium- Broth containing fresh animal tissue, such as rabbit kidney, spleen, testes or heart sup- ports growth of many anaerobes like Non pathogenic Treponemes
  • 40Only non pathogenic Treponemes can be cultivated in complex culture medium,
  • Pathogenic Treponemes cannot be grown in artificial culture medium.
10. Ans. (d) (Promotes growth of Staph.aureus and Candida) Ref: Bailey and Scott's diagnostic Microbiology 12/e p851
  • Both CLED and Mac Conkey media are Differential Media and they inhibit proteus swarming and both promote growth of Pseudomonas.
  • Mac Conkey media inhibits growth of all GPB, including Staphylococcus and Candida due to the incorporation of inhibitor substance Crystal violet.
  • Where as CLED supports growth of Staphylococcus and Candida, which are also urinary pathogens. Hence it is preferable to use CLED for urine samples in the laboratories.
Uses of Some Commonly Used Solid Media
Medium
Uses
Nutrient agar
  • Routine culture
  • Demonstration of pigmentation
  • Catalase, oxidase and agglutination test can be done on this plate
  • Demonstrate non-fastidious nature of the bacteria
MacConkey agar
  • Used for Routine culture of gram negative bacilli
  • Differential medium
  • Mildly selective medium
  • CLED agar-Used for same prupose, but mainly for urine samples.
Blood agar
  • Used for Routine culture as an Enriched medium
  • Culture of fastidious organisms such as Streptococcus species
  • Demonstration of hemolytic property
Chocolate agar
  • Enriched medium
  • Culture of Haemophilus influenzae, Neisseria
Mueller Hinton Agar (MHA)
  • For antibiotic susceptibility test by disk diffusion method
11. Ans. (c) (6%) Ref: Ananthnarayan 9th/42
Agar concentration routinely used-
  • In solid media- 2%
  • Semisolid media- 0.5-1%
  • To inhibit swarming- 6%
12. Ans. (a) (Blood Agar) Ref: Ananthnarayan 9th/42-43
  • Blood Agar is an enriched media.

Bacterial GeneticsCHAPTER 1.4

Bacterial nucleus contains double stranded circular DNA without any basic protein. They lack nuclear membrane or nucleolus. The bacterial DNA is haploid, replicates by simple fission and about 1 mm in length.
 
 
Plasmids
  • Extra chromosomal DNA substances.
  • Have property of independent replication
  • Circular and double stranded DNA molecules that encode traits that are not essential for bacterial viability.
  • Plasmids are of different types:
    • F factor,
    • R factors
    • Col factor
  • Plasmids confer new properties to recipient bacteria:
    • Resistance to one or several antibiotics,
    • Production of toxins,
    • Synthesis of cell surface structures required for adherence or colonization
  • Plasmids integrated with host chromosome are known as episomes.
 
METHODS OF TRANSFER OF DNA BETWEEN BACTERIAL CELLS
  1. Transformation:
    • Process of the transfer of free DNA itself from one bacterium to another.
    • Griffith experiment- done on Mice provides the evidence of transformation.
      • Live Noncapsulated Pneumococcus or Dead capsulated Pneumococcus can- not kill a mice due to transfer of genetic information for capsule synthesis from dead capsulated strain to live non capsulated strain.
    • When purified DNA is injected into the nucleus of a bacterial cell the process is called as transfection.
  2. Transduction: Transfer of a portion of the DNA from one bacterium to another mediated by a bacteriophage.
  3. Lysogenic conversion: Bacteriophage when integrated with the bacterial chromosome, this stage is called as lysogenic conversion. In this stage, the phage DNA may impart toxigenicity to the bacteria.
    • Bacteriophage encoded toxin-
      • Diphtheria toxin,
      • Verocytotoxin of E.coli
      • Cholera toxin
      • Botulinum toxin c and D,
      • Streptococcal pyrogenic exotoxin A and C
    • Transduction is of two types: generalized and specialized.
  1. Conjugation:
    • Process of transfer of DNA from the donor bacterium to the recipient bacterium.
    • Occurs between two closely related species (mostly in Gram negative bacteria)
    • Discovered by Lederberg & Tatum in E.coli (1946).
    • Donor bacteria (F +) possess fertility plasmid (or F factor) that codes for sex pilus or conjugation tube.
    • The sex pilus produces a bridge between donor & recipient cell through which a 42copy of F factor is transmitted. So that after the conjugation is over- Both donor & recipient will have fertility plasmids
    • Along with the F factor, drug resistance genes may get transmitted-Responsible for the spread of multiple drug resistance among bacteria.
    • HFR state (high frequency recombination state): It is stage where the donor's F-factor integrates with the chromosome. Such donor mates with the recipient with high frequency and following which more chromosomal genes are transferred; however the F-factor doesn't get transmitted.
    • F’ factor: When the integrated F factor reverts back from integrated state to free state, it carries few of the chromosomal gene called as F prime (F’ factor).
    • Sexduction: When F’ factor mates with a recipient; it transfers F factor and attached chromosomal genes.
Mutational Drug Resistance
Transferable Drug Resistance (Conjugation)
One drug resistance at a time
Multi drug resistance at a time
Low degree resistance
High degree resistance
Overcome by high drug/drug combinations
Cant'overcome
Spread by- Vertical spread only
Spread by- Horizontal spread to any spp
Virulence might be low
Virulence not decreased
 
Transposon/ Jumping Gene- (Transfer of DNA Within Bacterial Cells)
  • They are mobile DNA of 1-2kb.
  • They can transfer DNA from one site of the bacterial chromosome to another site or to a plasmid – in a cut and paste manner (transposition).
  • Don't have self-replicative power (differs from plasmid)
  • They code for drug resistance enzymes, toxins or a variety of metabolic enzymes.
  • Discovered by Barbara McClintok.
 
Polymerase Chain Reaction – PCR
  • Process of amplification of small amount of DNA
  • Requires:
    • Taq (Derived from plant Thermus acquaticus) Polymerase- which can with- stand the high temperature during the reaction
    • Primers- short oligonucleotide complementary to a small sequence of the tar- get DNA
    • Nucleotide, Mg++ ion
  • 3 Basic Steps:
    • DNA extraction from micro-organism
    • Amplification of extracted DNA- 3 steps
      • ✓ Denaturation (95°C),
      • ✓ Primer annealing (55°C),
      • ✓ Extension (72°C)
    • Gel electrophoresis of amplified product
 
Modification of PCR
  • Reverse transcriptase PCR (RT-PCR) – used for RNA Viruses
  • Nested PCR- Two round PCR by using two different primers against the same organism, it is more specific and sensitive.
  • Multiplex PCR- Uses >1 primers which can detect many organism in one reaction, Used for syndromic approach.
  • Real-time PCR (rt-PCR) - recent development.
    • Real time visualization of amplification process
    • Takes less time
    • Less contamination rate
    • Quantification is possible
 
43Applications of PCR
  • Amplify very few copies of a specific DNA, so it is more sensitive and specific.
  • Detect uncultivable or fastidious microorganisms.
  • Detect the genes responsible for drug resistance like M.tuberculosis, HIV etc
  • Provide useful information that may predict progression of the disease e.g. HIV.
  • Detect genetic diseases such as sickle cell anemia, phenylketonuria, and muscular dystrophy.
  • Quantification of genome-used for disease monitoring.
44MULTIPLE CHOICE QUESTIONS
1. PCR is used to: (AIMS Nov 2013)
  1. Detect target plasmids
  2. Amplify small amount of DNA
  3. Seal the cut ends of DNA
  4. Cleave the bacterial plasmid
2. Mechanism of direct transfer of free DNA: (NEET Pattern Based)
  1. Transformation
  2. Conjugation
  3. Transduction
  4. None
3. Reverse transcriptase is: (PGI June 2011)
  1. DNA dependent RNA polymerase
  2. RNA dependent DNA polymerase
  3. DNA dependent DNA polymerase
  4. RNA dependent RNA polymerase
  5. RNA polymerase
4. Enzyme(s) used in polymerase chain reaction is/are: (PGI June 2011)
  1. Restriction endonuclease
  2. DNA polymerase
  3. Alkaline phosphate
  4. RNA polymerase
  5. Reverse transcriptase
5. In transduction, the DNA transmitted by the vector to the bacteria belongs to? (DNB Dec 2010)
  1. Human cell
  2. Bacteriophage
  3. Another bacterium
  4. Virus
6. Phage mediate transfer of cDNA into host is known as: (DNB DEC 2012)
  1. Transduction
  2. Transformation
  3. Transmission
  4. Conjugation
7. Not true about bacterial drug resistance mechanism: (AI 2012) (AIIMS Nov 2011) (AIIMS May 2012)
  1. Most common mechanism is production of neutralizing enzymes
  2. If resistance is plasmid mediated, it is always transferred vertically
  3. Alteration of target seen in pneumococcal resistance
  4. Complete removal of target is cause of resistance to Vancomycin
8. All of the following are required in PCR except: (AIIMS Nov 2011)
  1. Deoxyribonucleotides
  2. Thermostable enzyme /DNA polymerase
  3. Dideoxyribonucleotides
  4. Magnesium/ssDNA/Template DNA
9. Chromosomal mutation can be identified by all except: (AIIMS Nov 2011)
  1. Single strand polymorphism
  2. Agarose gel electrophoresis
  3. Denaturating Gradient gel electrophoresis
  4. Dideoxy nucleotide trail sequencing
10. Northern bloting is used for separation of: (DNB June 2012)
  1. DNA
  2. RNA
  3. Proteins
  4. None
11. Horizontal transmission of ‘R’ factor is by: (JIPMER 2009)
  1. Transduction
  2. Transformation
  3. Conjugation
  4. Fusion
12. Polymerase chain reaction was discovered by: (TN 2009)
  1. Saiki
  2. Salk
  3. Wastson and Crick
  4. Thary B Mulis
13. Multiple drug resistance is spread by: (TN 2008)
  1. Transformation
  2. Transduction
  3. Mutation
  4. Conjugation
14. Virus mediated transfer of host DNA from one cell to another is known as: (AI 2005)
  1. Transduction
  2. Transformation
  3. Transcription
  4. Integration
15. The following phenomenon is responsible for antibiotic resistance in bacteria due to slime production: (AIIMS Nov 2003)
  1. Co-aggregation
  2. Biofilm formation
  3. Mutation involving in altered target site for antibiotics
  4. Mutation involving a target bypass mechanism
16. The role of plasmids in conjugation was first described by Lederberg and Tatum (1946) in: (AIIMS 2002)
  1. H. influenza
  2. Corynebacterium
  3. Pseudomonas
  4. Escherichia coli
17. For PCR, Thermus acquaticus plant is used to prepare: (Recent Question 2013)
  1. DNA polymerase
  2. RNA polymerase
  3. Primers
  4. Restriction endonuclease II
18. DNA hybridization is called: (Recent Question 2013)
  1. Southern blot
  2. Northern blot
  3. Eastern blot
  4. Western blot
45EXPLANATIONS
1. Ans. (b)(Amplify small amount of DNA) Ref: Ananthanarayan 9/e p65
  • The polymerase chain reaction (PCR) is a biochemical technology in molecular biology to amplify a single or a few copies of a piece of DNA, generating thousands to millions of copies of a particular DNA sequence.
2. Ans. (a) (Transformation) Ref: Ananthanarayan 9/e p59, 8/e p63
  • Transformation is the process of the transfer of free DNA itself from one bacterium to another.
3. Ans. (b) (RNA dependent DNA polymerase)
Ref: Ananthanarayan 9/e p571, 8/e p570, Field's Virology 5/e p2188, Harper 28/e p307
  • Reverse transcriptase has RNA dependent DNA polymerase activity and it uses the genome RNA as a template and 1st synthesizes single stranded DNA by reverse transcription of genomic RNA, then finally a double stranded DNA.
  • The HIV and other retroviruses are enveloped RNA viruses, characteristically possessing an RNA-dependent DNA poly- merase called reverse transcriptase.
4. Ans. (b) (DNA polymerase)
Ref: Ananthanarayan 9/e p65, 8/e p70-71, Bailey and Scott Diagnostic Microbiology 12/e p127-131
  • Taq Polymerase is a DNA Polymerase that amplifies the target gene into millions of copy without denaturation. It is one of the basic requirements of PCR along with other requirements like primers, Nucleotide and Mg++ ion.
About Other Options
  • Restriction endonuclease – Require for RFLP (Restricted Fragment Length Polymorphism)
  • Reverse transcriptase – Require for Reverse transcriptase PCR (RT-PCR)
  • Alkaline phosphate- Required for ELISA.
  • RNA polymerase – Not required.
5. Ans. (b) (Bacteriophage) Ref: Ananthanarayan 9/e p59, 8/e p63
  • Transduction: Transfer of a portion of the DNA from one bacterium to another mediated by a bacteriophage.
6. Ans. (a) (Transduction) Ref: Ananthanarayan 9/e p59, 8/e p63
  • Refer text
7. Ans. (b) (If resistance is plasmid mediated, it is always transferred vertically)
Ref: Ananthanarayan 9/e p63, 8/e p67, Harrison 18/e p1157, Koneman's diagnostic Microbiology 6/e p945
  • If resistance is chromosomally mediated, it is usually transferred vertically from parent to daughter bacteria.
  • If resistance is plasmid mediated, it is usually transferred by horizontal route mainly by conjugation.
About Other Options
  • Option a-Clinically, enzymatic drug inactivation is the most common mechanism for acquired microbial resist- ance by bacteria …… Koneman's diagnostic Microbiology 6/e p945
  • Most common mechanism of bacterial drug resistance-
  • Option c-Pneumococcal resistance:– Due to Alteration of target i.e. Penicillin binding protein(PBP) Harrison 18/e p1157, Koneman's diagnostic Microbiology 6/e p958
  • Option d- Resistance to Vancomycin- is due to complete removal of target
  • D alanyl-D alanine present the bacterial cell wall is the target site for Vancomycin, which binds there and inhibits it and thus inhibits the cell wall synthesis.
  • If the bacteria is resistant to Vancomycin- Then the target site i.e. D alanyl-D alanine gets altered to D alanyl-D lactate which has less affinity for Vancomycin. So Vancomycin can't bind and inhibit.
The four Major Mechanisms of Antibacterial Resistance
  • Enzymatic inactivation of antimicrobial agents – like beta lactamases, aminoglycoside modifying enzymes etc ………...........MC Mechanism
  • Change in receptor for antibiotic attachment- e.g. Altered PBP
  • Alternation of membrane permeability
  • Bypassing the blocked metabolic pathway – seen in Cotrimoxazole
8. Ans. (c) (Dideoxyribonucleotides)
Ref: Ananthanarayan 9/e p65, 8/e p70-71, Bailey and Scott's Diagnostic Microbiology 12/e p127-131
  • Dideoxyribonucleotides is not used in PCR .
Requirement of PCR
  • Taq Polymerase- Obtained from Thermus aquaticus plant which can withstand the high temperature during the reaction.
  • Primers- short oligonucleotide complementary to a small sequence of the target DNA
  • 46Deoxynucleotide, Mg++ ion
  • Target DNA present in the sample
9. Ans. (b) (Agarose gel electrophoresis): Ref: Prescott's Microbiology 6/e p251-54, Wikipedia
  • Agarose Gel electrophoresis is used to separate the DNA by charge or by size. It is usually performed to visualize the am- plified DNA after PCR, but may be used as a preparative technique prior to use of other methods such as mass spectrometry, RFLP, PCR, cloning, DNA sequencing, or Southern blotting for further characterization.
  • Please remember, it is just a method of visualizing the DNA after separating by size. So it is always used as a part of any molecular method to visualize the DNA. But alone it cannot be used to detect any mutations.
Mutations can be Detected by
Phenotypic Methods:
– Carcinogenicity Testing (Ames Test)
– Direct detection systems (Replica Plating Test)
Genotyping Method
– RFLP (restricted fragment length polymorphism
– DNA sequencing
– Single-strand conformation polymorphism
– Gradient Gel Electrophoresis
10. Ans. (b) (RNA) Ref: Ananthanarayan 9/e p65, 8/e p69
  • Northern blotting is used for detection of- RNA
  • Southern blotting is used for detection of- DNA
  • Western blotting is used for detection of- Protein
11. Ans. (c) (Conjugation) Ref: Ananthanarayan 9/e p60, 8/e p65-66
  • Conjugation is the process where there is transfer of genetic elements from one bacterium (male) to another (female) along sex pilus or conjugation tube- Horizontal genetic transfer.
12. Ans. (d) (Kary B Mulis) Ref: Ananthanarayan 9/e p65, 8/e p69
  • Polymerase chain reaction is a rapid automated method of amplification of specific DNA sequences
  • Invented by Kary B Mulis in 1983, for which he won Nobel Prize in 1993.
13. Ans. (d) (Conjugation) Ref: Ananthanarayan 9/e p60-61, 8/e p65-66, SC Parija's Microbiology 1/e p61
  • Resistance (R) factors are extra chromosomal plasmids responsible for spread of multiple drug resistance among bacteria.
  • They are circular double stranded DNA carry genes for variety of enzymes that can destroy antibiotics
  • R factor consists of 2 components: Resistance transfer factor (RTF) and resistant determinant (r).
  • The resistance transfer factor is responsible for conjugational transfer while each r determinant carries resistance for one of the several antibiotics.
14. Ans. (a) (Transduction) Ref: Ananthanarayan 9/e p59, 8/e p65, SC Parija's Microbiology 1/e p65
  • Transduction- transfer of a portion of the DNA from one bacterium to another mediated by a bacteriophage (Virus infecting bacteria).
  • The phage DNA within the recipient bacterial cell integrates into the bacterial cell DNA during a process called lysogenic conversion.
  • Lysogenic conversion confers a new property to the bacterial cell (can convert a non pathogenic bacteria into pathogenic).
  • Bacteriophages encode diphtheria toxin, botulin toxin, cholera toxin and erythrogenic toxin.
15. Ans. (b) (Biofilm formation) Ref: Internet-dentalcarestamford.com
  • Most bacteria live in complex communities called biofilms.
  • A biofilm is a well-organized community of bacteria that adheres to surfaces and is embedded in an extracellular slime layer.
  • Once a bacterium attaches to a surface, it activates a whole different set of genes that gives the bacterium different characteristics from those that it had as a free-floating organism.
16. Ans. (d) (Escherichia coli) Ref: Ananthanarayan 9/e p60, 8/e p65
  • Bacterial conjugation was first described by Lederberg and Tatum (1946) in a strain of E.coli called K12 and it's the most extensively studied strain.
17. Ans. (a) (DNA polymerase) Ref: Mackie McCartney 14/e p227
The heat stable DNA polymerase enzyme used for PCR is derived from plant Thermus acquaticus.
18. Ans. (a) (Southern blot) Ref: Ananthanarayan 9/e p65
  • DNA DNA hybridization is called- Southern blot
  • RNA RNA hybridization is called- Northern blot
47Immunology
  • 2.1 Infection and Immunity
  • 2.2 Antigen, Antibody and Antigen Antibody Reaction, Complement
  • 2.3 Structure of Immune System and Immune Response
  • 2.4 Hypersensitivity Reaction
  • 2.5 National Immunization Schedule, Autoimmunity, Transplant & Cancer Immunology and Immunodeficiency Disorder48

Infection and ImmunityCHAPTER 2.1

 
 
Immunity
  • Refers to resistance exhibited by host towards the injury caused by microorganism or its products
 
Type
  • Innate immunity and Acquired/ Adaptive immunity
Innate Immunity
Acquired / Adaptive Immunity
  • Resistance to infection that an individual possess from birth by the genetic or constitutional makeup.
  • Which can be:
    • Species specific
    • Racial specific
    • Individual specific
  • Resistance to infection that an individual acquires during his life
  • Which can be:
    • Active or passive
    • Artificial or natural
  • Occurs in Minute
  • Occurs in Days
  • Diversity limited
  • Active against Wide range of infection
  • Non specific
  • Specific
  • No memory
  • Memory Present
Components of Innate Immunity
  • Phagocyte(monocyte, macrophage, neutrophils),
  • Natural Killer cell,
  • Mast cells, dendritic cells and epithelial cells
  • Alternate complement pathway
  • Mannose binding complement pathway
  • Acute phase protein(CRP, MBP, serum amyloid protein),
  • Normal resident flora antagonism,
  • Inflammation, fever
  • Skin and mucosal barrier
  • Pattern recognition receptors (PRR)- Toll-like receptors, C-type lectins, leucine-rich proteins, scavenger receptors, pentraxins, lipid transferases, integrins
  • Antimicrobial peptides-Defensins, cathelin, protegrin, granulysin, histatin, secretory leukoprotease inhibitor, and probiotics
  • Cytokines
Components of Acquired Immunity
  • T cell,
  • B cell,
  • Classical complement pathway
  • Antigen presenting cell
 
Factors Influencing Innate Immunity
  • Species specific
  • Racial specific
  • Individual specific (genetic influence)
  • Age
  • Hormonal influence
  • Nutrition.
Active Immunity
Passive Immunity
Produced actively by host immune system
Received passively
Induced by infection or immunogen
Induced by readymade antibody transferred
Long lasting
Short
Lag period present
No Lag period
Memory present
No Memory
Booster doses-useful
Subsequent doses-Less affective
Negative phase may occur
No Negative phase
In Immunodeficiency- not useful
useful
50
Primary Immune Response
Secondary Immune Response
Antibody response against primary antigenic challenge
Antibody response against subsequent antigenic challenge
Slow, appear late
Prompt, appear fast
Sluggish
Powerful
Short lived
Long lasting
Long Lag period (4-7days)
No / short lag period (1-3days)
No Negative phase
Negative phase may occur on subsequent doses when antibody level falls
Antibody produced in low titre
Antibody produced in high titre
Early antibody is IgM type –
More specific but less avid
Late antibody is IgG type –
Less specific but more avid
Antibody affinity- low
Antibody affinity- high
Antibody producing cells- Naive B cell
Antibody producing cells- Memory B cell
Antigen processed- T dependent and T independent Ag
Antigen processed- T dependent Ag
 
Local Immunity
  • Produced at mucosal surfaces- GIT or respiratory mucosa
  • Provided by IgA antibody
  • Induced by infection or by live vaccination.
 
Herd Immunity
  • Overall immunity of a community to a pathogen
  • If Herd immunity is good- chance of epidemic is less
  • Eradication of a communicable disease- depends on good Herd immunity
  • Provided by mass vaccination by live vaccination to all individual at same time.
  • Elements that contributes to Herd immunity are:
    • Occurrence of Clinical and sub clinical cases in herd
    • Ongoing immunization programme
    • Herd structure – includes Population.
  • Herd immunity occurs with the following vaccines:
    • Diphtheria
    • Pertussis
    • Measles, Mumps, Rubella
    • OPV
    • Small Pox.
 
Adoptive Immunity
  • Acquired by injection of immunologically competent T-lymphocytes known as Transfer factor
  • Used for treatment when the CMI is low- e.g. Lepromatous leprosy.
Acute Phase Reactant Protein: Synthesized in liver at steady concentration, but synthesis either increases or decreases exponentially during acute inflammatory conditions.
Positive Acute Phase Reactant Protein: Levels of such proteins increase during acute inflammation.
  • Serum Amyloid A
  • C- Reactive protein
  • Complement protein- factor B,D, properdin, C1-C9
  • Coagulation protein - fibrinogen, vW factor
  • Proteinase inhibitors- α1 antitrypsin
  • α1 acid glycoprotein
  • Mannose binding protein
  • Metal binding protein- Haptoglobulin, haemopexin, ceruloplasmin
Negative Acute Phase Reactant Proteins: Levels of some protein are decreased during acute inflammation. Their decrease helps liver to produce acute phase proteins
Example: Albumin, pre albumin, transferrin, histidine rich glycoprotein
 
51C-Reactive Protein
  • Acute phase reactant protein raised against any bacterial infection including pneumococcal infection
  • CRP precipitates with C carbohydrate antigen of Pneumococcus
  • It belongs to beta globulin family
  • Normal level- <0.2mg/dl
  • Level increase:
    • Insignificant increase(<1 mg/dl) - Heavy exercise, common cold, pregnancy, angina
    • Moderate increase(1-10 mg/dl)- in bronchitis, cystitis, malignancies, pancreatitis, myocardial infarction
    • Marked increase(>10 mg/dl)- in acute bacterial infection, major trauma and systemic vasculitis.
  • CRP can be detected by:
    • Precipitation method using C carbohydrate antigen
    • Passive (Latex) agglutination test- Precipitation reaction is converted to an agglutination reaction where carrier particle like Latex is used on which anti CRP antibody gets coated to detect CRP
    • Detection limit of Latex agglutination test – 0.6mg/dl
    • High-Sensitivity C-Reactive Protein- can be detected by ELISA based methods which may detect minute quantities of CRP, useful in cardiovascular diseases.
52MULTIPLE CHOICE QUESTIONS
1. Not true about innate immunity: (NEET Pattern Based)
  1. Not influenced by hormones
  2. Dependent on genetic constitution
  3. Identical twins have same degree of resistance
  4. Not influenced by exposure to antigen
2. All are true about innate immunity except: (NEET Pattern Based)
  1. Acts as first line of defense
  2. Complements are examples
  3. Non specific
  4. Not effected by genetic influences
3. Transfer factor is an example of: (NEET Pattern Based)
  1. Artificial active immunity
  2. Natural active immunity
  3. Adoptive immunity
  4. Artificial passive immunity
4. Components of innate immunity : (PGI Nov 2010)
  1. T lymphocyte
  2. B lymphocyte
  3. Complements
  4. NTh cells
  5. Integrins
5. Innate immunity is stimulated by which part of bacteria? (DNB Dec 2011)
  1. Carbohydrate sequence in the cell wall
  2. Flagella
  3. Bacterial cell membrane
  4. Nucleus
6. First barrier encountered for microorganism for common exposed sites: (AIIMS Nov 2011)
  1. Lysozyme
  2. Acidic ph
  3. Skin
  4. Glycogen
7. Innate immunity active against viral cells: (AI 2007)
  1. NTh cells
  2. Cytotoxic T cells
  3. B cells
  4. Memory B cell
8. All of the following are a part of the innate immunity except: (AIIMS May 2005)
  1. Complement
  2. NTh cells
  3. Macrophages
  4. T cells
9. Active immunity can be induced by: (PGI MAY 2013)
  1. Toxoids
  2. Subclinical infection
  3. Antitoxin
  4. Immunoglobulins
  5. Antigen exposure
10. True about passive immunity: (PGI MAY 2013)
  1. Cannot be given with active immunity
  2. Last for 4-5 days only
  3. It can be given before disease occurrence
  4. Can be transferred by antibodies from another Host
  5. Takes longer time to develop
11. Superantigen is produced by: (Recent Question 2013)
  1. Staphylococcus aureus
  2. Streptococcus pneumoniae
  3. Pseudomonas
  4. Clostridium
 
ACUTE PHASE REACTANTS AND CRP
12. Acute phase reaction in acute inflammation are: (PGI June 2003)
  1. Albumin
  2. Fibrinogen
  3. Haptoglobulin
  4. Gamma globulin
13. C-reactive proteins are: (PGI Dec 2000)
  1. Alpha – 1 globulin
  2. Beta – 1 globulin
  3. Alpha – 2 globulin
  4. Non – specific inflammatory protein
14. Span of C reactive protein half-life: (TNPG 2014)
  1. 18 hrs
  2. 2 hr
  3. 12 hr
  4. 15hr
53EXPLANATIONS
1. Ans. (a) (Not influenced by hormones) Ref: Ananthanarayan 9/e p78, 8/e p84, Kuby's Immunology 6/e p53
  • Innate immunity is influenced by hormones- Endocrine disorders like diabetes are associated with enhanced susceptibility to infection due to altered innate immunity.
  • Innate immunity is dependent on genetic constitution of the individual. Homogenous identical twins exhibit similar degree of innate immunity.
  • Innate immunity is nonspecific, it is not influenced by exposure to antigen.
2. Ans. (d) (Not effected by genetic influences) Ref: Ananthanarayan 9/e p78, 8/e p83, Kuby's Immunology 6/e p53
  • Innate immunity refers to the resistance to infection that an individual possess from birth by its genetic or constitutional makeup.
  • Other options are correct- for explanation, refer text.
3. Ans. (c) (Adoptive immunity) Ref: Ananthanarayan 9/e p81, 8/e p89, 150
Adoptive immunity
  • Acquired by injection of immunologically competent T-lymphocytes known as Transfer factor
  • Used for treatment when the CMI is low- e.g. Lepromatous leprosy.
4. Ans. (c) (d) (e) (Complements, NK cells, Integrins)
Ref: Ananthanarayan 9/e p79, 8/e p83, Kuby's Immunology 6/e p53, Harrison 18/e p2654
  • Complement pathways (alternate & mannose binding, NK cells, and Pattern recognition receptors like Integrins are components of Innate immunity
  • For the detail list of components of Innate immunity- Refer text.
5. Ans. (a) (Carbohydrate sequence in cell wall) Ref: Harrison 18/e p2654
  • Pattern recognition receptors are important component of innate immunity
  • Toll-like receptors are e.g. of PRR Protein Family which bind to carbohydrate antigens on bacterial and viral surfaces.
PRR(Pattern Recognition Receptor) Ligands
Toll-like receptors
Bacterial and viral carbohydrates
C-type Lectins
Terminal mannose & Carbohydrate on HLA molecules
Leucine-rich proteins
Lipopolysaccharide (LPS)
Scavenger receptors
Bacterial cell walls
Pentraxins- CRP, Amyloid
CRP- Phosphatidyl choline, Amyloid-Bacterial cell walls
Lipid transferases
LPS
Integrins
LPS
NOD-like Receptors
Viral DNA Bacterial muramyl dipeptide
6. Ans. (c) (Skin)
Ref: Bailey and Scott's Diagnostic Microbiology 12/e p29, Ananthanarayan 9/e p79, 8/e p85, Kuby's Immunology 6/e p5
  • Physical barriers (skin and mucous membranes) tend to prevent the entry of pathogens act as first line of defense against infection. ……………… Kuby's Immunology 6/e p5
  • The skin serves as a 1st line physical and chemical barrier to microorganisms.…. Bailey and Scott's 12/e p2954
Protective Characters of Skin and Skin Structures: (Bailey and Scott/P29, Box 3.1)
Outer Dermal layer:
  • Effective mechanical barriers to the entry of most microorganisms.
  • Sloughing out the outer layer periodically removes the attached bacteria
Hair Follicle, Sweat Gland and Sebum:
  • Provides acidic environment (pH 3–5) due to sebum secretion that retards growth of microbes.
  • Also secretes alcohol and other toxic lipids that inhibit bacteria
Skin Associated Lymphoid Tissue (SALT):
  • Mediates specific and non specific protection mechanisms against microorganisms that penetrates outer tissue layer.
Conjunctival Epithelium Covering eye:
  • Flushing action of tear removes microbes.
  • Tear contains lysozymes that kill bacteria.
7. Ans. (a) (NK cells) Ref: Ananthanarayan 9/e p137, 8/e p131
  • Natural killer cells are components of innate immunity … Kuby's Immunology 6/e p53
  • Natural killer cells possess spontaneous cytotoxicity towards virus infected cells and malignant cells and their cytotoxicity is not antibody dependent nor MHC restricted … … Ananthanarayan 8/e p131
  • About Other options- Cytotoxic T cells, B cells and Memory B cell are components of adaptive/acquire immunity.……. Kuby's Immunology 6/e p53
  • Components of Innate immunity- refer text.
8. Ans. (d) (T cells) Ref: Kuby's Immunology 6/e p53
  • Alternate pathways of Complement, NK cells and Macrophages (as phagocytes) are the components of innate immunity
  • B cell, T cell, Classical complement and Antigen presenting cell are components of adaptive/acquire immunity.
9. Ans.(a) (b) (e) (Toxoids, Subclinical infection, Antigen exposure) Ref: Ananthnarayan 9th /p81-82
  • Active immunity can be induced by any substance that actively stimulates the immune system to produce anti- body. Vaccines, toxoids, infection or antigen exposure can induce active immunity
  • Antitoxin and immunoglobulins are e.g. of Passive immunity.
10. Ans.(c) (d) (It can be given before disease occurrence, Can be transferred by antibodies from another Host) Ref: Ananthnarayan 9th /p82-83
  • Immunoglobulins can be given along with vaccine in post exposure prophylaxis- e.g. Rabies immunoglobulins
  • Passive immunity last for days to months
  • Immunoglobulins should be given immediately after the exposure but before the disease occurrence
  • Passive immunity can be transferred between individuals by Serum therapy (antibodies)
  • Passive immunity works immediately
11. Ans. (a) (Staphylococcus aureus) Ref: Stite's Medical Immunology, 10/e p152
 
ACUTE PHASE REACTANTS AND CRP
12. Ans. (a), (b), (c) (Albumin, fibrinogen, haptoglobulin) Ref: VenuGopal Jayapal's Immunology 1/e p14
  • Fibrinogen, haptoglobulin are e.g. of Positive Acute phase reactant protein where as albumin is e.g. of Negative acute phase protein.
13. Ans. (b), (d) (Beta globulin, Non – specific inflammatory protein) Ref: Kuby's Immunology 6/e p65
Refer text.
14. Ans. (a) (18 hrs) Journal: C-reactive protein: a critical update, J Clin Invest. 2003.
The plasma half-life of CRP is about 19 hours and is constant under all conditions of health and disease, so that the sole determinant of circulating CRP concentration is the synthesis rate.

Antigen, Antibody and Antigen Antibody Reaction, ComplementCHAPTER 2.2

 
ANTIGEN
  • Antigen- has two properties
    • Immunogenicity – capacity to induce antibody formation
    • Immunological reactivity- capacity to react with an antibody.
  • Hapten
    • Low molecular weight, defective antigen
    • Doesn't have Immunogenicity but retain immunological reactivity i.e. antigenic
    • Hapten becomes immunogenic by combing with carrier molecule.
    • Simple hapten- Univalent, non precipitating with its antibody, only blocks the site on antibody
    • Complex hapten - Polyvalent, precipitating with its antibody.
  • Epitope- Antigenic determinant
 
Antigenicity Depends on
  • Size – Larger size, more antigenic
  • Chemical – decreasing order of Antigenicity- Protein> carbohydrate>lipid & nucleic acid
  • Susceptibility to Tissue enzyme- increases Antigenicity- (latex and d amino acid being chemically inert, are non antigenic)
  • Foreignness- More the foreignness of the antigen, more is the Antigenicity
  • Route of entry
  • Genetic composition
  • Specificity
    • Tissue specific
    • Iso specific – Human RBC
    • Auto specific- HLA, self antigens
    • Organ specific.
Heterophile Specificity – antigens of different species cross react with each other
  • Paul Bunnel – EBV (Epstein Barr Virus) with sheep RBC
  • Weil felix - Proteus OX2,OX19,OX K with Rickettsia alkali stable polysaccharide
  • Cold Agglutination test – Mycoplasma with human O+ve RBC at 4oc
  • Streptococcus MG with Mycoplasma
  • Forssman antigen – Lipid-CHO complex, present in all except rabbit. So anti- Forssman antibody can be prepared in rabbit
  • Non treponemal test for syphilis like VDRL, RPR (Flocculation test)
    • Treponemal antibody detected by using Cardiolipin antigen prepared from beef heart
  • Autoimmune consequences
    • Acute rheumatic fever and glomerulonephritis (Streptococcal antigens and human tissue).
 
Superantigens
  • Act on region of TCR (T cell receptor)
  • Dont require antigen presentation by macrophage, directly stimulate non specific T cells
  • Leads to massive release of cytokines & polyclonal B cell activation
Example:
  • Staphyloccal toxin- TSST, Exfoliative toxin, Enterotoxins.
  • Streptococcal toxin- Str pyrogenic exotoxin A, B, C
  • Mycoplasma arthritidis
  • Yersinia enterocolitica.
  • M.tuberculosis
Fig. 2.2.1: 56Mode of action of various antigens
  • Viral superantigens – EBV, CMV, HIV and Rabies nucleocapsid
  • Fungal superantigens- Malassezia furfur.
T Independent Antigen
T Dependent Antigen
  • Chemically Simple- LPS, capsule, flagella
  • Chemically Complex- protein
  • Dose dependent Immunogenicity
  • Immunogenic over wide range of dose
  • Limited antibody response-IgM and IgG3
  • All Antibody can be raised
  • No memory
  • Memory present
  • No macrophage processing
  • Macrophage processing step is needed
  • Slowly metabolized
  • Rapidly metabolized
T Independent (TI) Antigen can be Classified
TI 1(LPS)-
Activate B cell polyclonally
• Activate both mature and immature B cell
• B cell against LPS don't show class switch over
TI 2 (capsular polysaccharide and flagella)-
• Don't activate B cell polyclonally
• Activate mature B cell only
• B cells show class switch over
T dependent Antigen
• Don't activate B cell polyclonally
• B cells against T dependent Antigen show class switch over
• Shows affinity maturation
 
ANTIBODY
  • Immunoglobulin is Y shaped, consisting of 2 Heavy and 2Light chain
  • Papain digestion of Immunoglobulin produces-
    • 2 Fab portion- antigen binding site
    • 1 Fc portion- perform biological function like complement attachment site, adherence to monocyte
  • Pepsin digestion of Immunoglobulin- resulting one fragment F(ab’)2
  • Heavy chain – 5 types- µ,, α, Ε, δ – One of these 5 chains present in an immunoglobulin (Isospecificity)
  • Light chain - 2 types- kappa/lambda - One of these 2 chains present in an immunoglobulin
  • Hinge region- rich in cysteine, praline.
 
Location of Immunoglobulin Chains
  • Heavy chains -chromosome -14
  • Light chain kappa -chromosome -2,
  • Light chain lambda -chromosome -22
  • After the synthesis, germ line recombination (rearrangement) of heavy and light chain occurs.
Fig. 2.2.2: 57Structure of immunoglobulin
IgG Antibody
Possess highest DHS
Daily production,
Half life (23days),
Serum concentration (decreasing order- GAMDE)- i.e. highest is IgG and lowest is IgE
• Types- IgG1 > 2 > 3> 4 (Decreasing order of the serum concentration)
• Responsible for-
– Precipitation,
– Neutralization,
– NK cell binding (to perform ADCC)
– Classical complement pathway binding (Except IgG4)
– Co-agglutination(Except IgG3)
– Opsonization
• Appear late, indicates past / chronic infection
• IgG avidity increases with time - So, detection of less avidity IgG indicates relatively recent infection
• Secreted in placenta (Except IgG2)
• Secreted in breast milk
• Helps in phagocytosis by binding to FcR on phagocytes (Except IgG2)
Ig E Antibody
• Heat labile,
• Lowest DHS
• Responsible for- Type I HSN,
• Homocytotropic (Species specific) antibody
• Reagin antibody
• Raised in helminthic infections
IgA Antibody
• 2nd most abundant
• Types
– IgA2 - Surface / mucosal IgA- Dimer joined by J chain and secretory piece (from epithelium), valency four
– IgA1 - Serum IgA - monomer, minor
• Responsible for-
– Alternate Complement
– Mucosal /local immunity
58Ig D Antibody
• Surface immunoglobulin on the surface of B cells
• Possess highest carbohydrate content
IgM Antibody
Possess highest MIS:
Molecular weight (900,000),
Intravascular distribution (blood Antibody) (80%),
Sedimentation coefficient (19),
• Pentameric with 10 valency
• Surface immunoglobulin on the surface of B cells
• 1st to appear following infection, indicates recent infection
• 1st to appear in intrauterine life also (20wk)-indicates congenital infection
• Responsible for (or Mediates)-
– Agglutination,
– Haemolysis,
– Opsonization,
– Classical complement pathway binding
Example:
– O Antibody in typhoid,
– Reagin Antibody (syphilis),
– Antibody of ABO), Rh system
 
Abnormal Immunoglobulin
  • Bence Jones Protein
    • Coagulate at 50c, redissolve at 70c
    • Elevated in Multiple myeloma
    • Due to Unchecked proliferation of single clone of plasma chain
    • Made up Light chain of immunoglobulin i.e. Kappa or lambda (but never both in same patient).
  • Waldenstrom macroglobulinemia – IgM
  • Heavy chain disease – Fc part of heavy chain
  • Cryoglobulinemia:
    • Precipitate at low temp, dissolve at room temp
    • Seen with – HCV, myeloma, SLE, autoimmune disease.
 
Immunoglobulin Specificity
  • Idiotypic specificity:
    • Based on antigenic determinant in paratope known as idiotopes
    • Occurs due to change in variable region of the immunoglobulin.
  • Isotypic specificity:
    • Difference b/t Immunoglobulin of different classes and subclasses present in all individual in a given species – IgG /A/M/D/E
    • Occurs due to change in constant region of the heavy chain.
  • Allotypic specificity:
    • Difference between immunoglobulin of same classes between different individual in same species.
    • Occurs due to allelic difffernces between the individuals.
    • Example – kappa light chain and heavy chain of IgG (γ), IgA (α)
    • No allotype is there for lambda or µ, ε, δ heavy chains or λ light chain.
 
ANTIGEN ANTIBODY REACTION
 
General Features
  • Antigen Antibody Reaction is Specific
  • Entire molecules, not fragments react
  • Ag or Ab don't denature during reaction
  • Combination occurs on the surface
  • Combination is firm but reversible-Bound together by weaker bonds such as ionic bond, Vander waal's forces and hydrogen bond rather than stronger bond like covalent bond.
  • Ag and Ab can combine in varing proportion, but optimum reaction at equilibrium (Marrack's hypothesis)
 
59Principle-Marrack's (Lattice) Hypothesis
  • Zone of equivalence- Antigen antibody reaction takes place at its best when the numbers of antigen and antibody are equal to each other.
  • Antigen antibody reaction doesn't take place properly if antigen or antibody level is excess.
  • Prozone phenomena or antibody excess occurs in:
    • Enteric fever (Salmonella Typhi)
    • Brucellosis
    • Leptospirosis
    • Syphilis
  • Post zone phenomena (Antigen excess) occurs in -
    • Cryptococcus
 
Precipitation
  • Definition- {Soluble Antigen + Antibody} at suitable temperature and pH → leads to formation of insoluble precipitate / floccules
  • Ring test
    • Ascoli thermo precipitation test (anthrax),
    • Lancefield grouping (Streptococcus).
  • Slide flocculation test – VDRL, RPR
  • Tube flocculation test – Kahn test, standardization of toxin
  • Immuno-diffusion (In gel):
    • Produces visible band, so interpretation is easy
    • Can be preserved
    • Differentiate between antigens.
  • Examples of immunodiffusion in gel
    • Single diffusion in one dimension (Oudin procedure)
    • Double diffusion in one dimension (Oakley Fulthorpe procedure)
    • Single diffusion in two dimension (Radial immunodiffusion)
    • Double diffusion in two dimension – e.g. include Elek gel precipitation (C.diphtheriae toxigenicity testing), and Eiken test (E.coli).
  • CIEP (countercurrent immunoelectrophoresis) - used for detection of αFP, Antigen of Cryptococcus and Meningococcus
  • Rocket electrophoresis(one dimension single immunoelectrophoresis)
  • Laurell's two dimension immunoelectrophoresis.
 
Agglutination
  • Definition- {Insoluble Antigen + Antibody)- at suitable temperature and pH leads to clumps formation
Example
  • Widal test – Enteric fever
  • Standard agglutination test Brucella
  • Microscopic agglutination test – Leptospira
  • Weil felix – Rickettsia
  • Paul Bunnel test-– EBV
  • Blood grouping
  • Coombs test for incomplete (IgG) Ab
    • Rh incompatibility-
      • ✓ Fetal Rh Ab- detected by direct Coombs test
      • ✓ Maternal Rh Ab- detected by indirect Coombs test
    • Also Used in Brucellosis.
  • Passive agglutination test:
    • Solubleantigeniscoatedoncarrierparticlelike RBC(indirecthemagglutination test/IHA), latex (latex agglutination Test)
    • So that a precipitation reaction can be converted to an agglutination reaction and hence better visualized.
    • E.g. –TPHA (syphilis), Rose Waaler test (Rheumatoid arthritis).
  • RPHA (Reverse Passive Hemagglutination test)-
    • Antibody is coated on a surface of carrier particle like RBC to detect antigens
    • E.g.-HBsAg.
 
60Complement Fixation Test (CFT)
  • First step- Test serum (antibody) + antigen + guinea pig complement
  • Second step- indicator system ( sheep RBC + amboceptor/ rabbit antibody to sheep RBC)
    • If patient serum has no antibody- complements are not utilized in the 1st step, so available for the 2nd step, hence there is complement mediated hemolysis of sheep RBC
    • If patient serum has antibody - complements are utilized in the 1st step, so not available for the 2nd step, hence there is no hemolysis of sheep RBC.
  • Examples
    • Wasserman test (Syphilis)
    • CFT for viral diseases and other infectious disease.
  • Indirect CFT- Used for testing certain avian (duck & parrot) and mammalian (horse, cat) RBCs which don't fix to guinea pig complements.
  • Other complement dependent serological test-
    • TPI, Sabin Feldman test(Toxoplasmosis)
    • Cytocidal or cytolytic tests- Vibrio
    • Immunoadherence – Vibrio & T.pallidum.
 
Neutralization Test
  • Toxin antitoxin neutralization test-
    • Schick test- Diphtheria toxin
    • Naegler reaction
    • Streptolysin O neutralization test
  • Viral neutralization test
  • Bacteriophage neutralization test - By Plaque inhibition test.
 
Opsonization
  • Enhanced phagocytosis by coating the microbial surfaces by opsonins.
  • Examples of opsonin molecules include:
    • Antibodies: IgG and IgM
    • Components of the complement system: C3b, C4b, and iC3b
    • Mannose-binding lectin (initiates the formation of C3b)
  • Most important components for opsonization- C3b & Fc (IgG).
 
Newer Methods Using on Lebelled Molecules
  • ELISA (Enzyme linked immunosorbent assay)- Enzyme tagged
  • RIA (Radioimmuno assay) – Radioactive isotope lebelled
  • CLIA(Chemiluminoscent linked immuno assay)-– Chemiluminoscent compound like acridinum
  • IFA (Immunoflouroscence assay) – Flouroscent dye lebelled.
 
Newer Methods Using NCM (Nitrocellulose Membrane)
ICT (Immunochomatographic Test):
  • Also known as Rapid test/dip stick or strip test or lateral flow assay
  • Uses Nitrocellulose membrane(NCM)
  • Used in Malaria, and other infectious diseases.
Western Blot – 3 Steps
  • PAGE (Polyacrylamide gel electrophoresis)→ Electrophoretically mobilize and separate the antigen fragments
  • Nitrocellulose membrane (NCM) blotting → antigenic fragments are blotted on NCM
  • Enzyme immunoassay – to detect the antibodies in serum of the patient.
 
COMPLEMENT SYSTEM
  • Non specific proteins
  • Not species specific
  • Constitutes 5% of serum protein
  • Heat labile −56c for 30min
  • Binds to Fc region of antibody- IgM (binds strongly) followed by IgG3 → 1 →261
Fig. 2.2.3: Complement (Opsonization) pathways
MBL- Mannose binding Lectin
MASP- MBL-associated serine proteases
Site of Synthesis of Complements
  • Liver- C3, C6, C9
  • GIT- C1
  • Macrophage- C2, C4
  • Spleen-C5, C8
Classical Pathway
Alternate Pathway
Lectin Pathway
Activator
Antigen antibody complex
Zymosan, Endotoxin
IgA, IgD, IgG4
Cobra venom
Nephritic factor
Carbohydrate residue of bacterial cell wall (mannose binding protein) that binds to host lectin antigen.
1st complement activated
C1
C3b
C4
C3 covertase
C4b2a
C3bBb
C4b2a
C5 covertase
(C3 covertase + 3b)
C4b2a3b
C3bBb3b
C4b2a3b
Complement level in the serum
All C1-C9: Low
C1,C4,C2- Normal
Others- Low
C1- Normal
Others- Low
Immunity
Acquired Immunity
Innate Immunity
Innate Immunity
 
Biological Role of Complement
  • Lysis of target cell
  • Chemotaxis- C5a & C3a
  • Anaphylaxis- C5a & C3a
  • 62Mediate HSN II & III
  • Opsonization- C3b
  • Autoimmune diseases
  • Immune adherence- EBV receptors
  • Endotoxic shock
  • Kinin like activity (↑vascular permeability)- C2b.
Deficiency of Complement
Syndrome Associated
C1Esterase inhibitor
Hereditary angioneurotic edema
Early components of classical pathway (C1,C2,C4)
SLE and other collagen vascular disease Pyogenic infection
C3 and its regulatory protein
Severe recurrent pyogenic infection
Complement C5-9 (membrane attack complex)
Gram –ve diplococcic (Neisseria), Toxoplasma
D,I
Pyogenic infections
Properdin
Neisseria infections
H
Hemolytic uremic syndrome
63MULTIPLE CHOICE QUESTIONS
 
ANTIGEN
1. Superantigen causes: (NEET Pattern Based)
  1. Enhancement of phagocytosis
  2. Polyclonal activation of B cells
  3. Antigen presentation by Macrophage
  4. Activation of complement
2. Which part of bacteria is mostly antigenic? (NEET Pattern Based)
  1. Protein
  2. Carbohydrate
  3. Lipid
  4. Nucleic acid
3. T independent antigen acts through: (NEET Pattern Based)
  1. Macrophages
  2. B cells
  3. CD8 T cells
  4. CD4 T cells
4. Superantigens are produced by all the following pathogens, except: (AI 2010, APPG 2014, Similar MCQ)
  1. Staphylococcus aureus
  2. Enterococcus faecalis
  3. Rabies virus
  4. HIV
5. A super antigen is a bacterial product that: (AI 2008)
  1. Binds to B7 and CD28 co-stimulatory molecules
  2. Binds to the beta chain of TCR stimulating T cell activation
  3. Binds to the CD4 + molecule causing T cell activation
  4. Is presented by macrophages to a larger – than: normal number of T helper CD4 + lymphocytes
6. Superantigens are: (PGI June 2005)
  1. Erythrotoxin of Staph. aureus
  2. Cl. difficile toxin
  3. Staphylococcal toxic shock syndrome toxin
  4. Cholera toxin
7. All of the following statements about carbohydrate antigen are true except: (AIIMS Nov 2008)
  1. It has lower immunogenicity
  2. Memory response is seen
  3. Cause polyclonal B cell stimulation
  4. Does not require stimulation by T cells
8. The protection against small pox by previous infection with cowpox represents: (AIIMS May 2004)
  1. Antigenic cross reactivity
  2. Antigenic specificity
  3. Passive immunity
  4. Innate immunity
9. Which of the following statements is true about hapten? (AI 2004)
  1. It induces brisk immune response
  2. It needs carrier to induce immune response
  3. It is a T: independent Antigen
  4. It has no association with MHC
10. Heterophile antibody is found in: (NEET Pattern Based)
  1. Weil Felix test
  2. Widal test
  3. Standard agglutination test
  4. All
11. All of the following interaction occurs between antigen antibody reaction except: (Recent Question 2013)
  1. Ionic bond
  2. Covalent bond
  3. Hydrogen bond
  4. Vander waal forces
 
ANTIBODY
12. Action of papain on an IgG molecule produces: (DNB Dec 2010)
  1. 2 Fc fragments and 1 Fab fragment
  2. 1 Fc fragment and 2 fab fragments
  3. 2 Fc fragments and 2 fab fragments
  4. 1 Fc fragment and 1 fab fragment
13. Immunoglobulin isotype class switch over is determined by: (NEET Pattern Based)
  1. Constant region of heavy chain
  2. Variable region of heavy chain
  3. Constant region of light chain
  4. Variable region of light chain
14. Immunoglobulin that changes in variable region is called as: (NEET Pattern Based)
  1. Allotypic variation
  2. Iostypic variation
  3. Idiotypic variation
  4. Isotypic variation
15. Variable portion of an immunoglobulin: (NEET Pattern Based)
  1. Amino terminal
  2. Carboxy terminal
  3. Acid terminal
  4. Amoxy terminal
16. Vaccination is based on the principle of: (AI 2012)
  1. Agglutination
  2. Phagocytosis
  3. Immunological memory
  4. Clonal detection
17. Antibody diversity is due to: (PGI Dec 2008)
  1. Gene rearrangement
  2. Gene translocation
  3. Antigenic variation
  4. CD40 molecules
  5. Mutation
18. Which of the following statements is true about isotypic variation? (AIIMS Nov 2008)
  1. These result due to subtle amino acid changes resulting from allelic differences
  2. 64These result due to changes in amino acid in heavy and light chain at variable region
  3. Changes in heavy and light chain in constant region is responsible for class and subclass of immunoglobulins
  4. These are areas in antigen that bind specifically to antibody
19. All of the following statements about hybridoma technology are true except: (AIIMS Nov 2008)
  1. Specific antibody producing cells are integrated with myeloma cells
  2. Myeloma cells with mutation in salvage pathway grows well in HAT medium
  3. Aminopterin, a folate antagonist, inhibits denovo pathway
  4. HGPRT ase and thymidylate synthetase are required for salvage pathway
20. Which of the following is associated with class specific antigenic determinants of an immunoglobulin? (AIIMS May 2004)
  1. L:chain
  2. H:chain
  3. J:chain
  4. Variable region
21. Antigen binding site on antibody is: (PGI June 2002)
  1. Hinge region
  2. Constant region
  3. Variable region
  4. Hyper variable region
  5. Idiotype region
22. In which of the following(s), the two allele are inherited together: (PGI Nov 2014)
  1. Idiotype
  2. Genotype
  3. Phenotype
  4. Allotype
  5. Isotype
23. Which of the following statements is true regarding kappa, lambda and heavy chain immunoglobulins? (PGI 2001)
  1. Coded in the same site of a chromosome
  2. Coded in different sites of same chromosome
  3. The chains are formed by genetic rearrangement after maturation
  4. Different chains of same immunoglobulins are coded by different chromosomes
  5. Different chains of same immunoglobulins are coded by same chromosome
24. When Papain cleaves IgG antibody, it produces: (Recent Question 2013)
  1. 2 Fab and 1 Fc
  2. 2 Fab and 2 Fc
  3. 1 Fab and 1 Fc
  4. 1 Fab and 2 Fc
 
IgG
25. True about an immunoglobulin: (NEET Pattern Based)
  1. IgG has max conc. in serum
  2. IgM has max conc. in serum
  3. IgA has max conc. in serum
  4. IgE has max conc. in serum
26. Ig that helps in Opsonization: (NEET Pattern Based)
  1. IgA
  2. IgG
  3. IgD
  4. IgE
27. The serum concentration of which of the following human IgG subclass is maximum: (AI 2005)
  1. IgG 1
  2. IgG 2
  3. IgG3
  4. IgG4
28. Which of the following immunoglobulins can cross placenta? (DNB DEC 2012, PGI June 2001)
  1. IgA
  2. IgM
  3. IgG
  4. IgD
29. True about antibody: (PGI MAY 2013, APPG2012, Similar MCQ)
  1. IgM is produced in primary response
  2. IgD protects mucosa
  3. IgE is main antibody in secondary response
  4. IgG is main antibody in secondary response
  5. IgA protects body surface
30. Which antibody deficiency causes recurrent infection by organisms with polysaccharide capsule? (Recent Question 2013)
  1. IgA
  2. IgG1
  3. IgG2
  4. IgM
31. True of the following is /are: (PGI June 2001)
  1. IgA crosses placenta
  2. Half life of IgG is 23days
  3. IgD is heat stable
  4. IgE has highest carbohydrate content
 
IgM
32. Activator of classical pathway of complement? (NEET Pattern Based)
  1. IgA
  2. IgG
  3. IgM
  4. IgE
33. Pentameric antibody with a J chain is? (NEET Pattern Based, DNB Dec 2010, DNB June 2009)
  1. IgA
  2. IgG
  3. IgM
  4. IgE
34. Which immunoglobulin acts as receptor on B cell ? (DNB Dec 2010)
  1. Ig G
  2. Ig A
  3. Ig M
  4. Ig E
35. In-utero infection leads to raise of which immunoglobulin first? (AI 2003, DNB DEC 2012)
  1. IgG
  2. IgA
  3. IgG
  4. IgM
36. 65Rheumatoid arthritis is best diagnosed by: (AIIMS Nov 2011) (Modified repeat of PGI 1998)
  1. Anticitrulline antibody
  2. Ig G antibody
  3. Ig A antibody
  4. Ig M antibody
37. The most avidly complement fixing antibody is: (AIIMS May 2002)
  1. IgA
  2. IgG
  3. IgM
  4. IgE
38. Which antibody is elevated in primary immune response? (Recent Question 2013)
  1. IgA
  2. IgM
  3. IgG
  4. IgE
39. Which is the first antibody is elevated in fetal life? (Recent Question 2013)
  1. IgA
  2. IgM
  3. IgG
  4. IgE
 
IgA
40. Synthesis of an immunoglobulin in membrane bound or secretory form is determined by: (AIIMS May 2012)
  1. One turn to two turn joining rule
  2. Class switching
  3. Differential RNA processing
  4. Allelic exclusion
41. Immunoglobulin in peyer's patch is: (DNB June 2011)
  1. IgM
  2. IgG
  3. IgA
  4. IgD
42. Immunoglobulin present in local secretions is: (DNB DEC 2012, DNB DEC 2009)
  1. IgG
  2. IgA
  3. IgM
  4. IgD
43. The secretory component of immunoglobulin molecule is: (PGI 2001)
  1. Formed by epithelial cells of lining mucosa
  2. Formed by plasma cell
  3. Formed by epithelial cell and plasma cell
  4. Secreted by bone marrow
 
IgE
44. Immunoglobulin that is inactive by heating is? (DNB DEC 2012)
  1. IgG
  2. IgA
  3. IgM
  4. IgE
45. Immunoglobulin that is elevated in helminthic infection? (NEET Pattern Based)
  1. IgG
  2. IgA
  3. IgM
  4. IgE
46. IgE is secreted by: (PGI 2005)
  1. Mast cell
  2. Basophil
  3. Eosinophils
  4. Plasma cells
  5. Neutrophils
47. Which immunoglobulin is scarce in human serum: (PGI June 2005)
  1. IgA
  2. IgG
  3. IgM
  4. IgD
  5. IgE
 
ABNORMAL IMMUNOGLOBULIN
48. Which precipitates at 50 to 60 C but re dissolve on heating : (AI 2012; AI 2000)
  1. Bence jones proteins)
  2. Heavy chain
  3. Both light and heavy chains
 
ANTIGEN ANTIBODY REACTION
49. Naegler reaction is example of: (NEET Pattern Based)
  1. Precipitation
  2. CFT
  3. Agglutination
  4. Neutralization
50. Coombs test is example of: (NEET Pattern Based)
  1. Coombs
  2. Agglutination
  3. Neutralization
  4. CFT
51. Paul Bunnel test is example of: (NEET Pattern Based)
  1. Agglutination
  2. Precipitation
  3. Neutralization
  4. CFT
52. Which are example of agglutination test: (NEET Pattern Based)
  1. Widal test
  2. VDRL test
  3. Kahn test
  4. Ascoli's test
53. Which of the following acts as an opsonin? (DNB June 2011)
  1. C3a
  2. C3b
  3. C5a
  4. LTB4
54. Immunoglobulins bound to the surface of bacteria mediates phagocytosis by- (Phagocytosis causes complement activation by: (AIIMS Nov 2011)
  1. C3b and Fc
  2. Receptor mediated endocytosis
  3. Lysozomal burst
  4. Oxidase action
55. Rose Waaler test done in: (PGI Dec 2006)
  1. Rheumatoid arthritis
  2. SLE
  3. Sjogren's syndrome
  4. Polymyositis
  5. Rheumatic fever
56. 66The following methods of diagnosis utilize labeled antibodies except: (AIIMS May 2005)
  1. ELISA (Enzyme Linked Immunosorbent Assay)
  2. Hemagglutination inhibition test
  3. Radioimmunoassay
  4. Immunofluorescence
57. Heterophile agglutination is/are used in all test except: (PGI MAY 2013)
  1. Widal test
  2. Weil-Felix reaction
  3. Paul -Bunnel test
  4. ELISA
  5. Cold agglutination test
58. Skin test based on neutralization reaction is/are: (PGI June 2004)
  1. Casoni test
  2. Lepromin test
  3. Tuberculin test
  4. Schick test
59. Prozone phenomenon is due to:
  1. Excess antigen (NEET Pattern Based, PGI June 2003)
  2. Excess antibody
  3. Hyperimmune reaction
  4. Disproportionate antigen - antibody levels
60. Prozone phenomenon is a feature is: (PGI 2001)
  1. Tularemia
  2. Legionnaire's disease
  3. Plague
  4. Brucellosis
61. Antigen antibody reaction is seen maximum in? (AIIMS May 10)
  1. Excess antibody
  2. Excess antigen
  3. Antigen & antibody are equal
  4. Antigen & antibody are low
 
COMPLEMENT SYSTEM
62. Complements act by: (NEET Pattern Based)
  1. They activate Adeny l cyclase
  2. Inhibits elongation factor
  3. Destruction of cell wall
  4. Increased permeability of cell membrane
63. Powerful Activator of classical complement pathway: (NEET Pattern Based)
  1. IgA
  2. IgG
  3. IgM
  4. IgD
64. Complement formed in liver: (NEET Pattern Based)
  1. C2, C4
  2. C3, C6, C9
  3. C5, C8
  4. C1
65. C-3 convertase in alternate complement pathway: (NEET Pattern Based)
  1. C4b2a
  2. C3b
  3. C3bBb
  4. C3a
66. Central component of complement system is? (NEET Pattern Based, DNB June 2011)
  1. C3a
  2. C3b
  3. C4
  4. C5
67. Which of the following acts as a chemoattractant? (DNB DEC 2012)
  1. C3a
  2. C3b
  3. C5b
  4. LTB4
68. Snake venom acts by ? (DNB June 2010)
  1. Classical pathway
  2. Alternate pathway
  3. Both
  4. None
69. Recurrent facial/oropharyngeal /laryngeal edema in a patient has low C4 & C3 and normal factor B. The pathogenesis is most likely due to: (NEET Pattern Based, AIIMS May 2005)
  1. Immune complex disease
  2. C1 esterase inhibitors deficiency
  3. Hereditary deficiency of C2
  4. Classical pathway activation by IgM
70. Which of the following best denotes classical complement pathway activation in immuno inflammatory condition: (AIIMS 2004)
  1. C2, C4, C3 decreased
  2. C2 and C4 normal, C3 is decreased
  3. C3 normal and C2 C4 decreased
  4. C2, C4, C3 all are elevated
71. C3 convertase acts on: (JIPMER 2006)
  1. C4b2b
  2. C4b2B3a
  3. C4b
  4. C3
67MULTIPLE CHOICE QUESTIONS
 
Antigen
1. Ans. (b) (Polyclonal activation of B cells) Ref: Journal-Superantigen/IJMM/2004/Vol.22/Iss.4
  • Superantigens directly bind non specifically to Vβ region of TCR of T cells and that leads to massive release of cytokines which in turn causes polyclonal activation of B cells and hypergammaglobulinemia.
2. Ans. (a) (Proteins) Ref: Ananthanarayan 9/e p88, 8/e p91
  • Decreasing order of antigenicity-
    • ✓ Highest antigenic is protein followed by carbohydrate, lipid and nucleic acid.
3. Ans. (b) (B cells) Ref: Ananthanarayan 9/e p90
  • T -independent antigens directly bind non specifically to surface immunoglobulins of B cells which in turn causes polyclonal activation of B cells and hypergammaglobulinemia.
4. Ans. (b) (Enterococcus faecalis) Ref: Journal-Superantigen/IJMM/2004/Vol.22/Iss.4
  • Refer text
5. Ans. (b) Binds to the beta chain of TCR stimulating T cell activation
Ref: Journal-Superantigen/IJMM/2004/Vol.22/Iss.4
  • Refer text
6. Ans.(c), (Staphylococcal toxic shock syndrome toxin) Ref:Journal-Superantigen/IJMM/2004/Vol.22/Iss.4
Already explained, Refer text
  • Erythrogenic toxin, though is a superantigen but is produced by Streptococcus (Not by Staph.aureus)
7. Ans. (b) (Memory response is seen)
Ref: Venu Gopal Jayapal's Immunology 1/e p42, Ananthanarayan 9/e p90, 8/e p93
  • Carbohydrate antigen like Lipopolysaccharide (LPS) is an example of T Independent Antigen
  • T Independent Antigen -
    • Directly stimulates B cell for antibody production without participation of T cell.
    • ✓ Dose dependent Immunogenic with Limited antibody response-IgM and IgG3
    • ✓ Lack memory response
    • ✓ LPS can cause polyclonal B cell activation
8. Ans. (a) (Antigenic cross-reactivity) Ref: Ananthanarayan 9/e p89, 8/e p93
  • Small pox and cow pox both belong to Poxviridae family. Due to antigen cross reactivity between them, antibody response against cow pox vaccine can protect against small pox.
  • Other e.g of Heterophile specificity – Refer text
9. Ans. (b) (It needs carrier to induce immune response)
Ref: Ananthanarayan 9/e p87, 8/e p91, Kuby's Immunology 6/e p77
  • Hapten – doesn't have Immunogenicity but retain immunological reactivity (i.e. antigenic)
  • They are incapable of inducing immune response to produce antibody, but when combines with larger carrier molecule, become immunogenic.
  • Haptens are T dependent antigens ……… Ananthanarayan 9/e p87, 8/e p94
10. Ans. (a) (Weil Felix test) Ref: Ananthanarayan 9/e p90, 8/e p93
  • Weil Felix test is an example of heterophile agglutination test. Ricketssial antibodies are detected by using Proteus Ox2, Ox19 & OxK antigens.
6811. Ans. (b) (Covalent bond) Ref: Ananthanarayan 8/e p104 & 9/e p103
The antigen antibody reaction is reversible and the antigen and antibody molecules are bound together by weaker bonds such as ionic bond, Vander Waal's forces and hydrogen bond rather than stronger bond like covalent bond.
 
Antibody Structure and Properties of Antibody
12. Ans. (b) (1 Fc fragment and 2 fab fragments) Ref: Ananthanarayan 9/e p94, 8/e p96
  • Papain digestion of Immunoglobulin produces-2 Fab portion and 1 Fc portion
  • Pepsin digestion of Immunoglobulin- resulting one fragment F(ab’)2
13. Ans. (a) (Constant region of heavy chain) Ref: Ananthanarayan 9/e p94, 101,8/e p96
  • Isotypic specificity- Difference b/t Immunoglobulin of different classes and subclasses present in all individual in a given species – IgG /A/M/D/E . Occurs due to change in constant region of the heavy chain
14. Ans. (c) (Idiotypic variation) Ref: Ananthanarayan 9/e p94, 100, 8/e p96
  • Idiotypic specificity- Due to the change in the idiotopes (antigenic determinant in paratope) of different antibodies. It occurs due to change in variable region of the immunoglobulin.
15. Ans. (a) (Amino terminal) Ref: Ananthanarayan 9/e p94, 8/e p96
  • Variable region end- Amino terminal (NH4)
  • Constant region end- carboxy terminal (CHO)
16. Ans. (c) (Immunological memory) Ref: Ananthanarayan 9/e p83
  • Vaccination leads to production of memory cells against the immunogens which play an important role in prevention of the infection by producing antibodies on subsequent exposure of the organism.
17. Ans. (a), (e) (Gene rearrangement, mutation) Ref: Kuby's Immunology 6/e p 123
  • Somatic hypermutation and Recombination of V-(D)-J segments joining (Gene rearrangement) are one of the important mechanisms involved for in Antibody diversity
Antibody Diversity
There are 1010 antibodies that can be generated against various antigenic stimuli.
This antibody diversity is possible due to-
  • Presence of Multiple germ-line gene segments
  • Recombination of V-(D)-J Segments joining
  • Junctional flexibility
  • P-region nucleotide addition (P-addition)
  • N-region nucleotide addition (N-addition)
  • Somatic hypermutation
  • Combinatorial association of light and heavy chains
18. Ans.(c)(Changesinheavyandlightchaininconstantregionisresponsibleforclassandsubclassofimmunoglobulins)
Ref: Ananthanarayan 9/e p99, 8/e p103
  • Isotypic specificity occurs due to changes in heavy and light chain in constant region is responsible for class and subclass of immunoglobulins.
  • Immunoglobulin specificity can be three type: Idiotypic specificity, Isotypic specificity and Allotypic specificity (for detail refer text)
19. Ans. (b) (Myeloma cells with mutation in salvage pathway grows well in HAT medium)
Ref: Venu Gopal Jayapal's Immunology 1/e p80
“Myeloma cells lacks HGPRT enzyme, hence they cannot grow well in HAT medium.”
Hybridoma technology
Done for mass production of monoclonal antibody
Principle
  • A normal activated antibody producing B cell is fused with a myeloma cell (cancerous plasma cell capable of indefinite growth) to produce a hybrid cell called hybridoma.
  • Hybridoma cells have property of both normal activated antibody producing B cell and myeloma cell (immortal), hence can be used to produce antibody in mass.
69Procedure
  • Mouse is injected with desire antigen.
  • After a period of immunization, mouse spleen B cells (HGPRT+ and Ig+) are isolated and mixed with myeloma cells (HGPRT-ve and Ig –ve) and allowed to grow in HAT media.
  • 3 Types of cells will be generated:
    • Unfused spleen cells- can grow but don't survive long as they are not immortal
    • Unfused myeloma cells- can't grow as they lack HGPRT to perform salvage pathway for purine synthesis
    • Hybrid cell (fused myeloma cell with spleen cells)- can grow and survive long.
HAT medium- contains Hypoxanthine, Aminopterine, Thymine
  • Purine synthesis in mammalian cell (spleen B cell) occurs by either de novo or salvage pathways.
  • Aminopterine blocks the de novo pathway so that the cell has to perform the salvage pathway to synthesis purines for its survival.
  • Salvage pathway requires HGPRT (hypoxanthine guanine phosphoribosyl phosphate) and thymine kinase
  • So any cell that lacks HGPRT cannot grow on HAT medium.(e.g. Myeloma cell)
20. Ans. (b) (H-chain) Ref: Ananthanarayan 9/e p94, 8/e p97
Immunoglobulin is classified to various classes like IgG/ IgA/ IgM/ IgD/ IgD based on the differences between constant regions of Heavy chain.
Immunoglobulin class
IgG
IgA
IgM
IgD
IgE
Heavy chain type
Gamma chain
Alfa chain
Mu chain
Delta chain
Epsilon chain
21. Ans. (d), (e) (Hyper variable region, Idiotype region) Ref: Ananthanarayan 9/e p96, 8/e p 97,101
  • Immunoglobulin molecule consists of two heavy chain and two light chain
  • Both heavy chain and light chain are divided to constant region and variable region
  • In Variable region - The amino acid sequence is variable determining the immunological specificity of antibody molecule
  • Hyper variable region - hotspots within variable region where the aminoacid variability is more. (3 spots in L and 4 spots in H chain are hyper variable)
  • Complementarity determining region-(paratope)- the site on hypervariable region that makes actual contact with an epitope.
  • Idiotope- specific antigenic determinant on paratope against which antibody can be raised.
22. Ans. (d) (Allotypes) Ref: Ananthanarayan 9/e p96
ALLOTYPES, or allelic variants within the constant regions, are known to exist for some of these isotypes, and are inherited in a Mendelian co-dominant fashion in allelic manner (“allelic type”).
23. Ans. (c), (d) (The chains are formed by genetic rearrangement after maturation, Different chains of same immunoglobulins are coded by different chromosome)
Ref: Venu Gopal Jayapal's Immunology 1/e p77. Journal: Immunoglobulin genes and the origin of antibody diversity, Rev Fr Transfus Hemobiol. 1992 Jan; 35(1):47-65
  • Heavy chains are located on human chromosome 14
  • Light chain kappa chains is located on human chromosome 2,
  • Light chain lambda chains is located on chromosome 22
  • Germ line recombination (rearrangement) heavy and light chain occurs
Germ Line Recombination Heavy and Light Chain
  • The light chain kappa and lambda multi gene family have 3 segments V, J, C gene segments on chromosome 2 and 22 respectively. (V-variable, J- junctional, C- constant)
  • Kappa chain has- 40V k, 5J k and 1C k segments.
  • Lambda chain has- 30Vλ, 4Jλ and 1Cλ segments
  • The heavy chain multi gene family has 4 segments V, D, J, C gene segments on human chromosome 14. (V-variable, D-diversity, J- junctional, C- constant)
  • Heavy chain has- 51VH, 27DH, 6 JH, a series of CH each encodes one constant region of Ig isotype.
  • During development of B cell, the immunoglobulin gene rearrangement occurs in an orderly way.
    • ✓ First heavy chain is formed by combination of any 1 gene segment of V, D, J and C segments. (1st DJ joining followed by V-DJ joining followed by VDJ-C joining)
    • 70✓ Then the light chain (either kappa or lambda) is formed by combination of any 1 gene segment of V, J and C segments. (1st VJ joining followed by VJ-C joining)
    • ✓ Then, heavy and light chain segments joint to form the complete immunoglobulin.
24. Ans. (a) (2 Fab & 1 Fc) Ref: Ananthanarayan 9/e p95
  • Papain cleaves antibody to- 2 Fab & 1 Fc
  • Pepsin cleaves antibody to- 2 (Fab)’
 
IgG Immunoglobulin
25. Ans. (a) (IgG has max conc. in serum) Ref: Ananthanarayan 9/e p96, 8/e p98
  • Decreasing order of Serum level of various immunoglobulin is GAMDE- i.e. highest is IgG and lowest is IgE
  • Decreasing order of Serum level of IgG subtypes - IgG1 > 2 > 3> 4
26. Ans. (b), (IgG) Ref: Ananthanarayan 9/e p96
  • Most important opsonization- C3b & Fc (IgG)
  • Examples of opsonin molecules include:
    • ✓ Antibodies: IgG and IgM
    • ✓ Components of the complement system: C3b, C4b, and iC3b
    • ✓ Mannose-binding lectin (initiates the formation of C3b)
27. Ans. (a) (IgG1) Ref: Ananthanarayan 9/e p97, 8/e p98
  • Serum level of IgG subtypes- IgG1-65%, IgG2-23%, IgG3-8%, IgG4- 4%
28. Ans. (c) (IgG) Ref: Ananthanarayan 9/e p96, 8/e p98
  • IgG is secreted in placenta, breast
  • Among the four subclasses- IgG2- poorly crosses placenta
29. Ans. (a) (d) (e) (IgM is produced in primary response, IgG is main antibody in secondary response, IgA protects body surface) Ref: Ananthnarayan 9th /p96-98
Refer chapter review for detail.
30. Ans. (c) (IgG2 deficiency) Ref: Jawetz 25/e p130, Robins Pathology 7/e p 144-147
Refer chapter 2.5, Q.No-8.
31. Ans. (b), (c) (Half life of IgG is 23days, Ig D is heat stable) Ref: Ananthanarayan 9/e p96, 8/e p98
  • IgG crosses placenta where as both IgG and IgA are present in breast milk
  • Half life of IgG is 23days (highest)
  • IgD is heat stable (only IgE is heat labile)
  • IgD has highest carbohydrate content
  • IgG induces leukotrienes release during inflammation
 
IgM Immunoglobulin
32. Ans. (c) (IgM) Ref: Ananthanarayan 9/e p98, 8/e p99
  • Classical complement is activated by both IgG and IgM. However, IgM is a powerful activator than IgG
33. Ans. (c) (IgM) Ref: Ananthanarayan 9/e p98, 8/e p99
  • IgM is pentameric having a valency of 10, the five molecules of IgM are joined together by J-chain.
34. Ans. (c) (IgM) Ref: Ananthanarayan 9/e p98, 8/e p99
  • Immunoglobulin acts as receptor on B cell (i.e. surface immunoglobulin)- IgM and IgD
35. Ans. (d) (IgM) Ref: Ananthanarayan 9/e p98, 8/e p99
  • IgM is phylogenetically oldest immunoglobulin and the earliest to be synthesized by the fetus, beginning by about 20 weeks.
7136. Ans. (a) > (d) (Anticitrulline antibody, IgM antibody) Ref: Ananthanarayan 8/e p175,Wikipedia
  • Anti-citrulline antibodies
    • Autoantibodies that are frequently detected in the blood of rheumatoid arthritis patients.
    • ✓ Research suggests that in the joints of patients with rheumatoid arthritis, proteins may be changed to citrulline as part of the process that leads to inflammation of the rheumatoid joint.
    • When the citrulline antibody is found in a patient's blood, there is a 90%-95% likelihood that the patient has rheumatoid arthritis.
  • Rheumatoid arthritis is also diagnosed by detecting RF (Rheumatoid factor)
  • RF is a IgM antibody directed against Fc portion of IgG antibody.
  • RF is usually a 19s IgM, though IgA or IgG have been identified ….. Ananthanarayan 8/e p175
  • RF is detected by passive agglutination tests like:
    • Rose Waaler test
    • Latex agglutination test
37. Ans. (c) (IgM) Ref: Ananthanarayan 9/e p98, 8/e p100,118
  • IgM is a better activator of Classical complement pathway than IgG
  • Classical complement pathway starts when an antibody binds to C1q.
  • C1q has six binding sites and atleast two binding sites have to be recognized by antibody for affective activation which can be mediated by one molecule of IgG or two molecule of IgM.
  • Among the subclasses, IgG3 can affectively activate complement but others poorly activate.
  • Alternate pathway of complement- activated by IgA
38. Ans. (b) (IgM) Ref: Ananthanarayan 9/e p146
  • IgM- elevated in primary immune response
  • IgG- elevated in secondary immune response
39. Ans. (b) (IgM) Ref: Ananthanarayan 9/e p98
  • IgM- First antibody is elevated in fetal life
  • IgG- Only antibody that crosses placenta
 
IgA Immunoglobulin
40. Ans. (c) (Differential RNA processing) Ref: Kuby's Immunology 6/e p130-131
  • “Differential RNA processing of a common primary transcript determines whether the secreted or membrane form of an immunoglobulin will be produced”……. Kuby 6/e p131
    Membrane-bound or secreted form of immunoglobulin:
  • Mature naive B cells produce only membrane-bound antibody (IgD or IgM), whereas differentiated plasma cells produce secreted antibodies (IgA).
  • Membrane-bound or secreted form of immunoglobulin is synthesized by alternate RNA splicing.
  • RNA splicing- It is a post transcription modification by which the introns (that don't code for proteins) are deleted and the exons are connected to form mRNA from which the proteins are formed.
  • Post-transcriptional processing of immunoglobulin primary transcripts is required to produce functional mRNAs
  • Membrane-bound or secreted form of immunoglobulin differ in the amino acid sequence of the heavy-chain carboxyl-terminal domains.
    • ✓ The secreted form has a hydrophilic sequence of about 20 amino acids in the carboxylterminal domain;
    • ✓ Membrane-bound form with a sequence of about 40 amino acids containing a hydrophilic segment that extends outside the cell, a hydrophobic transmembrane segment, and a short hydrophilic segment at the carboxyl terminus that extends into the cytoplasm.
41. Ans. (c) (IgA) Ref :Hashizume T etal: Infect Immun. 2008 Mar;76(3):927-34. Epub 2007 Dec 17
  • Peyer's patches are composed of 30-40 lymphoid follicles present in intestinal sub mucosa
  • They are required for intestinal secretory immunoglobulin A responses which provides local or intestinal immunity or mucosal immunity.
  • Peyer's patches also contain Intra epithelial CD8 lymphocyte containing TCR δ receptors
42. Ans. (b) (IgA) Ref : Ananthanarayan 9/e p97, 8/e p99-100
  • Refer text
7243. Ans. (a) (Formed by epithelial cells of lining mucosa) Ref: Ananthanarayan 9/e p97, 8/e p99
  • Surface IgA is dimeric, both the monomers are joined by J chain and secretory piece
  • J chain is synthesized by the plasma cell (present in other polymeric immunoglobulin like IgM)
  • Secretory component (rich in glycine) is produced by mucosal epithelium.
 
IgE Immunoglobulin
44. Ans. (d), (IgE) Ref: Ananthanarayan 9/e p98, 8/e p100
  • Only heat labile immunoglobulin is – IgE
45. Ans. (d), (IgE) Ref: Ananthanarayan 9/e p99, 8/e p100
  • Helminthic infection is characterized by an increase of IgE antibodies.
46. Ans. (d), (Plasma cell) Ref: Ananthanarayan 9/e p99, 8/e p100
  • IgE, like other immunoglobulins is secreted by plasma cells.
  • IgE after secreting from plasma cell, gets bound to mast cell by Fc portion. When antigen comes and binds to fab region of IgE, it in turn stimulates mast cell and mast cell degranulation occurs. (Type I hypersensitivity reaction)
47. Ans. (e), (IgE) Ref: Ananthanarayan 9/e p96, 8/e p98 Table 12.1
  • Decreasing order of Serum concentration and daily production of immunoglobulin-
    • ✓ IgG> IgA >IgM > IgD> IgE
    • ✓ Highest – IgG
    • ✓ Lowest – IgE
 
Abnormal Immunogloblin
48. Ans. (a), (Bence jones proteins) Ref: Ananthanarayan 9/e p99, 8/e p100
Bence Jones Protein-
  • Abnormal immunoglobulin found in urine
  • Coagulates at 50°c, redissolves at 70°c
  • Elevated in Multiple myeloma
 
Antigen Antibody Reaction
49. Ans. (d) (Neutralization) Ref: Ananthanarayan 9/e p112, 8/e p111, 120
  • Naegler reaction is an alfa toxin anti alfa toxin neutralization test
50. Ans. (b) (Agglutination) Ref: Ananthanarayan 9/e p109, 8/e p111, 120
  • Comb's test is an agglutination test done in Rh incompatibility
51. Ans. (a) (Agglutination) Ref: Ananthanarayan 9/e p109, 8/e p111, 120,
  • Paul Bunnel test is an heterophile agglutination test done in infectious mononulceosis.
52. Ans. (a) (Widal test) Ref: Ananthanarayan 9/e p109, 8/e p111, 120,
  • Widal test is an agglutination test
  • VDRL- Slide flocculation test (Precipitation test)
  • Kahn test- Tube flocculation test (Precipitation test)
  • Ascoli's thermo precipitation test- Ring precipitation test done for Anthrax antigen detection
53. Ans. (b) (c3b) Ref: Ananthanarayan 9/e p112, 8/e p111,120, Robbins Basic Pathology 7/e p42-43
  • Opsonins (like Fc of IgG and complement factors like C3b, C4b and ic3b) play a very important role in uptake of bacteria by binding to the specific molecules on the surface of bacteria thus facilitating the engulfment by the phagocyte (which bear the receptors for the opsonins).
54. Ans. (a) (c3b and Fc) Ref: Ananthanarayan 9/e p112, 8/e p111, 120, Robbins Basic Pathology 7/e p42-43
  • Refer text
7355. Ans. (a) (Rheumatoid arthritis) Ref: Ananthanarayan 9/e p110, 8/e p109,175
Rose Waaler test
  • A type of passive agglutination test to detect the RA factor for the diagnosis of Rheumatoid arthritis
  • RA factor- IgM antibody against Fc fragment of IgG.
  • Sheep RBC are coated with sub agglutinating dose of anti erythrocyte antibody (amboceptor) and used as antigen in the agglutination test to detect RA factor.
  • However, currently Latex and bentonite are used as carrier particle for IgG to detect RA (IgM) factor.
  • Detail of Passive agglutination test & RPHA (Reverse Passive Hemagglutination test)- Refer text
Fig. 2.2.4: Opsonization
56. Ans. (b) (Hemagglutination inhibition test) Ref: Ananthanarayan 9/e p112, 113, 117, 8/e p112-115
  • Hemagglutination inhibition test is detected by inhibition of hemagglutination of anti RBC antibody present in patient's serum with respective RBC antigen.
Methods of diagnosis utilizing labeled antibodies
  • Labeled antibodies are used for-
  • Antibody detection- labeled anti human gamma globulin is used to detect human gamma globulin present in patient's serum.
  • Antigen detection- labeled specific antibodies are used to detect corresponding antigens from the sample.
Procedure
Antibody Labeled With
Detection Method
ELISA
Enzyme like Peroxidase
Colorimetric detection of intensity of color product (chromogen) produced by the enzymatic reaction.
Immunfluroscence
Fluorescent dye like fluoroscein or auramine, rhodamine
Fluorescence detected by fluorescence microscope
Chemiluminescence Immunoassay (CLIA)
Chemiluminescent compounds like luminal or acridinium ester
Luminometer detection
Radioimmunoassay
Radio isotope
Scintigraphy by gamma camera
57. Ans. (a) (d) (Widal, ELISA) Ref: Ananthnarayan 9/e p90
Refer chapter review to know the list of heterophile agglutination tests
58. Ans. (d) (Schick test) Ref: Ananthanarayan 9/e p112, 8/e p111,237
  • Neutralization is a reaction in which antigens and antibodies neutralize each other.
  • The reaction is used to identify toxins and antitoxins, as well as viruses and viral antibodies.
  • Examples of Neutralization test – Refer text
59. Ans. (b) (Excess antibody) Ref: Ananthanarayan 9/e p105, 8/e p104
  • Antigen antibody reaction results when a large lattice is formed consist of alternate antigen and antibody molecules which occur in the zone of equivalence.
  • In the zone of antigen or antibody excess, the lattice doesn't enlarge as the valencies of either antibody or antigen respectively are fully satisfied.
    • 74Antibody excess- known as Prozone phenomena
    • Antigen excess- known as Post zone phenomena
60. Ans. (d) (Brucellosis) Ref: Ananthanarayan 9/e p343, 8/e p 345
  • Prozone phenomena or antibody excess occurs in – Enteric fever, Brucellosis, Leptospirosis, Syphilis
  • Post zone phenomena (Antigen excess) occurs in – Cryptococcus
61. Ans. (c) (Antigen & antibody are equal) Ref: Ananthanarayan 9/e p105, 8/e p104
  • Antigen antibody reaction results when a large lattice is formed consist of alternate antigen and antibody molecules which occur in the zone of equivalence.
 
Complement System
62. Ans. (d) (Increased permeability of cell membrane) Ref: Ananthanarayan 9/e p122, 8/e p122
  • In all three complement pathways, the final product is formation of membrane attack complex (C6-9) which form pores on the target cell membrane that leads to increased permeability & lysis of cell.
63. Ans. (c) (IgM) Ref: Ananthanarayan 9/e p122, 8/e p122
  • Only IgM followed by IgG 3>1>2 can fix to classical pathway of complements.
64. Ans. (b) (C3, C6, C9) Ref: Ananthanarayan 9/e p122, 8/e p122
  • Site of Complement synthesis- Liver- C3,C6,C9, GIT- C1, Macrophage- C2,C4, Spleen-C5,C8
65. Ans. (c) (C3bBb) Ref: Ananthanarayan 9/e p122, 8/e p122
  • C3 covertase of classical & lectin pathway is C14b2a and alternate pathway is C3bBb
66. Ans. (b) (C3b) Ref: Ananthanarayan 9/e p122, 8/e p122
  • C3b (also called as C5 convertase) is the central cytokine involved in all three complement pathways which activates C5 following which the lytic stage gets initiated and C6-9 get deposited to form membrane attack complex.
67. Ans. (a) (C3a) Ref: Ananthanarayan 9/e p122, 8/e p122
Role of Byproducts of complements:
  • Chemotaxis and Anaphylaxis- - C5a & C3a
  • Opsonization- C3b
  • Kinin like activity (↑vascular permeability)- C2b
68. Ans. (b) (alternate pathways) Ref: Ananthanarayan 9/e p123, 8/e p122
  • Activator of the alternate pathway:
    • ✓ Zymosan, Endotoxin
    • ✓ IgA, IgD, IgG4
    • ✓ Cobra venom
    • ✓ Nephritic factor
  • Activator of the classical pathway- Antigen antibody complex
  • Activator of the mannose binding pathway- Carbohydrate residue of bacterial cell wall (mannose binding protein)
69. Ans. (b) (C1 esterase inhibitors deficiency) Ref: Ananthanarayan 9/e p122, 8/e p122
  • This a case of hereditary angioneurotic edema
  • Points in favor- Recurrent facial / oropharyngeal / laryngeal edema in a patient with low C4,low C3 and normal factor B.
C1 esterase inhibitors deficiency is known as hereditary angioneurotic edema:
  • Characterized by – episodic edema of the subcutaneous tissue of mucosa of GIT and respiratory tract.
  • Mechanism – C1 esterase inhibitors deficiency leads to autocatalytic activation of C1 which in turn activates C4 and C2 leading to C2 kinins released which is the principle mediator of angioneurotic edema
  • Treatment – free plasma, epsilon aminocaproic acid
7570. Ans. (a) (C2, C4, C3 decreased) Ref: Ananthanarayan 9/e p121, 8/e p119
C2, C3, and C4 all are utilized in Classical pathway, hence their serum level will be low.
Classical pathway:
  • Starts by Activation of C1 by antigen antibody complex
  • Activated C1 in turn activates C4 to C4a (anaphylotoxin) and C4b
  • C1 4b in turn activates C2 (in presence of Mg ion)to form C2a and C2b (kinin like, increases vascular permeability)
  • C14b2a is known as C3 convertase which activates C3 to C3a (chemotactic and anaphylotoxin) and C3b
  • C14b2a3b is known as C5 convertase which activates C5 to C5a (chemotactic and anaphylotoxin) and C5b
  • Late components add to C5b – to form C56789 (membrane attack complex) which forms pores on cell membrane causing cell lysis and also activates bystander cells to make them susceptible to lysis.
71. Ans. (d) (C3) Ref: Ananthanarayan 9/e p122, 8/e p119
  • C3 convertase (C14b2a) acts on C3 to split to C3a and C3b.
  • C3a is chemotactic and anaphylotoxin
  • C3b joins the pathway to form C14b2a3b which is known as C5 convertase

Structure of Immune System and Immune ResponseCHAPTER 2.3

 
LYMPHOID SYSTEM LYMPHOID ORGAN + LYMPHOID CELLS
Lymphoid Organ
  • Central / Primary – Thymus and Bursa of Fibricus (birds) / Bone Marrow (human)
  • Peripheral/Secondary- Spleen, Lymph Node, MALT (GIT and Respiratory mucosa)
Lymphoid Cells: T Cell, B Cells, NK Cell and Dendritic Cell
 
Thymus
  • Developed from 3rd/4th pharyngeal pouch
  • Divided to :
    • Cortex contains T lymphocytes(thymocytes), nurse cell (epithelial cells)
    • Medulla contains epithelial cells, lymphocytes (thymocytes), Hassal's corpuscle (Concentric layers of degenerated epithelial reticular cells)
  • Thymic epithelial cells produce hormones like thymulin, thymopoietin and thymosin.
  • Progenitor T cells passed through thymus during embryonic life
  • 1% of them released out, rest destroyed as they are self reacting T cells (central tolerance)
  • Released T cells are educated Acquire Thy Ag
  • Defect in thymus – leads to low CMI
    • Runt disease affecting nude mice
    • DiGeorge syndrome
T Cell Development
  • Starts from Yolk Sac → fetal liver → Bone marrow→ Thymus
Development of T Cell in Thymus
  • CD7 +ve Pro T cell enter thymus
  • Pro T cell converts to Pre T cell (CD2 and CD3 +ve)
  • Pre T cell converts to Immature T cell – (CD1, 4, 8 and T cell receptor +ve).
  • Immature T cells are also called as double positive T cells.
  • Self reacting T cells are deleted
Fig. 2.3.1: Structure of thymus
77
Fig. 2.3.2: Structure of lymph nod
  • Mature T cells – (loose CD1) – CD4 T cells and CD8 T cells
  • T cell receptor
    • TCR – made up α β or ϒδ
    • TCR α β – most common (95%)
    • TCR ϒδ –present on Intra epithelial lymphocyte and intracellular organism like MTB.
 
Bursa of Fabricus and Bone Marrow
  • Bursa of fabricus – Birds and Bone marrow – humans
  • All lymphocytes originates in BM
  • T cell goes to thymus for maturation
  • B cell proliferates in BM.
 
Lymph Node
  • T dependent area – Paracortical area
  • B dependent area – Cortical follicle & Medulla.
 
Spleen
  • Divided to red pulp and white pulp area
  • White pulp contains – Periarteriolar lymphoid sheath- PALS surrounding central artery of spleen (T cell zone) and marginal zone present next to PALS (B cell area).
 
Lymphocytes
  • Short lived lymphocyte – life span 2 week, Effector cells
  • Long lived lymphocyte – life span 2-3 years, memory cells
  • Markers:
    • B cell markers- CD 10, CD 19, CD21/CR-2(EBV receptor), CD 22, CD 23
    • T cell markers CD1, CD2, CD3, CD4, CD5 (on Tc), CD7, CD8, CD28 (receptor for B7 on APC), CD32 (receptor for Fc of IgG), CD35/CR1 (C3b receptor), CD40
    • NK cell marker- CD 16, CD56
    • Pan T cell marker- CD3
    • Pan lymphocyte marker- CD 45 (present on T,B and NK cell), Memory cell marker- CD45RO.
    • T regulatory cells- CD4 & CD25- suppress immune system
78
Fig. 2.3.3: Structure of spleen
Fig. 2.3.4: Structure of MALT
Fig. 2.3.5: Structure of M-cell
Property
T-cell
B-cell
Origin
Bone marrow
Bone marrow
Maturation
Thymus
Bone marrow
Peripheral lymphocytes
65-85%
15-25%
Antigen recognition receptors
T cell receptors
Membrane-bound Ig
CD markers
CD1,2,3,4,7,8
CD19,10,21,22,23
EAC rosette (receptors for C3), (CR2-/CD21 EBV receptors)
Absent
Present
SRBC rosette (E rosette), CD2(measles receptor)
Present
Absent
Thymus specific antigens
Present
Absent
Surface immunoglobulin (IgM, IgD)
Absent
Present
Receptor for Fc fragment of IgG
Absent
Present
Microvilli on the surface
Absent
Present
Blast transformation with
Phytohemeagglutinin and concovalin
Endotoxin
 
T lymphocyte
CD4 T Helper Cells:
  • Recognizes antigens presented with MHC II molecule
  • 60% of Total lymphocytes
    • TH1 cell – secrete IL2, TNF β and IFN ϒ
      • ✓ IFN ϒ– Activate macrophage, class switch over of B cells to produce IgG2a,IgG3
      • ✓ IL2 – T cell growth factor (activation of Th, Tc & DTH Tcell), NK cell → LAK cell
      • ✓ TNF β
    • TH2 cell – secrete IL4, 5, 6, 10,13
      • ✓ IL4 – Inhibit Th1, Chemo attractant, class switch over of B cells to produce IgE, IgG1
      • ✓ IL5 –Chemo attractant to Eosinophils.
Class Switch Over of B Cells
  • Depending on the cytokine stimulus provided by the Tells, B cells undergo class switch over of B cells to produce different classes of immunoglobulin.
Cytokine Involved
Class Switch Over of B Cells to Produce
IFN ϒ
IgG2a, IgG3
TGF- β
IgA, IgG2b
IL-4
IgE, IgG1
IL-2, IL-4,IL-6
IgM
CD 8 Expressing Cells:
  • Recognizes antigens presented with MHC I molecule
  • 30% of Total lymphocytes
    • T Cytotoxic cells (Tc cells) – causes Cytotoxic lysis of target cells
    • T Supressor cells (Ts cells) – suppresses immune response.
 
NK Cell
  • Natural killer cell / null cell / large granular (LGL)/ lymphokine activated cell
  • NK cell→ LAK transformation is induced by IL2
  • Constitutes 5-10% of total lymphocyte
  • Possess indented nuclei and several granules
  • NK cells are Not MHC restricted
  • Possess CD16 and CD 56 markers
  • Used in treatment of – renal cell Ca
  • Mediates ADCC – Antibody dependent cell mediated cytotoxicity
    • CD16 (FcR like) recognizes Fc portion of IgG antibody bound to a target cell
    • Then, NK cell release perforin and granazyme which lyses the target cells
    • Responsible for Cytotoxicity to virus infected cell and malignant cells, parasitic infection.
 
HLA / MHC
  • Transplantation antigen- determining histocompatibility
  • Play a central role in antigen recognition
  • Coded by-short arm of chromosome 6 in humans
  • Highly polymorphic – 24 alleles – HLA A, 50-HLA B
  • MHC Alleles – co dominantly expressed
  • Class I MHC
    • Present in all nucleated cells
    • Present peptide antigen to Tc cells (CD8 T Cells)
    • Required for-Processing of virus infected cells and tumour cells
    • Function - Graft rejection, Cell mediated cytolysis, Hormone receptor
    • Heterodimer- made up of four globular domains
    • Three α chain globular domain (coded by chr-6) & β2 microglobulin (coded by chromosome-15)
      • ✓ α 1- α 2 Groove : acts as Ag peptide binding groove
      • ✓ α 3 : acts as CD8 Binding groove.
  • Class II MHC
    • Present peptide antigen to Th cells (CD4 T Cells)
    • Regulates immune response
    • Plays a central role in initiation of the immune response to transplantation antigens
    • Function – GVH and MLR
    • Heterodimer- made up of four globular domains- α 1,α2, β1, β2
      • ✓ α 1- β1 Groove : acts as Ag peptide binding groove
      • ✓ β2 : acts as CD4 Binding groove
  • Class III MHC genes –genes coding C2,C4, properdin, factor B, C3 convertase, TNF, HSP.
80
 
Macrophages
Fig. 2.3.6: Structure of MHC-I and MHC-II molecules
  • Derived from the bone marrow
  • Has role in : Phagocytosis , Secretion of cytokines and Antigen presentation
  • Examples of macrophages:
    • Peripheral blood –Monocytes
    • Liver – Kupffer cells,
    • Brain – Microglia,
    • Kidney – Mesangial cells,
    • Bone – Osteoclasts
    • Lung – Alveolar macrophage
    • Connective tissue – Histiocytes
    • Inflammation site – Multinucleated cell and epitheloid cell.
  • Kill cell by:
    • Phagocytosis and phagolysozome fusion and lysosomal degranulation
    • Also by generating oxygen free radical.
 
Dendritic Cell
  • Act as APC, Transport the presented antigen to lymph node
  • Possess long membrane extensions
  • Bone marrow derived (different lineages from common myeloid progenitor)
  • Possess high MHCII, also possess B7
  • Non phagocytic cells
  • Types:
    • Interdigitating DCs- in T cell area
    • Interstitial DCs- in organs
    • Langer Hans cells DCs- in skin
    • Circulating DCs- in blood & lymph.
 
Follicular DC
  • Special type, Not produced in BM
  • Present in lymphoid follicles of LN
  • Doesn't have MHCII, But have FcR for antibodies
  • Helps in maturation and diversification of B cells & memory B cell.
 
Antigen Presentation
Cytosolic Pathway
Endocytic Pathway
For Intracellular Antigen
Virus infected cell / tumor cell
MHCI – CD8 T cell interaction
For Extracellular Antigen Macrophage / dendritic cell / B cell MHCII – CD4 T cell interaction
 
81IMMUNE RESPONSE
82
 
Adjuvant
  • Enhances immunogenicity of Antigen
  • Example:
    • Freund's incomplete Antigen - Antigen coated to water phage of Water in oil emulsion– leads to slow release of Antigen, hence enhances immunogenicity.
    • Freund's complete Ag – Adding M.tuberculosis Muramyl dipeptide antigen
    • Aluminium hydroxide or phosphate
    • LPS of B.pertussis
    • Corynebacterium granulosum.
 
Immunological Tolerance
  • A condition in which contact with an antigen specifically abolishes the capacity to mount an immune response against a particular antigen when given subsequently
  • Central Tolerance
    • Provided by Thymus
    • By removal of self reacting T cell in embryonic life
  • Peripheral Tolerance
    • Cellular anergy
    • Provided by CTLA4
    • It blocks the co-signal (B7-CD28 signal).
 
CMI Detected by
  • Lymphocyte transformation test
  • Migration inhibition test.
 
Transfer Factor
  • It is a T lymphocyte extract
  • Low Molecular weight,
  • Stable, resistant and non antigenic
  • Polypeptide-polynucleotide complex,
  • Enbloc Transfer of CMI to all Antigen to which the donor is sensitive.
Cytokine
Secreted by
Targets and Effects
Some Cytokines of Innate Immunity
Interleukin 1 (IL-1)
Monocytes, macrophages, endothelial cells, epithelial cells
Vasculature (inflammation); hypothalamus (fever); liver (induction of acute phase reactant proteins)83
Cytokine
Secreted by
Targets and Effects
Tumor Necrosis Factor-α (TNF-α)
Macrophages
Vasculature (inflammation); liver (induction of acute phase proteins); loss of muscle, body fat (cachexia); induction of death in many cell types; neutrophil activation
Interleukin 12 (IL-12)
Macrophages, dendritic cells
NK cells; influences adaptive immunity (promotes TH1 subset)
Interleukin 6 (IL-12)
Macrophages, endothelial cells
Liver (induces acute phase proteins): influences adaptive immunity (proliferation and antibody secretion of B cell lineage)
Interferon α (IFN-α) (This id a family of molecules)
Macrophages
Induces an antiviral state in most nucleated cells; increases MHC class I expression; activates NK cells
Interferon β (IFN-β)
Fibroblasts
Induces an antiviral state in most nucleated cells; increase MHC class I expression; activates NK cells
Some Cytokines of Adaptive Immunity
Interleukin 2 (IL-2)
T cells
T-cell & B cells proliferation;
NK cell activation and proliferation
Interleukin 4 (IL-4)
TH2 cells; mast cells
Promotes TH2 differentiation; isotype switch to lgE
Interleukin 5 (IL-5)
TH2 cells
Eosinophil activation
Interleukin 25 (IL-25)
Unknown
Induces secretion of TH2 cytokine profile
Transforming growth factor β (TGF-β)
T cells, macrophages, other cell types
B-cell proliferation; promotes isotype switch to lgE; inhibits macrophages
Interferon γ (IFN-γ)
TH1 cells; CDB* cells; NK cells
Activates macrophages; Increases expression MHC class I and class II molecules; increase antigen presentation
Cytokine/Function
TH1
TH2
Cytokine Secretion
IL-2
+
IFN-γ
++
TNF-β
++
GM-CSF
++
IL-3
++
++
IL-4
++
IL-5
++
IL-10
++
IL-13
++
Functions
Help for total antibody production
+
++
Help for lgE production
++
Help for lgG2a production
++
+
Eosinophil and mast-cell production
++
Macrophage activation
++
Delayed-type hypersensitivity
++
Tc-cell activation
++
84MULTIPLE CHOICE QUESTIONS
 
LYMPHOID ORGAN
1. Lymphoreticular system includes: (PGI June 08)
  1. T-cells
  2. B-cells
  3. Platelets
  4. Macrophages
  5. Neutrophils
2. All are peripheral lymphoid organ except: (PGI May 10)
  1. LN
  2. Spleen
  3. MALT
  4. Thymus
  5. Bone marrow
3. T cell dependent region in lymphnode: (NEET Pattern Based)
  1. Cortex
  2. Medulla
  3. Paracortical area
  4. Mantle layer
4. Neonatal thymectomy leads to: (AI 2002)
  1. Decreased size of germinal center
  2. Decreased size of paracortical areas
  3. Increased antibody production by B cells
  4. Increased bone marrow production of lymphocytes
 
T LYMPHOCYTES
5. The following features are common to T and B-lymphocytes except: (AI 2012)
  1. Site of origin
  2. Site of differentiation
  3. Stimulated by Cytokines
  4. Memory
6. Which of the following features is not shared between ‘T cells’ and ‘B cells’? (AI 2012, AIIMS Nov 2012)
  1. Positive selection during development
  2. Class I MHC expression
  3. Antigen specific receptors
  4. All of the above
7. Cell mediated immunity is by virtue of: (DNB June 2011)
  1. Helper T cell
  2. Suppressor T cells
  3. Cytotoxic T cells
  4. All above
8. Common between B and T cells: (PGI June 2007)
  1. Origin from same cell lineage
  2. Site differentiation
  3. Antigenic marker
  4. Perform Both humoral and cellular immunity
  5. Further differentiation seen
9. Rosette formation sheep RBC is mediated by: (AI 2011, AIIMS Nov 08)
  1. T cells
  2. B cells
  3. NK cells
  4. Macrophages
10. Most potent stimulator of Naive T cells:
  1. Mature dendritic cells (AI 2011, AIIMS Nov 08)
  2. Follicular dendritic cells
  3. Macrophages
  4. B cells
11. All of the following are functions of CD4 helper cells, except: (AI 2009)
  1. Immunogenic memory
  2. Produce immunoglobulins
  3. Activate macrophages
  4. Activate cytotoxic cells
12. Which one of the following is a pan T lymphocyte: (AI 2003)
  1. CD2
  2. CD3
  3. CD19
  4. CD25
13. Memory T cells can be identified by using the following marker: (AI 2003)
  1. CD45 RA
  2. CD45 RB
  3. CD45 RC
  4. CD45 RO
14. Apart from T and B lymphocytes, the other class of lymphocytes is: (PGI 2002)
  1. Macrophages
  2. Astrocytes
  3. NTh cells
  4. Langerhans cells
 
B CELL
15. Which one of the following immunoglobulins constitutes the antigen binding component of B – cell receptor?
  1. IgA (PGI 2005)
  2. IgD
  3. IgM
  4. IgG
 
NK CELL
16. Regarding NTh cells, false statement is: (AI 2001)
  1. It is activated by IL – 2
  2. Expresses CD3 receptor
  3. It is a variant of large lymphocyte
  4. There is antibody induced proliferation of NTh cells
17. 85All are true about NK cells except: (PGI Nov 10)
  1. Possess CD 16
  2. Possess CD 56
  3. Important role in virus infected cell
  4. Produces complement factors
18. Perforins are produced by: (PGI 2001)
  1. Cytotoxic T cells
  2. Suppressor T cells
  3. Memory helper T cells
  4. Plasma cells
  5. NK cells
 
MACROPHAGE
19. Macrophages are major source of: (DNB June 2010)
  1. IL-1
  2. IL-5
  3. IL-7
  4. IFN-Y
20. All of these are antigen presenting cells (APC's) except:
  1. T cells (PGI 2002)
  2. B cells
  3. Fibroblasts
  4. Dendritic cells
  5. Langerhans cells
 
MHC COMPLEX
21. The role played by MHC 1 and 2 is to: (AIIMS Nov 2013, AIIMS May 2014, AIIMS Nov 2014)
  1. Stimulate interleukin production
  2. Immunoglobulin class switch over
  3. Transduce the signal to T cell following antigen binding
  4. Presenting the antigen for recognition byT cell antigen binding receptors
22. MHC molecules are coded by: (NEET Pattern Based)
  1. Chromosome-6
  2. Chromosome-7
  3. Chromosome-8
  4. Chromosome -9
23. Gene coding for MHC-I include: (NEET Pattern Based)
  1. A,B,C
  2. DR
  3. DP
  4. DQ
24. Peptide binding site on MHC-I for presenting processed antigen to CD8 T cells is formed by: (AI 2010)
  1. Junction of Proximal domains made up of Alpha subunit
  2. Junction of Distal domains made up of Alpha subunit
  3. Junction of Proximal domains made up of beta subunit
  4. Junction of Distal Proximal domains made up of beta subunit
25. MHC II are presented on :(PGI Nov 10)
  1. Macrophages
  2. Dendritic cells
  3. Lymphocytes
  4. Eosinophils
  5. Platelet
26. Cell type which lacks HLA antigen is: (AIIMS May 2005)
  1. Monocyte
  2. Thrombocyte
  3. Neutrophil
  4. Red blood cell
27. MHC class III genes encode: (AI 2003)
  1. Complement component C3
  2. Tumor necrosis factor
  3. Interleukin 2
  4. Beta 2 macroglobulin
28. T helper cell recognizes: (PGI 2002)
  1. MHC class I
  2. MHC class II
  3. Processed peptides
  4. Surface Ig
29. MHC class II antigens are located on which of the following cells: (PGI 2000)
  1. Platelets
  2. Dendritic cells
  3. RBC
  4. T Cells
  5. B Cells
30. MHC III codes for: (Recent MCQ 2013)
  1. Complements
  2. Interleukin
  3. Prostaglandins
  4. Interferon
 
CYTOKINES
31. Which of the following is not pyrogenic in nature?
  1. IL 1 (AI 2012)
  2. IL 18
  3. TNF alpha
  4. INTERFERON alpha
32. Which of the following chemical mediators of inflammation is an example of a C-X-C or alpha chemokine? (AIIMS 2003)
  1. Lipoxin LXA – 4
  2. Interleukin IL – 8
  3. Interleukin IL – 6
  4. Monocyte chemo attractant protein MCP -1
33. IL – I produces: (AI 2002)
  1. T lymphocyte activation
  2. Delayed wound healing
  3. Increased pain perception
  4. Decreased PMN release from bone marrow
34. Interleukin–1 (IL–1): (AIIMS 2001, DNB 2002)
  1. Is the product of both lymphocytes and macrophages
  2. Is the sole signal required for proliferation of T-lymphocytes
  3. Is an example of an antigen –specific factor
  4. Induces the maturation of IL – 2 producing lymphocytes
35. IL- 1 is produced by: (PGI 2000)
  1. Macrophage
  2. Helper T lymphocytes
  3. B cells
  4. Cytotoxic T-cells
36. 86IL – 2 produced by: (AI 2000)
  1. T cells
  2. B cells
  3. Monocytes
  4. Neutrophils
37. Most important chemical mediators of gram negative septicaemia: (Recent MCQ 2013)
  1. IL-2B
  2. TNF-α
  3. TGB-β
  4. IL-20
 
INTERFERON
38. True about interferon: (NEET Pattern Based)
  1. Viral protein
  2. Virus specific
  3. Host protein
  4. Inactivated by nucleases
39. True about interferon: (PGI June 2005)
  1. It is virus specific
  2. It is bacteria specific
  3. Produced from Bacteria
  4. Effective against viral infection
  5. It is species specific
40. Interferon gamma secreted by :(DNB Dec 2009)
  1. Activated T-cell
  2. CD 8 cells
  3. RBC
  4. Neutrophils
 
ADJUVANT
41. The main aim of an adjuvant is to increase: (NEET Pattern Based)
  1. Distribution
  2. Absorption
  3. Antigenicity
  4. Metabolism
 
TOLERANCE
42. Which one of the following statements best describes immunological tolerance? (AIIMS 2009)
  1. Immunologic maturity of the host does not play a major role
  2. It occurs only with polysaccharide antigens
  3. It is related to the concentration of antibody
  4. It is prolonged by administration of immunosuppressive drugs.
87EXPLANATIONS
1. Ans. (a), (b), (d), (e) (T-cells, B-cells, Macrophages, Neutrophils) Ref: Ananthanarayan 9/e p128, 8/e p123, 132, Kuby's Immunology 6/e p29
Lymphoreticular System- Consists of:
  • Lymphoid System – Lymphoid Organ + Lymphoid Cells (T-cells, B-cells) (Detail-Refer text)
  • Reticular endothelial system- Macrophage and Microphage (Neutrophils, Eosinophils)
2. Ans. (d), (e) (Thymus, Bone marrow) Ref: Ananthanarayan 9/e p128, 8/e p123, 132, Kuby's Immunology 6/e p295
Lymphoid Organ:
  • Central / Primary – Thymus and Bursa of fabricus (birds) / Bone Marrow (human)
  • Peripheral/Secondary- Spleen, Lymph Node, MALT (GIT and Respiratory mucosa)
3. Ans. (c) (Paracortical area) Ref: Kuby's Immunology 6/e p45-46
Lymph Node
Spleen
• T cell area- Paracortical area
• B cell area- Cortex and medulla
• T cell area- Periarteolar lymphoid sheath
• B cell area- marginal zone
4. Ans. (b) (Decreased size of paracortical areas) Ref: Ananthanarayan 9/e p130, 8/e p125, 126, Kuby's Immunology 6/e p 45,47
  • “Thymus is the site of T cell maturation, so neonatal thymectomy leads to decrease production of mature T cells hence all the Thymus dependent area of lymphoid organs will undergo degeneration.”
 
Cells of Immune System- T cell
5. (b) (Site of differentiation) Ref: Ananthanarayan 9/e p130,135, 8/e p128
  • Site of differentiation- T cells differentiate in thymus where as B cells in bone marrow.
  • Site of origin – Both T & B cells originate from a common lymphoid stem cell in bone marrow.
  • Stimulated by Cytokines- IL2 stimulates T cells whereas IL4, 5, 6 stimulates B cells.
  • Memory- T & B cell can transform to memory cells which helps in secondary immune response.
6. Ans. (a) (Positive selection during development) Ref: Ananthanarayan 9/e p130, 8/e p128
  • During embryonic life, for deletion of self reacting clones, T cells undergo positive selection followed by negative selection where as B cells undergo only negative selection.
About Other Options
  • Class I MHC molecules are expressed on all nucleated cells. Both T & B cells being nucleated, so they express class I MHC. Whereas class II MHC are expressed on all APCs (so present on B cells but not on T cells).
  • Antigen specific receptor:
    • ✓ Antigen specific receptor on T cell- TCR (T cell receptor)
    • ✓ Antigen specific receptor in B cell- BCR (B cell receptor) i.e. surface immunoglobulins like IgM and IgD
887. Ans. (d) (All of the above) Ref: Ananthanarayan 9/e p152
  • Cell mediated immunity refers to specific immune response mediated by effector T cells (both T-helper, Cytotoxic T cells and Suppressor T cells) generated against an antigen.
8. Ans. (a), (e) (Origin from same cell lineage, Further differentiation seen) Ref: Ananthanarayan 9/e p133-36, 8/e p130, Kuby's Immunology 6/e p 29, 34
  • Both T and B cells originate from same lymphoid progenitor cell lineage in bone marrow.
  • Site of differentiation –T cell- in Thymu s and B cell- In bone marro w
  • Antigenic marker are also different for T & B cells (Refer text)
  • Immune response- T cell mediates- cellular immunity and B cell mediates- humoral immunity
  • Further differentiation:
    • ✓ T cell differentiated to TH (TH1 and TH2) and Tc cells
    • ✓ B cell differentiated to plasma cells
9. Ans. (a) (T cells) Ref: Ananthanarayan 9/e p133
  • Sheep RBC Rosette- T cells have CD2 receptors on their surface that bind to sheep erythrocytes to form rosettes where as B cells don't.
  • EAC Rosette- B cells can bind to sheep RBC coated with antibody & complement to form EAC Rosette due to presence of C3 receptor (CR2) on their surface.
10. Ans. (a) (Mature dendritic cells) Ref: Venu Gopal Jayapal's Immunology 1/e p28
  • Mature dendtritic cells possess higher level of MHC-II & costimulatory B-7 molecules, hence they are the most potent stimulator of T cells.
  • Detail-Refer text
11. Ans. (b) (Produce immunoglobulins) Ref: Venu Gopal Jayapal's Immunology 1/e p110
  • When resting Th cell binds to a MHC classII- antigen complex on APC, that initiates the activation of Th cell.
  • The activated Th cell divides many times to produce effector Th cell and memory Th cell.
  • Effector Th cell further divides to Th1and2 and perform the following actions-
    • ✓ Activation and proliferation of Tc cell
    • ✓ Regulate monocyte macrophage system
    • ✓ Helps in activation of B cell to produce plasma cell that secretes immunoglobulins, (but never produce immunoglobulins)
12. Ans. (b) (CD3) Ref: Venu Gopal Jayapal's Immunology 1/e p110
  • CD3 is a Pan T cell marker
13. Ans. (d) (CD45 RO) Ref: Venu Gopal Jayapal's Immunology 1/e p114
  • Virgin T cells express 220 kDa CD45RA
  • Memory T cells express 180 kDa CD45RO
14. Ans. (c), (NK cell) Ref: Ananthanarayan 9/e p137, 8/e p130, Kuby's Immunology 6/e p 29
  • About 5-10% of T cells lack the markers and features of both T and B cells and are known as Natural killer cells.
  • Lymphoid cells include- T cell, B cell, NK cell, follicular DC
 
B cell
15. Ans. (b),(c), (IgD, IgM) Ref: Ananthanarayan 9/e p136, 8/e p100
  • IgD, IgM occur on surface of unstimulated B lymphocytes and serves as recognition receptors for antigens.
 
NK cell
16. Ans. (b), (Expresses CD3 receptor) Ref: Ananthanarayan 9/e p137, 8/e p131, Kuby's Immunology 6/e p35
  • NK cell Expresses CD16, CD56 receptors
  • CD3 receptor is a pan T cell marker
17. Ans. (d), (Produces complement factors) Ref: Ananthanarayan 9/e p137, 8/e p131
  • NK cell Expresses CD16, CD56 receptors
  • Complement factors are produced by liver & macrophages.
8918. Ans. (a), (e) (Cytotoxic T cells, NK cells) Ref: Ananthanarayan 9/e p136,137, 8/e p131, Kuby's Immunology 6/e p356, 358
Both NK cell and Tc share in their mechanism of killing the target cell-
  • NK cell mediated cell death occurs due to ADCC
  • Antibody dependent cytotoxicity
    • ✓ NK cellpossess Fc receptor that recognizes Fc portion of antibody bound to a target cell (like virus infected cell and malignant cells, parasites)
    • ✓ Then, they release perforin and granazyme
    • ✓ Perforin makes pores and granazyme lyses the target cells
  • TCR of Tc cell also recognizes the antigen-MHC I complex of target cell. Then it forms pore in target cell membrane by perforin and release the granazyme inside the target cell.
 
Macrophage
19. Ans. (a) (IL-1) Ref: Kuby's Immunology 6/e p209
Macrophage produces various secretory molecules:
  • Enzymes- Lysozyme, proteases, elastases, collagenases, plasminogen activator,
  • Cytokines-IL-1, 8, 12, TNF-α, TGF-β, prostaglandins
  • Platelet derived growth factor (PDGF), Platelet activated factor (PAF),
  • CSF (colony stimulating factor), ACE(Angiotensin Converting Enzyme)
  • Complements- C2, C4
  • Reactive O2 species- H2O2, NO, OH
20. Ans. (a) (T cell) Ref: Kuby's Immunology 6/e p209
  • T cell is not an antigen presenting cell, rather, antigen after processing by an antigen presenting cell is presented to a T cell.
  • Whereas B cell, dendritic cell, fibroblast and langerhans cell (a type of dendritic cell) are the examples of Antigen presenting cells.
Antigen presenting cells (APC's) includes
Professional Antigen Presenting cells
Nonprofessional Antigen Presenting cells
  • Dendritic cells (several types)
  • Macrophage
  • B cells
  • Glial cells (brain)
  • Fibroblasts (skin)
  • Pancreatic beta cell
  • Thymic epithelial cells
  • Vascular endothelial cells
  • Thyroid epithelial cells
 
MHC Complex
21. Ans. (d) (Presenting the antigen for recognition by T cell antigen binding receptors) Ref: Ananthanarayan 9/e p140
The major function of MHC I and II molecule is to present the peptide antigen to the T cell receptors of T cells.
  • MHC I molecule presents the peptide antigen to T helper cells.
  • MHC II molecule presents the peptide antigen to Tc cells.
22. Ans. (a) (Chromosome-6) Ref: Ananthanarayan 9/e p139
  • MHC genes are located on short arm of Chromosome-6
23. Ans. (a) (A, B, C) Ref: Ananthanarayan 9/e p139
  • Class I MHC gene includes- A,B,C
  • Class II MHC gene includes – DP, DQ, DR
  • Class III MHC gene includes- genes for certain complement factors, Heat shock protein, TNF
90
24. Ans. (b) (Junction of Distal domains made up of Alpha subunit) Ref: Kuby's Immunology 6/e p194
Binding sites of MHC I & II molecules:
  • The antigen binding site-
    • ✓ For MHCI- groove between α1 and α2
    • ✓ For MHCII- groove between α1 and β1
    • ✓ Bothe these grooves lie in the distal part of MHC molecules
  • The co-receptor binding site
    • ✓ For MHCI- α3 domain binds to CD8 of Tc cell
    • ✓ For MHCII- β2 domain binds to CD4 of Th cell
    • ✓ Bothe these grooves lie in the proximal part of MHC molecules
25. Ans. (a) (b) (Macrophages, Dendritic cells) Ref: Ananthanarayan 9/e p142, 8/e p134
  • MHC I molecules are present on all nucleated cells- (RBC and platelets are non nucleated, so MHC I molecules are not found)
  • MHC II molecules are more restricted in distribution, present on macrophages, dendritic cells and B cells.
26. Ans. (d) (Red blood cell) Ref: Ananthanarayan 9/e p139, 8/e p134
  • Refer the previous explanation
27. Ans. (b) (Tumor necrosis factor) Ref: Ananthanarayan 9/e p140, 8/e p133
Class III MHC encodes for- C2, C4, Properdin, Factor B, C3 convertase, Tumor necrosis factor (TNF α and β) HSP (heat shock protein)
28. Ans. (b), (c) (MHC class II, Processed peptides) Ref: Kuby's Immunology 6/e p194
  • T helper cell recognizes processed antigen peptides presented along with MHC II
  • T Cytotoxic cell recognizes processed antigen peptides presented along with MHC I
29. Ans. (b), (e) (Dendritic cells, B cell) Ref: Ananthanarayan9/e p140, 8/e p134
MHC II molecules are more restricted in distribution, present on macrophages, dendritic cells, activated Tcell, B cells.
30. Ans. (a) (Complements) Ref: Ananthanarayan 9/e p140
  • MHC III codes for- Complements C2, C4, C3 convertase, factor B, properdin, HSP and TNF.
 
Cytokines
31. Ans. (b) (IL18) Ref: Ananthanarayan 9/e p154, 8/e p93-94, Kuby's Immunology 6/e p279,
Journal- Circulating Cytokines as Mediators of Fever, Mihai G. Netea et al, Clin Infect Dis. (2000) 31 (Supplement 5): S178-S184.
  • Each of the cytokines IL-1, TNF-α and IL-6 acts on the hypothalamus to induce a fever response ……. Kuby's Immunology 6/e p279
  • Bacterial LPS stimulates macrophages and endothelial cells to release TNF and IL1 which in turn activates IL-6. Finally Prostaglandin E2 (PGE2) is stimulated which is the main mediator of fever.
  • PGE2 is the ultimate mediator of the febrile response. PGE2 acts on neurons in the preoptic area (POA) through the prostaglandin E receptor 3 (EP3).
  • EP3-expressing neurons in the POA innervate the hypothalamus. Fever signals releases from hypothalamus stimulates the sympathetic output system, which evokes non-shivering thermogenesis to produce body heat and skin vasoconstriction to decrease heat loss from the body surface.
91
Cytokine
EP Activity
Comment
IL-1
+++
The most potent EP in humans; both IL-1α and IL-1β are EP (endogenous pyrogens)
TNF-α
++
Possible role for both soluble and membrane-bound form.
IL-6
++
IL-6 acts distally of TNF and IL-1 in the cytokine cascade
IFN
+/++
IFN-α is the potent; IFN-β less pyrogenic; IFN-γ may be pyrogenic through induction of IL-1 and TNF
32. Ans. (b) (Interleukin IL – 8) Ref: Harrison 17/e p2030, Wikipedia
The major role of chemokines is to act as a chemo attractant to guide the migration of cells. Members of the chemokine family are divided into four groups depending on the spacing of their first two cysteine residues.
33. Ans. (a) (T lymphocyte activation) Ref: Kuby's Immunology 6/e p307, Venu Gopal Jayapal's Immunology 1/e p125
Cytokine
Secreted by
Targets and Effects
Interleukin 1 (IL-1)
Monocytes,
Macrophages,
Endothelial cells,
Epithelial cells,
Dendritic cell
Vasculature (inflammation);
Hypothalamus (fever);
Liver (induction of acute phase proteins)
Th cell- activation to produce Il2
B cell- maturation and proliferation
Chemo-attractant
34. Ans. (d) > (a) (Induces the maturation of IL–2 producing lymphocytes, is the product of both lymphocytes and macrophages) Ref: Kuby's Immunology 6/e p307, Venu Gopal Jayapal's Immunology 1/e p125 Already explained
35. Ans. (a), (c) (Macrophage, B cell) Ref: Kuby's Immunology 6/e p307, Venu Gopal Jayapal's Immunology 1/e p125 Already explained
36. Ans. (a) (T cell) Ref: Kuby's Immunology 6/e p307, Venu Gopal Jayapal's Immunology 1/e p125
Cytokine
Secreted by
Targets and Effects
Interleukin 2 (IL-2)
Th1 cell
Proliferation of Th and Tc cells
NK cell transformation to LAK
T cell growth factor,
Activate DTH Tcell
B cell class switch over to IgG2
Cytokine
Secreted by
Targets and Effects
IL 3
Th cell
Hematopoietic growth factor- multi CSF
IL4
Th2 cell
Inhibit Th1,Class switch to IgE, IgG
IL5
Th2 cell
Enhances growth of Eosinophilsclass switch to IgA
IL6,10,13
Th2 cell
Same as IL4
IL8
Macrophage
Neutrophil chemo attractant
IL12
Macrophage
Promotes Th1 induction and Inhibit Th2
9237. Ans. (b) (TNFα) Ref: Stite'sMedical Immunology, 10/e p152, Kub's Immunology 6/e p192
  • Bacterial septic shock apparently develops because bacterial cell-wall endotoxins stimulate macrophages to overproduce IL-1 and TNF-α to higher levels that cause septic shock
  • TNFα is the principle cytokines involved in gram negative septic shock.
  • TNFα is primarily responsible for Shwartzman reaction, in which repeated injection of LPS (obtained from gram negative cell wall) into a solid tissue leads to hemorrhagic infarction.
 
Interferon
38. Ans. (c) (Host protein) Ref: Ananthanarayan 9/e p449, 8/e p447
Interferon:
  • Host coded proteins produced by cells on induction by viral or non-viral inducers.
  • Not virus specific. Interferon induced by one virus can confer protection against infection by same or unrelated virus.
  • They are species specific- Interferon produced by one species can protect only cells of same or related species against viral infections.
  • Has no direct action on viruses but acts on other cells of same species rendering them refractory to viral infections.
  • Mechanism- on exposure to Interferon, cells produce translation inhibiting protein (TIP) which selectively inhibits translation of viral mRNA without affecting cellular mRNA.s
  • RNA viruses are better inducers than DNA viruses.
  • Potent Viral Inducers- Togavirus, vesicular stomatitis virus, Sendia virus, NDV
  • Potent Non viral Inducers- intracellular organism like Mycobacteria, Mitogens (endotoxin), synthetic polymers
  • Types of Interferon:
Interferon type
Produced by
Inducer
Function
Interferon α (Nonglycosylated protein)
Leukocyte
Virus
Antiviral activity
Interferon β (Glycosylated protein)
Fibroblast
Virus
Antiviral activity
Interferon γ (Glycosylated protein)
T lymphocyte
Antigen Mitogen
Immunomodulator Anti-proliferator
39. Ans. (d) (e) (Effective against viral infection, It is species specific) Ref: Ananthanarayan 9/e p449, 8/e p447
  • Refer the previous explanation
40. Ans. (a) (Activated T- cell) Ref: Ananthanarayan 9/e p449, 8/e p447 Interferons are:
  • Inactivated by proteolytic enzymes but not by nucleases & lipases.
  • Resistant to pH (except IFN-γ), Heat stable
  • M.wt-17,000. Non dialyzable and non sedimentable.
  • Poorly antigenic, so not detected by serological tests, but by biological assays like ability to inhibit plaque formation by a sensitive virus.
41. Ans. (c) (Antigenicity) Ref: Ananthanarayan 9/e p150, 8/e p143 Adjuvant- Any substance that enhances immunogenicity of Antigen
42. Ans. (d) (It is prolonged by administration of immunosuppressive drugs) Ref: Ananthanarayan 9/e p157, 8/e p151
  • Specific immunological unresponsiveness to self antigens is called tolerance.
    • ✓ Tolerance is more common in immature immune system (Infants and neonates)- Immunologic maturity of the host play a major role.
    • ✓ Tolerance occurs with soluble antigens and haptens than particulate antigens like polysaccharide antigens.
    • ✓ Tolerance is related to the concentration of antigen
    • ✓ Tolerance is prolonged by administration of immunosuppressive drugs.

Hypersensitivity ReactionCHAPTER 2.4

  • Hypersensitivity is defined as injurious consequences in the sensitized host, following contact with specific antigen.
Character
Immediate
Delayed
Type
I, II, III
IV
Time
Minutes to hours
Days
Mediator
Antibodies
T cells
Route
Any route
Intradermal
Passive transfer
With serum
With transfer factor
Desensitization
Easy, but short lived
Difficult, but long lasting
Fig. 2.4.1: Various types of hypersensitive reactions
 
TYPE I HYPERSENSITIVITY REACTION
Mechanism
Sensitization phase
Priming dose of antigen (allergen)→ processed by APC → antigenic peptide presented to T cell → Th 2 → secretes IL4 → Acts on B cell → IgE produced → Mast cell coated with Ig E (Fc)
Effector phase
Shocking dose → IgE (Fab) binds to Antigen → Mast cell degranulation
 
94Mediators of TYPE I Hypersensitivity Reaction
Primary Mediators (Preformed in Mast cell)
Secondary Mediators (Newly Formed upon Stimulation)
• Histamine, heparin
• Serotonin
• Eosinophil chemotactic factor (ECF-A)
• Neutrophil chemotactic factor (NCF-A)
• Proteases
• Platelet-activating factor
• Leukotrienes
• Slow reactive substance of anaphylaxis, (SRS-A)
• Prostaglandins
• Bradykinin
• Cytokines-Interleukin and TNF, TGF, and GM-CSF
• Anaphylotoxin released from complement activation
 
Example of Type 1 HSN
  • Atopy
    • Asthma, Allerigic rhinitis (hay fever)
    • Food allergy, Atopic eczema.
  • Anaphylaxis
  • Theobald smith phenomena
  • Schultz Dale phenomena
  • Prausnitz Kustner reaction
  • Casoni test (hydatid disease)
  • Tropical Pulmonary Eosinophilia (TPE)
  • Loeffler's pneumonia (Ascaris)
  • Ground itch (Hook worm)
  • Leakage of hydatid fluid
  • Cercarial dermatitis/swimmer's itch (Schistosoma)
  • Wheal and flare reaction.
 
TYPE II HYPERSENSITIVITY REACTION
  • Compliment dependent cytolysis
    • Transfusion reaction,
    • Erythroblastosis fetalis,
    • Auto Immune hemolytic anemia
    • Pernicious anemia.
  • Compliment dependent inflammation (C3a, C5a)
    • Good Pasteur syndrome
    • Pemphigus vulgaris
    • Bullous pemphigus
    • Rheumatic fever
    • Vasculitis.
  • Compliment dependent phagocytosis
  • ADCC – (Type VI)- By NK cell mediated , Useful for parasite removal, graft rejection
  • Antibody dependent cellular dysfunction – (Type V)
    • Stimulation – Grave's disease (LATS)
    • Inhibition – Myasthenia gravis (Anti Acetyl choline receptor Antibody).
  • Other examples:
    • Anemia in malaria
    • Black water fever in malaria following quinine therapy
    • Myocarditis in Chagas’ disease.
 
TYPE III HYPERSENSITIVITY REACTION
  • Immune complex mediated HSN reaction- Immune complex gets deposited in tissues
  • Usually complement gets bound to complex – so serum level of complement falls
  • Neutrophils are attracted and release enzymes
  • 2 older forms- Arthus reaction- local reaction and Serum sickness- systemic reaction.
 
Example of Type III HSN
  • Hypersensitivity Pneumonitis (farmer's lung)
  • 95Post streptococcal glomerulonephritis
  • Sub acute bacterial endocarditis
  • Connective tissue disorder- SLE, Rheumatoid Arthritis, PAN (Polyarteritis nodosa)
  • Hyper acute graft rejection
  • Lepra reaction type 2 (ENL- Erythematous Nodosum Leprosum)
  • Nephrotic syndrome in Plasmodium malariae
  • Katayama fever in schistosomiasis
  • African trypanosomiasis, Schick test
  • Onchocerciasis
  • Henoch Schonlein Purpura
  • Microbial antigen forms immunocomplexes:
    • Streptococcus pyogenes,
    • M. leprae,
    • Treponema,
    • Plasmodium,
    • Trypanosoma,
    • HBV, HCV, EBV, Dengue.
 
TYPE IV / DELAYED HYPERSENSITIVITY REACTION
Mechanism of delayed-type (type IV) hypersensitivity
In the sensitization phase
  • After initial contact with antigen (e.g., peptides derived from intracellular bacteria), T helper cells proliferate and differentiate into TH1 cells which secretes cytokines
In the effector phase
  • After subsequent exposure of sensitized TH1 cells to antigen, the TH1 cells secrete a variety of cytokines and chemokines.
Fig. 2.4.2: Type IV hypersensitivity reaction
  • 96These factors attract and activate macrophages and other nonspecific inflammatory cells.
  • Activated macrophages are more effective in presenting antigen, thus perpetuating the DTH response, and function as the primary effector cells in this reaction.
  • Mediators
    • CD4 Th1 cell secretions- IL2, IFNγ and TNF-β
    • Chemokines- IL-8, Monocyte chemotactic factor (MCAF), Migration inhibitory factor (MIF).
  • Example
    • Tuberculin test-Mantoux/Heaf,
    • Lepromin test
    • Lepra reaction type I
    • Contact dermatitis- Nickel, poison ivy, poison oak, picryl chloride exposure
    • Cutaneous basophil hypersensitivity (Jones Mote reaction)
    • Frie test – done in LGV
    • Diabetes mellitus type 1
    • Multiple sclerosis
    • Peripheral neuropathies
    • Coeliac disease and Crohn's disease
    • Chronic transplant rejection
    • Graft-versus-host disease
    • Hashimoto's thyroiditis
    • Montenegro test (Leishmania)
    • Elephantiasis (in filariasis)
    • Granulomatous disease in-tuberculosis, sarcoidosis, schistosomiasis etc.
97MULTIPLE CHOICE QUESTIONS
 
TYPES I HYPERSENSITIVITY
1. Type I hypersensitivity is mediated by which of the following immunoglobulins? (NEET Pattern Based)
  1. IgA
  2. IgG
  3. IgM
  4. IgE
2. Wheal and flare is which type of hypersensitivity reaction? (NEET Pattern Based, DNB 2010)
  1. Type I
  2. Type II
  3. Type IV
  4. Type V
3. Injection of ovalbumin in rabbit induces production of which of the following antibody? (AI 2011)
  1. IgE
  2. IgA
  3. IgD
  4. IgM
4. Spring catarrh is which type of hypersensitivity reaction: (PGI June 2005)
  1. Type I
  2. Type II
  3. Type II and III
  4. Type IV
  5. Type V
5. Mast ct cell synthesize and / or secrete: (PGI June 2003)
  1. Adrenaline
  2. Ach
  3. Histamine
  4. Heparin
  5. Neutrophilic chemotactic factor
6. True about anaphylaxis: (PGI Dec 2000)
  1. Type – I reaction
  2. Large amount of histamine released
  3. Cytokines like IL, IL and GMCSF are released
  4. Mediated through allergen specific IgE
7. Type 1 hypersensitivity includes all of the following except: (PGI 2000)
  1. Autoimmune hemolytic anemia
  2. Anaphylaxis
  3. Extrinsic asthma
  4. Hay fever
8. Anaphylaxis is mediated by: (PGI May 2013)
  1. 5-hydroxytryptamine
  2. Heparin
  3. Prostaglandin
  4. Anaphylotoxins from complement activation
  5. Platelet activating factor
9. Allergic rhinitis is an example of: (Recent MCQ 2013)
  1. Type I hypersensitivity reaction
  2. Type II hypersensitivity reaction
  3. Type III hypersensitivity reaction
  4. Type IV hypersensitivity reaction
 
TYPES II HYPERSENSITIVITY
10. The type of Hypersensitivity reaction in Myasthenia gravis (AI 2012)
  1. Type I
  2. Type II
  3. Type III
  4. Type IV
11. Type 2 hypersensitivity reaction seen in: (Recent Question 2013)
  1. Myasthenia gravis
  2. Good pasture syndrome
  3. Sarcoidosis
  4. Grave's disease
  5. Hyper acute graft rejection
12. 2 days after penicillin administration, antibodies developed that resulted in hemolysis. Which type of hypersensitivity reaction is this? (AIIMS Nov 2014)
  1. Type-I
  2. Type-II
  3. Type-III
  4. Type-IV
13. Graves disease is an example of which type immunologic response: (PGI 2001)
  1. Type I
  2. Type II
  3. Type III
  4. Type IV
  5. Type V
 
TYPES III HYPERSENSITIVITY
14. The hypersensitivity reaction involved in the hyper acute rejection of renal transplant is: (AIIMS May 2005)
  1. Type I
  2. Type II
  3. Type III
  4. Type IV
15. Skin transplant was done from sister to brother. After few years, brother to sister skin transplant was done, but rejection occurred. What is this phenomena called as? (Recent MCQ 2013)
  1. Schwartzman reaction
  2. Theobald smith phenomena
  3. Eichwald Silmser effect
  4. Schultz Dale phenomena
 
TYPES IV HYPERSENSITIVITY
16. Which of the following is type IV hypersensitivity reaction? (NEET Pattern Based; PGI June 2005)
  1. Arthus reaction
  2. Serum sickness
  3. Schwartzman reaction
  4. Granulomatous disease
17. Skin tests are used for which hypersensitivity reactions? (PGI June 2007)
  1. I
  2. II
  3. II
  4. IV
  5. V
98EXPLANATIONS
 
Types I Hypersensitivity
1. Ans. (d) (IgE) Ref: Ananthanarayan 9/e p162, 8/e p163
  • Type 1 hypersensitivity reaction is dependent on IgE antibody (also known as Reagin antibody or cytotropic antibody).
2. Ans. (a) (Type I) Ref: Ananthanarayan 9/e p164, 8/e p163, Mosby's Medical Dictionary, 8th edition
  • Wheal-and-flare reaction is an example of immediate hypersensitivity reaction
Wheal-and-flare reaction:
  • Skin eruption that may follow injury or injection of an antigen. It is characterized by swelling and redness caused by a release of histamine.
  • The reaction usually occurs in three stages, beginning with the appearance of an erythematous area at the site of injury, followed by development of a flare surrounding the site; finally a wheal forms at the site as fluid leaks under the skin from surrounding capillaries.
3. Ans. (a) (IgE) Ref: journal:Int Arch Allergy Immunol. 1996 Mar;109(3):223-8.
  • Injection of ovalbumin in rabbit induces an immediate Type 1 hypersensitivity reaction which leads to production of IgE antibody.
  • OVA or hen's egg alum immunized mice produce anti-OVA antibodies predominantly of the IgG1 and IgE isotypes.
  • However, Ovalbumin-liposome conjugate induces IgG but not IgE antibody production.
4. Ans. (a) (Type I)
Ref: Journal : The Direction of Specific Efforts with Allergic Conjunctival Diseases in Japan- JMAJ 52(3): 173–177, 2009
Allergic Conjunctivitis
  • Phlyctenular kerato-conjunctivitis (type 4 hypersensitivity).
  • Spring catarrhal (type 1hypersensitivity).
Spring Catarrhal (Vernal Conjunctivitis)
  • It is a bilateral, seasonally recurrent, chronic conjunctivitis due to type I hypersensitivity allergic reaction to an external antigen.
  • The antigen reacts with IgE that will stimulate mast cell leading to release of histamine and other mediators of the allergic response.
  • The antigen is usually unknown, but the ultraviolet rays, heat and dust are contributing factors.
5. Ans. (c), (d), (e) (Histamine, Heparin, Neutrophil chemotactic factor)
Ref: Ananthanarayan 9/e p164, 8/e p165, Kuby's Immunology 6/e p380
  • IgE Sensitized Mast cell, on contact with specific allergen undergo degranulation to release primary mediators like heparin, histamine and Neutrophilic chemotactic factor (NCF-A)
  • Epinephrine (Adrenalin) is the drug of choice for systemic anaphylactic reactions.
  • For detail- Refer text.
6. Ans. (a), (b), (c), (d) (Type – I reaction, Large amount of histamine released, Cytokines like IL 4,IL 5 and GMCSF are released, Mediated through allergen specific Ig E)
Ref: Ananthanarayan 9/e p164, 8/e p164, Kuby's Immunology 6/e p381
Type I Hypersensitivity Reactions can be Systemic or Localized
  • Systemic anaphylaxis
    • ✓ Shock-like and often fatal, occurs within minutes
    • IgE mediated
    • ✓ Mediators released- both primary and secondary (like IL4,IL5 and GMCSF)
  • Localized anaphylaxis (atopy)
    • ✓ Allergic rhinitis (hay fever)
    • 99✓ Asthma
    • ✓ Atopic dermatitis (eczema)
    • ✓ Food allergies.
7. Ans. (a) (Autoimmune hemolytic anemia) Ref: Ananthanarayan 9/e p166, 8/e p164, Kuby's Immunology 6/e p381
  • Autoimmune hemolytic anemia is an example of Type II Hypersensitivity Reaction.
8. Ans. ALL (a) (b) (c) (d) (e) Ref: Ananthnarayan 9th /p164
Refer chapter review to know the list of mediators of anaphylaxis (i.e. type-I hypersensitivity reaction)
9. Ans. (a) (Type I hypersensitivity reaction) Ref: Ananthanarayan 9/e p165
  • Allergic rhinitis or Hay fever is e.g. of Type I hypersensitivity reaction.
 
Types II Hypersensitivity
10. Ans. (b), (Type II) Ref: Ananthanarayan 9/e p165, 8/e p162,
  • Myasthenia gravis is an Examples of Type II Hypersensitivity Reaction.
11. Ans. (a) (b) (d) (Myasthenia gravis, Good pasture syndrome, Grave's disease) Ref: Ananthnarayan 9th/p166
Refer chapter review for explanation.
12. Ans. (b) (Type-II) Ref: Journal: The facts about penicillin allergy: a review. J AdvPharmTech& Res 2010.
Penicillin in high doses can induce immune mediated hemolysis via the hapten mechanism in which antibodies are targeted against the combination of penicillin in association with red blood cells. Complement is activated by the attached antibody leading to the removal of red blood cells by the spleen. This is an example of drug induced type-II hypersensitivity reaction.
13. Ans. (b), (e) (Type II, Type V) Ref: Ananthanarayan 9/e p166,168, 8/e p162, Kuby's Immunology 6/e p388-390
  • Grave's disease is an Examples of Type II Hypersensitivity Reaction
  • According to some author, it is included in Type V HSN
  • Binding of anti thyroid stimulating hormone (anti-TSH) antibody to TSH stimulates the thyroid cells to produce thyroid hormone.
 
Types III Hypersensitivity
14. Ans. (c) (Type III) Ref: Ananthanarayan 9/e p166, 8/e p162, Kuby's Immunology 6/e p388-90
  • Hyper acute graft rejection is an examples of Types III Hypersensitivity reaction where as chronic graft rejection and Graft- versus-host disease is Types IV Hypersensitivity reaction.
15. Ans. (c) (Eichwald Silmser effect) Ref: Ananthanarayan 9/e p185
  • While transplants between members of a highly inbred strain of animals are successful, an exception is seen when the donor is a male and the recipient a female.
  • Such grafts are rejected as the grafted male tissue (XY) will have antigens determined by the Y chromosome which will be absent in the female (XX) recipient.
  • Grafts from the female to the male will succeed.
  • This unilateral sex linked histoincompatibility is known as the Eichwald-Silmser effect.
 
Types IV Hypersensitivity
16. Ans. (d) (Granulomatous disease) Ref: Ananthanarayan 9/e p166, 8/e p162
  • Granulomatous disease– type IV Hypersensitivity reaction
  • Arthus reaction, Serum sickness and Schwartzman reaction – type III Hypersensitivity reaction.
17. Ans. (a), (d) (Type I, IV) Ref: Ananthanarayan 9/e p162, 8/e p162, Kuby's Immunology 6/e p388-90
  • Type I skin test- Casoni skin test
  • Type IV skin test- Tuberculin, Lepromin and patch test.

National Immunization Schedule, Autoimmunity, Transplant & Cancer Immunology and Immunodeficiency DisorderCHAPTER 2.5

 
NATIONAL IMMUNIZATION SCHEDULE
National Immunization Schedule (NIS) for Infants, Children and Pregnant Women (India)
Vaccine
When to give
Dose
Route
Site
For Pregnant Women
TT-1
Earty in pregnancy
0.5 ml
Intra-muscular
Upper Arm
TT-2
4 weeks after TT-1*
0.5 ml
Intra-muscular
Upper Arm
TT-Booster
If received 2 TT doses in a pregnancy within the last 3 years*
0.5 ml
Intra-muscular
Upper Arm
For Infants
BCG
At birth or as early as possible till one year of age
0.1 ml (0.05 ml until 1 month age)
Intra-dermal
Left Upper Arm
Hepatitis B
At birth or as early as possible within 24 hours
0.5 ml
Intra-muscular
Antero-lateral side of mid thigh
OPV-0
At birth or as early as possible within the first 15 days
2 drops
Oral
Oral
OPV 1, 2 and
At 6th week, 10th week and 14th week
2 drops
Oral
Oral
DPT 1, 2 and 3
At 6th week, 10th week and 14th week
0.5 ml
Intra-muscular
Antaro-lataral side of mid thigh
Hepatitis B 1, 2 and 3
At 6th week, 10th week and 14th week
0.5 ml
Intra-muscular
Antaro-lataral side of mid thigh
Measles
9 completed months-12 months (glve up to 5 years if not received at 9-12 months age)
0.5 ml
Sub-cutaneous
Right upper Arm
Vitamin A (1st dose)
At 9 months with measles
1 ml (1 lakh IU)
Oral
Oral
For Children
DPT booster
16-24 months
0.5 ml
Intra muscular
Antero lateral side of mid thigh
OPV Booster
16-24 months
2 drape
Oral
Oral
Measles (2nd dose)
16-24 months
0.5 ml
Sub-cutaneous
Right upper Arm
Japanese Encephalitis**
16-24 months with DPT/OPV booster
0.5 ml
Sub-cutaneous
Left upper Arm
Vitamin A*** (2nd to 9th dose)
16 months with DPT/OPV booster. Then one dose every 6 months up to the age of 5 years
2 ml (2 lakh IU)
Oral
Oral
DPT Booster
5-6 year
0.5 ml
Intra-musecular
Upper Arm
TT
10 years and 16 years
0.5 ml
Intra-musecular
Upper Arm
Note:
(a) Interval between 2 doses of DPT, OPV and hepatitis B should not be less than one month.
(b) Minor cough, cold and mild fever are not a contraindication to vaccination.
(c) If the child has diarrhoea, give a does of OPV, but do not count the dose and ask the mother to return in 4 weeks for the missing does.
 
101AUTOIMMUNITY
Phenomenon of immune response against self antigens, either humoral or cell mediated.
 
Mechanisms of Autoimmunity
  • Sequestered antigens – lens protein
  • Altered self antigens – neoantigens (physical, chemical or biological influences)
  • Cross reacting antigens e.g. Streptococci with heart muscle
  • Molecular mimicry e.g. Coxsackie virus with myocardium
  • Polyclonal B cell activation
  • Inappropriate expression of class II MHC molecules
  • Cessation of tolerance and emergence of forbidden clones
  • Enhanced T helper cell and decreased T suppressor cell activity
  • Defects in Idiotype – anti Idiotype network.
 
Classification of Autoimmune Diseases
  • Hemocytolytic
    • Autoimmune hemolytic anemia- by drugs
    • Leucopenia
    • Thrombocytopenia
  • Localized (Organ Specific)
    • Direct cell damage
      • ✓ Hashimoto's thyroiditis,
      • ✓ Good pasture's syndrome
      • ✓ Post vaccine Encephalomyelitis
    • Stimulating antibodies- Grave's disease
    • Blocking antibodies- myasthenia gravis, pernicious anemia
  • Systemic (Non Organ Specific)
    • Antibodies directed against a broad range of antigens
    • E.g.-SLE, PAN, SJorgen's syndrome, Rheumatoid arthritis.
 
IMMUNODEFICIENCY DISORDER
  • Conditions where the defense mechanisms of the host are impaired leads to repeated microbial infections of varying severity and enhanced susceptibilities to malignancies.
 
Primary Immunodeficiency
  • Humoral
    • X – linked agammaglobulinemia
    • Transient hypogammaglobulinemia of infancy
    • Common variable immunodeficiency
    • Selective Immunoglobulin deficiencies
    • Immunodeficiency with hyper-IgM.
  • Cellular
    • Thymic hypoplasia
    • Chronic mucocutaneous candidiasis
    • Purine Nucleoside Phosphorylase deficiency.
  • Complement deficiencies disorders
    • C3 deficiencies – recurrent pyogenic infections
    • C6, C7, C8 – Neisserial infections
    • C1 esterase inhibitor deficiency –Hereditary angioneurotic edema.
  • Disorders of phagocytosis
    • Celiac Higashi syndrome
    • Leukocyte G6PD deficiency
    • Job's syndrome
    • 102Tuftin deficiency
    • Lazy leukocyte deficiency
    • Hyper IgE syndrome
    • Actin binding protein deficiency.
 
CANCER IMMUNOLOGY
TSTA: Tumor specific Transplantation antigen
  • These antigens are present on malignant cells but absent in correspondent normal cells
  • Two types-
    • Chemical induced TSTA- Transplantation specific, i.e. different tumors possess different TSTA even induced by same carcinogen.
    • Virus induced – Virus specific, i.e different tumors produced by same virus possesses same TSTA.
TATA: Tumor associated Transplantation antigen
  • They are present normally in less quantities, but elevated in malignancy
    • Alfa feto protein (Hepatoma)
    • Carcino embryonic Ag (in Ca colon)
    • Prostate specific Ag (Ca prostate)
    • CA 125 (in Ca Breast)
    • Beta HCG (in Hydatidiform mole).
 
Immunomodulators
  • Monoclonal Antibody- against tumor antigen
  • Tumor vaccine
  • Corynebacterium parvum, BCG
  • Thalidomide, Levamisole.
 
TRANSPLANT IMMUNOLOGY
Type of transplants
  • Autograft- within the same individual- e.g. skin grafting- Rejection is minimum
  • Isograft/ syngenic graft- Between identical twins
  • Allograft- Between two members of same species
  • Xenograft - Between two members of different species- Rejection is maximum.
 
Rejection Pathways
  • Direct pathway–Recipient T cells recognize intact donor APC containing MHC molecules
  • Indirect pathway–Recipient APC s process the donor-MHC antigen before presenting it to recipient T cells
  • Primary graft rejection–First set response-12 to 14 days
  • Second set rejection–Immunological memory, faster rejection-in 5-6 days.
Type
Time Taken
Mechanism
Hyper acute
Minute to hour
Preformed Antibody
Acute
Days to weeks
CD4 T cell activation
Chronic
Months to years
Fibrosis & widespread arteriopathy (T cell)
 
Post Transplant Infection
Peritransplant (<1 month)
  • Wound infections
  • Herpesvirus
  • 103Oral candidiasis
  • Urinary tract infection.
Early (1–6 months)
  • Pneumocystis jerovecii
  • Cytomegalovirus
  • Legionella
  • Listeria
  • Hepatitis B
  • Hepatitis C.
Late (>6 months)
  • Aspergillus
  • Nocardia
  • BK virus (polyoma)
  • Herpes zoster
  • Hepatitis B
  • Hepatitis C.
 
GVH (Graft Versus Host Reaction)
  • Occurs when Graft mounts an immune response against host tissue
  • GVH – Occurs when
    • Graft contains immunocompetent cells
    • Recipient has transplant Ag that is absent in graft
    • Recipient has not rejected graft
    • Recipient cannot respond immunologically.
  • Examples where GVH can take place:
    • Embryos or neonates, adults in immunosuppression
    • Runt disease.
  • Skin, liver and GI tract - main target organs of GVH.
 
Prevention of Transplant Rejection
  • ABO – typing
  • HLA – typing
    • Serological methods
      • ✓ Microcytotoxicity, Mixed leukocyte reaction
    • Molecular methods- PCR – RFLP.
104MULTIPLE CHOICE QUESTIONS
 
NATIONAL IMMUNIZATION SCHEDULE
1. Which vaccine can be given to pregnant women: (Recent Question 2013)
  1. Hepatitis B
  2. Meningococcus
  3. Rabies
  4. Measles
  5. BCG
2. According to national immunization schedule, which of the following is recommended for a child of 5 year of age? (AIMS Nov 2013)
  1. Pentavalent vaccine and vitamin A
  2. DT booster
  3. DT, OPV and vitamin A
  4. DPT booster and vitamin A
 
AUTOIMMUNITY
3. Autoimmunity can be caused due to all of the following except: (AIIMS May 2005)
  1. The pressure of forbidden clones
  2. Expression of cryptic antigens
  3. Negative selection of T- cells in the thymus
  4. Inappropriate expression of the MHC proteins
 
IMMUNODEFICIENCY
4. Chronic granulomatous disorder is due to defect in: (NEET Pattern Based)
  1. B-cell
  2. NADPH oxidase
  3. IgA
  4. T-cell
5. Chediakhigashi syndrome, defect is: (NEET Pattern Based)
  1. Fusion of lysosome
  2. T-cells
  3. B-cells
  4. Complement
6. Purine Nucleoside phosphorylase deficiency: (NEET Pattern Based)
  1. Humoral immunity deficiency
  2. Acquired immunity deficiency
  3. SCIDs
  4. Cell mediated immunity deficiency
7. Decreased IgM, bleeding tendency with eczema is seen in: (AI 2012)
  1. Wiskott-Aldrich syndrome
  2. Chronic granulomatous disease
  3. Job's syndrome
  4. Chediak Higashi syndrome
8. True about wiskottaldrich syndrome is: (DNB June 2011)
  1. Primary defect is B cell defect
  2. Primary defect is T cell defect
  3. Wasp gene mutation
  4. Defect in Phagocytosis
9. In a 5 year old boy who has history of pyogenic infections by bacteria with polysaccharide-rich capsules, which of the following investigations should be done? (AI 2012, AIIMS May 2012)
  1. IgA deficiency
  2. IgG1 deficiency
  3. IgG2 deficiency
  4. IgA and IgG2 deficiency
10. Adenosine deaminase deficiency is seen in the following: (AI 2005, 2001)
  1. Common variable immunodeficiency
  2. Severe combined immunodeficiency
  3. Chronic granulomatous disease
  4. Nezel of syndrome
11. SC 1D which is true: (NEET Pattern Based)
  1. Adenosine deaminase deficiency
  2. Decreased circulating lymphocytes
  3. NADPH oxidase deficiency
  4. Cl esterase dificiency
12. The commonest Primary immunodeficiency is: (PGI June 2005)
  1. Common variable immunodeficiency
  2. Isolated IgA immunodeficiency
  3. Wiskott – Aldrich syndrome
  4. AIDS
13. Which one of the following statements is correct regarding chronic granulomatous disease: (AIIMS Nov 2004)
  1. It is an autosomal dominant disease
  2. It is characterized by abnormal bacterial phagocytosis
  3. Recurrent streptococcal infections are usual in this disease
  4. Nitroblue tetrazolium test is useful for screening
14. All are true regarding agammaglobulinemia except: (PGI 2001)
  1. Loss of germinal centre in lymph node
  2. Normal cortical lymphocytes
  3. Normal cortical lymphocytes in paracortex and medulla
  4. Decreased red pulp in spleen
  5. Immunodeficiency (cell mediated)
15. Which is found in DiGeorge's syndrome: (PGI 2001)
  1. Tetany
  2. Eczema
  3. Mucocutaneous Candidiasis
  4. Absent B and T cells
  5. Total absence of T cells
16. Disorders of phagocytosis are all except: (PGI May 2013)
  1. Job's syndrome
  2. Chediak-Hegashi syndrome
  3. Myeloperoxidase deficiency
  4. Wiskott-Aldrich Syndrome
  5. Tuftsin deficiency
17. 105Wiskott Aldrich syndrome is characterized by: (Recent MCQ 2013)
  1. Eczema
  2. Elevated IgM
  3. Neutrophilia
 
TRANSPLANTATION IMMUNOLOGY
18. A woman with infertility receives an ovary transplant from her sister who is an identical twin. What type of graft is? (AI 2005)
  1. Xenograft
  2. Autograft
  3. Allograft
  4. Isograft
19. Hyperacute graft rejection is seen in: (Recent MCQ 2013)
  1. Bone marrow
  2. Kidney
  3. Liver
  4. Heart
106EXPLANATIONS
 
National Immunization Schedule
1. Ans. (a), (b), (c) (Hepatitis B, Meningococcus, rabies) Ref: Park 22nd 98/, 21st/p98
All live vaccine such as Measles and BCG are contraindicated in pregnancy
2. Ans. (d) (DPT booster and vitamin A) Ref: Park 22/e p115
According to national immunization schedule in India
  • DPT schedule- 1st/2nd/3rd dose at 6/10/14weeks and two boosters at 16-24months and 5-6yr
  • Vitamin A- 1st dose at 9month (along with measles), 2nd dose at 16–24months (along with DPT booster) and the 3rd to 9th dose given every 6month till 5yrs.
  • OPV schedule- Zero dose at birth, then 1st/2nd/3rd dose at 6/10/14weeks and booster at 16-24months.
 
Autoimmunity
3. Ans. (c) (Negative selection of Tcell in thymus) Ref: Harrison 17/e p2071-2073
Negative selection of T-cell is a process by which self reactive T-cells are killed in thymus - this is one of the mechanism of preventing development of Autoimmunity.
The other options are the mechanisms of preventing Autoimmunity
  • Forbidden clones with self reactivity arise as a result of failure of negative selection of autoreactive T-cell.
  • Cryptic antigens are the antigenic epitopes, which are normally not exposed, gets exposed to the immune system as a result of injury or infection and leads to development of immune response.
  • Inappropriate expression of MHC proteins also leads to development of self destructive immune response.
 
Immunodeficiency
4. Ans. (b) (NADPH oxidase) Ref: Ananthanarayan 9/e p172, 8/e p159
Chronic granulomatous disease
  • Due to defect in phagocytosis (results from absence of NADPH oxidase)
  • ↑Recurrent catalase +ve pyogenic infection (catalase –ve organisms are handled normally)
  • Screening test used- Nitroblue tetrazolium (NBT) reduction test is negative.
5. Ans. (a) (Fusion of lysosome) Ref: Ananthanarayan 9/e p172, 8/e p159
  • Chediak Higashi syndrome-
    • ✓ Results from ↓phagolysosome function due to ↓ lysosomal enzyme transfer to vacuole
    • ✓ ↓Pigmentation, photophobia, nystagmus, giant inclusions in leucocytes, ↑pyogneic infection.
6. Ans. (d) (Cell mediated immunity deficiency) Ref: Ananthanarayan 9/e p172, 8/e p157
  • Purine Nucleoside phosphorylase deficiency – is an autosomal recessive inherited trait, leads to low CMI & ↑Recurrent infection (Candidiasis), hypoplastic anemia & ↓uric acid.
7. Ans. (a) (Wiskott-Aldrich syndrome) Ref: Ananthanarayan 9/e p172, 8/e p158, Harrison 17/e p2060
Wiskott-Aldrich syndrome
  • X-linked disease-associated with WASp gene mutation (98%)
  • Combined immunodeficiency (Both B & T cell defect)
  • Defect in the ability to mount an IgM response to capsular polysaccharides of bacteria.
    Associated with:
    • ✓ Elevated IgA levels,
    • 107✓ Normal IgE levels, and
    • ✓ Low IgM levels.
  • Triad of symptoms includes:
    • ✓ Recurrent pyogenic Infections,
    • ✓ Thrombocytopenic Purpura,
    • Eczema
8. Ans. (c) (Wasp gene mutation) Ref: Ananthanarayan 9/e p172, 8/e p158.
  • Refer the above explanation
9. Ans. (c) (IgG2 deficiency) Ref: Jawetz 25/e p130, Robins Pathology 7/e p144-147
  • IgG2 subclass is the predominant antibody raised against the polysaccharide capsular antigens and deficiency of IgG2 subclass is commonly associated with recurrent pyogenic infection due to bacteria with polysaccharide capsule like Str.pneumoniae or H.influenzae.
According to the memory recall of some other students….
The question asked as- “Boy with history of recurrent sinopulmonary infections by bacteria with polysaccharide-rich capsules”.
  • For this question… the answer could be. “Both IgG2 & IgA deficiency”-
  • IgA deficiency- Due to history of recurrent sinopulmonary infections
    • ✓ IgA is the major immunoglobulin in mucosal secretions and weakened mucosal defenses predispose patients to recurrent sinopulmonary infections and diarrhea… Robins 7th /p 144-147
  • IgG2deficiency- Due to infections by bacteria with polysaccharide-rich capsules.
10. Ans. (b) (Severe combined immunodeficiency) Ref: Ananthanarayan 9/e p174, 8/e p158.
  • Severe combined immunodeficiency includes many syndromes with severe deficiency of both humoral and CMI responses.
  • They are inherited in autosomal recessive mode.
  • Primary defect is at the level of early precursors of immunocompetent cells in the fetal liver and bone marrow
  • Severe combined immunodeficiency types includes:
    • ✓ X linked(MC type, 50-60%)- due to mutation in γ chain of cytokine receptor (MC is IL7 receptor)
    • ✓ Jak3 signal defect
    • ✓ Bare lymphocyte syndrome (↓MHCII)
    • ✓ Swiss type agammaglobulinemia
    • ✓ Reticular dysgenesis of de Vaal
    • Adenosine deaminase (ADA) deficiency
    • ✓ In ADA deficiency, there will be total absence to mild abnormalities of B and T cell function.
11. Ans. (a) (Adenosine deaminase deficiency) Ref: Ananthanarayan 9/e p172, 8/e p158,
Refer Q.No 8.
12. Ans.(b) (Isolated IgA immunodeficiency) Ref: Ananthanarayan 9/e p173, 8/e p 158 and Harrison 17/e p2058
  • Among immunodeficiency diseases, primary immunodeficiency is relatively common.
  • The most common is- isolated IgA deficiency, occurs in approximately 1 in 600 individuals (in Europe and North America) and it is reported in about 0.2% of normal populations.
  • Next most common disorder - Common variable immunodeficiency, characterized by pan hypogamma- globulinemia.
  • Both of these immunodeficiency states often become clinically evident in young adults.
  • The more severe forms of primary immunodeficiency are relatively rare, have their onset early in life, and all too frequently result in death during childhood.
13. Ans. (d) (Nitro blue tetrazolium test is useful for screening) Ref: Ananthanarayan 9/e p175, 8/e p 159
Chronic granulomatous disease (CGD)
  • Inherited as an X-linked trait or as an autosomal recessive trait.
  • Clinical manifestations are usually apparent before the end of the second year of life.
  • Associated with deficiency of an enzyme, NADPH oxidase.
  • This enzyme deficiency causes-↓oxidative burst leads to ↓ killing of intracellular pathogens.
  • 108Process of phagocytosis is normal, but leucocytes fail to undergo degranulation following phagocytosis.
  • Catalase positive bacteria like Staphylococcal infections are recurrent in these patients, where as catalase negative bacteria like Streptococcal and Pneumococcal infections are not frequent as they are handled normally.
  • The quantitative nitro blue tetrazolium (NBT) test is employed to confirm the diagnosis of CGD- Leucocytes from the patients fail to reduce nitro blue tetrazolium (NBT) during phagocytosis.
14. Ans. (b), (c), (d) (Normal cortical lymphocytes, Normal cortical lymphocytes in paracortex and medulla and decreased red pulp in spleen) Ref: Ananthanarayan9/e p171-72, 8/e p155
Agammaglobulinemia/ X-linked agammaglobulinemia (Bruton's agammaglobulinemia).
  • It is the 1st immunodeficiency have been recognized
  • There is defective B-cell (Humoral) immunity with normal T-cell / cell mediated immunity.
  • The disease presents as recurrent serious infection with pyogenic bacteria, like Pneumococci, Streptococci, Meningococci, and Pseudomonas etc.
  • Patients respond normally to viral infections.
  • Live microbial vaccines should not be given to the patients with any type of primary immunodeficiency.
  • All classes of Ig are grossly depleted.
  • Lymph node biopsy reveals marked decrease in bursa dependent areas (Cortical areas and medullary cord)
  • In spleen there is depletion of B-cell dependent areas like- Perifollicular region, mantle layer and germinal centre and not red pulp.
15. Ans. (a), (c) (Tetany and Mucocutaneous Candidiasis) Ref: Ananthanarayan 9/e p173, 8/e p157
DiGeorge's syndrome (Thymic hypoplasia):
  • It is a type of cellular immunodeficiency
  • Developmental defect involving endodermal derivatives of 3rd and 4th pharyngeal pouches
  • Presence of aplasia or hypoplasia of thymus and parathyroid glands
  • It is not hereditary and does not show familial incidences
  • Probably due to intrauterine infections
  • Clinical features
    • ✓ Enhanced susceptibility to fatal viral, fungal (candidiasis) and bacterial infections
    • ✓ Fallot's tetrology and other anomalies in heart and great vessels
    • Neonatal hypocalcemic tetany- due to hypoparathyroidism
    • Circulating T-cells are reduced in number (not totally absent) and B-cells are not affected.
16. Ans. (d) (Wiskott-Aldrich Syndrome) Ref: Ananthnarayan 9th /p172
Refer chapter review for the detail list of disorders of phagocytosis.
17. Ans. (a) (Eczema) Ref: Ananthanarayan 9/e p174
  • Wiskott Aldrich syndrome is characterized by-
    • ✓ Eczema, thrombocytopenic purpura and recurrent infection.
    • ✓ X linked, boys are effected
    • ✓ Both CMI and AMI deficiency
    • ✓ ↓ Thymus & paracortical area of LN
    • ✓ Low IgM, but normal or elevated IgA or IgG
 
Transplantation Immunology
18. Ans. (d) (Isograft) Ref: Ananthanarayan 9/e p183, 8/e p178
See text.
19. Ans. (b) (Kidney) Ref: Ananthanarayan 9/e p184, 166
  • Hyperacute graft rejection is seen in kidney and skin grafting and is preformed antibody mediated (Type III Hypersensitivity reaction)
109Systemic Bacteriology
  • 3.1 Staphylococcus
  • 3.2 Streptococcus and Pneumococcus
  • 3.3 Neisseria
  • 3.4 Corynebacterium & Bacillus
  • 3.5 Clostridium
  • 3.6 Mycobacterium
  • 3.7 Enterobacteriaceae (E.coli, Klebsiella, Proteus, Shigella, Salmonella, Yersinia)
  • 3.8 Vibrio
  • 3.9 Pseudomonas & Other Non Fermenters and Haemophilus, Bordetella, Brucella (HBB)
  • 3.10 Spirochaete
  • 3.11 Rickettsia, Chlamydia and Mycoplasma
  • 3.12 Miscellaneous Bacteria110

StaphylococcusCHAPTER 3.1

 
GRAM POSITIVE COCCI
 
Micrococcaceae Family
  • Catalase positive
  • GPC cluster/tetrad- as cell division takes place in multiple planes. (‘Staphylo’ means grape like)
  • Staphylococcus, Micrococcus.
 
Streptococcaceae Family
  • Catalase negative
  • GPC chain/pair- as cell division takes place in single plane
  • Sterptococcus, Pneumococci, Enterococcus.
 
VIRULENCE FACTORS OF STAPHYLOCOCCUS AUREUS
  • Cell wall factors
  • Enzyme
  • Toxin
  • Hemolysin.
Antigenic structure and Virulence factors
Cell Wall Associated Structures
Activity
Peptidoglycan
  • More thicker,
  • Confers cell rigidity and induces inflammatory response
Teichoic acid
  • Helps in adhesion to mucosal surfaces and prevent opsonisation
Clumping factor/Bound coagulase
  • Responsible for slide coagulase reaction
Protein A
  • Antiphagocytic, anti complementary, chemotactic
  • Binds to Fc region of IgG leaving Fab region free to bind to Antigen - Basis of Co- agglutination reaction
Extracellular factors
Activity
α Hemolysin
  • Inactivated at 70°C reactivated paradoxically at 100°C (due to denaturation of a heat labile inactivator at 100°C)
  • Leucocidal, Cytotoxic, dermonecrotic, lethal
112
β Hemolysin
  • Sphingomyelinase
  • Lyses sheep RBC, but not human or rabbit RBC
  • Exhibits hot-cold phenomenon
γ Hemolysin
  • Bicomponent proteins, Lyses rabbit sheep and human red blood cells
δ Hemolysin
  • Detergent like, Lyses rabbit, sheep and human red blood cells
Leucocidins / Panton valentine toxin
  • Two components F and S
  • Damage PMN and macrophages
  • Associated with Community acquired MRSA
Synergohymenotropic toxin
  • Bicomponent toxin - γ and Leucocidins
Epidermolytic toxin (Exfoliative toxin)
  • Mainly belong to phage group II
  • 2 TYPES:
    • ✓ A: heat stable, chromosomal
    • ✓ B: heat labile, Plasmid mediated
  • Scalded skin syndrome (Nikolsky's sign-epidermal layer separated)
  • Severe- Ritter disease (newborn), TEN (adult)
  • Milder- Pemphigus neonatorum, bullous impetigo
Enterotoxins
  • Produced by 50% of clinical isolates
  • Cause food poisoning
  • Incubatory Period: 1–6hr due to preformed toxin
  • Vomiting occurs due to Vagus N and vomiting centre stimulation
  • Heat stable (not destroyed after heating food)
  • Multiple antigenic type (A-E, G-I, K-M) (MC- type A)
Toxic shock syndrome toxin
  • Most strains belong to phage group I
  • Enterotoxins F/pyrogenic exotoxin C (MC) followed by Enterotoxin B,C
  • Associated with use of vaginal tampon
  • Anti TSST1 Antibody is protective
  • Rash, fever, hypotension and Multi organ failure
  • Treatment- Clindamycin (reduces toxin synthesis)
Extracellular enzymes- Coagulase, heat stable thermonuclease, Deoxyribonuclease, Lipase, Protease, Staphylokinase (fibrinolysin), Hyaluronidase
Tube coagulase
Slide coagulase
Due to coagulase enzyme
Due to clumping factor
Requires CRF in plasma
Doesn't Requires CRF in plasma
Done in tube
Done in slide
Test +ve:- coagulum formed /Plasma clotted
Test +ve :- clumps formed
Staph.lugdunensis -ve
S.lugdunensis +ve
Staph. schleferi +ve
Staph. schleferi -ve
Other coagulase positive species (both tube & slide)-
  • Staph. hyicus
  • Staph.intermedius
Staphylococcus aureus is characterized by:
  • Coagulase
  • Heat stable thermo nuclease
  • DNAse
  • Phosphatase (also by S.epidermidis)
  • Golden yellow pigmentation (beta carotene)
  • Hemolysis on blood agar
  • Ferment mannitol
  • Black colony on K tellurite agar
  • Gelatin liquefy
  • Protein A
  • Clumping factor
113
  • MRSA ➔ Methicillin resistant S taphylococcus aureus
    • Definition - Methicillin MIC 8 µg/ml or more, Oxacillin MIC 4 µg/ml or more
    • Resistance is chromosomally mediated (mec A gene)
    • PBP gets altered PBP 2a (MC mechanism)
  • BORSA- Borderline Oxacillin resistant S.aureus- MIC is Mildly elevated, occurs due to hyperproduction of β lactamase. (BORSA strains)
  • Penicillin binding Protein (PBP)-
    • Normal protein present in all bacterial cell membrane.
    • Transpeptidase in nature
    • Required for cross linking of the peptidoglycan layer (essential for cell wall syn- thesis)
    • It is also the site of attachment for- beta lactam drugs to bind & inhibit
    • In MRSA- MecA gene alters PBP to PBP2a (which has less affinity for beta lactams)
    • Hence, MRSA strains are resistant to all beta lactams
  • Treatment-
    • Vancomycin is DOC (but it should not be started first, given only if others are resistant)
    • Others- Teichoplanin, Linezolid, streptogramin,
    • Cipro/levofloxacin, cotrimoxazole, clindamycin, mupirocine, minocycli- ne
    • Newer – Daptomycin, ceftobiprole, tigecycline, oritavancin
  • Types of MRSA-
    • Community acquired- mediated by mec A subtype IV and Possess PV Toxin, multidrug resistance
    • Hosp acquired - mediated by mecA subtype I, II, III, mono drug resistance
  • Detection of MRSA-
    • Antibiotic susceptibility test on Muller Hilton agar (Disk diffusion test)
      • ✓ By using Methicillin or Oxacillin disc or cefoxitin disc
  • Conditions to be maintained-
    • On media containing 4% NaCl,
    • Incubation at 30°C,
    • Full 24hr incubation
VRSA- Vancomycin Resistant S taphylococcus aureus – MIC 32 µg/ml or more
VISA - Vancomycin Intermediate S taphylococcus aureus - MIC 8–16 µg/ml
  • Mechanism – ↑thickness of cell wall (MC)
  • Treatment – same drugs given for MRSA except Vancomycin and Teichoplanin.
 
Culture Media
  • 5% sheep blood agar: S. aureus shows pin head shaped colony with narrow zone of β haemolysis
  • MacConkey agar: Minute pink Lactose fermenting colonies
  • Selective medium
    • Salt milk agar (8–10% NaCl)
    • Mannitol salt agar-with 7.5% NaCl
    • Ludlam's lithium chloride and tellurite media
  • Media for pigment: Milk agar, Glycerol monoacetate agar, Tween agar, incubation at 22°c
  • Golden yellow pigmentation on nutrient agar: Due to β-carotene
114S.aureus is the MC agent for
  • Skin and soft tissue infection
  • Botryomycosis
  • Tropical pyomyositis – S. aureus, (acute bacterial myositis – Group A Streptococ- cus) (Overall - S.aureus)
  • Osteomyelitis and septic arthritis (MC- knee)
  • Postoperative parotitis
  • Paronychia
  • Epidural abscess
  • Surgical wound infection
  • Pneumatocele
  • Folliculitis, furuncle, carbuncle, Mastitis and Hidradenitis suppurativa
Endocarditis
  • MC cause of Native valve endocarditis – S.aureus
  • MC cause of Prosthetic valve endocarditis
    • Early prosthetic valve endocarditis (<12months) - S.epidermidis
    • Late prosthetic valve endocarditis (>12months)- Strept Viridans
    • Overall MC cause of prosthetic valve endocarditis- S.epidermidis
  • MC cause of Endocarditis in IV drug users: -
    • Rt sided – S. aureus,
    • Lt sided – Enterococcus > S. aureus
    • Over all-S. aureus
  • MC cause of Sub acute endocarditis – Strept Viridans
Drug resistance in Staphylococcus aureus:
  • Penicillin resistance (90%)
    • Penicillinase production(β lactamase)
    • Destroys penicillin
    • Plasmid mediated.
  • MRSA (30–50%):
    • Alternation of penicillin binding protein
    • Chromosomal (MecA).
Points to remember:
  • MC method for typing of S.aureus – Phage typing (pattern method)
  • Mechanism of Penicillin resistance- production of β lactamase
  • Penicillin resistance in S.aureus transferred by – Transduction(MC) > conjugation
  • MC site of colonization – Anterior nares and Skin, (perineum, axilla, groins)
  • MC way of spread of infection in hospital- through the hands of hospital staff
  • Most effective way to prevent the hospital infection – hand washing.
 
Coagulase Negative Staph. (CONS) - Normal Flora of Skin
S. epidermidis
  • MC CONS- Accounts for 60–70% of CONS
  • Produce polysaccharide glycocalyx (slime) (Biofilm production)
  • Hence can adhere to any implanted foreign body like valvular shunts, prosthetic device
  • Infections:
    • Endocarditis with insertion of valvular prosthesis
    • Ventricular shunt infections
    • Stitch abscess.
S. saprophyticus:
  • Causes UTI in young sexually active female.
  • Novobiocin Resistant.
115MULTIPLE CHOICE QUESTIONS
1. Cephalosporin with anti MRSA activity: (AIIMS Nov 2014)
  1. Ceftriaxone
  2. Aztreonam
  3. Cefazolin
  4. Ceftobiprole
2. Drug(s) used in MRSA is /are: (PGI Nov 2014)
  1. Linezolid
  2. Cephalothin
  3. Vancomycin
  4. Meropenam
  5. Piperacillin + Tazobactum
3. Staph aureus causes: (NEET Pattern Based)
  1. Erythrasma
  2. Acne vulgaris
  3. Chancroid
  4. Bullous impetigo
4. Staphylococcal scalded skin syndrome is caused by: (NEET Pattern Based)
  1. Hemolysin
  2. Coagulase
  3. Enterotoxin
  4. Epidermolytic toxin
5. Catalase positive, beta-hemolytic staphylococcus: (NEET Pattern Based)
  1. S. aureus
  2. S. epidermidis
  3. S. saprophyticus
  4. None
6. Catalase positive coagulase negative beta hemolytic bacteria: (NEET Pattern Based)
  1. Strep pyogenes
  2. Staph aureus
  3. Coagulase negative staph
  4. Enterococci
7. What is the best way to control the outbreak of MRSA infection in a hospital ward? (AIIMS 2012)
  1. Vancomycin is given empirically to all the patients
  2. Frequent fumigation of wards
  3. Wearing mask before invasive procedure
  4. Washing of hands before & after treating the patients
8. Synergohymenotropic toxins of Staphylococal consists of: (PGI June 2011)
  1. a toxin
  2. β toxin
  3. γ toxin
  4. d toxin
  5. Panton – Valentine toxin
9. A person had infection due to gram positive organism treated with methicillin and then culture sensitivity shows resistance to it. Hence all can be given in MRSA except: (AIIMS Nov 2011) (AI 2012)
  1. Cotrimoxazole
  2. Cefaclor
  3. Vancomycin
  4. Ciprofloxacin
10. Most common cause of pyomyositis is: (DNB Dec 2011)
  1. Streptococcus pyogenes
  2. Pseudomonas
  3. Staphylococcs aureus
  4. E. Coli
11. Preformed toxin produces diarrhea in which organism? (DNB June 2010)
  1. Staphylococcus aureus
  2. Vibrio cholera
  3. Salmonella
  4. Escherichia coli
12. In a post operative ward, 4 out of 10 patients developed wound discharge, which on culture was found to be positive for coagulase positive gram positive cocci. Antimicrobial susceptibility showed that the strain was resistant to Methicillin. On surveillance cultures, a health care personnel attending to the patients was found to be a nasal carrier for the same agent. The following are true regarding the agent responsible for the outbreak: (AI 2011)
  1. The major route of spread causing the outbreak is air borne
  2. The resistance to methicillin is plasmid mediated
  3. The organism has an alteration in its penicillin binding proteins.
  4. It will be sensitive to treatment with antibiotics containing amoxicillin+ clavulinic acid combination.
13. Toxic shock syndrome is caused by: (JIPMER 2011 & 2010, PGI Dec 2007)
  1. Streptococcus pyogenes
  2. Staphylococcus aureus
  3. Strept. albicans
  4. Enterococcus.durans
14. All are true about penicillin resistance except: (AI 2010)
  1. β lactamase – MC mechanism of resistance
  2. Alteration in PBP is imp for gram –ve also
  3. Alteration in permeability /penetration of antibiotics – imp for gram –ve only
  4. β lactamase – imp for gram +ve and -ve bacteria
15. All are true about PBP except: (AI 2010)
  1. PBP is localized in outer cell wall
  2. PBP is essential for cell wall synthesis
  3. PBP acts as carboxypeptidase and transpeptidase
  4. Alteration in PBP – basis of MRSA
16. All true about S.aureus except: (AI 2010)
  1. MC source of infection – cross infection from infected persons in hospital
  2. 30% of general population is healthy carriers
  3. TSS and epidermolysin are Superantigens
  4. MRSA- chromosomally mediated
17. A patient has prosthetic valve replacement and he develops endocarditis 8 months later. Organism responsible is: (AIIMS Nov 2010)
  1. Staphylococcus aureus
  2. Streptococcus Viridans
  3. Staphylococcus epidermidis
  4. HACETh
18. 116A 25 year old man with 3 weeks fever presented with tricuspid valve vegetation. Patient is intravenous drug abuser. Most common cause of endocarditis in this patient is: (AIIMS Nov 2009)
  1. Staph. aureus
  2. Candida albicans
  3. Pseudomonas
  4. Strep. Viridian
19. True about methicillin resistance staphylococcus aureus (MRSA): (PGI June 2009)
  1. Isoxazolyl penicillin is highly effective
  2. All MRSA are multidrug resistance
  3. Vancomycin is effective
  4. MRSA are more virulent then sensitive strains
  5. Resistance develop due to altered binding protein
20. A diabetic patient developed cellulitis due to S. aureus, was found to be methicillin resistant on the antibiotic sensitivity testing. All of the following antibiotics will be appropriate except: (AI 2006)
  1. Vancomycin
  2. Imipenem
  3. Teichoplanin
  4. Linezolid
21. All of the following statements are true regarding Staphylococci except: (AIIMS Nov 2004)
  1. A majority of infections caused by coagulase- negative Staphylococci are due to Staphylococcus epidermidis
  2. B-lactamase production in Staphylococci is under plasmid control
  3. Expression of methicillin resistance in Staphylococcus aureus increases when it is incubated at 37 C on blood agar.
  4. Methicillin resistance in Staphylococcus aureus is independent of β-lactamase production
22. The following are characteristic features of staphylococcus food poisoning except: (AIIMS 2004)
  1. Optimum temperature for toxin production is 37 C
  2. Intradietic toxin is responsible for intestinal symptoms
  3. Toxin can be destroyed by boiling for 30 minutes
  4. Incubation period is 1–6 hours
23. A patient in an ICU is on a CVP line. His blood culture shows growth of gram positive cocci which are Catalase positive and coagulase negative. The most likely etiological agent is: (AIIMS May 2003)
  1. Staphylococcus aureus
  2. Staphylococcus epidermidis
  3. Streptococcus pyogenes
  4. Enterococcus faecalis
24. Staphylococcus aureus differs from Staphylococcus epidermidis by: (AI 2002)
  1. Is coagulase positive
  2. Forms white colonies
  3. A common cause of UTI
  4. Cause endocarditis in drug addicts
25. Staph. aureus causes vomiting in 6–8 hours. The mechanism of action is: (AIIMS May 2002)
  1. Stimulation of cAMP
  2. Vagal stimulation
  3. Stimulation of cGMP
  4. Acts through ganglioside GM receptor
26. Staphylococcal food poisoning includes the following feature: (AIIMS 2001)
  1. Incubating period of 24 hours
  2. Requires immediate antibiotics
  3. Common with dairy products
  4. Heat labile enterotoxin
27. A child after consuming food in a party complains of diarrhea within 1–5 hours. The diagnosis is: (NEET Pattern Based, AIIMS 2001, 1996, 1995)
  1. S. aureus
  2. Streptococcus
  3. Clostridium perfringes
  4. Clostridium botulinum
28. Common source of Staphylococcus in hospital: (PGI June 2001)
  1. IV fluids
  2. Infective wounds
  3. Hands of hospital personnel
  4. Bed linen
  5. Instruments
29. Staphylococcus can cause: (PGI June 2001)
  1. Ecthyma
  2. Erythrasma
  3. Furuncle
  4. Impetigo contagiosum
  5. Sycosis barbae
30. Which of the following organism is implicated in the causation of botryomycosis? (PGI 2001)
  1. Staphylococcus aureus
  2. Staphylococcus albus
  3. Pseudomonas aeruginosa
  4. Streptococcus pneumoniae
  5. Streptococcus pyogenes
31. Food poisoning after 1–5 hr of consumption of ice cream, identify the causative agent? (Recent Question of 2013)
  1. Staphylococcus aureus
  2. Bacillus cereus
  3. Salmonella
  4. Vibrio parahemolyticus
32. MC phage type of Staphylococcus aureus: (Recent Question of 2013)
  1. 79/80
  2. 3A/3B
  3. 80/81
33. A 25 year old female presented to the hospital on the third day of menstruation with complaints of high fever, vomiting and a rash on her trunk and extremities. On investigation she had leucocytosis and a negative blood culture. She is diagnosed as: (MHPG 2014)
  1. Staphylococcal food poisoning
  2. Scalded skin syndrome
  3. Toxic shock syndrome
  4. Varicella zoster infection
117EXPLANATIONS
1. Ans. (d) (Ceftobiprole) Ref: Journal: Ceftobiprole: a review of a broad-spectrum and anti-MRSA cephalosporin. Am J ClinDermatol. 2008;9(4):245–54.
  • Fifth-generation cephalosporins such as ceftobiprole and ceftaroline are effective against MRSA.
  • Ceftobiprole has additional activity against penicillin-resistant Streptococcus pneumoniae, Pseudomonas aerugi- nosa, and Vancomycin resistant Enterococci(VRE).
2. Ans. (a) (c) (Linezolid), (Vancomycin) Ref: Harrison 18th/chapter- table 135.3
  • MRSA strains are resistant to all β lactam drugs.
3. Ans. (d) (Bullous impetigo) Ref: Ananthanarayan 9 e/p202, 8 e/p198
  • Bullous Impetigo is a cutaneous condition that characteristically occurs in the newborn, presenting with bullae.
  • Bullous impetigo is caused by Staphylococcus aureus, which produces exfoliative toxins, whereas non-bullous impetigo is caused by either Staphylococcus aureus, or Streptococcus pyogenes.
4. Ans. (d) (Epidermolytic toxin) Ref: Ananthanarayan 9 e/p202, 8 e/p198
  • Staphylococcal scalded skin syndrome is due to Epidermolytic or exfoliative toxin.
5. Ans. (a) (S. aureus) Ref: Ananthanarayan 9 e/p202, 8 e/p198
  • S. aureus is- Catalase positive, beta-hemolytic and coagulase +ve.
6. Ans. (c) (Coagulase negative staph) Ref: Ananthanarayan 9 e/p202, 8 e/p198
  • Coagulase negative staph (CONS) are Catalase positive, non hemolytic and coagulase – ve. However, some CONS like St.hemolyticus and St.warneri are beta hemolytic.
  • S. aureus is- Catalase positive, beta-hemolytic and coagulase +ve
  • Strept.pyogenes- Catalase negative, beta-hemolytic and coagulase -ve
  • Enterococci- Catalase negative, non hemolytic and coagulase -ve.
7. Ans. (d) (Washing of hands before & after treating the patients) Ref: Harrison 18 e/p1166
  • Hand washing is the most effective way to prevent hospital acquired infections.
  • Prevention of the spread of S. aureus infections in the hospital setting involves hand washing and careful attention to appropriate isolation procedures……. Harrison 18 e/p1166.
8. Ans. (c) (e) (γ toxin, Panton – Valentine toxin) Ref: Ananthanarayan 9 e/p202, 8 e/p198
  • Gamma hemolysin and Panton Valentine toxin are membrane active toxins of Staph. aureus that are composed of two com- ponents (bicomponent toxins) and together called as Synergohymenotroic toxins.
9. Ans. (b) (Cefaclor) Ref: Koneman's diagnostic Microbiology 6th /p649–655
  • MRSA strains are resistant to all β lactam antibiotics as it results from alternation of penicillin binding protein. (For detail- refer chapter review).
10. Ans. (c) (Staphylococcus aureus) Ref: Harrison 18 e/p1164
  • Pyomyositis is an unusual infection of skeletal muscles that is seen primarily in tropical climates but also occurs in immunocompromised and HIV-infected patients. Pyomyositis presents as fever, swelling, and pain overlying the involved muscle.
  • Tropical pyomyositis – MC cause is S. aureus, (acute bacterial myositis – MC cause is Group A Streptococcus) (Overall - MC cause of pyomyositis is S.aureus).
11. Ans. (a) (Staphylococcus aureus) Ref: Harrison 18 e/p1165
  • E.g of Preformed toxin i.e toxin secreted in food- S.aureus enterotoxin, Bacillus cereus emetic type of enterotoxin and botulinum toxin.
11812. Ans. (c) (The organism has an alteration in its penicillin binding proteins.) Ref: Ananthanarayan 9 e/p205, 8 e/p197
  • Coagulase positive gram positive cocci – indicates Staphylococcus aureus
  • Resistant to Methicillin – indicates MRSA (Methicillin resistant S taphylococcus aureus)
  • Health care personnel of the hospital found to be nasal carrier for the same agent- Indicates that the infection has spread to the patient from health care personnel of the hospital
  • “The MC way of spread of infection in hospital- through the hospital staff (not from other patient from the hospital)”
  • Mechanism of Resistance of MRSA – due to chromosomally mediated Mec A gene which codes for altered PBP 2a (Penicillin Binding protein or receptor 2a) which has less affinity for β lactam drugs.
About Other Options
  • The major route of spread causing the outbreak is – through the hands of hospital staff
  • The resistance to Methicillin is chromosomally mediated
  • Since MRSA coded altered PBP 2a (Penicillin Binding protein 2a) has less affinity for β lactam drugs, so MRSA strains are resistant to all β lactam drugs. Even combination of β lactam and β lactamase inhibitor will not work.
13. Ans. (a), (b) (Streptococcus pyogenes, Staphylococcus aureus)
Ref: Ananthanarayan 9 e/p202,213, 8 e/p197, 209, Topley 10/e p813, 857, Harrison 17/e p875, 886, Harrison 18/e p1166
  • Staph TSST1 (enterotoxin F / pyrogenic Exotoxin C) –causes vaginal tampon associated TSS
  • Staph enterotoxin B,C – Non menstrual cases of TSS
  • Streptococcus pyrogenic exotoxin A,B,C- can also cause TSS
Also know-
  • Streptococcal TSS – Bacteraemic, ↑soft tissue necrosis (necrotizing fasciitis), but rash less common
  • Staphylococcal TSS – Rash more common, bacteraemia less common, less tissue necrosis
14. Ans. (b) (Alteration in PBP is imp for gram -ve also) Ref: Koneman's diagnostic Microbiology 6/e p649–655
Penicillin resistance is due to
Mechanism of Beta lactam resistance
Shown by
Penicillinase production
Gram +ve & -ve bacteria
Altered PBP
Gram +ve bacteria
Altered permeability (active efflux pumps)
Gram -ve bacteria
15. Ans. (a) (PBP is localized in outer cell wall) Ref: Koneman's diagnostic Microbiology 6/e p649–655
  • PBP (Penicillin Binding protein or receptor)is localized in outer cell membrane (not in the cell wall).
  • PBP acts as transpeptidase or carboxypeptidase enzyme (essential for cell wall peptidoglycan synthesis).
  • Penicillin binds to PBP and inhibits transpeptidation thus inhibiting peptidoglycan synthesis.
  • In MRSA –Mec A gene alters PBP to PBP 2a which has less affinity for β lactam drugs.
16. Ans. (a) (MC source of infection–cross infection from infected persons in hospital)
Ref : Harrison 18 e/p1160, 17/e p873, 879
  • MC source of infection – cross infection from hands of hospital staffs
  • Most effective way to prevent the hospital infection – hand washing
  • 25–30% of general population is healthy carriers of S.aureus
  • MC site of colonization –anterior nares, Skin (perineum, axilla, groins)
  • E.g. of Staphylococcal super antigen- TSS toxin and Epidermolytic toxin and enterotoxin
  • MRSA- chromosomally mediated while Penicillinase production is plasmid mediated.
17. Ans. (c) (Staphylococcus epidermidis) Ref: Harrison 18 e/p1053, 17/e p790
Refer text (chapter review) for explanation.
18. Ans. (a) (S.aureus) Ref: Harrison 18 e/p1053, 17/e p790
Refer text (chapter review) for explanation.
19. Ans. (c), (d), (e) (Vancomycin is effective, MRSA are more virulent then sensitive strains, Resistance develop due to altered binding protein) Ref: Topley Wilson 10/e p801
Option a- MRSA strains are resistant to all β lactam antibiotics
Also remember- Ceftobiprole is the only β lactam affective against MRSA
Option b- Most but not all MRSA are multi drug resistant.
119Community associated MRSA don't carry non- β lactam antibiotics resistant genes where as hospital acquired MRSA carry many non- β lactam antibiotics resistant genes. Topley Wilson 10/e p801
Option c- Vancomycin is the drug of choice for MRSA
Option d- Is MRSA more virulent than MSSA...........?
Still it is under controversy and different school of thoughts have different opinions.
Plenty of journals and references are there who support and oppose these statements.
“Numerous clinical studies have indicated, based on mortality rates, that MRSA strains are more virulent than MSSA strains However, laboratory examinations of pathogenic mechanisms and virulence factors in strains of MRSA and MSSA have generated conflicting data, probably the heterogenic nature of the MRSA strains.”
………… Journal : Clin Microbiol Infect. 2007 Sep;13(9):843–5.
“Community associated MRSA possess Panton valentine gene which makes it more virulent…Jawetz 24 e/p226
So what I can conclude is,
  • Though some people don't agree completely as laboratory data are conflicting but based on mortality data and observation of possessing PV toxin by community-MRSA, You can bravely go with MRSA is more virulent th an MSSA’.
Option e– Mechanism of resistant in MRSA is due to altered penicillin binding protein which has low affinity for penicillin, coded by MecA gene.
20. Ans. (b) (Imipenem) Ref: Koneman's diagnostic Microbiology 6th /p649–655
  • MRSA strains are resistant to all β lactam antibiotics
  • Also remember- Ceftobiprole is the only β lactam affective against MRSA
    ………Journal: JW Infectious Disease, 2008
21. Ans. (c) (Expression of meth icillin resistance in Staphylococcus aureus increases when it is incubated at 37°C on blood agar).
Ref: Harrison 18 e/p1163, 17th /879 & Ananthanarayan 9th/201, 8 e/p197, Jawetz 24 e/p224 & 25 e/p185.
  • Option a - >75% of infection caused by CONS are due to S.epidermidis Jawetz 24 e/p224
  • Option b - β-lactamase production is plasmid mediated where as PBP2a (MecA) gene is chromosomal.
  • Option c- MRSA expression increases by 4% NaCl, incubation at 30°C for full 24 hours Ananthanarayan 8 e/p197
  • Option d - Altered PBP2a is the main mechanism of MRSA. However, hyper production of β-lactamase also contributes to borderline MRSA. (But Option ‘c’ is a better answer than ‘d’)
22. Ans. (c) (Toxin can be destroyed by boiling for 30 minutes) Ref: Ananthanarayan 9 e/p202, 8 e/p198
Staphylococcal enterotoxin is heat stable. So cooking the contaminated food and leaving at room temp for some time leads to toxin accumulation.
About Other Options
Option a- Optimum temperature for toxin production is same as Optimum temperature for S.aureus growth i.e. at 37°C. So it seems to be a correct statement.
Remember- Optimum temperature for pigment production is 22°C and for MRSA expression is 37°C
Option b- St.aureus enterotoxin is intradietic in nature i.e. it is preformed, secreted in the diet
Option d- IP of S.aureus food poisoning is 1–6 hour due to preformed toxin
23. Ans. (b) (Staphylococcus epidermidis) Ref: Ananthanarayan 9 e/p206,8 e/p202
  • Gram +ve cocci, Catalase+ve – Suggestive of Staphylococcus
  • Coagulase –ve, infection of CVP line - Suggestive of Staphylococcus epidermidis
24. Ans. (a) (is coagulase positive) Ref: Ananthanarayan 9 e/p204,8 e/p202, Mackie McCartney 14th /p247
  • Coagulase enzyme is only produced by S.aureus not by S.epidermidis
  • About Other Options
  • Both S.aureus and S.epidermidis can form white colony (Though S.aureus forms golden yellow colony but few strains can also produce white colony)
  • UTI is mainly caused by S.saprophyticus (S.aureus and S.epidermidis can also cause UTI)
  • Though S.aureus is the MC agent of endocarditis in drug addicts but even S.epidermidis also can cause it.
12025. Ans. (b) (Vagal stimulation) Ref: Jawetz 24 e/p226 & 25 e/p188, Harrison 17th /p875 & 18 e/p1165
Jawetz 25th/p188 -“The emetic effect of enterotoxin is probably the result of central nervous system stimulation (vomiting center) after the toxin acts on neural receptors in the gut.”
Harrison 18 e/p1165 - The toxin stimulates the vagus nerve and the vomiting center of the brain. It also appears to stimulate intestinal peristaltic activity.”
26. Ans. (c) (Common with dairy products) Ref: Ananthanarayan 9 e/p202,8 e/p198
“Staphylococcal food poisoning occurs toxin producing S.aureus strains colonizing in hands of food handlers leading to production of preformed toxin in food mainly meat, fish, milk or milk products.”
Also see the detail of the enterotoxin from the table of the text.
About Other Options
  • Option a- IP of S.aureus food poisoning is 1–6 hr not 24 hour
  • Option b- Antibiotics can't prevent the disease as it is due to preformed antitoxin
  • Option d- Staphylococcal enterotoxin is heat stable. So cooking the contaminated food and leaving at room temp for some time leads to toxin accumulation.
27. Ans. (a) (S.aureus) Ref: Ananthanarayan 9 e/p202,8 e/p198
Causes diarrhea with in 1–5 hours of intake of food- S.aureus and Bacillus cereus (Emetic type) Hence the other options can be excluded
28. Ans. (b), (c) (Infective wounds, Hands of hospital personnel) Ref: Ananthanarayan 9 e/p204,8 e/p200
  • MC source of Staph infection- carriers and patients
  • Animal and inanimate objects are less important. (Option a, d and e ruled out)
29. Ans. (a), (c), (d), (e) Ecth yma, Furuncle, Impetigo contagiosa, Sycosis barbae)
Ref: Ananthanarayan 9 e/p203, 8 e/p199, p239
  • Option b, c- S.aureus causes skin and soft tissue infection like folliculitis, furuncle, abscess, carbuncle, impetigo, paronychia and less often cellulitis.
  • Option e- Sycosis barbae means chronic folliculitis of beard hair follicle. (MC agent is S.aureus)
  • Option a- Ecthyma is a more serious form of impetigo in which the infection penetrates deeper into the skin's second layer, the dermis.
  • Remember- Ecthyma gangrenosum is due to Pseudomonas aeruginosa-Ananthanarayan 8 e/p316
  • Option b- Erythrasma is caused by Corynebacterium minutissimum.-Ananthanarayan 9 e/p242,8 e/p239
30. Ans. (a), (c), (e) (S.aureus, Pseudomonas, Strept. Pyogenes)
Ref: Jagdish Chander's mycology, 3rd /p158/table 11.6
Agent of Botryomycosis-
  • S.aureus (MC agent)
  • Others- Streptococcus, Pseudomonas, E.coli, Proteus, CONS, Peptostreptococcus, Streptococcus spp
31. Ans. (a) (Staphylococcus aureus) Ref: Harrison 18/e p1088, 17/e p877, 78 and B.Arora's Microbiology 3/e p701
  • Staphylococci can multiply at a wide range of temperatures; thus, if food is left to cool slowly and remains at room temperature after cooking, the organisms will have the opportunity to form enterotoxin.
  • Outbreaks following picnics where potato salad, mayonnaise, and cream pastries have been served offer classic examples of staphylococcal food poisoning.
  • Staphylococcal food poisoning has an incubation period of < 6hr.
32. Ans. (c) (80/81) Ref: Ananthanarayan 8/e p203
Staphylococcus aureus phage type 80/81 is the most common type associated with hospital infections.
33. Ans. (c) (Toxic shock syndrome) Ref: Ananthnarayan 9th /p202
  • Refer chapter review.

Streptococcus and PneumococcusCHAPTER 3.2

  • Catalase –ve, gram positive cocci arranged in chains
 
CLASSIFICATION
On the basis of Hemolysis, Streptococci can be divided into 3 groups:
  1. ∝ Haemolytic – (Partial or green hemolysis)- E.g. Str. Viridans, Str.pneumoniae
  2. β Haemolytic – (Complete or yellowish hemolysis) – E.g. β haemolytic Streptococcus
  3. γ Haemolytic – no hemolysis seen. E.g. Enterococci
Lance field's grouping (for β haemolytic Streptococci):
On the basis of group specific carbohydrate antigen in cell wall, the b haemolytic Streptococci are further divided in to 20 serological types A to V except I & J;
Carbohydrate Antigen extracted by –
HCl (Lancefield's method), Formamide (Fuller's method), Enzymatic (Maxted's) or autoclaving.
Griffith Typing (for Streptococcus Group A / Str. pyogenes):
Streptococcus Group A is subdivided based on proteins M protein (>80 M types are present)
Species
Lancefield Group
Typical Hemolysis
Important Lab Characteristics
S. pyogenes
A
Beta
Bacitracin – sensitive, PYR test positive
S. agalactiae
B
Beta
Bacitracin – resistant, PYR test negative
Hippurate hydrolysis positive, CAMP test positive
 
STREP. PYOGENES (GROUP A)
Cultural characters:
  • Transport media- Pike's media
  • Blood agar - Pinpoint colony with wide zone of β Haemolysis
  • Selective media:
    • Crystal violet- Blood agar
    • PNF medium (Polymyxin B Neomycin Fusidic acid)
  • Virulent colony produces matt like colony whereas avirulent colony produces glossy colony.
  • Liquid medium- produces granular turbidity (S.aureus in liquid medium produc- es uniform turbidity)
Virulence factor
Activity
Lipoteichoic acid
Mediates adherence to fibronectin receptors on epithelial cells
M protein
Mediates adherence to epithelial cells, inhibits phagocytosis
Hyaluronic acid capsule
Inhibits phagocytosis
Erythrogenic toxin (Streptococcal pyrogenic exotoxin-SPE)
3 Types (SPE A, B and C)- All are e.g. of Superantigens
Type A&C bacteriophage coded, Btoxin chromosomal
Dick test -ID injection produces erythema in susceptible individuals
Schultz Charlton reaction (blanching of rash after injection of antibodies)- diagnostic for scarlet fever
Streptolysin O
Produced by group A, C and G Streptococci
Oxygen labile, Serum stable
Active in reduced state, Inactive in oxidized form
Similar to Pneumolysin, tetanolysin, perfringolysin
It is strongly antigenic and rise of ASO >200u, indicates recent streptococcal infection (except pyoderma & glomerulonephritis)
Streptolysin S
Oxygen stable, Serum soluble
Hemolysis on the surface of an aerobic blood agar plate.
Not antigenic.
Streptokinase
Fibrinolysin (activate plasminogen)
Rapid spread- By preventing the formation of fibrin barrier.
Therapeutically used in treatment of coronary thrombosis.
Deoxyribonuclease
Streptodornanse (4 types- A, B,C,D)
Anti-DNAase B > 300–350u is useful for the retrospective diagnosis of skin infections (pyoderma) & glomerulonephritis where ASO is usually low Liquify thick pus. Applied therapeutically in empyema
(Streptodornanse and streptokinase combination is used)
Hyaluronidase
M type 4 and 22 (non capsulated)
Serum opacity factor
Lipoproteinase enzyme
Pathogenicity
 
Suppurative
Group A Streptococcus MC agent for:
  • Pharyngitis/sore throat - Associated with lower M types.
  • Scarlet fever:
    • Pharyngitis and Sandpaper rashes, strawberry tongue, Pastia's lines- prominent rashes in skinfolds
    • Due to erythrogenic toxin (Dick test +ve)
  • Impetigo (pyoderma):
    • Seen in chlidren, poor hygiene, warm climate,
    • Characterized by pustular lesion that becomes honeycomb like crusts, no fever, painless.
    • Associated with Higher M types, and nephritogenic strains
  • Cellulitis and Erysipelas – Bright red appearance of the involved skin, peau d'orange texture due to involvement of superficial lymphatics, MC site- malar area of the face. Seen in older people.
  • Necrotizing fasciitis / streptococcal hemolytic gangrene- 60% is due to Gr A Strep- tococcus (flesh eating bacteria)
Other Complications caused by Group A Streptococcus:
  • Puerperal sepsis (Gr B Streptococcus is MC cause)
  • Toxic shock syndrome (Bacteraemia is common in Streptococcus compared to Staphylococcal TSS)
  • Streptococcal myositis (St.aureus is MC cause of myositis), Pneumonia and Empyema
  • Otitis media, Quinsy, Ludwig's angina, meningitis, pneumonia (post viral).
Treatment
  • Pharyngitis- Benzathine penicillin G / penicillin V
  • Erysipelas/Cellulitis- mild- Procaine penicillin, severe- Penicillin G
  • Pneumonia- penicillin G + drainage of empyema
  • Necrotizing fasciitis- Surgical debridement (most crucial) + Penicillin G + Clindamycin
  • Streptococcal TSS- Penicillin G + Clindamycin + immunoglobulin
Treatment of asymptomatic carrier-
  • Penicillin V + Rifampicin- for pharyngeal carrier
  • Vancomycin + Rifampicin- for rectal carriers
 
123Non suppurative sequelae
  • Due to antigenic cross reactivity, antibody produced against previous Strep- tococcal infection antigens, cross reacts with human tissue to produces lesions. This accounts for non-suppurative complications (like acute rheumatic fever and glomerulonephritis).
Streptococcal Ag
Mammalian Ag
Disease
Hyaluronic acid
Synovial fluid
Reactive arthritis
Cell wall protein M
Myocardium
ARF(Acute rheumatic fever)
Cell wall Ccarbohydrate
Cardiac valves
ARF (Acute rheumatic fever)
Cytoplasmic membrane
Vascular intima
Acute glomerulonephritis (AGN)
Peptidoglycan
Skin antigens
Guttate psoriasis
Property
Acute rheumatic fever
AGN
Site
Throat
Skin> Throat
Prior sensitization
Essential
Not
Serotype
Any
Pyodermal- 49,53–55, 59–61
Pharyngitis – 1,12
Immune response
Marked
Moderate
Complement level
Unaltered
Low
Genetic susceptibility
Present
No
Repeated attack
Common
Not so
Penicillin prophylaxis
Indicated
Not so
Course
Progressive
Spontaneous resolution
Prognosis
Variable
good
 
GROUP B STREPTOCOCCI
S. AGALACTIAE CAUSES
  • Puerperal sepsis
  • Early onset and late onset neonatal disease
    • Early onset disease:
      • Occur in infants within 1 week of birth (mainly first 48hr),
      • Acquired by -Colonization during or before birth from the colonized maternal genital tract
      • Risk factors -Prematurity and maternal factors(prolonged labor)
      • Presentation – Similar to neonatal sepsis. MC- pneumonia and/or respiratory distress syndrome followed by meningitis. Typical findings include respiratory distress, lethargy, and hypotension.
    • Late-onset infections:
    • Occur in infants 1 week to 3 months of birth.
    • Acquired by- contact with a colonized mother, nursery personnel > during delivery
    • Meningitis is the most common manifestation of late-onset infection and in most cases is associated with a strain of capsular type III.
  • Presumptive identification:
    • CAMP +ve (98–100%): CAMP factor is a phospholipase produced by GrB Streptococcus that causes synergistic hemolysis with β-lysin produced by certain strains of S. aureus.
    • Hippurate hydrolysis test +ve (99%)
    • Bacitracin resistant (92%),
    • PYP & bile esculin negative (99–100%)
    • Pigment production
    • Capsulated- Nine capsular serotypes are found.
  • Treatment:
    • Penicillin is the agent of choice for all GBS infections. GBS is less sensitive to penicillin than GAS, requiring higher doses. Vancomycin is an acceptable alternative for penicillin-allergic patients.
 
124GROUP D STREPTOCOCCI
  • Differentiated from other Streptococci by ability to grow in presence of 40% bile and hydrolyze aesculin.
  • They can be further divided into Enterococci and Nonenterococci.
Cultural characters of Enterococci
  • Growth in presence of:
    • 6.5% NaCl,
    • 40% bile,
    • at pH 9.6 and
    • at 45°C.
  • Other important characteristic:
    • PYR +ve
    • Esculin hydrolysis positive
    • Usually nonhemolytic but may show α or β hemolysis).
Resistance in Enterococci:
  • Most strains are resistant to penicillin, aminoglycoside and sulfonamides.
  • Resistance-Overcome by Combination therapy with penicillin and aminoglyco- side (Synergistic)
  • Vancomycin is usually indicated in the resistant cases but resistance to vancomy- cin is also reported.
  • Vancomycin resistant Enterococci (VRE):
    • Mediated by Van gene -which alters the target site of vancomycin (i.e. D-Alanine D-Alanine side chain of peptidoglycan layer is altered to D-Ala- nine D-Serine or D-Alanine D-Lactate) and this altered side chain has less affinity for vancomycin.
    • Van C genotype is exhibited by E.casseliflavus and E.gallinarum - They show intrinsic resistant to vancomycin.
  • Most common species associated with human disease Enterococcus fecalis
  • More drug resistant species is - Enterococcus faecium
  • Disease- UTI, endocarditis, intra abdominal infection, meningitis.
Non-Enterococcus – e.g. S.bovis – Penicillin sensitive, inhibited by 6.5% NaCl, 40% bile, at pH 9.6 and at 45°C.
 
Viridans Streptococci
  • α hemolytic, mouth commensal,
  • S.mutans – causes dental caries,
  • S sanguis – causes endocarditis
  • S. milleri group- produces suppurative infections, differ in hemolytic pattern (may be α, β or nonhemolytic)
  • Treatment- Usually sensitive to penicillin (except neutropenic patients with bacteremia where vancomycin is given).
 
S.PNEUMONIAE
S pneumoniae
S viridians
Morphology
Lanceolate or flame shaped
Round / oval
Arrangement
Gram positive cocci in Pairs
Gram positive cocci in long chains
Capsule
Present (detected by Quellung reaction)
Absent
Blood agar at 48 hour
Draughtsman or carrom coin colony
Convex
Liquid Medium
Uniform turbidity
Granular turbidity
Bile solubility
Soluble in bile
Insoluble in bile
Inulin fermentation
Fermenter
Non fermenter
Optochin
Sensitive
Resistant
Animal Pathogenicity
Pathogenic
Non-pathogenic
125Antigenic structure
  • Capsular polysaccharide Antigen:
    • SSS (Specific soluble substance)
    • 90 capsular types- Detected by Quellung reaction when reacts to specific antisera mixed with methylene blue
  • C -Carbohydrate Antigen –So named because CRP precipitates with this Antigen
  • Pneumolysin-produces α hemolytic but under anaerobic condition – produces β hemolytic colonies
  • Autolysin
    • Amidase (autolytic in nature)
    • Cleaves peptidoglycan layer
    • Responsible for - draughtsman shape colony and bile solubility.
Pathogenicity
  • Can be Commensal in nasopharynx and throat- Pneumococcal carriage ranging from 20% to 50% for children <5 years of age and from 5% to 15% for young and middle-aged adults
  • Pneumonia (MC cause of pyogenic lobar pneumonia)
    • In adults – type 1 – 8 (most virulent type-3 followed by 7),
    • In Children – 6, 14, 19, 23
  • Meningitis (MC cause of meningitis)
  • Others – Otitis media (MC pneumococcal syndrome), sinusitis, peritonitis etc.
  • Penicillin resistance– due to altered PBP 2b (chromosomal).
Treatment
  • Meningitis- Vancomycin + Cefotaxime
  • Invasive infection excluding meningitis- Penicillin G
  • Otitis media- Amoxycillin.
Vaccine
  • Polysaccharide conjugate vaccine –
  • 23 valent, covers 90% of strains
  • Not useful for <2 year as it is T independent antigen.
  • Contraindicated - lymph reticular malignancy, pregnancy and children below 2 years
  • Indication-
    • ✓ Splenectomy,
    • ✓ Sickle cell patient,
    • ✓ Diabetes
    • ✓ Chronic heart/lung/renal/liver disease
    • ✓ Coeliac disease
    • ✓ Immunodeficiency.
Conjugated vaccine
  • 7 valent- covers 60% of strains
  • Conjugation with carrier molecule increases immunogenicity, so can be given < 2 year
126MULTIPLE CHOICE QUESTIONS
1. Catalase negative beta hemolytic streptococci is: (NEET Pattern Based)
  1. Str. pneumoniae
  2. Str. pyogenes
  3. Str. viridans
  4. Enterococcus
2. Boy presented with skin ulcer on leg. Culture reveals beta hemolytic Streptococci. Culture from school children with sore throat some days back also revealed beta hemolytic Streptococci. What is the characteristic which can tell both the strains are same or different? (AIIMS Nov 2010)
  1. C carbohydrate Ag
  2. M protein
  3. Emu protein
  4. Mec A gene
3. Lancefield grouping of streptococci is done by using: (AIIMS Nov 2007, June 1998)
  1. M Protein
  2. Group C peptidoglycan cell wall
  3. Group C carbohydrate antigen
  4. Staining properties
4. True about streptococcus: (PGI Dec 2003)
  1. Lancefield classification is based on M protein
  2. Group ‘G’ not found in human
  3. Group ‘B’ causes neonatal meningitis
  4. Group ‘C’ can be isolated from vaginal flora
  5. Group ‘D’ causes urinary tract infection
5. Lancefield classification is based on: (Recent Question of 2013)
  1. M protein
  2. T protein
  3. R protein
  4. Group C carbohydrate
 
GROUP A STREPTOCOCCUS
6. 11 Year old child presented with sore throat since 3 days, which medium is used to culture the throat swab: (NEET Pattern Based)
  1. Blood agar
  2. L. J. medium
  3. Stewart medium
  4. Chocolate agar
7. False regarding Streptococcus pyogenes: (NEET Pattern Based)
  1. Causes necrotizing fascitis
  2. Beta haemolytic
  3. M protein is virulence factor
  4. Resistant to bacitracin
8. Necrotizing fascitis is caused by: (NEET Pattern Based)
  1. Staphylococcus aureus
  2. Beta hemolytic streptococci
  3. Clostridium perfringens
  4. Pneumococcus
9. All are true about streptococcus pyogenes except: (AIIMS May 2011)
  1. Streptolysin O is produced by Type A,C and G
  2. Streptodornanse depolymerizes DNA
  3. Streptolysin O is hemolytically active in reduced state
  4. Pyrogenic exotoxin A production is plasmid mediated
10. Which streptococcal antigen cross reacts with synovial fluid? (AI 2008)
  1. Carbohydrate (group A)
  2. Cell wall protein
  3. Capsular hyaluronic acid
  4. Peptidoglycan
11. A child presents with infective skin lesion of the leg. Culture showed hemolytic colonies which were gram + ve cocci in chains. The test to confirm the organism is: (AI 2012, AIIMS Nov 2006; AIIMS Nov 2011, AIIMS May 2007, AI 2007)
  1. Bile solubility
  2. Optochin sensitivity
  3. Bacitracin sensitivity
  4. Catalase positive
12. Treatment for streptococcal necrotizing fasciitis: (PGI June 2008)
  1. Surgical debridement
  2. Penicillin
  3. Clindamycin
  4. Metronidazole
  5. Vancomycin
13. An outbreak of Streptococcal Pharyngitis occurred in a village. In order to carry out the epidemiological investigations, it is necessary to perform the culture of the throat swab of the patients suffering from the disease. The transport media of choice would be: (AIIMS Nov 2004)
  1. Salt mannitol media
  2. Pike's media
  3. Stuart's media
  4. Cary Blair media
14. Streptococcal Toxic shock syndrome is due to the following virulence factor: (AIIMS May 2003, Dec 1998, AI 2002)
  1. M protein
  2. Pyrogenic exotoxin
  3. Streptolysin O
  4. Carbohydrate cell wall
15. The commonest organism causing cellulites is: (AIIMS Nov 2002)
  1. Streptococcus pyogenes
  2. Streptococcus faecalis
  3. Streptococcus Viridans
  4. Microaerophilic streptococci
16. All are true about streptococcus except: (AI 2001)
  1. M protein is responsible for production of mucoid colonies
  2. M protein is responsible for virulence
  3. Mucoid colonies are virulent
  4. No resistance to penicillin has been reported
17. Group A hemolytic pharyngitis is due to: (DNB June 2010)
  1. Local infection
  2. 127Systemic toxicity
  3. Attachment to mucosa
  4. Local toxins
 
GROUP B STREPTOCOCCUS
18. A child presents with sepsis. Bacteria isolated showed β hemolysis on blood agar, resistant to bacitracin and a positive CAMP test. The most probable organism is: (AI 2001, 2010)
  1. S.pyogenes
  2. S.agalactiae
  3. Enterococcus
  4. Pneumococcus
19. Neonatal meningitis acquired during passage through birth canal is due to: (TN 2002)
  1. Streptococcus agalactiae
  2. S. equisimilus
  3. S.pyogenes
  4. Pnemococci
20. Which of the following organism shows positive CAMP test: (Recent Question of 2013)
  1. Group A Streptococcus
  2. Group B Streptococcus
  3. Group C Streptococcus
  4. Group D Streptococcus
 
ENTEROCOCCUS
21. Which group of streptococcus grow at 60 C: (NEET Pattern Based)
  1. A
  2. B
  3. C
  4. D
22. Not true about Enterococcus: (AI 2008)
  1. Common species are E. faecium and faecalis
  2. Causes peritonitis
  3. Universally susceptible to penicillin
  4. Causes intraabdominal abscess
23. False about gram +ve cocci is: (AI 2008)
  1. Staph. saprophyticus causes UTI in females
  2. Most Enterococci are sensitive to penicillin
  3. Non pathogenic strains are coagulase negative
  4. Neonatal meningitis causing Streptococci hydrolyses hippurate
24. A patient admitted to an ICU is on central venous line for the last one week. He is on ceftazidime and amikacin. After 7 days of antibiotics he develops a spike of fever and his blood culture is positive for gram positive cocci in chains, which are catalase negative. Following this, vancomycin was started but the culture remained positive for the same organism even after 2 weeks of therapy. The most likely organism causing infection is: (AIIMS Nov 2011, AIIMS May 2006, AIIMS Nov 2006, AI 2007)
  1. Staphylococcus aureus
  2. Viridans streptococci
  3. Enterococcus faecalis
  4. Coagulase negative Staphylococcus
25. Which of the following organisms, when isolated in the blood, requires the synergistic activity of penicillin plus an aminoglycoside for appropriate therapy: (AIIMS Nov 2004)
  1. Enterococcus faecalis
  2. Staphylococcus aureus
  3. Staphylococcus pneumoniae
  4. Bacteroides fragilis
26. 45 years old Ramlal has intra abdominal sepsis. The causative organism was Vancomycin, gentamycin and ampicillin resistant. It grows in presence of 6.5% NaCl and arginine. Bile esculin hydrolysis is +ve. Which of the following is this organism? (AI 2001, AIIMS Nov 2001)
  1. Streptococcus agalactiae
  2. Enterococcus faecalis
  3. Streptococcus bovis
  4. S.pyogenes
 
STREPTOCOCCUS VIRIDANS
27. A patient of RHD developed infective endocarditis after dental extraction. Most likely organism causing this is: (AIIMS Nov 01)
  1. Streptococcus Viridans
  2. Streptococcus pneumoniae
  3. Streptococcus pyogenes
  4. Staphylococcus aureus
 
PNEUMOCOCCUS
28. A person presents with pneumonia. His sputum was sent for culture. The bacterium obtained was gram positive cocci in chains and alpha haemolytic colonies on sheep agar. Which of the following will help in confirming the diagnosis: (AIIMS May 2012)
  1. Novobiocin
  2. Optochin
  3. Bacitracin
  4. Oxacillin
29. The following statements are true regarding Streptococcus pneumoniae except: (AI 2011)
  1. It is bile-sensitive
  2. The capsule of S.pneumoniae allows establishment of infection
  3. It is an etiological agent of pneumonia and otitis media
  4. Pneumococcal meningitis is the least virulent of the major bacterial meningitides
30. 65 years old patient presented to the emergency with high grade temperature and increased respiratory rate. He complained of pain in the chest and had developed cough with expectoration. His sputum was sent to the laboratory for gram straining which showed the presence of pus cells and gram positive cocci in pair. The culture on the blood agar medium was also positive. Which of the following laboratory tests will help to differentiate the specific pathogen from the other commensal gram positive cocci? (AIIMS Nov 2009)
  1. Bacitracin sensitivity
  2. Catalase test
  3. Bile solubility
  4. Coagulase test
31. ‘C’ in C reactive protein stands for: (AI 2011)
  1. Capsular polysaccharide in Pneumococcus
  2. Concanavalin-a
  3. Calretinin
  4. C- carbohydrate antigen
32. 128An infant had high grade fever and respiratory distress at the time of presentation to the emergency room. The sample collected for blood culture was subsequently positive showing growth of α haemolytic colony. On Gram staining these were gram positive cocci. In the screening test for identification, the suspected pathogen is likely to be susceptible to the following agent: (AI 2007, AIIMS May 2006)
  1. Bacitracin
  2. Novobiocin
  3. Optochin
  4. Oxacillin
33. The sputum specimen of a 70 year old male was cultured on a 5% sheep blood agar. The culture showed the presence of
α-haemolytic colonies next day. The further processing of this organism is most likely to yield: (AIIMS Nov 2005)
  1. Gram positive cocci in short chains, catalase negative and bile resistant
  2. Gram positive cocci in pairs, catalase negative and bile soluble
  3. Gram positive cocci in clusters, catalase positive and coagulase positive
  4. Gram negative coccobacilli catalase positive and oxidase positive
34. In a splenectomized patient there is increase of infection by all the organisms except :
  1. Pneumococci (NEET Pattern Based, PGI 2000, SGPGI 2005)
  2. Thlebsiella
  3. H. influenzae
  4. Staph. aureus
35. Streptococcus pneumoniae true is: (SGPGI 2005)
  1. Vaccine is made from capsular polysaccharide
  2. Vaccine is routinely given to Indian children
  3. Catalase and Oxidase positive
  4. Bile insoluble and Optochin sensitive
36. Most common causative organism for lobar pneumonia is: (AIIMS 2004)
  1. Staphylococcus aureus
  2. Streptococcus pyogenes
  3. Streptococcus pneumoniae
  4. Haemophilus influenzae
37. Polysaccharide capsule resistance to phagocytosis is seen in (NIMHANS 2001, JIPMER 1990)
  1. N.meningitis
  2. Th.pneumoniae
  3. Streptococcus pneumoniae
  4. All
38. In a patient of orbital cellulitis, organism on culture show greenish colonies and Optochin sensitivity. The most likely organism is: (AI 2000)
  1. Streptococcus Viridans
  2. Staphylococcus
  3. Pseudomonas
  4. Pneumococcus
39. The most common organism causing acute otitis media is: (MHPG 2014)
  1. H. influenza
  2. S. pneumoniae
  3. M. catarhalis
  4. S. aureus
129EXPLANATIONS
1. Ans. (b) (Str. pyogenes) Ref: Ananthanarayan 9/e p209, 8/e p205
  • Str. Pyogenes- Catalase negative and beta hemolytic
  • Str. Pneumonia and Str. Viridians- Catalase negative and alfa hemolytic
  • Enterococcus - Catalase negative and non hemolytic.
2. Ans. (a) (C-carbohydrate Ag) Ref: Ananthanarayan 9/e p209, 8/e p205, Jawetz 24/e p636 and 25/e p195
  • β hemolytic Streptococci was obtained from two cases-Boy with skin ulcer and children with sore throat.
  • To know the relatedness between Beta hemolytic Streptococci, Lancefield grouping is done
  • Lancefield grouping is based on C-carbohydrate Ag.
Classification of Streptococcus- Refer text (chapter review) for explanation.
3. Ans. (c) (Group C carbohydrate antigen) Ref: Ananthanarayan 9/e p209, 8/e p205, Jawetz 24/e p636 and 25/e p195
  • β hemolytic Streptococcus is classified to group A-V based on C carbohydrate antigen (Lancefield classification)
  • M protein is used to further classify group A Streptococcus (Griffith typing).
4. Ans. (c), (e) (Group ‘B’ causes neonatal meningitis, Group ‘D’ causes urinary tract infection)
Ref: Ananthanarayan 9/e p209, 216, 217 and 8/e p205, 213, 215, Jawetz 24/e p636 and 25/e p195
  • Option a- Lancefield classification based on C carbohydrate antigen, NOT M protein
  • Option b- Group ‘G’ are normal flora of throats of human being, occasionally cause tonsillitis, UTI
  • Option c- Group ‘B’ i.e. S.agalactiae causes neonatal meningitis, can be isolated from vaginal flora
  • Option d- Group C is predominantly animal pathogen, occasionally causes URTI in humans.
  • Option e- Enterococci belongs to Group D Streptococci and are commonly involved in UTI, endocarditis and abdominal infection.
5. Ans. (d) (Group C carbohydrate) Ref: Ananthanarayan 8/e p205 and 9/e p209
  • Lancefield classification is based on- Group C carbohydrate
  • Griffith typing is based on-M protein
 
GROUP A STREPTOCOCCUS
6. Ans. (a) (Blood agar) Ref: Ananthanarayan 9/e p213, 8/e p208
  • Streptococcus pyogenes is the MC cause of sore throat. Blood agar is used for isolation of S.pyogenes.
  • Crystal violet blood agar is the selective media and Pikes media is the transport media for S.pyogenes.
7. Ans. (d) (Resistant to bacitracin) Ref: Ananthanarayan 9/e p213, 8/e p208
  • Streptococcus pyogenes (Group A) is sensitive to bacitracin where as Str.agalactiae (Group B) is resistant to bacitracin.
8. Ans. (b) (Beta hemolytic streptococci) Ref: Ananthanarayan 9/e p213, 8/e p208
  • Streptococcus pyogenes is the causative agent of necrotizing fasciitis.
9. Ans. (d) (Pyrogenic exotoxin A production is plasmid mediated) Ref: Ananthanarayan 9/e p213,8th /p208
“Streptococcal Pyrogenic exotoxin A and C production is bacteriophage mediated where as Streptococcal Pyrogenic exotoxin B is chromosomally mediated.”
About Other Options
  • Streptolysin O and S are produced by Group A, C and G
  • Streptodornanse is chemically DNAse in nature and depolymerizes DNA
  • Streptolysin O is inactive in oxidized form but active in reduced form.
10. Ans. (c) (Capsular hyaluronic acid) Ref: Ananthanarayan 9/e p212, 8/e p208
  • Capsular hyaluronic acid cross reacts with synovial fluid
  • Structural components of Strepto pyogenes cross reacts with human tissues-Refer text (chapter review).
13011. Ans. (c) (Bacitracin sensitivity) Ref: Ananthanarayan 9/e p215, 8/e p206
  • Hemolytic colony with gram positive cocci in chain is suggestive of β hemolytic Streptococcus.
  • Option c- Bacitracin sensitivity is used to differentiate between β hemolytic Streptococci Group A- Bacitracin sensitive and Group B- Bacitracin resistant
  • Option a and b- Bile solubility and Optochin sensitivity are used to differentiate Pneumococcus and S. Viridans
  • Option d- Catalase test is used to differentiate Streptococcus and Staphylococcus.
12. Ans. (a), (b), (c) (Surgical debridement, Penicillin, Clindamycin) Ref: Harrison 17th/p886 and 18/e p1176
  • Treatment of Necrotizing fasciitis- Surgical debridement (most crucial) + Penicillin G + Clindamycin
  • If single option to be selected – then Answer should be Surgical debridement
  • Harrison 17/e p 886 states- “Drainage and debridement are central to the management of necrotizing fasciitis; antibiotic treatment is a useful adjunct, but surgery is life-saving”.
13. Ans. (b) (Pike's media) Ref: Ananthanarayan 9/e p215, 8/e p211
  • The transport media for Streptococcus is -Pike's media (blood agar + crystal violet + Na azide)
  • Selective media for Streptococcus is - crystal violet blood agar.
About Other Options-
  • Salt mannitol media - Selective media Staphylococcus aureus
  • Stuart's media - transport media for Gonococcus
  • Cary Blair media- transport media for enteric pathogens like Vibrio.
14. Ans. (b) (Pyrogenic Exotoxin) Ref: Ananthanarayan 9/e p213, 8/e p208
  • Streptococcal Pyrogenic Exotoxin A,B,C – responsible for Toxic shock syndrome
  • Principle virulence factor for Streptococcus – M protein.
15. Ans. (a) (Streptococcus pyogenes) Ref: Internet source-PDR health/disease and condition
  • This is again a controversial
  • The most common cause of cellulitis is - Group A streptococcus
  • Staphylococcus aureus, also causes cellulitis, and is second in occurrence to Group A streptococcus.
16. Ans. (a) (M protein is responsible for production of mucoid colonies) Ref: Ananthanarayan 9/e p212, 8/e p206, 207
  • Option a, c- Capsule (NOT M protein) is responsible for production of Mucoid colony, corresponding to viru- lence.
  • Option b- M protein is also responsible for virulence by inhibiting phagocytosis.
  • Option d- No resistance to penicillin has been reported for Group A Streptococcus, but for other Streptococcus reports of resistance to penicillin has been reported.
17. Ans. (c) (mucosal attachment) Ref: Harrison 18/e p1172
The capsular polysaccharide may also play a role in GAS colonization of the pharynx by binding to CD44, a hyalu- ronic acid–binding protein expressed on human pharyngeal epithelial cells.
 
GROUP B STREPTOCOCCUS
18. Ans. (b) (S.agalactiae) Ref: Ananthanarayan 9/e p216, 8/e p207, Jawetz 25/e p202 and 24/e p240
    • Already explained
  • Points favoring to S.agalactiae:
    • Septicemia in a child
    • β hemolysis on blood agar
    • Resistant to bacitracin and a positive CAMP test
  • S.pyogenes- sensitive to bacitracin and CAMP test is negative.
19. Ans. (a) (Streptococcus agalactiae) Ref: Ananthanarayan 9/e p216,8/e p207, Jawetz 24th /p240 and 25/e p202
  • Already explained.
20. Ans. (b) (Group B Streptococcus) Ref: Ananthanarayan 8/e p215 and 9/e p217
CAMP test is positive for- Group B Streptococcus, Listeria, Clostridium perfringens (reverse CAMP +ve), Vibrio Eltor.131
 
ENTEROCOCCUS
21. Ans. (d) (D) Ref: Ananthanarayan 8/e p206
  • Group D Streptococci like Enterococci can grow > 60°c.
22. Ans. (c) (Universally susceptible to penicillin) Ref: Gillespie's Bacteriology 2/e p65
  • Enterococci shows intrinsic resistance to Penicillin, low level aminoglycoside – overcome by combination of Penicillin and aminoglycoside
  • Enterococci are the normal flora of GIT, so any breach in GIT mucosa leads to spillage causing peritonitis and intraabdominal abscess
  • E. faecalis is the most common species followed by E. faecium encountered in clinical specimen.
23. Ans. (b) (Most Enterococci are sensitive to penicillin) Ref: Gillespie Bacteriology 2/e p65 Enterococci show intrinsic resistant to Penicillin
24. Ans. (c) (Enterococcus faecalis) Ref: Gillespie Bacteriology 2/e p65
This is case of VRE (Vancomycin resistant Enterococci)
Points in favor:
  • Gram+ve cocci in chain and Catalase –ve points towards Streptococcaceae family
  • Resistant to aminoglycoside, cephalosporins and vancomycin
  • S, aureus and CONS are catalase +ve, hence they are ruled out.
25. Ans. (a) (Enterococcus faecalis) Ref: Gillespie Bacteriology 2nd /p65
Resistance in Enterococci
  • Intrinsic resistance to Penicillin, low level aminoglycoside – overcome by combination of Penicillin and amino- glycoside
  • Penicillin resistance is due to PBP gets altered to PBP-5
  • Intrinsic resistance to cotrimoxazole and Vancomycin (VanC type in Ent. casseliflavus / gallinarum)
  • Acquired resistance to high level aminoglycoside, chloramphenicol, tetracycline, Vancomycin, quinolone, macrolide
  • Mechanism of low level aminoglycoside resistance (MIC < 2000µg/ml)- due to limited drug intake
  • Mechanism of high level aminoglycoside resistance (MIC ≥ 2000µg/ml)- due to acquiring aminoglycoside modifying enzymes
  • Enterococcus faecium is more resistant than Enterococcus faecalis.
26. Ans. (b) (Enterococcus faecalis) Ref: Gillespie Bacteriology 2/e p65
Points in favor-
  • Intraabdominal sepsis.
  • Vancomycin, Gentamicin and ampicillin resistant
  • Grows in presence of 6.5% NaCl and arginine, Bile esculin hydrolysis is +ve.
Group D Streptococci - can be further divided into Enterococci and Nonenterococci.
  • Enterococci - Growth in presence of 6.5% NaCl, 40% bile, at pH 9.6 and at 45°C.
  • Nonenterococci – cannot grow in presence of 6.5% NaCl, 40% bile, at pH 9.6 and at 45°C.
 
STREPTOCOCCUS VIRIDANS
27. Ans -a (Streptococcus Viridans) Ref: Harrison 17/e p790, 18/e p1052–53
  • MC cause of Native valve endocarditis – S.aureus
  • MC cause of Sub acute endocarditis – MC agent Streptococcus Viridans
  • MC Streptococcus Viridans causing endocarditis. -S. sanguis
  • Following tooth extraction, transient bacteremia occurs and Streptococcus Viridans gets lodged into pre damage valve.
  • So, prophylactic antibiotic is implemented before tooth extraction.132
 
PNEUMOCOCCUS
28. Ans. (b) (Optochin) Ref: Ananthanarayan 9/e p222, 8/e p220
  • α haemolytic colonies and gram positive cocci – isolated from a sputum of a patient with fever and respiratory distress - Suggestive of :
    • ✓ Pneumococcal Pneumonia or
    • ✓ Pneumonia due to Str.viridans
  • To differentiate Pneumococcus from Str.viridans- Optochin sensitivity test is done.
29. Ans. (d) (Pneumococcal meningitis is the least virulent of the major bacterial meningitis)
Ref: Ananthanarayan 9/e p223, 8/p219–222
Among the major bacterial meningitides, Pneumococcus is one of the most virulent organism
About Other Options
Pneumococcus is-
  • Capsulated – principle virulence factor
  • Bile soluble
  • MC cause of lobar pneumonia and otitis media.
30. Ans. (c) (Bile solubility) Ref: Ananthanarayan 9/e p222, 8/e p219
Points in favor-
  • Fever and ↑respiratory rate, chest pain and cough with expectoration – Suggestive of Pneumonia
  • Gram positive cocci in pair was isolated from Sputum - Suggestive of Pneumococcus
  • Test to differentiate Pneumococcus from commensal like Strept Viridans- Bile solubility
  • Refer text for differentiating properties between Pneumococcus and S.viridans.
    It can be differentiated from other commensal in sputum i.e. Strept Viridans:
    • ✓ Optochin sensitivity (Ethyl hydrocuprein)
    • ✓ Bile solubility
  • Out of this, Bile solubility is a better Option because:
    • ✓ Few strains of Strept Viridans also can be sensitive to Optochin while few strains of Pneumococcus can be resistant.
    • ✓ Bile solubility is a constant property of Pneumococcus, hence is of diagnostic importance.
      ……..Ananthanarayan 8th /p219
31. Ans. (d) (C- carbohydrate antigen) Ref: Ananthanarayan 9/e p222, 8th /p221
C reactive protein:
  • Acute phase substance produced by hepatocyte
  • Produced in response to in any inflammatory condition, any bacterial infection, tissue destruction
  • It disappears once the inflammatory condition subsides
  • So named, because it is found to react with C- carbohydrate antigen of Pneumococcus
  • But It is not an antibody and its apparent antibody like relationship with ‘C’ antigen is only fortuitous
  • CRP – detected by Latex agglutination test
  • Used as an index of response to treatment of acute rheumatic fever and other conditions.
32. Ans. (c) (Optochin) Ref: Ananthanarayan 9/e p221, 8/e p220
  • α haemolytic colonies and gram positive cocci – isolated from a sputum of a patient with fever and respiratory distress - Suggestive of Pneumococcal Pneumonia
    Optochin sensitivity
    • ✓ Done to differentiate Pneumococcus from commensal like Strept Viridans
    • ✓ Optochin sensitivity (Ethyl hydrocuprein) - 5µg disc is used
    • ✓ Pneumococcus- Sensitive
    • ✓ Strept Viridans- Resistant.
33. Ans. (b) (Gram positive cocci in pairs, catalase negative and bile soluble) Ref: Ananthanarayan 9/e p222, 8/e p219
  • α haemolytic colonies grown from a sputum of a patient with fever and respiratory distress – Might be pointing towards Pneumococcal Pneumonia
  • Pneumococcus will be-Gram positive cocci in pairs, Catalase negative and Bile soluble.133
34. Ans. (d) (Staph. aureus) Ref: Ananthanarayan 9/e p18, 200, 8th /p111, 120, 219
  • Capsulated organism can resist killing by preventing phagocytosis.
  • Spleen has an important role in removing these capsulated organisms where opsonization takes place.
  • Certain complements (opsonins) can attach to these organisms and once delivered to spleen, their uptake is facilitated by complement receptors like CR1, 2 etc present on splenic phagocyte surface (Process known as Opsonization).
  • Hence, in a splenectomized patient there is increase risk of infection by capsulated organisms.
  • Among the options, all are capsulated except S.aureus.
35. Ans. (a) (Vaccine is made from capsular polysaccharide) Ref: Ananthanarayan 9/e p225, 8/e p219–23
  • Pneumococcal Vaccine is made from capsular polysaccharide
  • Vaccine is not routinely given but given to certain high risk groups.
  • Pneumococcus- Catalase and Oxidase negative
  • Pneumococcus- Bile soluble and Optochin sensitive.
36. Ans. (c) (Streptococcus pneumoniae) Ref: Ananthanarayan 9/e p222, 8/e p219
  • Streptococcus pneumoniae is the Most common causative organism for lobar pneumonia
  • Adult- MC serotype associated - Type 1–8
  • Children- Type 6,14,19,23 are frequently associated serotypes
  • Most Virulent- Type 3.
37. Ans. (d) (All) Ref: Ananthanarayan 9/e p18,225, 8/pe 219
  • All capsulated organisms can resist phagocytosis.
  • Polysaccharide capsule seen in – Pneumococcus, N. meningitidis, Klebsiella.
38. Ans. (d) (Pneumococcus) Ref: Ananthanarayan 9/e p222, 8/e p219
  • Greenish colonies on blood agar and Optochin sensitivity- Suggestive of Pneumococcus.
39. Ans. (b) (S. pneumoniae) Ref: Ear Infections/CDC Website
  • Streptococcus pneumoniae followed by Haemophilus influenzae and Moraxella catarrhalis are the most common bacterial isolates from the middle ear fluid of children with acute otitis media.

NeisseriaCHAPTER 3.3

Catalase and oxidase +ve Diplococci
N. meningitidis
N. gonorrhoeae
Capsulated
Non- Capsulated
Lens shaped/Half moon shaped (Diplococci with adjacent sides flattened)
Kidney shaped
Ferment Glucose and Maltose
Only ferments Glucose
Rarely have plasmids
Usually possess plasmid coding drug resistance genes
Exists in Both intra and extracelular forms
Predominantly intracellular form
 
N. MENINGITIDIS (MENINGOCOCCUS)
 
Antigenic Determinants and Virulence Factors
  • Capsular polysaccharide → inhibits phagocytosis, typed to 13 serogroups
    • MC serogroup causing disease → A B, C, Y, 29E, W135
    • Serogroups are further classified based on OMP.
  • Endotoxin and LOS (Lipo oligo saccharide)
    • It differs from LPS of Enterobacteriaceae by lacking the repetitive O side chain
    • Produce generalized Shwartzman reaction and Waterhouse – Friderichsen Syndrome
  • IgA 1 protease
  • Pili → allow to adhere and antiphagocytic
  • OMP → Outer Membrane Protein.
 
Pathogenicity
  • MC group associated-
    • Group A and E- Cause epidemic disease mainly in sub-Saharan Africa
    • Group A, B, C, Y, W 135- Cause Sporadic cases worldwide;
    • Group B has a propensity to cause hyperendemic disease
  • Humans- only reservoir
  • MC source – human carriers
  • Nasopharyngeal carrier-
    • Children mainly - 5-10% (inter epidemic),
    • 70-80% (during epidemic)
  • Mode of transmission- Droplet infection
  • MC Route- hematogenous (MC) followed by olfactory N and conjunctiva
  • Deficiency of terminal complement components (C5- C9): increase risk of Neisseria and Toxoplasma infection
  • Outbreak in Hajji Pilgrims in 1990 – due to group A, W135
  • High prevalence area– sub-Saharan belt of Africa
  • Seasonal variation- MC in winter spring
  • Risk factors- Overcrowding, Smoking, viral and Mycoplasma infection of respiratory tract
  • Case fatality ratio is 80% (falls to 10% if early treatment is started).
 
Clinical Feature
  • Rashes: A non-blanching rash (petechial or purpuric) develops in >80% of cases.
  • Fulminant meningococcemia:
    • Septic shock, DIC, rashes (Purpura fulminans), multi organ failure
    • Water House – Friderichsen Syndrome (Rashes, shock, DIC, B/L adrenal hemorrhage)
    • Occurs due to endotoxin/LOS (Not capsule)
  • 135Meningitis- common in 3-5 year age
  • Chronic Meningococcemia- Repeated episodes of petechial rash, fever, arthritis, and splenomegaly
  • Postmeningococcal Reactive Disease- Immune complex disease develops 4–10 days later, with manifestations like arthritis, rash, iritis, pericarditis, polyserositis and fever.
 
Diagnosis
  • Culture media – Chocolate agar, Mueller Hinton agar, Thayer martin media - 35°c at 5-10% CO2
  • Transport media- Stuart's medium
  • Sample – CSF, Blood (early stage) and nasopharyngeal swab (for carriers)
  • LDH and neuraminidase in CSF
  • Antibody – retrospective evidence only.
 
Treatment
  • Cefotaxime and ceftriaxone → DOC for treatment
  • Rifampicin (DOC), ciprofloxacin – for carriers and prophylaxis.
 
Vaccine
Polyvalent vaccine containing → A,C,Y, W-135 immunity lasts for 3 years
  • Dose-50 µg single dose, immunity starts in 10days, lasts for 3 years
  • Group A meningococcal polysaccharides vaccine -effective in preventing disease at all ages
  • Vaccination to travelers like Hajj pilgrimage
  • No vaccine for Group B:
    • As Group B capsule is poorly immunogenic and
    • Group B capsule is made up Sialic acid residue in capsule cross react with human brain tissue (fatal autoimmune consequences)
  • Not useful – below 3 year (capsule is poorly immunogenic < 3 year as it is T inde- pendent Antigen)
  • C/I- pregnancy.
 
N. GONORRHEA (GONOCOCCUS)
  • Based on Pili divided to - 4 types (T1-T4)
  • Intra species typing- Auxotyping(Typing based on nutritional requirement)
  • Incubation Period- 2–8 days.
 
Antigenic Structure
  • Pili - imp role in attachment, are antiphagocytic
  • Outer membrane protein – protein I, II, and III
    • Protein I→ Serves as a basis for serotyping
    • Protein II → associated with adherence, present in opaque colony (opacity associated OMP)
  • IgA 1 protease – also present in Meningococcus, Pneumococcus, H.influenzae
  • Lipo-Oligosaccharide (LOS) → Toxicity is due to endotoxic effect
  • Transferrin and lactoferrin binding protein.
 
Clinical Feature
  • Babies born to infected mother:
    • Ophthalmianeonatorum– Occursinfirst 2-5days(48hrs), purulentandcrusted discharge,
    • Chlamydia (after 1st week, mucous discharge, MC cause)
  • Causes septic arthritis in adult (MC knee joint)
  • Male- MC manifestation is urethritis(doesn't involve testes)
  • Female- cervicitis(adult vagina is resistant to Gonococcus, so Less severe in female)
  • Pharyngeal Gonorrhea - following oral-genital sexual exposure
  • Anorectal Gonorrhea- in homosexuals
  • 136DGI (deep gonococcal Infection):
    • Mainly polyarthritis (rarely endocarditis)
    • Risk factors-Menstruation and late complement deficiencies
    • MC associated with PorB.1A serotype expressing AHU auxotype- (Require arginine, hypoxanthine, and uracil).
  • Complication – Watercan perineum, Fitz Hugh Curtis syndrome (perihepatitis),
  • MC seen in people with Blood group B.
 
Lab Diagnosis
Sample:
  • Urethral discharge, endocervical swab (high vaginal swab not recommended), rec- tal swab (if h/o anal sex)
  • Blood culture and synovial fluid culture- for DGI.
Transport media
  • Charcoal impregnated swabs/ medium (Stuart / Amies media)
  • For longer holding period- CO2 generating system (JEMBEC system).
Gram staining:
  • For males: Gram staining of urethral discharge is more sensitive (90%)-Based on which treatment can be started
  • For females: Gram staining is less sensitive (50-60%) due to presence of commensal Neisseriaspp. ingenitaltractmayconfoundwiththeinterpretation, So, Endocervical culture is recommended.
Culture media in acute gonorrhoeae: Chocolate agar and Mueller Hinton agar
Selective media are useful in chronic cases:
  • Gonococci are difficult to grow than Meningococci
  • Thayer Martin (VCN- Vancomycin, Colistin, Nystatin)
  • Modified Thayer Martin (VCNT- VCN + Trimethoprim)
  • Modified New York city medium
  • Martin Lewis Media.
 
Treatment
  • DOC- Single-dose regimen of Cefotaxime and ceftriaxone
  • Treat both the partners, also include azithromycin for Chlamydia
  • Mostly resistant to Penicillin due to Penicillinase production.
137MULTIPLE CHOICE QUESTIONS
 
MENINGOCOCCUS
Pathogenesis and diagnosis
1. Waterhouse friedrichsen syndrome is a complication seen in infection with: (TNPG 2014)
  1. Neisseria gonorrhoeae
  2. Neisseria meningitidis
  3. Escherichia coli
  4. Mycobacterium tuberculosis
2. A woman is having sore throat with high grade fever with headache, nausea and vomiting. On examination, she is having RR of 36/min, temperature of 39 degrees and BP of 80/50 mm Hg. On her arm some red spots are seen distal to BP cuff. The probable diagnosis is: (AIIMS May 2013)
  1. N. meningitidis
  2. Brucella suis
  3. Brucella abortus
  4. Staph. aureus.
3. Which among the following differentiates Neisseria meningitidis from Neisseria gonorrhoeae? (NEET Pattern Based, DNB Dec 2010)
  1. It is oxidase positive
  2. It ferments glucose
  3. It ferments maltose
  4. It reduces nitrates
4. Waterhouse-Friderichsen syndrome is seen in: (NEET Pattern Based)
  1. Pneumococci
  2. Pseudomonas
  3. N. meningitides
  4. Yersinia
5. Which of the following is not found in meningococci? (DNB June 2009)
  1. Peptidoglycan
  2. Plasmid
  3. Mitochiondria
  4. Capsule
6. Virulence factors for Meningococci: (PGI Dec 2007)
  1. Capsule
  2. Pili
  3. Endotoxin
  4. Coagulase
  5. M. protein
7. The following statements about meningococcal meningitis are true except: (AI 2003)
  1. The source of infection is mainly clinical cases
  2. The disease is more common in dry and cold months of the year
  3. Chemoprophylaxis of close contacts of cases is recommended
  4. The vaccines is not effective in children below 2 years of age
8. Which of the following is true about meningococcal infection:
  1. It is the most common cause of meningitis in children
  2. All strains are uniformly sensitive to penicillin (AI 2000)
  3. Vaccine is not affective in children <2 year
  4. In India, Serotype B is more common than others
9. Xavier and Yogender stay in the same hostel of same university. Xavier develops infection due to group B Meningococcus. After few days Yogender develops infection due to Group C Meningococcus. All of the following are true statements except: (AI 2002)
  1. Educate students about meningococcal transmission and take preventive measures
  2. Chemoprophylaxis to all against both group B and group C
  3. Vaccine prophylaxis of contacts of xavier
  4. Vaccine prophylaxis of contacts of yogendra
10. Conjugate vaccines are available for the prevention of invasive disease caused by all of the following except: (AIIMS 2004)
  1. H influenzae
  2. Strep pneumoniae
  3. N. meningitidis (group C)
  4. N.meningitidis (group B)
 
GONOCOCCUS
11. Gonorrhea can be identified by? (DNB Dec 2011)
  1. Growth on Macconkey medium
  2. Growth at 22 C
  3. By the fermentation of glucose
  4. Growth in 45% 60% bile
12. Which of the following is not true about Neisseria gonorrhoeae? (AIIMS Nov 2009)
  1. It is an exclusive human pathogen
  2. Some strains may cause disseminated disease
  3. Acute urethritis is the most common manifestation in males
  4. All strains are highly sensitive to penicillin
13. N. gonorrhoeae causes: (PGI Dec 2006)
  1. Urethritis
  2. Salpingitis
  3. Cervicitis
  4. Vaginitis
  5. Conjunctivitis
14. The virulence factor of Neisseria gonorrhoeae includes all of the following except: (AIIMS May 2003)
  1. Outer membrane proteins
  2. IgA protease
  3. M proteins
  4. Pili
15. All are true about Neisseria gonorrhoeae except: (AIIMS 2001)
  1. Gram positive cocci
  2. Cause stricture of urethra
  3. Involves seminal vesicles and spreads to Epididymis
  4. Drug of choice is ceftriaxone
16. 138Selective medium for Gonococci: (PGI June 2005)
  1. Thayer – martin medium
  2. LJ medium
  3. DCA medium
  4. MacConkey medium
17. Which is the true statement regarding gonococcal urethritis? (PGI Dec 2001)
  1. Symptoms are more severe in females than in males
  2. Rectum and prostate are resistant to gonococci
  3. Most patients present with symptoms of dysuria
  4. Single dose of ciprofloxacin is effective in treatment
  5. Commonly leads to arthritis
18. Which of the following literally means “flow of seed”? (AIIMS 2008)
  1. Anthrax
  2. Clostridia
  3. Gonorrhoea
  4. Proteus
19. A patient has history of sexual intercourse with a commercial sex worker 3 days back, has developed genital discharge resembling “flow of seed”. What medium should be used for culture of the discharge material? (Recent Question 2013)
  1. Mannitol salt agar
  2. Thayer martin media
  3. Potassium tellurite agar
  4. TCBS
139EXPLANATIONS
 
MENINGOCOCCUS
1. Ans. (b) (Neisseria meningitidis) Ref: Ananthnarayan 9/e p229
  • Refer chapter review for detail.
2. Ans. (a) (N. meningitidis), Ref: Harrison 18/e p1166, 1211-13, Ananthanarayan 9/e p 230, 8/e p224-227.
  • Features like ……Female with Fever, headache, hypotension, tachycardia and petechial rashes…………Points towards …………. a case of meningococcal septicemia……
  • S. aureus TSS may present with similar features, but can be ruled out because. there is no h/o use of vaginal tampons.
  • Also read these lines….
    • ✓ The most common form of infection with N. meningitidis is asymptomatic carriage of the organism in the nasopharynx.
    • ✓ Despite the location of infection in the upper airway, meningococcal pharyngitis is rarely reported; however, upper respiratory tract symptoms are common prior to presentation with invasive disease.
3. Ans. (c) (It ferments maltose), Ref: Ananthanarayan 9/ e p228, 8/e p225
  • Meningococcus can ferment glucose and maltose where as Gonococcus can ferment only glucose.
4. Ans. (c) (N. meningitidis) Ref: Ananthanarayan 9/e p 228, 8/p 225
  • Waterhouse-Friderichsen syndrome is a complication seen in infection with N.meningitidis, characterized by bilateral adrenal gland hemorrhage.
5. Ans. (c) (Mitochondria) Ref: Ananthanarayan 9/e p 10, 228
  • Mitochondria – It is not found in bacteria (prokaryote), it is a feature of eukaryotes
  • Capsule – Meningococcus is capsulated, bears a polysaccharide capsule
  • Peptidoglycan- Is present in Neisseria, but thinner
  • Plasmid – Meningococcus rarely bears plasmid.
6. Ans. (a), (b), (c) (Capsule, Pili, Endotoxin) Ref: Ananthanarayan 9/e p 228-29, 8/e p223
Virulence factor of Meningococcus:
  • Refer text (chapter review) for explanation.
    • ✓ Option d- Coagulase is a Virulence factor for Staphylococcus
    • ✓ Option e- M protein is a Virulence factor for Streptococcus.
7. Ans. (a) (source of infection is mainly clinical cases) Ref: Ananthanarayan 8/e p 224, 225
  • Option a- The source of meningococcal infection is mainly nasopharyngeal carriers
  • Nasopharyngeal carrier occurs in children mainly - 5-10% (inter epidemic), 20-90% (in epidemic)
  • Option b- The disease is more common in winter and spring (Seasonal variation)
  • Option c- Chemoprophylaxis of close contacts of cases is recommended (Rifampicin or ciprofloxacin)
  • Option d- The vaccines is not effective in children below 3 years of age because capsular polysaccharide being T independent Antigen, is poorly immunogenic <3 year.
8. Ans. (c) (Vaccine is not affective in children < 2year)
Ref: Ananthanarayan 9/e p 230, 8th /p223, Harrison 17/ e p 2621, 913 & 18/e p 3410, 1216
  • Vaccine is not affective in children <3 year (explained earlier)
  • Option a- It is the most common cause of meningitis in young adults between 2-20 year.
  • Harrison 17th /p2621 says:
    • S. pneumoniae is the most common cause of meningitis in adults >20 years of age, accounting for nearly half the reported cases
    • N. meningitidis accounts for 25% of all cases of bacterial meningitis and for up to 60% of cases in children and young adults between the ages of 2 and 20 years.
  • 140Option b- “The prevalence of meningococci with reduced susceptibility to penicillin has been increasing, and high-level penicillin resistance has been reported” …………Harrison 18/e p1216
    • ✓ However penicillin resistance is for Gonococcus is more common than Meningococcus.
    • ✓ Please remember “all strains of Group A Streptococcus i.e S. pyogenes are uniformly sensitive to Penicillin.”
  • Option d- In India, MC serotype is –
    • ✓ Group A- during epidemics,
    • ✓ C- in sporadic,
    • ✓ B-both
  • High prevalence reported from– sub-Saharan belt of Africa.
MC agents of meningitis-
  • Neonate
    • ✓ E.coli > Gr B Streptococcus > Other gram –ve bacilli > Listeria …Ref: Forfar and Arneils
    • ✓ Gr B Streptococcus > E.coli ………. Ref: Nelson's Paediatrics
  • 2-20 year- Meningococcus > Pneumococcus > H.influenzae
  • >20 year – Pneumococcus
  • Over all – Pneumococcus (~50%) > Meningococcus (~25%) > group B streptococci (~15%) > Listeria (~10%) >H. influenzae (<10%) …… Ref: Harrison 18/e p 3410
9. Ans. (c) (Vaccine prophylaxis of contacts of Xavier) Ref: Ananthanarayan 9/e p 230, 8/e p 227
  • Meningococcal Polyvalent vaccine contains à A,C,Y, W-135, immunity lasts for 3 years
  • No vaccine available for Group B:
  • As its capsule is poorly immunogenic- -α-2,8-N-acetylneuraminic acid is expressed on the surface of neural cells in the fetus such that the B polysaccharide is perceived as “self” and therefore is not immunogenic in humans. ……… Harrison 18/e p 1219
  • Also remember, vaccine is not effective in children below 3 years of age because capsular polysaccharide being T independent Antigen, is poorly immunogenic <3 year.
10. Ans. (d) (N. meningitidis group B) Ref: Ananthanarayan 9/e p 230, 8/e p 227
Already explained
  • N. meningitidis group B capsule is made up Sialic acid residue which cross reacts with human brain tissue (fatal autoimmune consequences).
  • Conjugate Vaccine - by conjugating the polysaccharide vaccine with a protein carrier molecule thus increasing the immunogenicity. So that it can be given to <3 year children.
  • Currently its available for H. influenzae type b, Pneumococcus, Meningococcus group A,C,Y,W135.
 
GONOCOCCUS
11. Ans. (c) (By the fermentation of glucose) Ref: Ananthanarayan 9/e p234, 8/e p230
  • Gonococcus cannot grow on MacConkey medium
  • Gonococcus can grow at 37°C but not at 22°C
  • Gonococcus can fermentation of glucose
  • Gonococcus cannot grow in presence of 40% bile (It is a property of Enterococcus)
12. Ans. (d) (All strains are highly sensitive to penicillin) Ref: Harrison 17/e p914, 916 & 18/e p1221 Ananthanarayan 9/e p233, 8 /e p230
Gonococci usually possess multiple plasmid coding drug resistance genes.
PPNG
Penicillinase producing N. gonorrhoeae
TRNG
Tetracycline resistance N. gonorrhoeae
CMRNG
Chromosomally mediated resistance in N. gonorrhoeae to Penicillin and Tetracycline
QRNG
Quinolone resistant N. gonorrhoeae
About Other Options
  • Option a- N. gonorrhoeae is an exclusively obligate human pathogen
  • Option b- Some strains may cause disseminated gonococcal disease (DGI) and complication like arthritis, endocarditis.
  • Option c- Acute urethritis is the most common manifestation in males while in female it is cervicitis.
14113. Ans. All i.e. (a), (b), (c), (d), (e) (Urethritis Salpingitis, Cervicitis Vaginitis, Conjunctivitis)
Ref: Ananthanarayan 9/e p232, 8/e p229
  • Refer text for (chapter review) explanation.
14. Ans. (c) (M proteins) Ref: Ananthanarayan 9/e p231, 8/e p227
Virulence factor for Gonococcus- Refer text for (chapter review) explanation.
15. Ans. (a) (Gram positive cocci) Ref: Ananthanarayan 9/e p 230, 8/e p227 Neisseria gonorrhoeae is gram –ve diplococcic (kidney shaped)
Option B and C- stricture of urethra seminal vesicles and spreads to epididymis (in males) Option D- DOC is ceftriaxone (3rd generation cephalosporin)
16. Ans. (a) (Thayer Martin medium) Ref: Ananthanarayan 9/e p231, 8/e p227
  • Refer chapter review
17. Ans. (C) (Most patients present with symptoms of dysuria) Ref: Harrison 17/e p916-918 & 18/e p1222-23
“Acute urethritis is the most common clinical manifestation of gonorrhea in males and major symptoms are Urethral discharge and dysuria, usually without urinary frequency or urgency ………Harrison 18/e p1222-23
About Other Options
  • Fluoroquinolones offered the advantage of antichlamydial activity when administered for 7 days
  • Third-generation cephalosporins have remained highly effective as single-dose therapy for gonorrhea
  • Adult vagina is resistant to Gonococcus, so infection is less severe in female
  • In males, it can involve rectum and prostate and epididymitis. (never involve testes).
DGI (gonococcal arthritis)-
  • Occurs in ~0.5–3% of untreated persons.
  • DGI is mostly caused by - PorB.1A serotype which are highly susceptible to penicillin, and had special growth requirements (i.e., the AHU auxotype) that made the organism more fastidious and more difficult to isolate.
  • Manifestations- Suppurative painful polyarthritis(MC Knee), Skin lesion, Endocarditis
  • Menstruation and Complement deficiencies C5 -C9)- risk factor for DGI
18. Ans. (c) (Gonorrhoea) Ref: Ananthanarayan 8/e p228
Gonorrhoea means flow of seed described by Galen in 1903.
19. Ans. (b)(Thayer martin medium) Ref: Ananthanarayan 9/e p233
  • Genital discharge resembling “flow of seed” Suggestive of Gonococcal urethritis
  • Selective media for culture of Gonococcus- Thayer martin medium

Corynebacterium & BacillusCHAPTER 3.4

 
CORYNEBACTERIUM DIPHTHERIAE
  • C. diphtheriae – known as Kleb Loeffler bacillus
  • Arrangement- Club shaped, Chinese letter and cuneiform arrangement
  • Possess metachromatic granules (Inorganic polyphosphates) - Babes Ernst or polar body or volutin granule
  • Special stain to demonstrate metachromatic granules - Albert, Ponder, Neisser stain
Metachromatic Granule is also produced By:
  • C.diphtheriae, C.xerosis
  • Gardenerella vaginalis
  • Spirillium
  • Mycobacterium spp
  • Enterobacter aerogene
  • Yeast.
On the basis of morphology on tellurite medium C. diphtheriae has four biotypes
McLeod classification
Gravis
Intermedius
Mitis
Colonies on PTA
Daisy head
Frogs eggs colony
Poached egg
Fermentation of starch
+ve
-ve
-ve
Toxigenic strains
100%
95–99%
80–85%
Occurrence
Epidemic
Epidemic
Endemic
Complication
Paralytic and Hemorrhagic
Hemorrhagic
Obstructive
Hemolysis
Variable
non Hemolytic
Hemolytic
Morphology
Short, no granules
Long barred, poor granules
Long curved, prominent granules
  • All 3 types are nitrate +ve
  • Belfanti (fourth biotype)– Nitrate –ve variant of Mitis
Culture:
  • Loffler's serum slope (Enriched medium)
    • Earliest growth (6–8 hours)
    • Granules best developed
    • Water of condensation can be used for animal pathogenicity
  • Potassium tellurite -0.04% (Selective medium)
    • Black colonies- produced in 2 days due to tellurite reduction.
    • Modification of PTA- Tinsdale's, Hoyle's medium, McLeod's medium
    • Best medium overall and best medium for detection of carriers, convalescent and cases.
Clinical diphtheria:
  • Clinical diphtheria is produced by 3 species
    • C. diphtheriae (with four biotypes),
    • C. ulcerans
    • C. pseudotuberculosis
  • All three produce Diphtheria toxin and black colony on PTA
  • C. diphtheriae – (urease negative) but C. ulcerans and C. pseudotuberculosis – (urease positive).
143Diphtheria toxin:
Vaccine
Antibiotic
Carrier
Not affective
Affective
Treatment of Diphtheria
Immunoglobulins are Affective
Not affective (Except early stage)
Cutaneous Diphtheria
Not affective
Affective
Schick test:
  • Susceptibility test before starting immunization (now obsolete)
  • Toxin -given intradermally on test forearm while the other forearm acts as control injected with heat inactivated toxin.
Schick test
Result
Interpretation
Negative
No reaction in both arm
Immune - No Immunization Required
Positive
Erythema and indurations on test arm,↑ size by 7th day
No reaction in control arm
Susceptible - needs immunization
Pseudo
Erythema and indurations on test and control arm but disappears by 5th day
Hypersensitive - No Immunization Required
Combined
Erythema and indurations on test and control arm but disappears only from control arm by 5th day
Susceptible and Hypersensitive - needs immunization
 
Pathogenicity
  • Natural infection only in man
  • Binds EGF(epidermal growth factor) like receptor
  • Pseudo membrane formation on tonsil, posterior pharyngeal wall- tightly adherent to the underlying tissues. Attempts to dislodge the membrane may cause bleeding
  • Source- carriers (Nasal carries are particularly dangerous - shed large no. of bacilli)
  • Faucial diphtheria– MC, laryngeal – dangerous, requires tracheostomy
  • Bull neck- neck edema and tonsillar swelling
  • Systemic:
    • Neurological complication- occurs in 1st/2nd week, Gradual improvement occurs
    • 1st muscle involved – palatopharynges
    • Ciliary paralysis occurs
    • Cranial neuropathies
    • Peripheral neuropathy
    • Myocarditis is permanent
  • 144Death- due to circulatory failure
  • Invasive infection – very rare
  • Precipitating factors- pre existing cardiac anomaly, IV drug abuser, cirrhosis.
CDC classification of diphtheria:
  • Confirmed respiratory diphtheria- Clinically compatible + lab proved+/ epidemiologically linked
  • Probable respiratory diphtheria- Only Clinically compatible.
 
Lab Diagnosis
  • Usually clinical, not to wait for lab confirmation
  • Albert staining and gram staining of throat swab and culture
  • Followed by toxigenicity testing.
Toxigenicity or Virulence test of C. diphtheria
  • In vivo tests: Guinea pig -Subcutaneous test and Intra dermal inoculation,
  • In vitro tests:
    • Elek's gel precipitation test
    • PCR detecting Tox gene
    • ELISA/ICT detecting toxin
    • Cytotoxicity on tissue culture.
 
Treatment
  • DOC of cases- penicillin,
    • Antibiotics do not neutralize circulating toxin.
    • So, antibiotics + antitoxin (horse ADS) given
  • DOC of carriers –Erythromycin
  • A case or a carrier is said to be noninfectious if two consecutive cultures properly obtained from nose and throat in 24 hr gap are negative.
 
Prevention
  • Vaccine- active immunization by toxoid: To prevent diphtheria, but can't prevent carrier stage
  • Herd immunity of >70% is required to prevent epidemic spread of diphtheria. (Park 20th /p145).
DPT- DT, TT and pertussis (whole cell or acellular component)
  • Adsorbed toxoid is more immunogenic than fluid toxoid
  • Aluminum phosphate Adsorbed toxoid- more immunogenic than hydroxide Ad- sorbed toxoid
  • Thiomersal (0.01%)used as preservative
  • Deep IM- Anterio-lateral aspect of thigh, middle 1/3rd (gluteal region is not pre- ferred as fat may inhibit DPT absorption)
  • 1dose (0.5ml)- contains
    • Glaxo- 25 Lf (DT), 5 Lf (TT), 20,000 million (Pertussis killed bacilli)
    • Kasuli- 30 Lf (DT), 10 Lf (TT), 32,000 million (Pertussis killed bacilli)
  • Storage- 2–80c, if accidentally frozen- then discard the vaccine
  • Schedule-5dose-3dose at 6/10/14 week followed by DPT booster at 16–24 week and booster DPT at 5 year
  • Protective Titre of antitoxin ≥ 0.01 unit/ml
  • dT (Adult dose)- contains 2Lf dose of diphtheria toxoid (compared to 25Lf in DT) – dT given in adults >12yr (3 doses, 0,1m, 1yr)
145
  • Both whole cell killed B.pertussis and acellular pertussis component increases potency of DT and TT (acts as adjuvant)
  • Quadruple vaccine – (DPT+ Hib) – DT and TT increases immunogenicity of Hib.
  • Whole cell killed B.pertussis is encephalogenic and short lasting immunity, asso- ciated with neurological complications and hypotonic hyporesponsive syndrome while acellual pertussis component (aP) is devoid of neurological complication.
  • Acellual pertussis component consists of pertussis toxoid, agglutinogen, fim- brial Antigen.
  • Absolute contraindication to DPT- Hypersensitivity and progressive neurological disorder (febrile seizure or cerebral palsy are not contraindications)
 
Non Diphtheria Corynebacterium
  • C. ulcerans – mimic respiratory diphtheria, transmitted by cow milk
  • C. pseudotuberculosis (Preisz Nocard bacilli) – affect sheep and horse
  • C. minutissimum (causes erythrasma)
  • C. jeikeium- multidrug resistant
  • C. parvum – immunomodulator.
 
BACILLUS
  • Aerobic gram positive spore forming bacilli
  • Most pathogenic- B.anthracis (others are called as Anthracoid bacilli).
 
BACILLUS ANTHRACIS
Historic relevance of B.anthracis:
  • Koch's postulates based on B.anthracis
  • First bacterial attenuated vaccine
  • First pathogenic bacteria seen under microscope
  • First communicable disease transmitted by infected blood
  • First to be isolated in pure culture.
 
Anthrax Vs Anthracoid Bacilli
Cultural characteristics of B.anthracis:
  • Non motile and Capsulated
  • Presumptive diagnosis:
    MC Fadyean reaction – polypeptide capsule is seen when stained with polychrome methylene blue
  • Confirm diagnosis – By Immunofluorescence microscopy
  • Gram staining : bamboo stick appearance, chain of bacilli
  • On Agar plate- medusa head appearance colony (under low power microscope)
  • Gelatin stab: inverted fir tree appearance
  • Solid media with penicillin – String of pearl appearance
  • Blood agar- non hemolytic colony
  • Selective medium – PLET media
  • Spores found in soil or in culture but never in animal body.
Virulence factor:
Anthrax toxin – 3 fraction
  • Edema factor -↑cAMP,
  • Protective factor-helps in binding (Antibody to it is protective),
  • Lethal factor
  • Alum precipitated toxoid vaccine- prepared by including protective antigen
  • Ascoli thermo precipitin test – to demonstrate anthrax Antigen
146Capsule-
  • Made up polyglutamate
  • Plasmid mediated,
  • Inhibits phagocytosis
  • Live attenuated spore vaccine (Sterne, Mazucchi)- prepared by deleting the capsule genes
 
Anthrax
It is a zoonotic disease (cattle and sheep) Man acquires infection through:
  • Cutaneous type- by small cuts or skin abrasion, use of shaving razors used for animal.
  • Pulmonary type- by spore inhalation
  • Intestinal type- by ingestion of meat (rarely)
  • Rarely by mechanical transmission through insects
Types of anthrax
  • It is a zoonotic disease (cattle and sheep)
  • Herbiverous animals are commonly affected than carnivorous.
  • G.pig and mice – Most, Sheep- and cattle- moderate.
  • Spores found in soil or in culture and never in animal
  • Anthrax - MC Agent of Bioterrorism → recently used by Afghanistan (spores enclosed in paper were mailed)
  • Mode of transmission- Direct contact (MC,95%) followed by inhalation, ingestion and Rarely by mechanical transmission through insects.
Cutaneous Anthrax
Pulmonary Anthrax
Hide porter's disease
Wool sorter's disease
Cutaneous exposure to infected animals
Inhalation of spores
Malignant pustule (Black eschar surrounded by non pitting edema, non-malignant)
hemorrhagic mediastinitis
Dock worker, butcher, abattoir, farmer
Wool factory
Most Common (95%)
Rare
Self limiting
Fatal
 
Bacillus Cereus
  • Motile, lacks capsule, causes food poisoning
  • Can also cause opportunistic infections like endocarditis, bacteremia, LRTI, meningitis
  • Can be a part of normal GIT flora
  • Presence of B.cereus atleast >105/gm of stool- considered diagnostic.
  • Selective media- MYPA (mannitol, egg yolk, phenol red, Polymyxin agar).
B.cereus
Diarrheal type
Emetic type
IP
8–16 hour
1–5 hour (preformed toxin)
Food
Cooked meat/vegetable
Rice (Chinese fried rice)
Toxin
Resemble heat labile toxin of E.coli
Resemble heat stable toxin of E.coli
Clinical feature
Diarrhea, fever, rarely nausea
Vomiting, abdominal cramps
Serotype
2, 6, 8, 9, 10, 12
1, 3, 5
147MULTIPLE CHOICE QUESTIONS
 
CORYNEBACTERIUM DIPHTHERIAE
 
Pathogenicity
1. Bull neck is seen in: (AIIMS May 2014)
  1. Tuberculous lymphadenitis
  2. Diphtheria
  3. Goitre
  4. Lymphoma
2. Lysogenic conversion is seen in: (NEET Pattern Based)
  1. Diphtheria
  2. Salmonella
  3. Staphylococcus
  4. E. coli
3. True about Corynebacterium diphtheriae: (AIIMS May 2007/NOV 2007. DNB Dec 2010, AI 2000/2011, NEET Pattern Based)
  1. All types produce toxin
  2. Toxin production is dependent upon critical concentration of iron
  3. Coded by plasmid
  4. Inhibit cAMP
4. Ehrlich phenomenon is seen in: (PGI June 2011)
  1. Mycobacterium T.B
  2. Proteus
  3. Staphylococcus
  4. Corynebacterium
  5. Mycoplasma
5. Protein synthesis is inhibited by the toxin(s) of: (AIIMS 2000, PGI May 2012)
  1. Pertussis
  2. Cholera
  3. Pseudomonas
  4. Diphtheria
  5. Shiga toxin
6. True about diphtheria are all EXCEPT: (DNB June 2011)
  1. Bull neck appearance
  2. Maculopapular rash
  3. High carrier state
  4. Causes pseudomembrane
7. Metachromatic granules are seen in? (DNB Dec 2011)
  1. Corynebacterium
  2. E.coli
  3. Yersinia
  4. Pseudomonas
8. False about C. diphtheriae is: (AI 2011)
  1. Toxin production is chromosome mediated
  2. Iron is required for toxin production
  3. Toxin inhibits protein synthesis
  4. Non toxigenic strain also causes infection
9. Clinical diphtheria is caused by: (PGI June 2009)
  1. Corynebacterium diphtheriae
  2. Corynebacterium parvum
  3. C.ulcerans
  4. Streptococcus pyogenes
  5. C.pseudodiphthericum
10. One unit of diphtheria antitoxin was defined as the smallest amount of antitoxin required to neutralize: (SGPGI 2009)
  1. 100 MLD of toxin
  2. 200 MLD of toxin
  3. 300 MLD of toxin
  4. 400 MLD of toxin
11. The type of Diphtheria with highest mortality is: (JIPMER 2009)
  1. Pharyngeal
  2. Nasal
  3. Laryngeal
  4. Conjunctival
12. True about diphtheria is: (SGPGI 2008)
  1. Cause cranial nerve palsies in 2nd and 3rd week
  2. Treatment with erythromycin
  3. It is gram negative organism
  4. Passive immunization is harmful and should not be tried
13. Which toxin acts by ADP ribosylation: (PGI June 2007)
  1. Botulinum toxin
  2. Shiga toxin
  3. V.cholerae toxin
  4. Diphtheria toxin
  5. Pertussis toxin
14. Which of the following is true about diphtheria except: (PGI 2000)
  1. Faucial diphtheria is more dangerous than laryngeal diphtheria
  2. Laryngeal diphtheria mandates tracheotomy
  3. Child is more toxic with faucial diphtheria
  4. Myocarditis may be a complication
  5. Palatal paralysis is irreversible
15. Culture media for Corynebacterium diphtheria: (JIPMER 2001)
  1. Loeffler's serum slope
  2. MacConkey
  3. Sabourauds agar
  4. Lowenstein Jensen medium
 
Lab diagnosis of diphtheria
16. In a completely and adequately immunized child against Diphtheria, the throat swab was collected. It showed the presence of Corynebacterium diphtheria like organisms on Albert staining. These organisms can have one of the following properties on further laboratory processing: (AIIMS Nov 2004)
  1. It can grow on Potassium tellurite medium
  2. It would show a positive Elek's gel precipitation test
  3. It can be pathogenic to experimental guinea pigs
  4. It can produce cytotoxicity in tissue cultures
17. Which Corynebacterium spp. can cause both cutaneous and respiratory diphtheria: (Recent Question of 2013)
  1. Corynebacterium diphtheriae
  2. Corynebacterium ulcerans
  3. 148Corynebacterium xerosis
  4. Corynebacterium hoffmanii
18. A 12 year-old child presents with fever and cervical lymphadenopathy. Oral examination shows a grey membrane on the right tonsil extending to the anterior pillar. Which of the following medium will be ideal for the culture of the throat swab for a rapid identification of the pathogen: (NEET Pattern Based, AIIMS Nov 2002)
  1. Nutrient agar
  2. Blood agar
  3. Loffler's serum slope
  4. Lowenstein Jensen medium
19. A child presents with a white patch over the tonsils, diagnosis is best made by culture in: (AI 2001)
  1. Loeffler medium
  2. L.J. media
  3. Blood agar
  4. Tellurite medium
20. Metachromatic granules seen in: (PGI 2000)
  1. C.diphtheriae
  2. Mycoplasma
  3. Gardenerella
  4. Chlamydia
  5. Staphylococcus
21. Positive schick's test indicates that person is: (NEET Pattern Based, AI 2002, AIIMS 2000, AIIMS Nov 2007)
  1. Immune to diphtheria
  2. Hypersensitive to diphtheria
  3. Susceptible to diphtheria
  4. Carrier of diphtheria
22. Herd immunity over…….% is required to prevent epidemic spread of diphtheria? (DNB 2000)
  1. 50%
  2. 55%
  3. 60%
  4. 70%
23. DOC for diphtheria carrier? (DNB 03)
  1. Erythromycin
  2. Tetracycline
  3. Penicillin
  4. DPT
24. Schick test doesn't indicate? (MP 2007, Tholkata 2003)
  1. Immunity to diphtheria
  2. Susceptibility to diphtheria
  3. Carrier of diphtheria
  4. Hypersensitivity to diphtheria
25. Diphtheria carriers are diagnosed by? (MP2000)
  1. Throat culture
  2. Gram's staining
  3. Albert's stain
  4. Schick test
 
BACILLUS ANTHRACIS
26. Malignant pustule is/are seen in infection with: (PGI June 2011, AIIMS DEC 2010, MAY 2003)
  1. Treponema pallidum
  2. Campylobacter granulomatosis
  3. Bacillus anthracis
  4. H.ducreyi
  5. Pseudomonas aeruginosa
27. An abattoir worker presented with a malignant pustule on his hand that progressed to form an ulcer. Smear was taken from the ulcer and sent to laboratory for investigation. The diagnosis is: (AIIMS Nov 2012)
  1. Cutaneous anthrax
  2. Carbuncle
  3. Ulcerating melanoma
  4. Infected rodent ulcer
28. Bacillus used to test efficacy of sterilization by autoclave is? (DNB Dec 2010)
  1. Bacillus subtilis
  2. Bacillus pumilis
  3. Bacillus stearothermopilus
  4. Coxiella burnetti
29. True about anthrax: (PGI June 2009)
  1. Cause by Gram positive bacilli
  2. Soil reservoir
  3. Spore formation takes place inside the body
  4. More common in carnivorous than herbivores
  5. Penicillin is drug of choice
30. A man, after skinning a dead animal, developed a pustule on his hand. A smear prepared from the lesion showed the presence of Gram positive bacilli in long chains which were positive for McFadyean's reaction. The most likely etiological agent is: (AI 2004)
  1. Clostridium tetani
  2. Listeria monocytogenes
  3. Bacillus anthracis
  4. Actinomyces sp
31. True about Bacillus anthracis: (PGI Dec 2004)
  1. Zoonotic disease
  2. Man to man transmission possible
  3. Agent for bioterrorism
  4. Antibiotic has no role
  5. Mc Fadyean reaction seen
32. Inverted fir tree appearance is characteristic: (JIPMER 2004)
  1. Bacillus anthrax
  2. Haemophilus influenzae
  3. Yersinia pestis
  4. Brucella
33. Which of the following is true regarding anthrax: (PGI Dec 2001)
  1. Mc’ fadyean reaction shows capsule
  2. Humans are usually resistant to infection
  3. Less than 100 spores can cause pulmonary infection
  4. Gram stain shows organism with bulging spores
  5. Sputum microscopy helps in diagnosis
34. Anthrax bacilli differs from Anthracoid bacilli by being: (PGMEE 2006)
  1. Non- capsulated
  2. Strict aerobe
  3. Non-motile
  4. Haemolytic colonies on blood agar
35. All are true about cutaneous anthrax except: (JIPMER 2009)
  1. Extremely painful
  2. The whole area is congested and edematous
  3. Central crustation with black eschar
  4. Satellite nodule around inguinal region
36. 149A person working in an abattoir presented with a pustule on his hand that progressed to form an ulcer. Smear is taken from the ulcer and sent to laboratory for investigation. Which of the following is the best stain to determine the causative agent of the ulcer:(AIIMS Nov 2012, AI2007, AIIMS Nov 2006)
  1. Trichrome methylene Blue
  2. Carbol fuschin
  3. Acid fast stain
  4. Calcoflour white
37. Bacillus anthracis, false statement is: (Recent Question of 2013)
  1. Gram negative
  2. Bacilli in long chain
  3. Non haemolytic colony in blood agar
  4. Medusa head appearance on nutrient agar
38. Non industrial anthrax is transmitted by which of the following occupational exposure (s)? (PGI Nov 2014)
  1. Butcher
  2. Farmer
  3. Abattoir
  4. Sportsmen
39. Mc-Fadyean reaction is shown by: (APPG 2014)
  1. Bacillus anthracis
  2. Clostridium diphtheriae
  3. Escherichia coli
  4. Listeria monocytogenes
 
Bacillus Cereus
40. Characteristic of Bacillus cereus food poisoning is: (AIIMS 2010)
  1. Presence of Fever
  2. Presence of Pain abdomen
  3. Absence of Vomiting
  4. Absence of Diarrhea
41. Non invasive diarrhea is due to: (AI 2009)
  1. Shigella
  2. Salmonella
  3. B.cereus
  4. Y.enterocolitica
42. Incubation period for B. cereus: (PGI June 2008)
  1. 1–6 hrs
  2. 8–16 hrs
  3. 24 hrs
  4. > 24 hrs
  5. Weeks
43. Chinese restaurant syndrome after eating fried rice and vanilla sauce is due to: (APPG 2014, MHPG 2014)
  1. Clostridium perfringens
  2. Bacillus cereus
  3. Staphylococcus aureus
  4. Clostridium botulinum
150EXPLANATIONS
 
CORYNEBACTERIUM
1. Ans. (b)(Diphtheria) Ref: Ananthnarayan 9/p239
  • Bull neck is tonsillar swelling with neck edema, seen as a complication of faucial diphtheria.
2. Ans. (a) (Diphtheria) Ref: Ananthanarayan 8/e p 238–39
  • Lysogenic conversion –This is a stage, where the phage DNA integrates with bacterial DNA. In this stage, the phage DNA may code for toxin and imparts toxigenicity to the bacteria.
  • E.g. of toxins that are phage coded- Diphtheria toxin, cholera toxin, VT of E.coli, botulinum toxin C&D and Streptococcal pyrogenic exotoxin A&C.
3. Ans. (b) (Toxin production is dependent upon critical concentration of iron) Ref: Ananthanarayan 8/e p 238–39
  • Optimum conc. of iron (0.1mg/L) is required for expression of diphtheria toxin.
  • Among the biotypes of C.diphtheriae, all most all strains of gravis, 95–99% strains of intermedius and 80–85% strains of mitis are toxigenic.
4. Ans. (d) (Corynebacterium) Ref: Ananthanarayan 8/e p 238–39
  • The ability of diphtheria toxin and antitoxin to combine varying proportion is the basis of Ehrlich phenomenon.
  • During the diphtheria toxin-antitoxin interaction, it is said that diphtheria toxin would combine with more amount of antitoxin if added in instalment rather than adding all the toxin together.
  • This discrepancy is due to presence of toxin preparations in various amounts of Toxoid and ability of diphtheria toxin and antitoxin to combine in varying proportion. This is known as Ehrlich phenomenon
5. Ans. (c) (d) (e) (Pseudomonas) (Diptheria) (Shiga toxin) Ref: Ananthanarayan 9/e p 238, 8/e p234
Mechanism of action of exotoxin:
  • Toxin that increases cAMP- Pertussis toxin, Cholera toxin, heat labile toxin of E.coli, Anthrax toxin (edema factor)
  • Toxin that increases cGMP- Heat stable toxin of E.coli
  • Toxin that inhibits protein synthesis-
    • ✓ By inhibiting ribosome- Shiga toxin & Verocytotoxin of E.coli
    • ✓ By inhibiting elongation factor-2- Diphtheria toxin and Pseudomonas toxin
  • Neurotoxin –
    • ✓ Blocks the release of glycine & GABA- Tetanospamsin
    • ✓ Blocks the release of acetyl choline- botulinum toxin
6. Ans. (b) (Maculopapular rash) Ref: Harrison 18th/e p1189, Park 22nd /p152, 21st /p149
  • Rashes are not seen in diphtheria
  • Bull neck appearance- A few patients with diphtheria develop massive swelling of the tonsils and neck edema called as present with “bull-neck” diphtheria.
  • High carrier state- Carriers are the common source of infection, the ratio is 95 carriers to 5 cases. Nasal carriers are particularly dangerous- shed large no. of bacilli
  • Pseudomembrane - characteristic white to grey pseudomembrane is caused by diphtheria toxin–mediated necrosis of the respiratory epithelial layer, producing fibrinous coagulative exudate.
7. Ans. (a) (Corynebacterium) Ref: Ananthanarayan 9th/e p236,8th/e p233,
  • Metachromatic Granule is produced By- C.diphtheriae, C.xerosis (Refer chapter review for the detail
8. Ans. (a) (Toxin production is chromosome mediated) Ref: Ananthanarayan 9/e p238, 8/e p234
  • DT is coded by Bacteriophage (tox phage)
About other options-
  • Option b- Toxin synthesis depends on optimum iron concentration (0.1mg per litre) while 0.5mg/ml inhibits synthesis. Other factors for Toxin synthesis – pH, osmolarity, aminoacid conc.
  • 151Option c- Diphtheria toxin → causes ADP ribosylation of elongation factor 2→thus inhibits EF2 → inhibits protein synthesis
  • Option d- Toxin has no role for Cutaneous diphtheria and Skin /local lesions can be produced by non Toxigenic strain strains also.
Also remember - Bacteriophage coded Toxin
9. Ans. (a), (c) (Corynebacterium diphtheriae, C.ulcerans) Ref: Gillepsie Bacteriology 2nd/p123
  • Clinical diphtheria - caused by C. diphtheriae, C. ulcerans and C. pseudotuberculosis
All 3 produce DT and black colony on PTA
10. Ans. (a) (100 MLD of toxin) Ref: Ananthanarayan 8/e p238
Ehrlich defined- “Minimum Lethal Dose of DT as the least amount of toxin requires to kill a 250 gm Guinea pig in 4 days of inoculation,
One unit of diphtheria antitoxin is defined as the smallest amount of antitoxin required to neutralize 100 MLD of toxin.”
11. Ans. (c) (Laryngeal) Ref: Harrison 17/e p891 & 18/e p 1189
Formation of extensive pseudo membrane over larynx carries the risk of airways obstruction.
12. Ans. (a) (Cause cranial nerve palsies in 2nd and 3rd week) Ref: Harrison 17/e p892 & 18/e p1190, says - “Neurologic
  • Manifestations may appear during the first or second week of illness, typically beginning with dysphagia and nasal dysarthria and progressing to other signs of cranial nerve involvement.”
About other options
Option b- Prompt administration of diphtheria antitoxin is critical in the management of respiratory diphtheria.
  • Antibiotics have no role once the toxin is formed and are mainly used to prevent transmission to other susceptible contacts.
  • Procaine penicillin G given for treatment and erythromycin given for carriers. Option c- C.diphtheriae is a gram +ve bacilli
Option d- Passive immunization is critical in the management of respiratory diphtheria.
13. Ans. (c), (d), (e) (Cholerae toxin, Diphtheria toxin, Pertussis toxin) Ref: Ananthanarayan 9th/e p238, 8/e p235 ADP ribosylating toxin:
  • Cholera toxin /LT of E.coli, Pertussis Toxin - AB5
    • ✓ B subunit →attach GM1 ganglioside receptor
    • ✓ A subunit causes → ADP ribosylation of G protein→ ↑adenyl cyclase→ ↑cAMP
  • Diphtheria Toxin and Exotoxin A of Pseudomonas-
  • ADP ribosylation of EF2 → ↓ protein synthesis
14. Ans. (a), (e) (Faucial diphtheria is more dangerous than laryngeal diphtheria, Palatal paralysis is irreversible) Ref: Harrison 17/e p891, 16/p835 & 18/e p1189, Ananthanarayan 9/p239, 8/e p235
  • Option a- Formation of extensive pseudo membrane over larynx carries the risk of airways obstruction, so definitely it is more dangerous than Faucial diphtheria
  • Option b- Harrison 16th/p835–“Early intubation or tracheostomy is indicated when the larynx is involved or signs of impending resp. obstruction are detected.”
  • Option c- “systemic toxicity is more severe with extensive pseudo membrane which is more common in faucial diphtheria.
  • Option d- Myocarditis and polyneuropathy are late toxic complications.
  • Option e- Post diphtheritic Palatal paralysis occurs in 3–4 week but as a rule recovery is the rule.
15. Ans. (a) (Loeffler's serum slope) Ref: Ananthanarayan 9th/e p240,8/e p233
  • 152Loeffler's serum slope is enriched media used for C.diphtheriae, granules are produced after 6–8 hrs of incubation..
16. Ans. (a) (It can grow on Potassium tellurite medium) Ref: Journal: IJMM 2002/V20/Is-1
  • This child is a carrier for C.diphtheriae.
  • Adequate vaccination can prevent the disease but not the colonization.
“Mass immunization might result in disappearance of toxigenic strains from many populations but has no effect on carriage of non -Toxigenic strains.”
  • Since the child is completely vaccinated, antitoxin level in serum can always neutralize the toxin but it can't prevent the colonization of non toxigenic strain.
  • So, the organism can be grown on Potassium tellurite medium but since it might be a non toxigenic strain, so any test to detect the toxigenicity will be negative.
Toxigenicity test of C. diphtheria:
  • In vivo tests- Subcutaneous and Intra dermal test in Guinea pig.
  • In vitro tests: Elek gel precipitation test, PCR detecting Tox gene, ELISA/ICT detecting toxin, cytotoxicity test in tissue cultures.
17. Ans. (a) (Corynebacterium diphtheriae) Ref: Ananthanarayan 9/e p239
  • Corynebacterium diphtheria can cause both cutaneous and respiratory forms of diphtheria.
18. Ans. (c) (Loffler's serum slope) Ref: Ananthanarayan 9th/e p237, 8/e p233
  • Rapid identification of C. diphtheria - Loffler's serum slope (6–8 hour)
  • Best media for isolation in carriers/contacts/convalescent – PTA (48 hour)
  • Metachromatic granules best develops on - Loffler's serum slope
19. Ans. (d) (Tellurite medium) Ref: Ananthanarayan 9th/ e p237, 8/p233, Topley 10/p979
  • Rapid identification of C. diphtheria - Loffler's serum slope (6–8 hour)
  • Best media for isolation in carriers/contacts/convalescent – PTA (48 hour)
  • As PTA is a selective media, so it is useful in isolating C. diphtheria by inhibiting the normal commensals of throat.
  • Topley 10th/p979 says- “Primary plating media for cultivation of C. diphtheriae should be sheep blood agar plus one selective media.”
Also remember-
  • Black colony on PTA is also produced by other Coryneform.
  • So the best selective media is Tinsdale Media (Cysteine Tellurite Blood agar) where It produces Black colony with a halo (due to cysteinase activity)
20. Ans. (a), (c) (C.diphtheriae, Gardenerella) Ref: Ananthanarayan 9/e p 237, 8/e p233, Journal- ABE WIEDRA/1959/ V78 “Metachromatic Granules of Microorganisms”
  • For detail – Refer chapter review
21. Ans. (c) (Susceptible to diphtheria) Ref: Ananthanarayan 9/e p 241, 6/e p214
  • Positive test means that person is susceptible to diphtheria – So immunization is required.
  • Shick test - For detail – Refer chapter review
22. Ans. (d) (70%) Ref: Park 22/e p152, 21/ e p150
  • Herd immunity over 70% is required to prevent epidemic spread of diphtheria
23. Ans. (a) (Erythromycin) Ref: Park 22nd/p153, 21/p151
  • DOC or carriers of diphtheria- erythromycin
24. Ans. (c) (Carrier of diphtheria) Ref: Park 22nd/p147, 21st/p145
  • Positive Schick test- indicates susceptible to diphtheria
  • Negative Schick test- indicates immune to diphtheria
  • Combined reaction Schick test- indicates susceptible and hypersensitive to diphtheria
  • Pseudo reaction Schick test- indicates hypersensitive to diphtheria
15325. Ans. (a) (Throat culture) Ref: Park 22/e p152, 21/e p150
  • Carriers harbor the diphtheria bacilli in throat, hence can be effectively diagnosed by throat culture.
 
BACILLUS ANTHRACIS
26. Ans. (c) (Bacillus anthrax) Ref: Ananthanarayan 9/p247,8 /p245 Cutaneous lesions of B.anthracis is characterized by painless black eschar surrounded by non-pitting, gelatinous edema called as ‘malignant pustule’.
27. Ans. (a) (Cutaneous anthrax) Ref: Ananthanarayan 9th/e p247
  • Malignant pustule is the characteristic feature of cutaneous anthrax
28. Ans. (c) (Bacillus stearothermophilus) Ref: Ananthanarayan 9/e p32, Journal: Developments in the use of Bacillus species for industrial production, Marcus Schallmey et al, Can. J. Microbiol. 50: 1–17 (2004).
Industrial Application of Bacillus species-
  • Bacillus stearothermophilus – is used as a control of sterilization in autoclave
  • Bacillus subtilis sub species niger - is used as a control of sterilization of autoclave, hot air oven and plasma sterilization
  • Bacillus amyloliquefaciens - Secrete large quantities of enzymes like barnase (a ribonuclease), alpha amylase used in starch hydrolysis, the protease subtilisin used with detergents, and the BamH1 restriction enzyme used in DNA research.
  • B. thuringiensis-Insecticide, used to control Culex, Aedes, or Anopheles.
  • B. licheniformis- Secretes Bacitracin
  • B. polymyxa, B. colistinus- Secretes polymyxin and colistin
29. Ans. (a), (b) (Cause by Gram positive bacilli, Soil reservoir)
Ref: Jawetz 24th/p205 & 25th/p167, Ananthanarayan 8/e p243–248
  • Option a- Bacillus anthracis is a Gram positive bacilli
  • Option b- Reservoir is animal and soil. Animal gets infection by inhalation of spores in soil
  • Option c- Spore formation never takes place inside the body
  • Please remember “spore formation takes place in unfavorable conditions and human body is a favorable environment for the organism like Bacillus anthracis causing anthrax and Clostridium perfringens causing gas gangrene.”
    • Spore formation takes place in soil or in culture.
    • Anthrax spore formation
      • Encouraged by oxalate agar, 2% NaCl at 25–30°c
      • Inhibited by CaCl2
    • Spore stain –
      • Fulton and Schaeffer stain (malachite green and safranin)
      • Modified Ziehl Neelsen with 0.25% H2SO4
  • 154Option d- Anthrax is more common in herbivores like sheep and cattle than carnivorous
  • Option e - Jawetz 24th /p205 & 25th/p167 says- “Ciprofloxacin is recommended for treatment; penicillin G, along with gentamicin or streptomycin, has previously been used to treat anthrax.”
  • Ananthanarayan 8th/e p248 says- “penicillin G or streptomycin are no longer used for treatment which are replaced now by ciprofloxacin & Doxycycline.”
30. Ans. (c) (Bacillus anthracis) Ref: Jawetz 24 /p205 & 25/p167, Ananthanarayan 9/e p248,8/e p248 It is a case of cutaneous anthrax.
Points in favor: Exposure to a dead animal, pustule on hand and Gram positive bacilli in long chains positive for McFadyean's reaction
McFadyean's reaction is demonstration of capsule by polychrome methylene blue staining..
31. Ans. (a), (c), (e) (Zoonotic disease, Agent for bioterrorism, Mc Fadydean's reaction seen)
Ref: Ananthanarayan 9th/e p245–48, 8th/e p243–248
  • Option a- Anthrax is a Zoonotic disease, mainly transmitted from sheep and cattle.
  • Option b- Man to man transmission doesn't occur
  • Option c- Bacillus anthracis is a class I Agent for bioterrorism
  • Option d - Antibiotic has role if started early before toxin production.
  • Option e- McFadyean's reaction is demonstration of capsule of Bacillus anthracis
  • Class I agents of Bioterrorism –
    • ✓ Anthrax, Plague, Tularemia, Botulism
    • ✓ Small pox, and Viral hemorrhagic fevers
32. Ans. (a) (Bacillus anthracis) Ref: Ananthanarayan 9th/e p245, 8th/e p243
  • Inverted fir tree appearance in gelatin stab culture – seen by Bacillus anthracis
  • Bacillus anthracis laboratory findings- Refer text.
33. Ans. (a), (b), (c), (e) (M’ fadyean reaction shows capsule, Humans are usually resistant to infection, Less than 100 spores can cause pulmonary infection, Sputum microscopy helps in diagnosis)
Ref: Ananthanarayan 9/e p247–48, 8 /e p243, Greenwood 16/e p226, Harrison 17/e p1344 & 18/e p1769
  • Option b- Greenwood 16/e p226 says –“Humans are relatively resistant to infection with B anthracis”
  • Option c- Harrison 17/e p1344 & 18/e p1769 says “While an LD50 of 10,000 spores is a generally accepted number for anthrax it has also been suggested that as few as one to three spores may be adequate to cause disease in some settings.”
  • Option d- Gram stain shows organism with non bulging spores.
  • Bulging spores are seen for Clostridium
  • Option e - microscopy of the sample like lesion, blood, sputum reveals gram positive bacilli in chain
34. Ans. (c) (Nonmotile) Ref: Ananthanarayan 9th/e p249, 8th/e p247
  • Anthrax bacillus is capsulated, non motile and produce non hemolytic colony on blood agar.
  • Both Anthrax bacilli and Anthracoid bacilli are strict aerobe.
  • Also Refer the table on the text.
35. Ans. (a) (Extremely painful) Ref: Ananthanarayan 9/e p247, 8 /e p245
Cutaneous lesions in Anthrax are painless.
36. Ans. (a) (Trichrome methylene Blue) Ref: Ananthanarayan 9/e p248, 8/e p246
  • This is a case of Cutaneous anthrax (Points in favor- abattoir occupation, pustule andulcer on hand)
  • Provisional diagnosis of Anthrax – demonstration of Capsule by polychrome methylene blue (M’ fadyean reaction)
  • Confirmed diagnosis – by immunofluorescence microscopy
  • Cutaneous anthrax (Hide porter's disease) –MC (95%), but usually self limiting
  • Occupation associated- Dock worker, butcher, abattoir, farmer
37. Ans. (a) (Gram negative) Ref: Ananthanarayan 9/e p244
Bacillus anthracis is gram positive bacilli.
15538. Ans. (a), (b), (c) (Butcher, Farmer, Abattoir) Ref: Ananthnarayan 9/p247
  • Non industrial anthrax is transmitted by direct contact following occupational exposures seen in butchers, farmers and abattoirs
39. Ans. (a) (Bacillus anthracis) Ref: Ananthnarayan 9/p248
  • When specimen is stained with polychrome methylene blue, an amorphous purplish capsule around the blue bacilliseen, whichis characteristic of anthrax bacillus.
  • This is called as M”Fadyean's reaction and is employed for the presumptive diagnosis of anthrax in animals.
 
Bacillus Cereus
40. Ans. (b) (presence of Pain abdomen) Ref: Ananthanarayan 9/e p249, 8/e p245
  • Fever is not a consistent feature of B. cereus food poisoning.
  • Abdominal cramps may be seen in both emetic and diarrheal type of B. cereus food poisoning.
  • For detailed explanation refer text (chapter review).
41. Ans. (c) (B.cereus) Ref: Ananthanarayan 9/e p249, 8/e p247
Invasive diarrhoea - Shigella, EHEC, EIEC, Campylobacter, V.parahemolyticus, Salmonella, Y.enterocolitica
Non Invasive diarrhea- EPEC, ETEC, Cholera, B.cereus, S.aureus
42. Ans. (a), (b) (1–6 hours, 8–16 hours) Ref: Ananthanarayan 9/e p249, 8/e p247
Emetic type – IP 1–5 hour
Diarrheal type – IP 8–16 hour
43. Ans. (b) (Bacillus cereus) Ref: Ananthnarayan 9/p249
  • Chinese restaurant syndrome means- Food poisoning within 1–6hr of consumption of Chinese fried rice………… Most probable agent is Bacillus cereus.

ClostridiumCHAPTER 3.5

  • Gram Positive, spore forming, Anaerobes.
  • All are motile and exhibit stately motility except- Cl.tetani VI and Cl perfringens
  • All are Noncapsulated (except Cl.perfringes, Cl.butyricum)
  • Most of the Clostridia are found normally in feces-Cl. ramosum, Cl. perfringens, Cl. difficile
  • Spore – Most of them possess sub terminal spores except
    • Cl tetani -Spherical and terminal- Drum Stick Appearance
    • Cl. tertium - Oval and terminal – Tennis racket shaped
    • Cl.bifermentans - Central
  • Saccharolytic Vs Proteolytic species -
    • Saccharolytic species– turn RCM (Robertson Cooked meat broth) pink
    • Proteolytic species– turn RCM black with foul and pervasive odor
Saccharolytic – e.g Cl. botulinum C,D,E
Proteolytic- e.g Cl tetani
Both Saccharolytic and Proteolytic but predominantly Saccharolytic-
Cl. Perfringens, Cl.novyi, Cl.septicum, Cl.difficile
Both Saccharolytic and Proteolytic but predominantly Proteolytic- Cl. botulinum A,B,F
 
CL. PERFRINGENS
  • Non Motile, capsulated,
  • Spores are oval, sub terminal
  • They are Toxigenic and invasive
  • Possess 4 Major (Lethal) Toxins (a,b, ε, i) and 8 minor toxins
  • a toxin- Lecithinase or phospholipase-C: Principle virulence factor
  • Based on the toxin production, divided to 5 types (A-E)
  • Blood Agar Produces Target Haemolysis (double zone meholysis)
  • Litmus Milk reaction (+)
  • Shows Naegler reaction – due α toxin → opalescence on egg yolk media which can be inhibited by antisera
Infections:
 
Gas Gangrene
  • Definition (Oakley, 1954)-Rapidly spreading edematous myonecrosis occurring characteristically in association with severe wounds of extensive muscle masses contaminated with pathogenic clostridia particularly C.perfringens.
  • Agents of gas gangrene- (Usually Polymicrobial)-
    • Established causes – 60% due to Cl. perfringens type-A, 20–40% due to-Cl. novyi, Cl.septicum
    • Probable Pathogen- Cl histolyticum, Cl fallax
    • Doubtful Pathogen- Cl bifermentans, Cl sporogenes, CL.tertium, Cl.sordellii, Cl.aerofoetidium.
  • Incubation period- 10–48 hr- for Cl. Perfringens, 2–3 days- Cl.septicum, 5–6 days- Cl.novyi.
  • No pus cells in spite of extensive tissue damage
  • Pain and crepitus-characteristic
  • Gas gangrene strains of Clostridium perfringens don't produce spores-neither in tissue nor in media
  • 157Lab diagnosis:
    • Diagnosis of gas gangrene should be made on clinical ground, Lab diagnosis is only for confirmatory.
    • Gram staining:
      • ✓ Gram Positive Bacilli without spore- Cl. perfringens
      • ✓ Citron body and boat shaped Gram Positive Bacilli - Cl.septicum
      • ✓ Gram Positive Bacilli with oval sub terminal spore- Cl.novyi.
    • If C.perfringens is suspected- then it has to be confirmed by-
      • ✓ Naegler reaction
      • ✓ Reverse CAMP test.
  • Treatment – surgery (mainstay of treatment)
  • DOC-Penicillin + Clindamycin,
  • Hyperbaric O2
  • Anti Gas gangrene serum.
 
Other Infections d/t C.Perfringens
  • Food Poisoning – d/t Type A →
    • Enterotoxin (Similar to cholera & LT of E.coli)
    • Heat resistant spores -found contaminated with food
  • Necrotising enteritis (Pigbel): Due to beta toxin of type C of C.perfringens
  • Biliary tract infection
  • Urogenital infection
  • Brain abscess & meningitis
  • Endogenous gas gangrene following GI surgery
  • Thoracic infection and Panophthalmitis.
 
CLOSTRIDIUM TETANI
  • All are Motile (except type VI)
  • Shows swarming on blood agar.
Two toxins:
  • Tetanospasmin – Blocks release of inhibitory transmitters glycine and GABA→ leads to spastic paralysis
  • Telanolysin – No role in pathogenesis.
 
Tetanus
  • Route- injury, unhygienic surgery, abortion/delivery, otitis media
  • More common in warm climate and rural area with fertile soil
  • 1st symptom-↑masseter tone (trismus/lock jaw) then→ descending tetanus
  • Hands feet are spared
  • Mentation unimpaired, deep tendon reflex↑
  • In neonates, difficulty in feeding is the usual presentation
  • Autonomic disturbance is maximal during the second week of severe tetanus
  • Incubation Period – 6–10 days
  • Shorter Incubation Period→ graver the prognosis
  • Complications-
    • Risus sardonicus - characteristic, abnormal, sustained spasm of facial muscles, appears to produce grinning
    • Opisthotonus position
    • Respiratory muscles spasm-may cause airway obstruction
  • Noninfectious- no person to person spread
  • Diagnosis is always clinical, microscopy is unreliable.
    158Treatment: all type of wounds need surgical toilet followed by:
    Immunity Category
    Wound <6hr, clean, non- penetrating, no/negligible tissue damage
    Other wounds
    Category A
    Nothing required
    Nothing required
    Category B
    Toxoid 1 dose
    Toxoid 1 dose
    Category C
    Toxoid 1 dose
    Toxoid 1 dose + HTIG
    Category D
    Toxoid complete dose
    Toxoid complete dose+ HTIG
    • Category A: Taken complete course of TT/booster with in past 5 year
    • Category B: Taken complete course of TT/booster with in past > 5 year- <10 year
    • Category C: Taken complete course of TT/booster with in past >10 year
    • Category D: Not Taken complete course of TT/booster or immunity status not known
  • Antibiotics-
    • Antibiotics- play a role in early infection before expression of toxin (before 6 hr)
    • It is useful only to eradicate source of toxin, but no role if toxin is already produced
    • Metronidazole (DOC) >Penicillin for 7 days (start within 6 hour of infection).
  • Antitoxin-
    • HTIG (Human Tetanus Immunoglobulin)- 250IU IM-Effect lasts for 30days
    • ETIG (Equine Tetanus Immunoglobulin)-1500IU – last for 7–10 days, 2nd dose eliminated rapidly, serum sickness may develop.
  • Symptomatic treatment- Anti spasmodic.
 
Prevention
  • Primary immunization -DPT (5-doses) followed by 2 doses of TT (at 10 & 16 year)- Total 7 doses
  • Adult- 2 doses at 1m interval + 2 boosters at 1 year & 6 year - Total 4 doses
  • Protective titre- 0.01 IU/ml.
 
Other form of Tetanus
  • Maternal tetanus is defined by the WHO as tetanus occurring during pregnancy or within 6 weeks after the conclusion of pregnancy (whether with birth, miscar- riage, or abortion).
  • Neonatal tetanus is defined by the WHO as “an illness occurring in a child who has the normal ability to suck and cry in the first 2 days of life but who loses this ability between days 3 and 28 of life and becomes rigid and has spasms. (Also k/a- “8th Day disease”).
  • Neonatal tetanus is seasonal- July, August and September
  • 7 clean practices are proposed under RCH-II:-- Clean hand, clean surface, clean cord blade, clean cord tie, clean cord stump, clean towel, clean water.
  • Neonatal tetanus Elimination is based on-
    • Rate less than 0.1/1000 live births
    • TT2 coverage in pregnant women- >90%
    • Attended deliveries- > 75%
    • Classification of districts into “NT high risk, control and eliminated districts” is based on these three criteria.
  • TT in pregnancy-
    • 2 doses of TT- 1st as early as possible, 2nd dose- at least a month later and at least 3 weeks before delivery.
    • National immunization schedule- 2 doses of TT given between 16–36 weeks.
    • In high incidence areas- 2 doses of TT can be extended to all females in the reproductive age
    • If mother comes to you directly at delivery- then give only one dose of TT
    • Infants born to unimmunized mother- should receive ATS
    • Previously immunized mother- 1booster dose of TT is sufficient
    • There after → Need of booster doses every alternate pregnancy.
 
159CLOSTRIDIUM BOTULINUM
  • Botulinum toxin → Most toxic known to man (MLD → 1 mg)
  • Mechanism of action- Blocks release of acetylcholine →leads to flaccid paralysis
    • It causes proteolytic cleavage of a protein complex which is the binding site of Acetyl choline to the pre synaptic vesicle. resulting in a blockage of the release of the acetylcholine
    • Botulinum toxin A, E- causes cleavage of SNAP25
    • Botulinum toxin C1- causes cleavage of Syntaxin
    • Botulinum toxin B, D, F, G- causes cleavage of Synaptobrevin.
  • Botulinum toxin is subtyped to 8 types (A-G),
  • Type A, B and E → commonly causes Human disease
  • All subtypes are -neurotoxin, except C2-enterotoxin
  • Botulinum toxin Type C,D- bacteriophage coded
  • Botulinum toxin used therapeutically for the treatment for strabismus, blepharospasm
  • Botulinum toxin - Also produced by Cl. butyricum, Cl.baratti, Cl.argentinense
  • Toxin liberates as protoxin. It requires trypsin or other proteolytic enzyme to convert to active form
  • It differs from other Exotoxin as it produced intracellularly and appear outside only after autolysis of the cell
  • Blocking of Ach release is permanent, but the action is short lasting as the recovery occurs in 2–4 months once the new terminal axon spourts.
 
Types of Botulism
Food borne botulism
  • Due to Preformed toxin contaminated with Canned food
  • Symptoms- (5Ds)- Diplopia, dysphasia, dysarthria, constipation, descending (symmetric) paralysis, ↓DTR
  • No sensory involvement and No diarrhea
  • Wound botulism – due to contamination of spores to the wound surface
  • Infant botulism
    • Due to spore ingestion
    • Source-honey
    • Floppy Child syndrome
    • Affect usually <6months age.
 
CLOSTRIDIUM DIFFICILE
  • Causes Pseudo membranous Colitis
  • Risk factor-
    • Prolonged hospital stay
    • Prolonged intake of borad spectrum antibiotics like Clindamycin (MC), Ampicillin, cephalosporin
  • Toxins: A→Enterotoxin B→Cytotoxin, Both are required for the manifestation
  • Toxin Can be demonstrated – effect on Hep2 or ELISA
  • Diagnosis-
    • Culture –more sensitive, but less specific (Since it can be found as a part of normal flora)
    • Specific methods-
      • ✓ Toxin démonstration (Effect on Hep2 or ELISA)
      • ✓ Histopathological prove
      • ✓ PCR for tox B gene
      • ✓ Tox Ag (EIA)
      • ✓ Pseudomembrane by colonoscopy (Sn-50%)
  • T/t –DOC-Metronidazole (initial mild cases), Vancomycin (Severe, recurrent cases).
 
160NON-SPORING ANAEROBES
Gram +ve cocci
Peptostreptococcus and Peptococcus
Gram –ve cocci
Veillonella
Gram +ve bacilli
Bifidobacterium, Propionibacterium, Eubacterium, Lactobacillus, Mobilincus, Actinomyces
Gram -ve bacilli
Bacteroides, Prevotella, Porphyromonas, Fusobacterium
Spirochete
Treponema and Borrelia (Not Leptospira)
Non spring anaerobe as Common Normal flora
Skin
Propionibacterium
Mouth
Fusobacterium, Prevotella, Anaerobic cocci
Intestine
Clostridia, Bacteroides (MC commensal)
Stomach
Lactobacilli
Vagina
Anaerobic cocci, Lactobacilli
Common disease
Mobilincus- Bacterial vaginosis (also caused by Gardenrella)
Fusobacterium fusiformis- Agent of Vincent's angina (also caused by Borrelia vincentii)
Anaerobic cocci-
Puerperal sepsis and other female genital tract infection (also by B.fragilis and Prevotella)
Skin and soft tissue infection
B.fragilis- Abdominal infection
Non sporing anaerobe, Capsulated Gram Negative Bacilli
MC commensal in human intestine
MC anaerobe to cause infection
Endotoxin is less toxic than that of aerobic Gram Negative Bacilli
Fusobacterium necrophorum - agent of Lemierre's syndrome (is a form of thrombophlebitis)
161MULTIPLE CHOICE QUESTIONS
 
GENERAL
1. Characteristic of anaerobic bacteria is? (DNB June 2011)
  1. Foul smelling discharge
  2. Fail to grow in aerobic media
  3. Gas in tissue
  4. All of the above
2. Clostridia are? (DNB June 2010)
  1. Gram positive aerobe
  2. Gram positive obligate anaerobe
  3. Gram negative rods
  4. Gram negative bacillus
3. Non motile clostridia is: (JIPMER 2010, AI 2009)
  1. Cl. perfringens
  2. Cl.novyi
  3. Cl. botulinum
  4. Cl. difficile
4. Oval bulging terminal spores seen in: (AI 2008)
  1. Cl. tertium
  2. Cl. welchii
  3. Cl. perfringens
  4. Cl. Histolyticum
5. Sub terminal spores are found in: (PGI Dec 2008)
  1. Clostridium sordelli
  2. Clostridium sporogenes
  3. Clostridium difficile
  4. Clostridium tertium
  5. Clostridium botulinum
 
CLOSTRIDIUM PERFRINGENS
6. Best treatment for contaminated wound with necrotic material is: (AIIMS Nov 2013)
  1. Anti-gas gangrene serum
  2. Debridement
  3. Antibiotics
  4. Tetanus toxoid
7. Gas gangrene is / are caused by: (AIIMS May 2009, PGI June 2011)
  1. C.novyi
  2. C.septicum
  3. C.histolyticum
  4. C.perfringens
  5. C.tetani
8. Regarding Clostridium perfringens gas gangrene, false is: (PGI JUNE 2008, AIIMS May 2009)
  1. Common cause of gas gangrene
  2. Nagler reaction positive
  3. Most common toxin is Hyaluronidase
  4. Food poisoning strain of Cl. Perfringens produces heat resistant spores
9. Gastrointestinal enteritis necroticans is caused by: (PGI Dec 2000)
  1. Clostridium difficile
  2. Clostridium perfringens
  3. Botulinum
  4. Campylobacter jejuni
  5. Pseudomonas
10. The following statements are true regarding Clostridium perfringens except: (AI 2005)
  1. It is the commonest cause of gas gangrene
  2. It is normally present in human faeces
  3. The principal toxin of C. perfringens is the alpha toxin
  4. Gas gangrene producing strains of C.perfringens produce heat resistant spores
11. Regarding gas gangrene one of the following is correct: (SG PGI 2004, AI 2004)
  1. It is due to clostridium botulinum infection
  2. Clostridia species are gram-negative spore forming anaerobes
  3. The clinical features are due to the release of protein endotoxin
  4. Gas is invariably present in the muscle compartments
12. Most appropriate way to prevent gas gangrene is: (Recent Question 2013)
  1. Antitoxin
  2. Prophylactic parenteral antibiotics
  3. Extensive debridement
  4. Limb amputation
13. True about gas gangrene: (PGI May 2013)
  1. Underlying skin and muscles are normal
  2. Caused by tetanospasmintoxinxc
  3. Muscle rigidity & spasms are characteristic
  4. Most common organism implicated is C.perfringens
  5. Passive immunization does not help
 
CLOSTRIDIUM TETANI
14. Mechanism of action of tetanospasmin: (NEET Pattern Based)
  1. Inhibition of GABA release
  2. Inhibition cAMP
  3. Inactivation of Ach receptors
  4. Inhibition of cGMP
15. Which of the following is false for tetanus (AI 2011)
  1. Produces Heat resistant spores
  2. Person to person transmission doesn't not occur
  3. Incubation period is 6–10 days
  4. Three doses of primary vaccination is protective
16. Which of the following is a dead end infection? (DNB June 2010)
  1. Tetanus
  2. Vibrio
  3. Staphylococcus
  4. Hemophilus
17. Site of action of tetanus toxin: (JIPMER 2007)
  1. Pre synaptic terminal of spinal cord
  2. Post synaptic terminal of spinal cord
  3. 162Neuromuscular junction
  4. Muscle fibers
18. The following statements are true about DPT vaccine except: (AI 2004)
  1. Aluminium salt has an adjuvant effect
  2. Whole killed bacteria of Bordetella pertussis has an adjuvant effect
  3. Presence of acellular pertussis component increases its immunogenicity
  4. Presence of H. influenzae type b component increases its immunogenicity
19. True about tetanus: (PGI Dec 2004)
  1. Gram –ve spore forming organism
  2. Produces Telanolysin and Tetanospasmin
  3. Trismus and neck stiffness are early sign
  4. Generalized tonic-clonic seizure occurs on hyper stimulation
  5. Wound debridement is necessary
20. True about Clostridium tetani: (PGI Dec 2003)
  1. It is gram+ve
  2. Drum stick appearance
  3. Grows in aerobic environment
  4. It is gram-ve
  5. Toxin promotes GABA release
21. A 10 year old boy following a road traffic accident presents to the casualty with contaminated wound over the left leg. He has received his complete primary immunization before preschool age and received a booster of DT at school entry age. All of the following can be done except: (AIIMS 2001)
  1. Injection of TT
  2. Injection of human antiserum
  3. Broad spectrum antibiotics
  4. Wound debridement and cleaning
22. A person has received complete immunization against tetanus 10 years ago. Now he presents with a clean wound without any lacerations from an injury sustained 2.5 hours ago. He should now be given: (AI 2001, Tharnataka 2011)
  1. Full course of tetanus toxoid
  2. Single dose of tetanus toxoid
  3. Human Tet globulin
  4. Human Tet globulin and single dose of toxoid
23. The most effective way of preventing tetanus is: (AIIMS 2001)
  1. Surgical debridement and toilet
  2. Hyperbaric oxygen
  3. Antibiotics
  4. Tetanus toxoid
24. The causative organism can be isolated in which among the following conditions: (PGI Dec 2001)
  1. Serum in toxic shock syndrome
  2. Meningococcal rash
  3. Rheumatic valvulitis
  4. CSF in tetanus
  5. Diphtheritic myocarditis
25. All are done to prevent neonatal tetanus except? (AI 2007)
  1. Two TT doses to all pregnant
  2. TT to all females in the reproductive age
  3. TT to all new borne
  4. Penicillin injection to all new borne
26. Three doses of TT provide immunity for? (DPG 2006)
  1. 1 yr
  2. b. 5 yr
  3. 10 yr
  4. 15 yr
27. Wrong about Cl.tetani? (AIIMS Nov 2010)
  1. Main reservoir- Soil, human & animal intestine
  2. MC mode of transmission- Trauma & contaminated wound
  3. Herd immunity- not useful
  4. MC season –winter and dry
28. For neontatal tetanus elimination, the incidence should be….? (DNB 08)
  1. Less than 0.5 /1000 live birth
  2. Less than 1/ 1000 live birth
  3. Less than 0.1/ 1000 live birth
  4. Less than 10/ 1000 live birth
29. Period of communicability for tetanus? (UP 07, RJ 01)
  1. 7 days
  2. 14 days
  3. 21 days
  4. None
30. A person has got a non penetrating wound 11 hour back. He had taken complete course of TT 1 year back. What treatment is recommended next? (MP 09)
  1. Full course of tetanus toxoid
  2. Single dose of tetanus toxoid
  3. Nothing required
  4. Human Tet globulin and single dose of toxoid
31. For neontatal tetanus elimination. All are true except ? (MH 08)
  1. Incidence should Less than 0.1 /1000 live birth
  2. >90% TT coverage to the neonates
  3. TT2 coverage in pregnant women >90%
  4. Attended deliveries >75%
32. An adult with no immunization history presents with a clean non penetrating wound 2hrs back. What measure has to be taken? (Tharnataka 2011)
  1. Full course of tetanus toxoid
  2. Single dose of tetanus toxoid
  3. Nothing required
  4. Human Tet globulin and single dose of toxoid
33. False about tetanus? (Recent Question 2013)
  1. Opisthotonus position
  2. Trismus
  3. Unconsciousness till death
  4. Risus sardonicus
 
CLOSTRIDIUM BOTULINUM
34. Botulism is most commonly due to: (NEET Pattern Based)
  1. Egg
  2. Milk
  3. Meat
  4. Canned vegetables
35. Botulisim causes: (NEET Pattern Based)
  1. Descending flaccid paralysis
  2. Descending spastic paralysis
  3. Ascending paralysis
  4. Ascending spastic paralysis
36. Botulinum affects all except: (AI 2007, NEET Pattern Based)
  1. Neuromuscular junction
  2. Pre ganglionic junction
  3. Post ganglionic nerves
  4. CNS
37. 163An 18 year old male presented with acute onset descending paralysis of 3 days duration. There is also history of blurring of vision for the same duration. On examination, the patient has quadriparesis with areflexia. Both the pupils are non- reactive. The most probably diagnosis is: (AIIMS 2006)
  1. Poliomyelitis
  2. Botulism
  3. Diphtheria
  4. Porphyria
38. The following statements are true regarding botulism except: (AIIMS May 2003)
  1. Infant botulism is caused by ingestion of preformed toxin
  2. Clostridium botulinum A, B,C and F cause human disease
  3. The gene for botulinum toxin is encoded by a bacteriophage
  4. Clostridium baratti may cause botulism
39. Not true about botulinum toxin: (PGI Dec 2007)
  1. Short life span
  2. Increases acetylcholine release
  3. Used for the treatment of Blepharospasm, static and dynamic wrinkles
  4. Irreversibly decreases acetylcholine in neuromuscular junction
40. True about Clostridium tertium: (PGI June 2008)
  1. Gram variable
  2. Terminal spore
  3. Produces exotoxin
  4. Causes septicemic arthritis
41. Among the toxin produced by Clostridium botulinum, the non neurotoxic one is: (JIPMER 2000)
  1. A
  2. B
  3. C1
  4. C2
  5. D
 
CLOSTRIDIUM DIFFICILE
42. Cause of clostridium difficile associated diarrhea: (NEET Pattern Based)
  1. Trauma
  2. Dairy products
  3. Fried rice
  4. Antibiotic use
43. Pseudomembranous colitis is caused by? (DNB June 2012)
  1. Clostridium perfringens
  2. Clostridium difficile
  3. Clostridium tetani
  4. Clostridium botulinum
44. Clostridium difficile infection occurs after: (PGI June 2008)
  1. After prolong antibiotic therapy
  2. Pantoprazole increases the risk
  3. Associated with use of rectal thermometer
  4. Increased with proportion of hospital stay
45. Pseudo membranous colitis, all are true except: (AIIMS May 2007)
  1. Toxin A is responsible for clinical manifestation
  2. Toxin B is responsible for clinical manifestation
  3. Blood in stools is a common feature
  4. Summit lesions is an early histopathological finding
46. A patient of Acute lymphocytic leukemia with fever and neutropenia develops diarrhea after administration of amoxycillin therapy, which of the following organism is most likely to be the causative agent: (AIIMS Nov 2005)
  1. Salmonella Typhi
  2. Clostridium difficile
  3. Clostridium perfringens
  4. Shigella flexneri
47. True regarding pseudo membranous colitis are all except: (AI 2000)
  1. It is caused by clostridium difficile
  2. The organism is a normal commensal of gut
  3. It is due to production of phospholipase A
  4. It is treated by vancomycin
48. DOC for pseudo membranous enterocolitis: (AIIMS Nov 2014)
  1. Oral Vancomycin
  2. Penicillin
  3. Oral Ampicillin
  4. Clindamycin
 
NONSPORING ANAEROBES
49. With reference to Bacteroides fragilis the following statements are true, except: (AIIMS Nov 2012, AIIMS Nov 2011, AI 2007, AIIMS May 2006, AIIMS Nov 2006, AIIMS May 2003)
  1. B. fragilis is the most frequent anaerobe isolated from clinical samples
  2. B. fragilis is not uniformly sensitive to metronidazole
  3. The lipopolysaccharide formed by B. fragilis is structurally and functionally different from the conventional endotoxin
  4. Shock and disseminated intravascular coagulation are common in Bacteroides bacteremia
50. A patient presents with frontal abscess. Foul smelling pus is aspirated. Pus shows red fluorescence on ultraviolet examination. The most likely organism causing the frontal abscess is: (AIIMS May 2002)
  1. Bacteroides
  2. Peptostreptococcus
  3. Pseudomonas
  4. Acanthamoeba
164EXPLANATIONS
1. Ans. (d) (All of the above) Ref: Ananthanarayan 9/e p270, 8th/e p267
General feature of Anaerobic infection-
  • Foul smelling putrid Pus
  • Toxemia and fever not marked
  • Fails to grow aerobically- O2 is lethal to them
  • Usually Polymicrobial
  • Follows precipitating factors like trauma
  • Predisposing factor- DM, malnutrition, malignancy
  • Location near mucosal surface
  • Special feature like- Gas in specimen (Gas gangrene), black pigment that fluoresce (P. melaninogenica), sulfur granule (Actinomyces)
2. Ans. (b) (Gram positive obligate anaerobe) Ref: Ananthanarayan 9/e p251, 8th/e p267
  • Clostridia are Gram positive spore forming obligate anaerobe
3. Ans. (a) (Cl. perfringens) Ref: Ananthanarayan 9/e p252, 8/e p249
  • All Clostridia are motile except Cl. perfringens and Cl. tetani type VI.
  • Clostridia show stately type of motility.
  • All Clostridia are non capsulated except Cl. perfringens and Cl. butyricum.
4. Ans. (a) (Cl. tertium) Ref: Ananthanarayan 9/e p252, 8/e p249, Topley 10/e p1092
All clostridia produce bulging spore (spore are wider than the bacilli) except Cl. bifermentans and Cl. sordelli
Species
Nature of Spore
Cl tetani
Spherical and terminal spore (Drum Stick Appearance)
Cl. tertium and Cl.botulinum
Oval and terminal spore (tennis racket shaped)
Cl. bifermentans
Central /equatorial (spindle shaped), Non bulging
Cl. perfringens
Sub terminal and Club shaped
Clostridium sordelli
Sub terminal, oval, Non bulging
Clostridium sporogenes
Sub terminal, oval
Clostridium difficile
Sub terminal, oval
5. Ans. (a), (b), (c), (e) (Clostridium sordelli, Clostridium sporogenes, Clostridium difficile, Clostridium botulinum)
Ref: Ananthanarayan9/e p252, 8/e p249,Topley 10/e p1044, 1092 Refer the previous explanation.
Topley 10/e p1044 states –
“In Cl. botulinum and tetani, the spores are sub terminal to terminal giving rise to club or tennis racket appearance.”
So, what we suggest is, there is no clear cut distinction between terminal and sub terminal spores for Cl. botulinum and tetani. Both should be kept in mind and 1st preference should be “terminal” but if it is not there in the option, sub terminal can also be ticked.
 
CLOSTRIDIUM PERFRINGENS
6. Ans. (b) (Debridement) Ref: Harrison 18/e p1205
  • Contaminated wound with necrotic material is suggestive of higher risk of clostridial infection that may lead to gas gangrene.
  • Treatment of gas gangrene-
    • ✓ Emergent surgical exploration and thorough debridement are extremely important.
    • ✓ Hyperbaric oxygen therapy may be considered after surgery and antibiotics (Penicillin G plus Clindamycin (600–900 mg IV q6–8h) and anti-gas gangrene serum has been initiated.
1657. Ans. (a) (b) (c) (d) (C.novyi, C.septicum, C.histolyticum, C.perfringens) Ref: Ananthanarayan 9/e p257, 8/e p251
  • Cl. Perfringens Cl.novyi and Cl.septicum are the established Pathogens and Cl histolyticum is a Probable Pathogen among the agents of Gas gangrene. (For detail- Refer Chapter review)
8. Ans. (c) (Most common toxin is Hyaluronidase) Ref: Ananthanarayan 9/e p254, 8/e p251, Topley 10/e p1114
  • Most common toxin is ‘α toxin’ not ‘Hyaluronidase’
  • Cl. perfringens produces 4 Major (Lethal) Toxins – alfa, beta, iota, epsilon
  • Commonest cause of gas gangrene is Clostridium perfringens type A (60%)
About Other Options
Option b- Nagler reaction positive for Cl. perfringens and Cl.bifermentans
Nagler reaction:
  • Opalescence is produced surrounding the colony when Cl. perfringens is streaked on an egg yolk agar due to digestion of egg yolk protein by Lecithinase (α toxin).
  • Organism that secretes α toxin are-
  • Clostridium perfringens, Cl, sordelli, Cl novyi and Cl.bifermentans
  • But only Cl. perfringens and bifermentans toxin can be neutralized by Cl. perfringens antitoxin and give a positive Nagler reaction
Option d- Food poisoning strain of Cl. perfringens (mostly Type A) characterized by their marked heat resistant spores but feeble production of α and theta toxin.
Instead they produce heat labile enterotoxin (similar to LT of E.coli)
Also remember-
‘Gas gangrene producing Cl. perfringens strains don't produce Spores in tissue/media.
9. Ans. (b) (Cl. perfringens) Ref: Ananthanarayan 9/e p255–56, 8/e p254
Enteritis necroticans (Pigbel in New Guinea)-
  • Caused by Cl. Perfringens type-C strains with heat resistant spores
  • Spores germinates in intestine producing beta toxin
  • Usually following a pig meat diet along with a trypsin inhibitors like sweet potato
10. Ans. (d) (Gas gangrene producing strains of C. perfringens produce heat resistant spores)
Ref: Ananthanarayan 9/e p258, 8/e p249–254
  • “Gas gangrene producing strains of C.perfringens don't form spores in tissue/media
  • Whereas, Food poisoning strain of Cl. perfringens (mostly Type A) and Enteritis necroticans causing Cl. perfringens type-C strains are characterized by their marked heat resistant spores”
About Other Options-
  • Commonest cause of gas gangrene is Clostridium perfringens type A (60%)
  • “Some of the pathogenic Clostridia like Cl. perfringens, Cl. tetani and Cl difficile are normally present in human and animal faeces”
  • Predominant gastrointestinal Clostridia are Cl ramosum followed by Cl. perfringens.
  • The principal toxin of C. perfringens is alpha toxin
11. Ans. (d) (Gas is invariably present in the muscle compartments) Ref: Ananthanarayan 9/e p257–58, 8/e p255
Gas gangrene results when Clostridia invade muscle producing abundant toxin resulting in extensive necrosis of muscle compartments accompanying with gas.
About Other Options-
  • Option a - Gas gangrene is due to Cl. perfringens, but not Cl botulinum.
  • Option b- Clostridia are gram positive spore forming anaerobes
  • Option c- Gas gangrene is due the release no. of exotoxin like alfa toxin. More so, since it is gram+ve organism, it doesn't have endotoxin.
12. Ans. (c) (Extensive debridement) Ref: Ananthanarayan 9/e p258 Surgical debridement is the mainstay of treatment of gas gangrene.
16613. Ans. (d) (Most common organism implicated is C.perfringens) Ref: Ananthnarayan 9 /e p254
  • Most common organism implicated in Gas gangrene is Clostridium perfringens followed by C.novyi and C.septicum
  • Underlying skin and muscles are gangrenous
  • Gas gangrene is caused by α-toxin
  • Pain and crepitus in muscle are characteristic of Gas gangrene
  • Passive immunization – Anti α- toxin (anti gas gangrene serum) is of great value for the treatment of Gas gangrene.
 
Clostridium Tetani
14. Ans. (a) (Inhibition of GABA release) Ref: Ananthanarayan 9/e p262, 8/e p258–61, Park 22/e p115 & 21/e p113
  • Mechanism of action of tetanospasmin is: - Acts presynaptically & inhibits of glycine & GABA release that leads to spastic contraction of muscles.
15. Ans. (d) (Three doses of primary vaccination is protective) Ref: Ananthanarayan9/e p262, 8/e p258–61, Park 22/e p115 & 21/p113 According to National immunization schedule:
  • Primary course of immunization includes:
  • “3 doses of DPT at 4–8 week apart followed by booster of PDT at 16–24 month followed by booster of DPT at 5–6 year followed by booster of TT at 10 and 16 year.” ………Park 22nd /p115& 21 e/p113
About Other options:
  • C. tetani produces heat resistant spores
  • Person to person transmission doesn't not occur in tetanus
  • Incubation period of tetanus is 6–10 days.
16. Ans. (a) (Tetanus) Ref: Ananthanarayan 9/e p261,8th/e p260
  • Tetanus is non infectious i.e. there is no man to man transmission. Man acts as dead end.
Other options- All are transmitted by carriers
  • Vibrio-Fecal carries are common
  • Staphylococcal carriers – skin and anterior nare
  • Hemophilus carrier- in nasopharynx
17. Ans. (a) (Pre synaptic terminal of spinal cord) Ref: Ananthanarayan 9/e p261,8/e p258–61
  • Tetanospasmin – Blocks release of inhibitory transmitters glycine and GABA at CNS and Pre synaptic terminal of spinal cord → leads to spastic paralysis
  • The manifestations of Tetanus and strychnine poisoning are similar except Tetanospasmin acts at Pre-synaptic terminal where as strychnine acts at post synaptic terminal.
18. Ans. (d) (Presence of H. influenzae type b component increases its immunogenicity)
Ref: Ananthanarayan 9/e p330, 8/e p332)
  • In Quadruple vaccine (DPT+ Hib) – DT and TT components increase immunogenicity of Hib
  • “Phospho-ribosyl pyrophosphate (PRP) of Hib is poorly immunogenic in <2year and immunogenicity can be increased by adding protein carrier like DT and TT” …………Ananthanarayan 9/e p330,8/e p332
  • However, Hib never increases immunogenicity of DPT.
About Other Options-
  • Both Diphtheria and Tetanus toxoid are aluminum phosphate Adsorbed toxoid increases immunogenicity
  • Both whole cell killed B. pertussis and acelluar pertussis component increase potency of DT and TT (adjuvant effect).
  • The only difference is, “Whole cell killed B. pertussis is encephalogenic and immunity is short lasting while acellual pertussis component (DTaP) is devoid of neurological complication.”
19. Ans. (b), (c), (d), (e) (Produces tetanolysin and tetanospasmin, Trismus and neck stiffness are early sign, Generalized tonic-clonic seizure occurs on hyper stimulation Wound debridement is necessary)
Ref: Ananthanarayan 9/e p261, 8/e p257- 260
  • Option a- Cl. tetani is Gram +ve spore forming organism,
  • 167Option b- tetanolysin and Tetanospasmin are the principle toxin produced by Cl.tetani
  • Option c- 1st symptom-↑masseter tone(trismus/lock jaw) then→ descending tetanus
  • Option d- “Tetanus patient should be isolated to protect them from noise and light which may provoke convulsion” …………Ananthanarayan 8/e p260
  • Option e- Prophylactic measures include- Wound toilet, antibiotic and most importantly immunization.
20. Ans. (a), (b) (It is gram+ve, Drum stick appearance) Ref: Ananthanarayan9/e p259, 8/e p257
  • Cl. tetani is Gram +ve spore forming strict obligate anaerobe
  • Possess terminal round spore
  • Option e-Toxin inhibits GABA release
21. Ans. (b) (Injection of human antiserum) Ref: Park 22/e p286–87 & 21/e p287
  • Treatment of tetanus consists of (Wound debridement + Antibiotic + immunization)
  • For treatment of wound injury, we have to know the type of wound and the immune category that the patient belongs to.
  • This history is suggestive of-
    • ✓ 10 year old child Taken complete immunization with booster of DT at school entry age – indicates taken complete immunization >5 to <10 year back – So belongs to Immunity category B
    • ✓ Type of wound- contaminated (other wound category)
    • ✓ Treatment required for Category B with contaminated (other wound)- Toxoid 1 dose
  • Antibiotic has role to eradicate the source of toxin (No role after 6 hour when the toxin is already formed)
  • HT Ig has role in category C and D persons i.e. when the complete immunization taken >10 year or the immunization is not complete/unknown.
Treatment: All type of wounds need surgical toilet followed by: Immunization. (Refer chapter review)
22. Ans. (b) (Toxoid 1 dose) Ref: Park 22/e p286 & 21/e p287
  • Type of wound- clean wound without any lacerations from an injury sustained 2.5 hours ago
  • Patient belongs to Immunity category C - complete immunization taken 10 years ago
  • So, Treatment required for Category C with simple wound is - Toxoid 1 dose
23. Ans. (d) (Tetanus toxoid) Ref: Park 22/e p286, Ananthanarayan 9/e p 263,8/e p260
  • Treatment of tetanus consists of (Wound debridement + Antibiotic + immunization)
  • The most effective way of preventing tetanus is- Tetanus toxoid
  • The most effective way of preventing gas gangrene is- Surgical debridement and toilet
24. Ans. (a), (b) (Serum in Toxic Shock Syndrome, Meningococcal rash)
Ref: Topley 10/e p813, Harrison 16th/p828& 18/e p1117
  • Option a- Lab test for diagnosis of Toxic Shock Syndrome includes isolation of S.aureus from mucosal or normally body site (Topley 10/e p813) and isolation of Str. pyogenes from mucosal or normally body site like blood (Harrison 16th /p828)
  • Option b- “Meningococci may sometime be demonstrated in petechial lesions by culture.” …………Ananthanarayan 8/e p226
  • Rheumatic fever is due to cross reacting antigen between Strept M protein and human myocardium.
  • Diphtheria and tetanus - both can cause toxemia but never bacteremia.
25. Ans. (d) (Penicillin injection to all new borne) Ref: Park 22/p286, 21/p287
  • ATS is recommended to all neonates borne to unimmunized mother. (Detail- refer chapter review)
26. Ans (c) (10 yrs) Ref: Park 22/e p286, 21 e/p287
  • Individuals sustaining tetanus-prone wounds should be immunized if their vaccination status is incomplete or unknown or if their last booster was given >10 years earlier
  • Two doses of immunization provides immunity for several years….. Park 22/p285
27. Ans. (d) (MC season –winter and dry) Ref: Park 22/e p284–85,21 e/p283–84
  • Tetanus shoes seasonal variation, in India, >50% of cases occur in- July to September
  • Main reservoir- Soil, human & animal intestine
  • MC mode of transmission- Trauma & contaminated wound
  • Herd immunity- not useful for tetanus
28. Ans (c) (Less than 0.1/ 1000 live birth) Ref: Park 22/e p286,21 e/p287
16829. Ans (d) None Ref: Park 22/e p284,21 e/p285
Tetanus is NOT infectious from man to man.
30. Ans (b) (Single dose of tetanus toxoid) Ref: Park 22/e p286,21 e/p287
  • It is a case of Simple wound (Non penetrating wound 1hr back) and category C (h/o complete course of TT 11 yr).
  • The recommendation for simple wound and category C is - Single dose of tetanus toxoid
31. Ans (b) (>90% TT coverage to the neonates) Ref: Park 22/e p284,21 e/p283
  • TT coverage to the neonates is not included in the strategies for elimination of neonatal tetanus.
32. Ans (a) (Full course of tetanus toxoid) Ref: Park 22/e p286,21 e/p287
  • It's a case of simple wound (clean and non penetrating) and category-D (unimmunized)
  • The recommendation for simple wound and category D is - Full dose of tetanus toxoid
33. Ans. (c) (Unconsciousness till death) Ref: Ananthanarayan 9/e p261
 
Clostridium Botulinum
34. Ans. (d) (Canned vegetables) Ref: Park 22/e p217
  • The most common food associated with botulism are home preserved foods such as home canned vegetables, smoked or pickled fish, homemade cheese and similar low acid food.
35. Ans. (a) (Descending flaccid paralysis)
  • Botulism blocks acetyl choline release, hence causes Descending flaccid paralysis
36. Ans. (d) (central nervous system) Ref: Harrison 17th/p901, 16th /p843 & 18/e p1200 “Botulinum neurotoxin, acts on - peripheral cholinergic nerve terminals, including
  • ✓ Neuromuscular junctions,
  • ✓ Postganglionic parasympathetic nerve endings,
  • ✓ Peripheral ganglia.
  • ✓ The central nervous system is not involved”
37. Ans. (b) (Botulism) Ref: Harrison 17/e p901 &18/e p1201, Ananthanarayan 9/e p264,8/e p262
Points in favor-
Descending paralysis, blurring of vision, quadriparesis (flaccid paralysis), areflexia and non-reactive pupils
Also know-
  • Causes of Descending paralysis – Tetanus, Botulism, Polio, Diphtheria
  • Tetanus causes spastic paralysis where as botulism causes flaccid paralysis
38. Ans. (a) (Infant botulism is caused by ingestion of preformed toxin)
Ref: Harrison17th/p901&18/e p1200, Ananthanarayan 9/e p264, 8/e p262
  • “Infant botulism occurs due to ingestion of spores, toxin released inside, Whereas Food borne botulism occurs due to preformed toxin mixed with canned food”
About Other option-
  • Option b- Toxin types A, B, E, and (rarely) F cause disease in humans;
  • Type G has been associated with sudden death, but not with neuroparalytic illness,
  • Type C and D cause disease in animals.
  • Option c- Atleast Type C,D- are bacteriophage coded
  • Option d- “Rare strains of other clostridia species—C. butyricum and C. baratti—have been found to produce botulism toxin.” ………Harrison 17/p901 & 18/e p1200
39. Ans. (b) (Increased acetylcholine release) Ref: Harrison 17/p901 & 18/e p1200, Ananthanarayan 9/e p264,8/e p262
  • Botulinum toxin Site of action-Pre synaptic neuromuscular junctions, postganglionic parasympathetic nerve endings, and peripheral ganglia
  • 169BT is used in the Treatment of neuromuscular disorder like strabismus, blepharospasm, wrinkle, myoclonus. (Since Botulism toxin produces flaccid paralysis)
  • The action is short lasting and recovery occurs in 2–4 months once the new terminal axon spourts form and restore transmission.
  • Blocking of Ach release from nerve terminal is permanent and recovery occurs only by sprouting of new nerve terminals.
40. Ans. (a), (b), (d) (Gram variable, Terminal spore, causes septic arthritis)
Ref: Bailey and Scott 12th /p465 Table- 44.1
41. Ans. (d), (C2) Ref: Ananthanarayan9/e p264, 8/e p262
All subtypes (A-G) of Botulinum toxin are pharmacologically similar (neurotoxin) but antigenically different, except C2 (enterotoxin)
 
Clostridium Difficile
42. Ans. (d)(Antibiotic use) Ref: Ananthanarayan 9/e p265,8/e p263
  • Prolonged broad spectrum antibiotic therapy is the most important risk factor for Clostridium difficile associated diarrhea.
43. Ans. (b)(Clostridium difficile) Ref: Ananthanarayan 9/e p265,8/e p263
  • Pseudo membranous colitis is caused by Clostridium difficile
44. Ans. (a), (b), (c), (d) (After prolong antibiotic therapy, Pantoprazole increases the risk, Associated with use of rectal thermometer, Increased with proportion of hospital stay) Ref: Harrison 17th/p818 & 18/e p1091
Risk factors for Pseudo membranous colitis -
  • Prolonged broad spectrum antibiotic therapy, Length of hospital stay.
  • Older age, Greater severity of underlying illness,
  • Gastrointestinal surgery, Use of electronic rectal thermometers,
  • Enteral tube feeding, Antacid treatment, Proton pump inhibitors
Decreased Risk-
  • Patients pre colonized with C. difficile – Less Risk as they possess IgG antibody
  • Infant has less risk than adult as they lack mucosal toxin receptor
45. Ans. (c) (Blood in stool is a common feature) Ref: Harrison 17/e p818 &18/e p1092, Topley 10/e p1120 Harrison 17/e p 818 & 18/e p1092, says- Stools are almost never
Grossly bloody and range from soft and unformed to watery or mucoid in consistency, with a characteristic odor.
About Other option
  • Cl. difficile Toxins: A→Enterotoxin B→Cytotoxin
  • Both are required for manifestation.
  • In the earliest stage, Summit lesion i.e. tiny superficial intercryptal erosions may be found. …………….Topley10/e p1120
46. Ans. (b) (Clostridium difficile) Ref: Harrison 17/e p 8 & 1818/e p 1091, Ananthanarayan 9/e p265,8/e p263
  • Points favoring- diarrhea after administration of amoxicillin therapy
17047. Ans. (c) (It is due to production of phospholipase A)
Ref: Harrison 17/e p818–20 & 1818/e p1091–93, Ananthanarayan 9/e p265,8/e p263
C.difficile has two toxins -Toxin A-Enterotoxin and Toxin B- Cytotoxin
About Other options-
  • Option a- Pseudo membranous colitis is caused by Cl. difficile
  • Option b- Cl. difficile is found as a normal GIT flora, so merely isolation of the organism is not sufficient for diagnosis.
  • Harrison 17/e p903 states – Clostridia are present in the normal colonic flora at 109–1010/g.
  • C. ramosum is the most abundant, followed by C. perfringens.”
  • Option d- Treatment of choice –Metronidazole, Alternate- Vancomycin
48. Ans. (a) (Oral Vancomycin) Ref: Harrison 18/e p1093–94
Etiological agent for pseudomembranous enterocolitis is Clostridium difficile.
Treatment of Clostridium difficile infection-
  • Initial episode, mild to moderate cases- Oral metronidazole is the drug of choice (500 mg thrice a day for10–14 days)
  • Recurrent episodes or severe cases- Vancomycin is the drug of choice(500 mg, four times a day for 10–14 days)
  • Severe complicated or fulminant infection- Combination of vancomycin (given via nasogastric tube and by retention enema) and IV metronidazole has been recommended.
 
Nonsporing Anaerobe
49. Ans. (d) (Shock and disseminated intravascular coagulation are common in Bacteroides bacteremia)
Ref: Harrison 17/e p810 & 18/e p1332, 1338, Ananthanarayan 9/e p269, 8/e p267
“B. fragilis LPS are 100–1000 times less biologically potent than endotoxin associated with aerobic gram-negative bacteria. This accounts for the lower frequency of DIC and purpura in Bacteroides bacteremia than in facultative aerobic gram-negative bacillary bacteremia.”
About Other options-
Option a- “Bacteroides are the most common anaerobes isolated in clinical specimen.” ………Ananthanarayan 9/e p269,8/e p267
Option b- “Metronidazole is active against gram-negative anaerobes, including the B. fragilis group; resistance is rare but has been reported” ………Harrison 17/e p810 & 18/e p1332, 1338
50. Ans. (a) (Bacteroides) Ref: Ananthanarayan 9/e p269,8/e p267
  • Frontal abscess with foul smelling pus- Points towards anaerobic infection
  • Fluorescence on ultraviolet examination – indicates Prevotella melaninogenica which is a type of Bacteroides infection.
Bacteroides
Most common anaerobes isolated in clinical specimen.
Virulence factor- capsular polysaccharide and LPS
Classified to-
Porphyromonas (Asaccharolytic)
Prevotella (moderately Saccharolytic)
Bacteroides proper (Saccharolytic) – B. fragilis
Also know, the Anaerobic/Aerobic Ratio in human GIT flora- (Harrison 17/e p 903& 18/e p1332,1338)
1:1 (stomach, ileum, jejunum, saliva)
103: 1 (Terminal ileum and colon and Gingival crevices)
10:1 (Female genital tract)

MycobacteriumCHAPTER 3.6

 
ACID FAST ORGANISM
  • Acid fastness is due to mycolic acid and integrity of cell wall
    • Mycobacteria - TB/leprae/NTM
    • Parasites –
    • Nocardia
    • Cryptosporidium,
    • Rhodococcus
    • Cyclospora.
    • Spore
    • Isospora,
    • Sperm head
    • Tinea saginata egg & scolex
    • Legionella mecdedei
    • Hooklet of hydatid cyst
  • Mycobacterium – Name was derived from their branching filamentous form and mould like pellicle in liquid media
 
M TUBERCULOSIS COMPLEX – INCLUDE SPECIES
  • M. tuberculosis
  • M. bovis (bovine tubercle bacillus)
  • M. africanum (intermediate between M tuberculosis and M bovis)
  • M. microti (vole tubercle bacillus)
M tuberculosis
M bovis
Shape- Curved, long, beaded, less uniformly stained
Straight, short, stout, uniformly stained
Rough, tough, buff colony
White, smooth, moist colony
Eugonic growth
Dysgonic growth
Glycerol enhances growth
Glycerol inhibits growth
Resistant to TCH (Thiophene-2carboxylic acid hydrazide)
Sensitive
Niacin test +ve Nitrate test +ve,
Niacin -ve Nitrate -ve
Pathogenic to Guinea pig, but Not to rabbit
Pathogenic to both
Obligate aerobe
Microaerophilic
Both are equally pathogenic to human
 
Pathogenicity
  • Escape killing by macrophage by inhibiting phagolysozome fusion and HSN IV
  • Virulence factor- cord factor, lipoarabinomannan, HSP
CLINICAL FEATURE:
Pulmonary TB:
Primary TB:
  • Affect children,
  • Sub pleural focus in lower lobe of lungs (Ghon)+ hilar LN↑= primary complex
  • Ghon focus + Fibrosis and calcification- Ranke complex.
172Post primary TB:
  • Affect Adult,
  • Upper lobe focus (Simons focus),
  • LN spread rare, hematogenous spread seen
  • Necrosis, cavitations seen
  • Infra clavicular lesion- Assman focus.
Extrapulmonary TB-
  • MC site LN (cervical) followed by pleural TB
  • Skeletal TB- MC Site is spine
  • TB meningitis
  • GI TB- MC site caecum, ileum
  • TB pericarditis- MC cause of chronic constrictive pericarditis
  • Scrofuloderma- Skin lesion d/t breakdown of underlying TB foci (usually lymph node)
  • Lupus vulgaris-
    • MC TB skin lesion,
    • Female,
    • Face and neck,
    • Apple jelly nodule,
    • ↑ Risk for malignancy (Sq cell Ca)
 
Laboratory Diagnosis
Digestion and Decontamination procedures for sputum:
  • ✓ Petroff's method (4%Na OH)
  • ✓ Others- N acetyl cysteine, Oxalic acid
AFB smear:
Ziehl Neelsen staining- detection limit 104 bacilli/ml of sputum
  • ✓ RNTCP recommends to use 25% sulfuric acid
  • ✓ Used for monitoring treatment, assessing the severity, assessing the infectiousness of the patient
  • ✓ Two sputum samples are collected- early morning and spot
  • ✓ Sputum microscopy- Method of choice case finding tool for tuberculosis under RNTCP.
• Acid alcohol is used as decolorizer – for renal tuberculosis to differentiate M.tuberculosis (acid fast & alcohol fast) from M.smegmatis (commensal, acid fast but not alcohol fast)
Fluorescent staining- Auramine /rhodamine- More rapid method (Screened under low power)
Culture: Detection limit 10 to 100 viable organism,
  • Incubation – 6–8 week, Generation time 14 – 15 hours
  • Sputum Culture facility available at district and regional centers
  • Only done for chest symptomatic but smear negative patient, for drug sensitivity and monitoring.
Solid media-
  • ✓ Egg Based- Lowenstein Jensen, Petragnani, Dorset egg media
  • ✓ Blood Based Media – e.g. Tarshismedia
  • ✓ Agar Based Media –Transparent; colonies observed earlier- Middle brook 7H1 1and 7H10
  • ✓ 7H11 is preferred as it improves the recovery of INH resistant strains of M. tuberculosis
Liquid media– Middle brook 7H9, Dubos, Proskauer, Sula, Sauton –
• Used for drug susceptibility, Antigen and vaccine
Automated culture method – detects growth faster
  • ✓ BATEC–Radiometric detection of 14 CO2 using 14 C labeled substrates (palmitic acid)
  • ✓ BacT/ALERT–colorimetric detection of pH change by liberating CO2 by M tuberculosis
  • ✓ ESP – detection of change in pressure by production of CO2 by M tuberculosis
  • ✓ MGIT – Mycobacteria Growth Indicator Tube
  • ✓ Septi check system–(Biphasic) - Contain 3 solid media (modified LJ, middle brook 7H11 and chocolate agar)
  • ✓ PCR detecting IS6110 gene
  • ✓ Serology Not much useful
173Diagnosis of latent TB
Risk Group for whom ATT prophylaxis is recommended (Harrison 17/e p1019, 18/e p1357)
Cut off for +ve Tuberculin (mm)
  • HIV-infected persons or persons receiving immunosuppressive therapy
  • Close contacts of tuberculosis patients especially children
  • Persons with fibrotic lesions on chest radiography
>5mm
  • Recently infected persons (<2 years)
>10mm
  • Persons with high-risk medical conditions-
  • DM, hematologic diseases, injection drug use, end-stage renal disease
>10mm
  • Low-risk person (for employment purpose)
  • Otherwise treatment is not indicated for these low-risk persons.
>15mm
 
BCG → Bacilli Calmette Guerin
  • BCG strain- M bovis attenuated in glycerol bile potato medium, 239 subcultures done over 13 years
  • WHO recommends Danish 1331 strain of BCG
  • In India, it is prepared in Guindy, Chennai
  • BCG is available in lyophilized form
    • Hence, it has to be reconstituted by Normal saline as a diluent (Distilled water is never used),
    • It has to be administered within 1 hr of reconstitution
  • Dose and strength- 0.1ml containing 0.1mg TU (0.05ml- for neonates)
  • Alcohol- not used to wipe the skin
  • Site- above insertion of left deltoid
  • Route- By Intradermal route by using a 26 gauge tuberculin syringe
  • Phenomena after BCG-
    • After 2–3week- papule
    • 5–6-weeks- shallow ulcer covered with crust
    • 6–12 weeks- Permanent tiny round scar (4–8mm diameter)
    • 8–14 weeks- Mantoux test becomes positive
  • Protective Efficacy-0–80%, Duration-Short lasting immunity -last for 15–20yr
  • MC complication- ulceration, LN
  • Indication-
    • Direct BCG- BCG is given to Newborn soon after birth directly, no need after 2 year. Strategy followed by most of the developing countries.
    • 174Indirect BCG- BCG is given after performing tuberculin test, recommended after 1 year. BCG to sensitized individual might result in a local complications, similar to Koch's phenomenon
  • Contraindication- HIV+ve, AFB+ve mother, ↓Immunity, generalized eczema, pregnancy & malignancy.
  • Other BCG strains-
    • Montreal strain
    • Sanofi-Pasteur's BCG (using Glaxo 1077 strain)
    • Tokyo 172 strain
    • OncoTICE-used for bladder cancer,
Sensitivity Testing:
Drug Resistance- Main mechanism is due to mutation. Genes involved are:
  • Resistance Ratio
  • INH - Kat G gene, Inh A gene, ahpC
  • Absolute concentration
  • R - rpoB gene (RNA polymerase B)
  • Proportion method – RNTCP recommended
  • Z - Pnc A (Pyrazinamidase)
  • Radio metric method
  • E - emb A, B, C (Arabinosyl transferase)
  • Molecular methods
  • S - ribosomal protein subunit 12 (rpSL)
MRDTB
Resistant to INH and Rifampicin +/ Resistant to other 1st line drug
3.7% of new cases are MDRTB
Level higher in previously treated cases (20%)
60% of total MDRTB resides in BRICS countries- brazil, Russia, India, China, South Africa
XDRTB
MDRTB + Resistant to quinolone + Resistant to aminoglycoside (amikacin/ capreomycin/ kanamycin)
9% of MDR TB are XDR TB
Neonatal Tuberculosis:
  • Before delivery:
    • If mother chest X-ray and sputum AFB +ve then, mother is given ATT (HRE)
  • After delivery –If mother chest X-ray and sputum AFB +ve then-
  • 3 Dos:
    • Mother to be given ATT(HRE)
    • Baby (INH for 9–12 months)
    • Screening of household contacts
  • 3 Don'ts:
    • Don't separate baby from mother
    • Don't with hold of breast feeding
    • Don't give to baby BCG
 
NTM (NON TUBERCULOUS MYCOBACTERIA)/ATYPICAL MYCOBACTERIA
NTM differentiated from MTB complex by:
  • Resistance to paranitrobenzoic acid & TCH
  • Aryl sulfatase test +ve
  • Strong Catalase +ve
  • Non pathogenic for guinea pig but pathogenic for mouse
  • Resistant to antitubercular drugs
175Atypical (environmental) Mycobacteria
Runyon group
Species
I Photo chromogen
Pigmentation only in light
M marinum, M. asciaticum, M.simiae, M. kansasii, (Code- MASK)
II Scoto chromogen
Pigmentations in light and dark
M. scrofulaceum, M szulgai, S.gordonae (Code- SSG)
III Non-chromogen
No Pigmentation
MAC, M xenopi M. ulcerans
IV Rapid growers
Grows within a week
M chelonei, M fortuitum, M.smegmatis, M.abscessus
 
Diseases Caused by Atypical Mycobacterium
  • Post trauma/injection abscess - M chelonei, M fortuitum
  • Swimming pool or fish tank granuloma - M marinum
  • Buruli ulcer - M. ulcerans
  • Mycobacteria Causing Johnes Disease -M. paratuberculosis
  • Lymphadenopathy - M. scrofulaceum
  • Pulmonary disease - M kansasii
  • Disseminated disease- M avium intracellulare
  • Organisms requiring incubation at low temp of 25 to 33°C- M ulcerans
 
MYCOBACTERIUM LEPRAE
  • Non cultivable on artificial medium & cell line(doesn't follow the Koch's postulate)
  • Generation time- 20 days
  • Grows well in cooler part of body(skin, testes, peripheral nerve, anterior eye)
  • Acid Fast to 5% H2 SO4
  • Arrangement of the Lepra bacilli-
    • Intracellularly as parallel cigar bundles of bacilli bound with lipid like glia (globi) present inside foamy macrophage (Virchow's leprae cell)
  • Bacteriological index (BI)– total no of leprae bacilli
  • Morphological index (MI)- The % of solid uniformly stained live bacilli in tissues
  • MI is more meaningful for assessing the progress of patients on chemotherapy.
 
Epidemiology of Leprosy
  • Social disease or Hansen's disease- one of the oldest disease of mankind
  • Transmission- nasal secretion > direct contact>breast milk, vertical mode, tattooing
  • Incubation Period- 2–5 year
  • Not highly communicable, intimate and prolong contact necessary
  • Most affected area- Southeast Asia and Brazil- In India (MC-Bihar>Orissa>UP)
  • Situation in India-
    • India has achieved the status of ‘leprosy elimination in Dec 2005- < 1case/10,000.
    • 32 states have achieved the status except Bihar, Chhattisgarh and Dadra & Nagar Haveli
    • Prevalence of leprosy in India 0.72/10,000 population
    • MBL accounts for 48%, children- 9.9% only, Cure rate- 90–95%
 
Animal Model
  • Foot pad of mouse (Shepard model)
  • Nine banded armadillo is highly susceptible to leprosy, due to low body temperature.
  • Other - Korean chipmunk, Indian pangolin, Slender loris, European hedgehog
 
Antigenic Structure
  • LAM-B (Lipoarabinomannan B) is highly immunogenic and used in serodiagnosis of leprosy.
  • PGL – 1 (phenolic glycolipid 1)
 
176Classification
  • Ridley Jopling classification: TT, BT, BB, BL, LL
  • Madrid classification – LL, TT, borderline/dimorphous, indeterminate (early unstable type)
  • Indian classification – Madrid + pure neuritic type
Lepromatous
Tuberculoid
Multibacillary(Bacteriological index (4 -6+)
Paucibacillary(Bacteriological index (0–1+)
Skin lesion many, symmetric, poorly marginated, multiple infiltrated nodules and plaques, xanthoma- like or dermatofibroma papules; leonine facies and eyebrow alopecia
One or a few sharply defined annular asymmetric macules or plaques with a tendency toward central clearing, elevated borders
Nerve lesion- late
Early anesthetic skin lesion, enlarged thickened nerves, nerve abscess (MC in BT)
CMI low, AMI exaggerated
CMI & AMI both-normal
Lepromin test - ve
Lepromin test +ve
Lymphocyte transformation test -ve
Lymphocyte transformation test +ve
CD8+/CD4+ T cell ratio in lesions: 2:1
CD4+/CD8+ T cell ratio in lesions- 2:1
Humoral- Auto Antibodies +ve
Humoral- Auto Antibodies –ve
VDRL test -Biological false +ve
VDRL test –ve
PGL-1 (Phenolic Glycolipid) antibodies- 95% of cases elevated
PGL-1 antibodies- 60% of cases elevated
Macrophages- foamy type
Macrophages- epitheloid type
Langhans giant cells- not seen
Langhans giant cells- found
Involve any organ – except CNS and lungs, also warm area of skin (axilla, groin, scalp)
MC nerve involved- Ulnar >post auricular
Medial popliteal N- never involved
Lepra Reaction Type I-
Lepra Reaction Type II -
  • Type of hypersensitivity type IV,
  • Seen with borderline leprosy
  • Inflammation of previous lesions, neuritis
  • If occurs before t/t – down grading reaction occurs towards LL
  • If occurs after t/t – upgrading or reversal reaction occurs towards TT
  • MC feature- edema
  • Reversal reactions- ↑Th1 response,
  • MC nerve- Ulnar Nerve
  • Associated with ↑Tcell (γδ TCR)
  • Treatment DOC- glucocorticoid
  • ENL- Erythematous Nodosum Leprosum
  • Type of hypersensitivity type -III,
  • Seen with Lepromatous variety (BL, LL)
  • Usually follows sulfonamide therapy, but may precede treatment
  • MC- crop of painful erythematous papule
  • Central cytokine - TNFα & Th2 cytokines
  • T/T- DOC- glucocorticoid, thalidomide, clofazimine and antipyretics
  • Type III Lepra Reaction - Lucio phenomena- HSNIII, severe form, associated with untreated diffuse LL, both thalidomide, glucocorticoid are not affective, seen in people from Caribbean and Mexico.
 
Lepromin Reaction0.1 ml Lepromin given Intradermally to Inner Forearm
  • Early/Fernandez – Reading taken at 2–3 days- Induration like tuberculin (DTH)- indicates infection in past (not useful)
  • Late/Mitsuda reaction - Reading taken 3 weeks (21 days) → +ve if :-nodule of > 5mm
    • It Measure of CMI induced by injected Lepromin (doesn't say about the past exposure)
    • Uses of Lepromin test–
      • ✓ Classify lesions of leprosy,
      • ✓ Assess prognosis,
      • ✓ Assess resistance to leprosy in individuals
177Lab diagnosis:
  • Sample-Total 6 sample-Minimum 4 skin (slit skin smear from edge, NOT centre)+ ear lobule+ nasal mucosa
  • Acid fast staining with 5% sulfuric acid
  • Grading of smear is done based on MI.
  • Mouse food pad inoculation
  • Antibody to PGL1
 
Treatment
  • Paucibacillary leprosy:
    • Given for-I, TT, BT and if bacteriological index <2, skin lesion : 1–5, one nerve involvement
    • Regimen- Rifampicin (monthly)+ dapsone daily – for 6 months,
    • Follow up till 2 yrs.
  • Multibacillary leprosy:
    • Given for BB, BL, LL if bacteriological index ≥ 2, skin lesion : >5, nerve involvement : ≥1
    • Regimen- Rifampicin (monthly)+ dapsone daily +clofazemine daily, till 1 yr or smear –ve,
    • Follow up annually till 5 yrs.
Anti -leprosy Vaccines : (experimental)
  • BCG- protectivity 20–40%
  • Armadillo-derived killed M leprae
  • BCG and Killed M leprae
  • ICRC bacillus.
Leprosy is not amenable to eradicate because
  • Long and variable IP
  • Disputed mode of transmission
  • Subclinical cases
  • CMI low (in LL)
  • Absence of effective vaccine
  • Bacterial resistance
  • Complicated spectrum of disease
  • Social issues.
178MULTIPLE CHOICE QUESTIONS
 
MYCOBACTERIUM
1. Niacin test is not positive in: (PGI Nov 2014)
  1. Mycobacterium tuberculosis
  2. Mycobacterium cheloneii
  3. Mycobacterium bovis
  4. Mycobacterium simiae
2. True about tuberculin test: (PGI May 2013)
  1. Patients with sarcoidosis show anergy
  2. TB pericarditis patient may have positive test
  3. Miliary TB may come out as negative test
  4. Tuberculin test is the only way to diagnose pulmonary TB if sputum smear and microscopy is negative
  5. Used for measuring the prevalence of tuberculosis infection in a community
3. Xpert MTB/RIF test is/are used for: (PGI May 2013)
  1. For assessing resistance to isoniazid
  2. For assessing multi drug resistant TB
  3. For assessing rifampicin resistance
  4. Monitoring drug response in MDR TB
  5. Diagnosis of TB
4. Niacin test is positive in: (PGI Nov 2014)
  1. Mycobacterium tuberculosis
  2. Mycobacterium cheloneii
  3. Mycobacterium bovis
  4. Mycobacterium simiae
5. RNTCP diagnosis is based on: (AIIMS Nov 2014)
  1. Sputum microscopy
  2. Chest X ray
  3. LJ culture
6. Mycobacteria can be stained by: (PGI Nov 2014)
  1. Ziehlneelsen Stain
  2. Thinyoun Stain
  3. Auraminerhodamine Stain
  4. GMS (Gomori metanamine Stain)
  5. Albert stain
7. XDRTB is defined as: (PGI Nov 2014)
  1. Resistant to Amikacin + Ofloxacin
  2. Resistant to INH + Rifampicin
  3. Resistant to INH + Rifampicin + Amikacin
  4. Resistant to Rifampicin + Amikacin+ Ofloxacin
  5. Resistant to INH + Rifampicin + Amikacin + Ofloxacin
8. Which is used in digestion and decontamination of sputum in smear preparation: (PGI May 2013)
  1. NaOH
  2. ThOH
  3. NaCI
  4. ThCI
  5. N-acetyl-L-cysteine
9. The following media can be used to culture Mycobacterium tuberculosis EXCEPT: (APPG 2014)
  1. Dorset's egg
  2. Middlebrook 7H10
  3. LJ medium
  4. McLeod's Medium
10. Which of the following is acid fast: (Recent Question 2013)
  1. Nocardia
  2. Actinomyces
  3. Actinomadura
  4. Streptomyces
11. Most appropriate method to assess the incidence of tuberculosis in community? (AIIMS Nov 05, AI 07)
  1. Identify all positives to Tuberculin test
  2. Sputum examination of symptomatic patients
  3. Identify new converter to tuberculin test
  4. Screen all under 5r - for tuberculin test
12. Most appropriate method to assess the prevalence of tuberculosis in community? (AI 04, DNB 07)
  1. Tuberculin test
  2. Sputum examination
  3. Clinical examination
  4. Chest X ray
13. Fall in which of the parameters indicates decrease in tuberculosis problem in India? (DPG 04)
  1. Incidence of infection
  2. Incidence of disease
  3. Prevalence of infection
  4. Prevalence of disease
14. National Institute of Tuberculosis located in? (AIIMS Nov 03, DNB 03)
  1. Chennai
  2. Bangalore
  3. Delhi
  4. Chengalpettu
15. Prevalence of tuberculosis in India? (Orissa 2011, Tholkata 04)
  1. 25%
  2. 30%
  3. 40%
  4. 50%
16. Tuberculin unit contains (DNB 03)
  1. 0.0001 mg
  2. 1 unit of PPD RT3
  3. 0.1 mg of BCG
  4. None
17. Best criteria for tuberculosis diagnosis by WHO? (Bihar 05, DNB 01)
  1. Sputum examination
  2. Chest pain
  3. X ray finding
  4. Mantoux positive
18. True about tuberculosis? (MP 2000, MH 2002)
  1. Smear positive- requires >10,000 bacilli/ml
  2. Mantoux test can differentiate recent and past infection
  3. Can grow on ordinary media
  4. Drug sensitivity test- by disk diffusion
19. 179In India, a tubercular mother is advised for? (RJ 2003)
  1. BCG to baby
  2. ATT to mother
  3. With hold breast feeding
  4. Separated from mother
20. Multi drug therapy is given for tuberculosis because….? (MP 2002)
  1. To delay development of resistance
  2. To reduce toxicity
  3. To broaden antimicrobial spectrum
  4. To prevent toxin release from the organism
21. Which of the following statements is true about BCG vaccination? (AIIMS May 2011)
  1. Distilled water or normal saline is used as diluents for BCG vaccine
  2. The site for injection should be cleaned thoroughly with spirit
  3. Tuberculin test is positive after 6 weeks of vaccination
  4. WHO recommends Danish 1331 strain for vaccine production
22. Analysis of the IFN-gamma responses of whole blood cells from BCG-vaccinated or non-BCG-vaccinated donors or patients with tuberculosis, stimulated with PPD, ESAT- 6 or CFP-10 antigens, and evaluation of the specificity and sensitivity of the test INF-gamma assay in TB is done. Which of the following is not true? (AI 2012)
  1. 1st generation uses ESAT
  2. 2nd generation uses ESAT and CFP-10
  3. ESAT and CFP-10 cannot differentiate between myco- bacteria and other atypical mycobacteria
  4. Replacing the PPD test for screening of tuberculosis.
23. A 25 year old lady presented with fever of 1 month duration, ataxia and head ache. Brain imaging showed dilated ventricles with basal exudates, infarcts and hydrocephalus. What should be the probable diagnosis? (AI 2012)
  1. TB meningitis.
  2. Pyogenic meningitis
  3. Viral meningitis
24. Acid fast organism (s) is/are: (SG PGI 2003, APPG 2011, PGI May 2012)
  1. Atypical mycobacteria
  2. Rickttesia
  3. Nocardia
  4. Chlamydia
25. Method of testing resistance to drugs in TB are all expect: (AIIMS May 2004, DNB Dec 2010)
  1. Radiometric broth method
  2. Molecular method
  3. Disk diffusion method
  4. PCR
26. Sterile pyuria is present in? (AI 2011)
  1. Renal tuberculosis
  2. Chronic hydronephrosis
  3. Wilm's tumour
  4. Neuroblastoma
27. The best diagnostic procedure of M.tuberculosis: (SG PGI 2008)
  1. PCR
  2. Auramin rhodamine stain
  3. Sputum culture
  4. ESR
28. Mycobacterial species differentiated by all except: (JIPMER 2004)
  1. Catalase test
  2. Amidase
  3. Niacin
  4. Oxidase
29. True about Mantoux test: (PGI June 2003)
  1. < 5 cm always +ve
  2. Usually –ve after treatment
  3. Positive reaction in children < 2 yrs is not important than adult
  4. Usually read after 48–72 hours
  5. False +ve in post measles state
30. True regarding mycobacterium tuberculosis is: (PGI June 2002)
  1. Produces visible colonies in 1 weeks time on Lowenstein –Jensen media
  2. Decolorized by 20% sulfuric acid
  3. Facultative aerobe
  4. Niacin positive
31. Commonest mycobacterial infection in patient residing in tropical countries: (PGI 2002)
  1. M. leprae
  2. M. avium intracellulare
  3. M.tuberculosis
  4. M. kansasii
32. Basanti, 29 year aged female from Bihar present with active TB. She delivers baby. All of the following are indicated except: (AI 2001)
  1. Administer INH to baby
  2. Withhold breast feeding
  3. Give ATT to mother for 2 years
  4. Ask mother to ensure proper disposal of sputum
33. Selective media for TB bacilli is: (PGI 2001)
  1. NNN media
  2. Dorset media
  3. L.J. media
  4. Nutrient agar
  5. Mac Conkey media
34. True about Mantoux is: (PGI June 2001)
  1. False negative in fulminant diseases
  2. If once done, next time it is always positive
  3. Results are given in terms of positive and negative
  4. Indurations given in terms of length and breadth
  5. Always indicate active TB infection
35. Collection of urine sample of a patient of TB /kidney: (AIIMS June 2000)
  1. 24 hrs urine
  2. 12 hrs urine
  3. In early morning
  4. Any time
36. Tuberculin test is done for: (AI 2000)
  1. Previous or present sensitization to tuberculous protein
  2. Patient is resistant to TB
  3. Patient is susceptible to TB
  4. Individual is suffering from TB
37. Not easily cultivable but well viable & used in epidemiology are/is: (PGI Dec 2000)
  1. Staph
  2. Mycobacterium TB
  3. E. coli
  4. Salmonella
38. 180The tuberculosis bacilli was discovered by: (TN 2004)
  1. Robert Thoch
  2. Edward Jenner
  3. Louis Pasteur
  4. Jonas Salk
 
NON TUBERCULOUS MYCOBACTERIUM
39. Pigment producing atypical mycobacteria: (NEET Pattern Based)
  1. M. fortuitum and M. chelonae
  2. M. gordonae and M. szulgai
  3. M. xenopi and MAC
  4. M. ulcerans
40. Fish tank granuloma is caused by: (NEET Pattern Based, PGI June 2011)
  1. M. kansasi
  2. M.marinum
  3. M.paratuberculosis
  4. M.gordonae
  5. M.scrofulaceum
41. Rapidly frowing Atypical organism involved in lung infection ? (AIIMS May 2013)
  1. M.chelonae
  2. M.fortuitum
  3. M.abscessus
  4. M.kansasi
42. Which is not a mycobacteria tuberculosis complex organism? (NEET Pattern Based, DNB June 2010)
  1. M. africanum
  2. M. tuberculosis
  3. M. bovis
  4. M. Thansasii
43. All are rapid growers except: (SG PGI 2009)
  1. M. fortuitum
  2. M. chelonei
  3. M. avium intracellulare
  4. M. smegmatis
44. Scotochromogens are: (PGI Dec 2008)
  1. Mycobacterium gordonae
  2. Mycobacterium marinum
  3. Mycobacterium intracellulare
  4. Mycobacterium avium
  5. Mycobacterium kansasii
45. True about MOTT: (AIIMS 2008, 1992)
  1. Causes dissemination infection
  2. Occurs in person with normal immunity
  3. Decreases efficacy of BCG due to cross infection
  4. Person to person transmission is seen
46. Which one of the following statement is true regarding pathogenicity of Mycobacteria species: (AI 2006)
  1. M.tuberculosis is more pathogenic than M. bovis to the humans
  2. M. kansasii can cause a disease indistinguishable from tuberculosis
  3. M. africanum infection is acquired from the environmental source
  4. M. marinum is responsible for tubercular lymphadenopathy
 
M. LEPRAE
47. Leprosy can be transmitted by all except: (AI 04)
  1. Mother to child
  2. Insect
  3. Tattooing
  4. Breast milk
48. Leprosy can be considered as a public health problem if the prevalence exceeds…..: (AI 03, DNB 08)
  1. 1/10,000
  2. 2/10,000
  3. 5/10,000
  4. 10/10,000
49. At the end of paucibacillary multidrug therapy for 6months, if the skin lesions (persistence erythema and induration in the plaque) persists, then what is the next step according to Who guidelines? (AIIMS May 01)
  1. Stop anti-leprosy treatment
  2. Continue same treatment till erythema subsides
  3. Biopsy of the lesion to document activity
  4. Continue dapsone alone for 6month more
50. False about leprosy: (AI 04)
  1. Multi bacillary leprosy:- >5 lesions
  2. New case detection rate :- indicator of incidence
  3. Target elimination of leprosy:- prevalence <1/10,000 population
  4. Defaulter- Not taken treatment for >6months
51. ENL occurs in: (NEET Pattern Based, Tharnataka 07)
  1. Due to lepromin test
  2. Due to multi drug therapy
  3. In LL patients
  4. In TT patients
52. True about leprosy in India: (PGI Dec 08)
  1. Lepra bacilli- cannot survive outside humans
  2. Bacteria load – high in TT
  3. Insect can transmit
  4. Relapse rate :- indicator of drug efficacy
  5. If prevalence is high in childhood:- it means disease is under control
53. False about lepromin test: (AIIMS 2006, AIIMS 2007, AIIMS May 2010)
  1. Negative in children <6 month age
  2. It's a diagnostic test
  3. Used to classify leprosy
  4. BCG vaccine may convert a negative lepromin test to positive
54. Mitsuda reaction is read at: (DNB 04, Orissa o8)
  1. 3rd day
  2. 10th day
  3. 21st day
  4. 45th day
55. In multibacillary leprosy, after the treatment, follow up is done yearly for..: (DNB 08)
  1. 3 yr
  2. 5 yr
  3. 10 yr
  4. 2 yr
56. 181Treatment of leprosy according to WHO is done by all except? (Bihar 05)
  1. Dapsone
  2. Ciprofloxacin
  3. Clofazemine
  4. Rifampicin
57. In paucibacillary leprosy, dapsone is continued for: (UP 08, RJ 07)
  1. 6 months
  2. 9 months
  3. 12 months
  4. 3 months
58. Lepromin test is valuable for: (MP 09, AIIMS 2006, AI 2002, MP 01, MP 05)
  1. Diagnosis
  2. Response to treatment
  3. Epidemiological reason
  4. To test humoral immunity
59. Bacteriological index of 1+ indicates: (UP 2004, MP 03)
  1. <100 bacilli/Oil immersion field
  2. 1–10 bacilli/100 oil immersion field
  3. No Bacilli in all fields
  4. Bacilli in all fields
60. Which is NOT included in Madrid classification but included in indian classification? (MH 03)
  1. Inderminate leprosy
  2. Borderline leprosy
  3. Tuberculoid leprosy
  4. Pure neuritic type leprosy
61. Multibacillary leprosy, the bacteriological index is more than: (RJ 05)
  1. 1
  2. 2
  3. 3
  4. 4
62. Antibodies against PGL-1 are seen in: (DNB DEC 2012)
  1. M. leprae
  2. M tuberculosis
  3. Borrelia
  4. Brucella
63. Erythema nodosum leprosum (ENL) occur in: (PGI June 2011)
  1. Borderline leprosy
  2. Lepromatous leprosy
  3. Indeterminate type
  4. Histoid leprosy
  5. Tuberculoid type
64. Leprosy affects all except: (AI 2007)
  1. Testes
  2. Ovary
  3. Eye
  4. Nerve
65. The main cytokine, involved in erythema nodosum leprosum (ENL) reaction is: (AIIMS 06)
  1. Interleukin – 2
  2. Interferon –gamma
  3. Tumor necrosis factor – alpha
  4. Macrophage colony stimulating factor
66. The following drug is used for the treatment of type II lepra reaction, except: (AIIMS 06)
  1. Chloroquine
  2. Thalidomide
  3. Cyclosporine
  4. Corticosteroid
67. Subtype of leprosy with maximum number of TH -1 cells: (JIPMER 2006)
  1. TT
  2. BB
  3. LL
  4. Borderline leprosy
68. Under leprosy eradication programme the management of single lesion is? (AIIMS 2002)
  1. Single dose of Rifampicin and Dapsone
  2. Rifampicin and Dapsone for 6 months
  3. Rifampicin, ofloxacin and minocycline single dose
  4. Rifampicin, clofazemine and Dapsone for 1 months
69. Which of the following is true regarding globi in a patient with Lepromatous leprosy: (AI 2002)
  1. Consists of lipid laden macrophages
  2. Consists of macrophages filled with AFB
  3. Consists of neutrophils filled with bacteria
  4. Consists of activated lymphocytes
70. Exacerbation of lesions in patients of borderline leprosy is seen in: (PGI June 2001)
  1. ENL (erythema nodosum leprosum)
  2. Lepra reaction type I
  3. Jarisch – Herxheimer reaction
  4. Resolving leprosy
  5. Rifampicin and minocycline for 6 months
71. Lupus vulgaris is caused by: (Recent Question 2013)
  1. M tuberculosis
  2. M lepare
  3. M ulcerans
  4. M marinum
72. Criteria for diagnosing multibacillary leprosy include(s): (PGI May 2013)
  1. 2–6 skin lesion
  2. Skin smear 1+
  3. Skin smear 2+
  4. Deformity ±
  5. Eye lesion +
73. All are true statement regarding leprosy except: (PGI Nov2012)
  1. Multibacillary leprosy means person having 6 or more skin lesions
  2. Regular MDT means patients received 2/3rd of months of treatment schedule
  3. In paucibacillary leprosy> 2 nerves are involved
  4. Loss of sensation may present
  5. Lepra reaction if not treated can leads to permanent deformities
74. Leprae bacilli doubling time: (TNPG 2014)
  1. 20 hours
  2. 20 minutes
  3. 12days
75. Satellite papules are characteristic of following type of leprosy: (MHPG 2014)
  1. Borderline Tuberculoid (BT)
  2. Polar Tuberculoid (TT)
  3. Borderline Lepromatous (BL) D) Polar Lepromatous (LL)
182EXPLANATIONS
 
MYCOBACTERIUM TUBERCULOSIS
1. Ans. (a) (b) (d) (M.tub…, M.chelo…., M.simiae…) Ref: Ananthnarayan 9/e p347
Niacin test is positive in Mycobacterium tuberculosis, M.simiae and few strains of M.cheloneii
2. Ans. (a),(b),(c), (e) (Patients with sarcoidosis…, TB pericarditis.., Milliary TB.., Used for ….) Ref: Park 22nd /p172
  • Tuberculin may be false negative in conditions such as- early TB, late or miliary TB, patients with low CMI, sarcoidosis or following measles
  • In extrapumlonary TB like TB pericarditis also, tuberculin test is positive unless the patient is immunocompromised.
Interpretation of result of tuberculin test:
  • Adults: Positive test only confirms past infection but does not indicate presence of active stage of the disease. It is only used as epidemiological marker.
    • Marker of prevalence of tuberculosis-Defined as all tuberculin reactors in a community
    • Marker of incidence of tuberculosis- Defined as new converters to tuberculin test in a community
  • Children: Indicates active infection and used as diagnostic marker.
3. Ans. (c), (e) (For assessing rifampicin resistance, Diagnosis of TB) Ref: CDC website
The Xpert MTB/RIF is a cartridge-based, fully automated diagnostic test that can-
  • Identify Mycobacterium tuberculosis DNA and resistance to rifampicin (RIF) simultaneously
  • It works on the principle of PCR- i.e. Only detects the DNA but cannot quantify the DNA, hence NOT suitable for monitoring disease progression. Real time PCR is the best for this purpose.
  • Provides accurate results in less than two hours so that early treatment can be started
  • Has minimal bio-safety requirements, training, and can be done in any laboratories.
4. Ans- (a) (b) (d) (M.tuberculosis, M.cheloneii, M.simiae) Ref: Ananthnarayan 9th/p 347
Niacin test is not positive in Mycobacterium tuberculosis, M.simiae and few strains of M.cheloneii
5. Ans. (a) (Sputum microscopy) Ref: Park 22/e p170
Sputum smear microscopy is the recommended method for the diagnosis of tuberculosis under RNTCP.
6. Ans. (a), (b), (c) (ZiehlNeelsen, Kinyoun Stain, Auramine rhodamine) Ref: Ananthnarayan 9/e p353 Mycobacterium tuberculosis being acid fast can be stained by acid fast staining-
  • Ziehl-Neelsen staining (hot method) or its modifications such as
    • Kinyoun's stain (cold acid fast staining) or
    • Gabbet's staining method
  • Fluorescent staining such as Auraminerhodamine staining method
7. Ans. (e) (Resistant to INH + Rifampicin + Amikacin + Ofloxacin) Ref: Ananthnarayan 9/e p357
XDRTB is defined as at least MDR TB (i.e. rifampicin +isoniazid) +Resistant to one fluoroquinolone (ofloxacin) + one injectable second line aminoglycosides (Amikacin or kanamycin or capreomycin).
8. Ans. (a) (e) (NaOH, N-acetyl-L-cysteine) Ref: Ananthnarayan 9/e p352
Sputum and specimens from non-sterile sites need prior treatment for digestion (to liquefy the thick pus cells and homogenization) and decontamination (to inhibit the normal flora) and concentration (to increase the yield).
  • Petroff's method (4% NaOH)- Most commonly followed
  • NALC (N acetyl cysteine) + 2% NaOH – This is superior to Petroff's method for isolation. NALC liquefies the sputum and NaOH kills the normal flora. This method is better compatible with automated culture systems.
  • If for only smear microscopy, then formalin or hypochlorite can also be used as mucolytic and for killing the bacilli. However, they are not useful for culture or animal pathogenicity.
1839. Ans. (d)(McLeod's Medium) Ref: Ananthnarayan 9th/p354
McLeod's Medium is a selective medium for Corynebacteriumdiphtheria.
10. Ans. (a) (Nocardia) Ref: Ananthanarayan 9/e p393
  • Nocardia is branching gram positive and acid fast bacilli
11. Ans. (c) (Identify new converter to tuberculin test) Ref: Park 22/e p168, 21/e p166
  • Identify all positives to Tuberculin test- Marker of prevalence of tuberculosis
  • Sputum examination of symptomatic patients by Zn stain- Method of choice case finding tool for tuberculosis under RNTCP.
  • Identify new converter to tuberculin test- Marker of incidence of tuberculosis.
12. Ans. (a) (Tuberculin test) Ref: Park 22/e p168, 21e p166
  • Identify all positives to Tuberculin test- Marker of prevalence of tuberculosis.
13. Ans. (a) (Incidence of infection) Ref: Park 22/e p168,21/e p166
Refer chapter review.
14. Ans. (b) (Bangalore) Ref: Internet source
Important National centers for Tuberculosis in India:
  • Tuberculosis research centre- Chennai
  • National institute of tuberculosis- Bangalore
  • L R S Institute of Tuberculosis & Respiratory Diseases - Delhi
  • JALMA Institute for Leprosy & Other Mycobacterial diseases- Agra.
15. Ans. (c) (40%) Ref: Park 22/p167
  • Prevalence of tuberculosis infection in India -40% (i.e. 2 out of five Indians are Mantoux positive)
  • In June 2011, the revised estimate of tuberculosis in India:
    • ✓ Prevalence tuberculosis cases in India – 256 cases/100,000 population
    • ✓ Incidence tuberculosis cases in India - 185 cases /100,000 population.
16. Ans. (b) (1 unit of PPD RT3) Ref: Park 22/p172, 21/p168
  • WHO advocates PPD-RT3 with Tween-80
  • 1 TU is equivalent to 0.01ml of OT or 0.00002 mg of PPD-S.
  • 1 dose of 1 TU of PPD in 0.1 ml.
17. Ans. (a) (Sputum examination) Ref: Park 22/p170, 21/p169
  • Sputum examination of symptomatic patients by Zn stain- Method of choice case finding tool for tuberculosis under RNTCP.
18. Ans. (a) (Smear positive- Requires >10,000 bacilli/ml) Ref: Park 22/p170, 20/p161
  • For smear to be acid fast stain positive- it requires >10,000 bacilli/ml
  • Mantoux test cannot differentiate recent and past infection
  • M.tuberculosis is fastidious, it cannot grow on ordinary media
  • Disk diffusion test is NOT useful for drug sensitivity testing of M.tuberculosis.
19. Ans. (b) (ATT to mother) Ref: Park 22/p172, 21/p168
  • After delivery –If mother chest X-ray and sputum AFB +ve then
3 Dos-
3 Don'ts-
Mother to be given ATT(HRE)
Don't separate the baby from mother
Baby (INH for 9–12 months)
Don't with hold of breast feeding
Screening of household contacts
Don't give to the baby BCG
18420. Ans. (a) (To delay development of resistance) Ref: Park 22/e p171–72, 21/e p173–74
  • Resistance to anti-tubercular drug is mainly due to mutational drug resistance which can be overcome by combination of drugs.
21. Ans. (d) (WHO recommends Danish 1331 strain for vaccine production)
Ref: Park's 22/e p178–79, 21/e p176–77, 20/p171
  • WHO recommends Danish 1331 strain for vaccine production (M.bovis). In India, it is prepared in Guindy, Chennai
  • Normal saline is recommended as diluents for BCG vaccine as distilled water is irritant
  • The site for injection should be cleaned thoroughly with soap but disinfectant or antiseptic should not be used. If alcohol is used then it should be evaporated before the vaccination is given
    • Tuberculin test is positive after 8 weeks of BCG vaccination but in some it might require 14 weeks.
22. Ans. (c) (ESAT and CFP-10 Can't differentiate between Mycobacteria and other atypical Mycobacteria)
Ref: Harrison 18/e p1351
  • “IFN-γ Release Assays (IGRAs) are more specific than the Tuberculin Skin Testing (TST) as a result of less cross-reactivity due to BCG vaccination and sensitization by nontuberculous mycobacteria.”
  • Diagnosis of Latent M. Tuberculosis Infection:
    • ✓ Tuberculin Skin Testing
    • ✓ IFN-γ Release Assays
  • IFN-γ Release Assays (IGRAs)- (Harrison 18/e p1351)
    • ✓ In vitro assays that measure T cell release of IFN- γ in response to stimulation with the highly TB-specific antigens like excretory secretory antigen (ESAT-6) and Culture filtrate protein (CFP-10).
    • ✓ Two commercial assays are available-The T-SPOT. TB®, The QuantiFERON-TB Gold
    • ✓ IGRAs are more specific than the TST as a result of less cross-reactivity due to BCG vaccination and sensitization by nontuberculous mycobacteria.
    • ✓ These tests have shown better correlation than the TST with exposure to M. tuberculosis in contact investigations in low-incidence settings.
23. Ans. (a) (TB meningitis.) Ref: Harrison 18/e p3426
  • The combination of unrelenting fever of 1 month duration, ataxia and head ache with Brain imaging showing dilated ventricles with basal exudates, infarcts and hydrocephalus is highly suspicious for tuberculous meningitis. ………………………………….Harrison 18th p3426
24. Ans. (a) (c) (Atypical mycobacteria)(Nocardia) Ref: Ananthanarayan 9/e p359
  • ✓ Refer chapter review- for the list of acid fast organisms.
25. Ans. (c) (Disk diffusion method) Ref: Ananthanarayan 9/e p354
  • Disk diffusion method is used for drug susceptibility testing for most of the bacteria; however, it is not used for M.tuberculosis.
  • Various methods used for drug susceptibility testing for M.tuberculosis-
    • ✓ Resistance Ratio
    • ✓ Absolute concentration
    • ✓ Proportion method
    • ✓ Radio metric method
    • ✓ Molecular methods.
26. Ans. (a) (Renal tuberculosis) Ref: Harrison 17/e p1011 & 18/e p1347
“The documentation of culture-negative pyuria in acidic urine raises the suspicion of tuberculosis
27. Ans. (c) (Sputum culture) Ref: Tuberculosis 2007-From basic science to patient care
  • Culture is the Gold standard for diagnosis of tuberculosis with detection thresh hold of 10–100 bacilli/ml.
  • Also, it detects the viability of the organism.
  • It is more sensitive than Acid fast smear and approaches 100% specificity.
About Other Options
  • PCR though can detect even 1 bacilli/ml, but it is not the gold standard because:
    • ✓ False –ve PCR might occur by presence of PCR inhibitors in sample or
    • 185✓ False +ve PCR might occur due to contamination during the procedure
    • ✓ More so it can't detect the viability of the organism.
  • However PCR is useful in:
    • ✓ Extrapulmonary tuberculosis (where Culture is less sensitive)
    • ✓ For rapid diagnosis along with rapid drug sensitivity detection.
  • Staining (ZN and Auramin rhodamine stain) though it is rapid, but sensitivity is low (detection limit of 104 bacilli/ml.
  • ESR is non specific, can be elevated in no. of condition.
28. Ans. (d) (Oxidase) Ref: Ananthanarayan 9/e p347, 8/e p354
  • Oxidase is never used for differentiation between M.tuberculosis and NTM.
MTB can be differentiated from NTM by:
  • Sensitive to paranitrobenzoic acid & TCH
  • Aryl sufatase test -ve and Amidase –ve
  • Weak Catalase +ve
  • Niacin +ve.
29. Ans. (d) (Usually read after 48–72 hours)
Ref: Ananthanarayan 9/e p354–55, 8/e p356, Park 22/e p172–73, 20/e p163 & 21/e p168
  • Option a- Mantoux test: Induration <5mm- always-ve
  • Option b- Doesn't always turn –ve after treatment
  • Option c- Mantoux test is the indicator of – Active infection in infants
  • Option e- Mantoux test becomes false -ve in post measles state
    “A positive tuberculin test may occasionally revert –ve after INH”- Park 22/e p172,20th /p164
  • Diagnosis of latent tuberculosis- refer chapter review for explanation.
30. Ans. (d) (Niacin positive) Ref: Ananthanarayan 9/e p347, 8/e p348, 349
  • Human strains of M.tuberculosis is Niacin +ve where as no other Mycobacterium are +ve. (Except M. simiae, few strains of M. chelonei.)
About Other Options-
  • Option a- Produces visible colonies in 6–8 weeks time on Lowenstein – Jensen media
  • Option b- Resists Decolorization by 20% sulfuric acid (Acid fast)
  • Option c- M. tuberculosis is strict aerobe while M. bovis is Microaerophilic.
31. Ans. (c) (M.tuberculosis) Ref: Park 22/e p166, 20/e p161 & 21/e p165
  • In tropical country, the most common Mycobacterial infection is due to M.tuberculosis.
  • India ranks first globally accounting for 20% of total tuberculosis cases of the world.
32. Ans. (b) (Withhold breast feeding) Ref: Nelsen 17/e p971
  • If mother chest X-ray and sputum AFB +ve then, “mother to be given ATT(HRE) + baby (INH for 9–12m) + screening of household contacts
  • “Separation from mother or with hold of breast feeding is not recommended”.
33. Ans. (b), (c) (Dorset media, L.J. media) Ref: Ananthanarayan 9/e p 346, 8/e p348
  • Selective media for Tuberculosis- Refer chapter review.
34. Ans. (a) (False negative in fulminant diseases)
Ref: Park 22/e p172, 20/e p164 & 21/e p168, Ananthanarayan 9/e p354–55, 8/e p356
False –ve Mantoux test –Early/adv. TB, Miliary TB, Post measles and decreased immunity.
About Other Options
Option b-“Tuberculin test sensitivity slowly wanes over time and repeat test may appear negative in non infected persons. ………………Park 22/e p172, 20/e p164
  • However, it may enhance the intensity of response to a reactive person and this booster effect, a false –ve or equivocal test due to waning allergy may turn positive on repeat test in a diff site.
  • Tuberculin test doesn't Always indicate active TB infection”. …….Ananthanarayan 9th/p354–55,8/e p356
186Option c- Results are given in terms of positive (>10mm) and negative (<5mm) and doubtful/equivocal (6–9mm)
Option d- Indurations given in terms of breadth only
  • “Horizontal transverse diameter of the induration in mm using a plastic ruler or caliper”
Option e- Mantoux test is an Indicator of -Active infection in infants, past exposure in adult
35. Ans. (c) (In early morning) Ref: Mackie McCartney Practical Microbiology 14/e p86
“The 1st urine passed on the day i.e. Early morning sample is recommended for tuberculosis of urinary tract.”
  • Most commonly collected Sample for UTI- Midstream urine
  • Best Sample for UTI- supra pubic aspiration
  • Sample for UTI for infant- supra pubic aspiration
  • Best Sample when urethritis or prostatitis is suspected – initial flow of urine
  • If delay is expected to process, then – store by refrigeration at 4°c or add boric acid 1.8%.
36. Ans. (a) (Previous or present sensitization to tuberculous protein) Ref: Park 22/e p172, 20/e p164 & 21/e p168
Positive tuberculin test indicates –
  • Active infection in infants (suffering from TB)
  • Prevalence of infection
  • Past exposure to TB bacilli in adult (Previous or present sensitization to tuberculous protein)
Option b and c- “Tuberculin test doesn't indicate susceptibility or resistance to TB where as Lepromin test is used to assess individual resistance to Leprosy”.
37. Ans. (b) (Mycobacterium tuberculosis) Ref: Ananthanarayan 9/e p346, 353, 8/e p348
M.tuberculosis is slow growing takes 4–8weeks of time to grow in culture.
Other options – Staph, E.coli and Salmonella can be grown easily with in 24hour.
Also know– Generation time of “E.coli is 20 min, M.tuberculosis is 20 hour and M. leprae is 20 days.”
38. Ans. (a) (Robert Koch) Ref: Ananthanarayan 9/e p345, 8/e p347
“Robert Koch in 1882 isolated mammalian tubercle bacillus and proved its causative role in tuberculosis by satisfying Koch's postulate where as: Lepra bacillus was 1st described by Hansen.”
 
NON TUBERCULOUS MYCOBACTERIUM
39. Ans. (b) (M. gordonae and M. szulgai) Ref: Ananthanarayan 9/e p360, 8/e p357 Pigment producing atypical mycobacteria -
  • Photo chromogen -M marinum, M simiae, M. asciaticum, M kansasii,
  • Scotochromogen -M. scrofulaceum, M szulgai, S.gordonae.
40. Ans. (b) (M. marinum) Ref: Ananthanarayan 9/e p360, 8/e p359, Jawetz 25/e p290, 297
  • M. marinum (e.g. of Photochromogen) is originally isolated from fish is the causative agent of swimming pool or fish tank granuloma.
  • This condition is associated with development of superficial granulomatous lesions in the skin.
41. Ans. (c), (M.abscessus), Ref: Harrison 18/e p1369, Koneman's Diagnostic Microbiology 6/e p1105
  • Any of the rapidly growing Mycobacteria such as M chelonei, M fortuitum & M. abscesses can cause pulmonary infection but infection with M. Abscessus may be particularly dangerous ……….. Koneman's Diagnostic Microbiology 6th/p1105
  • M. kansasii ……. Belongs to photochromogen.
    • ✓ Can cause a clinical syndrome that strongly resembles tuberculosis, consisting of hemoptysis, chest pain, and cavitary lung disease.
  • MAC - most common causes of pulmonary nontuberculous mycobacterial infection.
  • The rapidly growing NTM, M chelonei, M fortuitum & M. abscessus, acquired via skin contamination from surgical instruments (especially in cosmetic surgery), injections, and other procedures. These infections are typically accompanied by painful, erythematous, draining subcutaneous nodules, usually without associated fever or systemic symptoms.
18742. Ans. (d) (M.Kansasii) Ref: Ananthanarayan 9/e p354, 359, 8/e p357–361, Jawetz 25/e p290, 297
M tuberculosis complex – include species
  • M. tuberculosis
  • M. bovis (bovine tubercle bacillus)
  • M. africanum
  • M. microti (vole tubercle bacillus)
43. Ans. (c) (M. avium intracellulare) Ref: Ananthanarayan 9/e p360, 8/e p357–361
Atypical (environmental) Mycobacteria- Refer chapter review for detailed explanation.
44. Ans. (a) (Mycobacterium gordonae) Ref: Ananthanarayan 9/e p360, 8/e p357
Scoto chromogen – Pigmentation is produced in light and dark as well. Example – M. scrofulaceum, M szulgai, S.gordonae
45. Ans. (a), (c) (Causes dissemination infection, Decreases efficacy of BCG due to cross infection)
Ref: Greenwood 16th /p216–17
Option b-“Most species are less virulent for humans than M. tuberculosis. Although men are frequently infected, overt disease is very uncommon except who are profoundly immunocompromised.” ………Greenwood 16th /p216
Option c- “Prior exposure to some NTM like M.vaccae, M.non chromogenicum may have conferred partial immunity on population thus have masked the potential benefit of BCG” ……………..Park 22/e p178–79, Journal- Clin Infect Dis /2004/V 39/ Iss-4
Option d- The NTM are distributed widely in the environment.
Both asymptomatic and symptomatic infections with NTM are acquired most often from childhood contact with soil, water. ………Harrison 17th /1027
46. Ans. (b) (Mycobacterium kansasii produce human disease indistinguishable from tuberculosis Ref: Jawetz 24/e p328 & 25/e p297, Harrison 18/e p1369
  • Mycobacterium kansasii produce human disease indistinguishable from tuberculosis”
  • M. kansasii i is the most pathogenic non tuberculous mycobacterial species affecting the lung, and the clinical features of M. kansasii disease resemble those of tuberculosis. ………..Harrison 17th /1030
  • NTM causing human infection- Refer chapter review.
About Other Options
Option a-M.tuberculosis is equally pathogenic than M.bovis to the humans and Guinea pig. However, M.bovis but not M.tuberculosis is pathogenic to rabbit.
Option c-M. africanum belongs to the members of Mycobacterium tuberculosis complex. It is commonly found in West African countries, causing up to a quarter of cases of tuberculosis in countries such as the Gambia. ‘It is an infection of humans only and is spread by an airborne route from individuals with open cases of disease.’ ………………….Source- Wikipedia
Option d-“M. marinum causes swimming pool granuloma/fish trank granuloma/ fish fancier's finger. Secondary lesions appear along with sermal lymphatics.” (Sporotrichoid spread) …..Greenwood 16th /p217
 
M.LEPRAE
47. Ans. (None of the options) Ref: Park 22/e p290
Mode of transmission of leprosy-
  • Droplet (aerosols)- MC mode
  • Direct contact transmission (skin-skin)
  • Indirect contact (soil, fomites, clothes and linens)
  • 188Mother to child (transplacental)
  • Insect
  • Tattooing
  • Breast milk
48. Ans. (a) (1/10,000) Ref: Park 22/e p290
  • Leprosy can be considered as a public health problem if the prevalence exceeds-1/10,000
  • Leprosy can be considered as eliminated if the prevalence < 1/10,000
  • India has been declared to eliminate leprosy in 2005
  • India so far has eliminated three diseases:
    • ✓ Dracunculiasis/guinea worm disease (2000)
    • ✓ Leprosy (2005)
    • ✓ Yaws (2006).
49. Ans. (a) (Stop anti-leprosy treatment) Ref: Park 22/e p298
  • Refer chapter review
50. Ans. (d) (Defaulter- Not taken treatment for > 6 months) Ref: Park 22/e p287–302
  • Multi bacillary leprosy:- bacteriological index ≥ 2, skin lesion : >5, nerve involvement : ≥1
  • New case detection rate :- indicator of incidence
  • Target for elimination of leprosy:- prevalence <1/10,000 population
  • Defaulter- Not taken treatment for ≥ 2 months.
51. Ans. (c) (In LL patients) Ref: Park 22/e p297
  • ENL occurs in type II lepra reaction in LL and BL patients.
52. Ans. (c) (Insect can transmit lepra bacilli) Ref: Park 22/e p287–302
  • Lepra bacilli- can survive outside humans in animals like foot pad of mice and armadillo.
  • Bacteria load – high in LL
  • Insect can occasionally transmit lepra bacilli
  • If prevalence is low in childhood:- it means disease is under control.
53. Ans. (b) (It's a diagnostic test) Ref: Park 22/e p294
  • Lepromin test is negative in children <6 month age, it may be positive at the end of 1st year.
  • Lepromin test a NOT used in diagnosis.
  • Lepromin test is used to classify leprosy and also a prognostic marker.
  • BCG vaccine is capable of converting a negative lepromin test to positive.
54. Ans. (c) (21st day) Ref: Park 22/e p293–94
  • Early reading (Fernandez) is taken at 3days; late reading (Mitsuda reaction) is taken at 3 weeks i.e. on 21st day.
55. Ans. (b) (5 yr) Ref: Park 22/e p299
  • In multibacillary leprosy, after the treatment, follow up is done yearly for 5 years
  • In paucibacillary leprosy, after the treatment, follow up is done yearly for 2 years.
56. Ans. (b) (Ciprofloxacin) Ref: Park 22/e p296
  • Common drugs given in treatment of leprosy that included in WHO protocol- Dapsone, rifampicin, clofazemine
  • Other drugs which can be given in treatment of leprosy- Ethionamide, quinolones, clarithromycin, minocycline.
57. Ans. (a) (6 months) Ref: Park 22/e p297
  • In paucibacillary leprosy, dapsone is continued for- 6 months
  • In multibacillary leprosy, dapsone is continued for- 12 months
58. Ans. (b) (Response to treatment) Ref: Park 22/e p294
  • Uses of Lepromin test–
    • ✓ Classify lesions of leprosy,
    • ✓ Assess prognosis,
    • ✓ Assess resistance to leprosy in individuals
18959. Ans. (b) (1–10 bacilli/100 oil immersion field) Ref: Park 22/e p293
  • Bacteriological index-
    • ✓ 0 indicates No Bacilli in all fields
    • ✓ 1+ indicates- 1–10 bacilli/100 oil immersion field
    • ✓ 2+ indicates-1–10 bacilli/10 oil immersion field
    • ✓ 3+ indicates-1–10 bacilli/ each oil immersion field
    • ✓ 4+ indicates- 10–100 bacilli/ each oil immersion field
    • ✓ 5+ indicates – 100–1000 bacilli/ each oil immersion field
    • ✓ 6+ indicates - >1000 bacilli/ each oil immersion field
  • Morphological index- No. of live bacilli detected by –Solid uniformly stained, parallel sides, rounded ends, length 5times than width.
  • Solid fragmented granular (SFG) percentage- % of solid fragmented and granular bacilli. It is the most sensitive indicator for monitoring response to treatment.
60. Ans. (d) (Pure neuritic type leprosy) Ref: Park 22/e p291
Classification of leprosy:
  • Ridley Jopling classificationà TT, BT, BB, BL, LL
  • Madrid classification – LL, TT, borderline/dimorphous, indeterminate (early unstable type)
  • Indian classification – Madrid + pure neuritic type
61. Ans. (b) (2) Ref: Park 22/e p291–92
  • Paucibacillary leprosy- Given for-I, TT, BT and if bacteriological index <2, skin lesion : 1–5, one nerve involvement
  • Multibacillary leprosy- Given for BB, BL, LL if bacteriological index ≥ 2, skin lesion : >5, nerve involvement : ≥1
62. Ans. (a) (M. Leprae) Ref: Harrison 18/e p1361
  • IgM antibodies to PGL-1 (phenolic glycolipid 1)-
    • ✓ Tuberculoid (TT, BT) Leprosy – 60%
    • ✓ Borderline (BB, BL) Leprosy – 85%
    • ✓ Lepromatous (LL) Leprosy- >90%
63. Ans. (a) (b) (Borderline Leprosy, Lepromatous leprosy)
Ref: Ananthanarayan 9/e p367, 8/e p368, Harrison 18/e p1363
  • Erythema nodosum leprosum (ENL) occurs exclusively in patients near the lepromatous end of the leprosy spectrum (BL-LL), affecting nearly 50% of this group….. Harrison 18/e p1363
64. Ans. (b) (Ovary) Ref: Harrison 17/e p1020–1024, Internet source
  • Grows well in cooler part of body skin, testes, peripheral nerve, anterior eye
  • It can involve any organ – except CNS and lungs, ovary and also warm area of skin(axilla, groin, scalp)
  • “LL leprosy, the anterior chamber of the eye is invaded by bacilli, and ENL may result in uveitis, with consequent cataracts and glaucoma. Thus leprosy is a major cause of blindness in the developing world.
  • M. leprae invades the testes, and ENL may cause orchitis.
  • Tuberculoid leprosy usually affects Peripheral Nerves
65. Ans. (c) (Tumor necrosis factor – alpha) Ref: Harrison 17/p 1024 & 18/e p1363
“Elevated levels of circulating tumor necrosis factor (TNF) have been demonstrated in ENL; thus, TNF may play a central role in the pathobiology of this syndrome”
66. Ans. (c) (Cyclosporine) Ref: Harrison 17/e p1026 & 18/p1366
Treatment of lepra reactions:
Type 2 lepra reactions
  • ENL is (mild) - Treated with antipyretics alone
  • Cases with many skin lesions, fever, malaise, and other tissue involvement- Glucocorticoid
  • ENL (recurring and persisting) - Treatment with thalidomide
  • Others- Clofazemine in high doses, Chloroquine KDT 5th /p714
Type 1 lepra reactions
  • Best treated with glucocorticoid
  • Clofazemine and thalidomide are ineffective
19067. Ans. (a) (TT) Ref: Harrison 17/e p1024 & 18/e p1362
“In Tuberculoid leprosy tissue, there is a 2:1 predominance of helper CD4+ over CD8+ T lymphocytes. In LL tissues, there is a 2:1 ratio of CD8+ to CD4+ T lymphocytes.”
68. Ans. (c) (Rifampicin, ofloxacin and minocycline single dose) Ref: Harrison 17/e p1025 & 18/e p1364–65
“For patients with single-lesion paucibacillary leprosy, the WHO recommends as an alternative a single dose of rifampicin (600 mg), ofloxacin (400 mg), and minocycline (100 mg).”
Treatment of Leprosy- Refer chapter review.
69. Ans. (b) (Consists of macrophages filled with AFB) Ref: Ananthanarayan 9th/p364,8/e p364
“Intracellularly, M.leprae are arranged as – parallel cigar bundles of bacilli bound with lipid like glia (globi) present inside foamy macrophage (Virchow's leprae cell)”
70. Ans. (b) (Lepra reaction type I) Ref: Ananthanarayan 9/e p367, 8/e p368, Harrison 17/e p1024–26 &18th/p1363
  • For details about Lepra reaction type I- Refer chapter review.
71. Ans. (a) (M.tuberculosis) Ref: Harrison 18/e p420
  • Lupus vulgaris (also known as Tuberculosis luposa) are painful cutaneous tuberculosis skin lesions with nodular appearance, most often on the face around the nose, eyelids, lips, cheeks, ears and neck. It is the most common M. tuberculosis skin infection.
72. Ans. (b),(c) (Skin smear 1+, skin smear 2+) Ref: Park 22/e p295
Criteria
Paucibacillary
Multibacillary
Skin lesions
1–5
6 or more
Nerve involvement
1
2 or more
Microscopy
Smear negative
Smear positive at any one site
Leprosy type
TT, BT and Intermediate
BB, BL, LL
73. Ans.(c) (In paucibacillary leprosy> 2 nerves are involved) Ref: Park 22/e p292–295, 21/e p292–99
  • Refer chapter review for detail.
  • Regular multidrug therapy of leprosy means patients should receive atleast 2/3rd of the total duration of treatment schedule (i.e. at least 8 full months out of 12 months).
74. Ans. (c) (12 days) Ref: Ananthnarayan 9/e p22–23
  • Refer chapter review.
75. Ans. (a) Borderline Tuberculoid (BT) Ref: Oxford Textbook of Medicine/p578
  • Satellite papules are characteristic of Borderline Tuberculoid (BT) type of leprosy.

Enterobacteriaceae (E.coli, Klebsiella, Proteus, Shigella, Salmonella, Yersinia)CHAPTER 3.7

 
Enterobacteriaceae
Tribe
Genus
Tribe1
Escherichieae
Escherichia, Shigella
Tribe2
Edwardsielleae
Edwardsiella
Tribe3
Salmonelleae
Salmonella
Tribe4
Citrobactereae
Citrobacter
Tribe5
Klebsielleae
Klebsiella, Hafnia, Serratia, Enterobacter, Pantoea
Tribe6
Proteeae
Proteus, Morganella, Providencia
Tribe7
Yersinieae
Yersinia
 
Family Character
  • Gram negative bacilli, Non fastidious
  • Glucose fermenting,
  • Reduce Nitrate to nitrites.
  • Aerobic and facultative anaerobe
  • Catalase +ve, (except Shigella dysenteriae 1)
  • Oxidase negative
  • Motile (exception Shigella, Klebsiella, Salmonella gallinarm-pullorum)
 
Classification: Based on Lactose Fermentation (on MacConkey Agar)
  • Lactose fermenter – E.g. Escherichia, Klebsiella- Pink colony on MacConkey agar
  • Non Lactose fermenter- E.g. Shigella, Salmonella, Proteus, Yesinia- Pale colony on MacConkey agar
  • Late Lactose fermenter – E.g. Shigella sonnei
 
ESCHERICHIA COLI
  • Motile by Peritrichous flagella,
  • Hemolytic on Blood agar- especially uropathogenic E.coli
  • Biochemical reactions: IMViC: ++-- (Indole +ve, MR +ve, VP-ve, citrate-ve)
  • Antigenic structure
    • Somatic Ag O-, Heat stable, LPS, 170 serotypes
    • Flagellar antigen H-, heat labile, 75 types
    • Capsular Ag K- attachment, 103 types
    • Fimbrial Ag F
  • Normal colon commensal strains: bélongs to early O groups (1, 2, 3, 4 etc)
  • Enteropathogenic strains- belong to later O groups (26,55,86,111 etc.)
Surface antigens
  • O antigen (LPS): endotoxic; inhibits phagocytosis
  • K antigen; protects from phagocytosis
  • Flagella (H) antigen- motility
Clinical infections
 
UTI (Uropathogenic E.coli)
  • MC cause of UTI- Uropathogenic E.coli
  • 192MC serotypes causing UTI- “Early” O groups 1, 2, 4, 6, 7, 18,75
  • Lower UTI (Cystitis and urethritis)-
    • ✓ Mostly due to ascending infection, lacks K Ag
    • ✓ Virulence factors- Hemolysin (Hemolytic on Blood agar), P fimbriae, Mannose resistant fimbria, Siderophore
    • ✓ Symptoms- Burning micturition, dysuria, urinary frequency, and su- prapubic pain.
Upper UTI (Pyelonephritis):
  • Mostly due to descending infection (hematogenous spread)
  • Virulence factor- Possess K antigen
  • Symptoms- fever, vomiting, abdominal pain.
E.coli is considered as a pathogen (not a commensal) if:
  • Bacteria count- >105 /ml of urine (Kass’ concept of “significant bacteriuria)
  • 1 pus cell/ 7 high power fields
  • 1 bacteria/ oil immersion field
Diagnosis:
If there is delay – urine can be refrigerated or stored with 1.8% boric acid
  • Screening:
    • Griess nitrite test,
    • Catalase,
    • triphenyl tetrazolium phenyl test,
    • leucocytes esterase test,
    • Wet mount (↑leucocytes).
  • Semi quantitative culture (standardized loop technique)
  • Quantitative culture(Pour plate method)
  • UTI is diagnosed when Bacteriuria:
    • 105 /ml of urine (In asymptomatic),
    • 104 /ml of urine (In symptomatic, MSU),
    • 102 /ml of urine (In catheterized patient),
  • Any count is significant:
    • For suprapubic aspiration,
    • Gram positive organism,
    • If patient is receiving antibiotics.
 
Diarrheogenic E. coli
LT: (heat labile)
ST (heat stable)
VT (verocytotoxin/ Shiga like toxin)
Resemble cholera toxin in structure- AB5
A-↑ cAMP,
B-binds GM1 ganglioside
↑ cGMP
Inhibit ribosome and Inhibit protein synthesis
Plasmid coded
Plasmid coded
Phage coded
Toxin Demonstration by-
  • Ligated rabbit ileal loop test - +ve at 18 hrs
  • Adult rabbit skin test +ve
  • Steroid production in Y1 mouse adrenal cell culture
  • Elongation in Chinese hamster ovary cells
Toxin Demonstration by-
  • Ligated rabbit ileal loop test
  • +ve at 6 hrs
  • Intra gastric infant mouse test +ve
Toxin Demonstration by-
  • Identical to Shiga toxin, but not neutralized by shiga antitoxin
  • Cytotoxic to Vero cells
ST and LT can also be differentiated by- Latex Agglutination, ELISA, DNA probe
1931) EPEC- (Entero pathogenic E.coli)-
  • Causes infantile Diarrhea, sporadic diarrhea in adults, Person-to-person spread
  • Nontoxigenic, non invasive
  • Adhere to intestinal mucosa and disrupt brush border– attaching and effacing lesion on intestinal epithelium
2) ETEC- (Entero toxigenic E.coli)-
  • Acute watery diarrhea in infants and adults:
  • MC cause of traveler's diarrhea
  • Pathogenesis: 1)Toxins- LT, ST 2) Fimbrial proteins (Colonization Factor Ag), Noninvasive
  • Diagnosis –Typing, Demonstration of toxins (Refer the previous table)
3) EIEC- (Entero invasive E.coli)-
  • Dysentery like disease in all ages
  • Pathogenesis: Epithelial cell invasion by virulence marker Antigen (VMA)
  • Called atypical E coli (lactose Nonfermenter), resembles Shigella flexneri
  • Diagnosis – VMA ELISA, HeLa cell invasion assay, Sereny's test (Guinea pig inoculation→conjunctivitis)
4 EHEC -(Entero hemorrhagic E.coli)-
  • MC serotype associated-O157:H7, but rarely by O6, O26, O55, O91, O103, O111, O113
  • Transmitted by contaminated food- undercooked ground beef (MC)
  • Frequent in industrialized countries (others are common in developing regions)
  • Low infective dose- <102 CFU
  • Manifestations- Grossly bloody diarrhea, abdominal pain and fecal leukocytosis, absence of fever
  • EHEC secretes VT which causes Capillary microangiopathy that leads to –
    • ✓ Hemorrhagic uremic syndrome (HUS)- Bloody diarrhea, thrombocytopenia, renal failure & encephalopathy
    • ✓ HC (hemorrhagic colitis)
Diagnosis:
Sorbitol MacConkey agar- Do not ferment sorbitol and rhamnose, β-glucuronidase negative, PCR to detect VT1 and VT2 genes, Demonstration of VT1 and VT2 in Vero cell lines
5) EAEC- ((Entero aggregative E.coli)
  • Manifestations-Persistent diarrhea, Traveler's diarrhea, acute diarrhea, especially in developing countries
  • Most are “O” un-type able but “H” type able strain
  • Aggregated in a “Stacked Brick” formation on Hep2 cell lines
  • Produces EAST 1 toxin(enter aggregative heat stable Enterotoxins)
  • E. coli O104:H4 is an enteroaggregative strain, that has caused major outbreaks in Germany in 2011. Peculiarity is, it produces shiga toxin and can cause HUS.
6) DAEC-(Diffusely adherent E.coli)- Adhere to epithelial cells in diffuse pattern
Traveler's diarrhea:
  • Enterotoxigenic Escherichia coli - Most common
  • Enteroaggregative E. coli – 2nd Most common
  • Unknown – 20% - 3rd Most common
  • Others agents include:
    • Shigella and EIEC
    • Salmonella
    • Campylobacter
    • Aeromonas
    • Plesiomonas
    • Vibrio cholerae
    • Rotavirus
    • Norwalk virus
    • Entamoeba histolytica
    • Giardia lamblia
    • Cryptosporidium
    • Cyclospora
Other manifestations of E.coli:
  • GNBs are the most common cause (60–70% of cases) of hospital-acquired pneu- monia (E.coli - third or fourth most commonly isolated GNB in hospital-acquired pneumonia)
  • 194E. coli is one of the two leading causes of neonatal meningitis (the other being group B Streptococcus)- possess the K1 capsular antigen
  • Cellulitis/Musculoskeletal Infection
  • Endovascular Infection
  • Bacteremia (urinary tract is the most common source)
Treatment for E.coli infections:
  • UTI- fluoroquinolones
  • Diarrhea- fluid and electrolyte balance, No Antibiotics (antibiotics may aggravate EHEC)
  • Extra-intestinal E.coli- multidrug resistant, antibiotics given according to the sus- ceptibility testing, Carbapenems and amikacin are the most predictably active agents.
 
KLEBSIELLA
  • Nonmotile, Lactose fermenter
  • Capsulated, produce mucoid colonies
  • K. pneumoniae – Urease +ve - causes pneumonia, UTI, abdominal, wound and sur- gical site infection
  • K.ozaenae, - causes ozaenae (foul smelling nasal discharge)/ atrophic rhinitis
  • K.rhinoscleromatis – causes rhinoscleroma.
 
PROTEUS
  • Motile, Non Lactose fermenter
  • Tribe character- PPA+ve (phenyl alanine deaminase test +ve)
  • Urease+ve
  • H2S+ve – P.mirabilis and P.vulgaris
  • Swarms on blood agar
  • Fishy/seminal smell
  • Diene's phenomenon – to know the relatedness between different strains of Proteus
  • Forms struvite stone in bladder in alkaline urine
  • Non motile strains of Proteus (Basis of Weil felix) - OX 19, OX2 (Pr vulgaris), OX K (Pr mirabalis).
Organism which swarm- Proteus, Clostridium tetani, V. parahemolyticus, V. alginolyticus and Serratia.
 
SHIGELLA
  • Non Motile, Non Lactose fermenter, causes dysentery
  • 4 Species- Sh. dysenteriae, Sh. flexneri, Sh. boydii, Sh. sonnei.
  • 195Sh. dysenteriae Type 1:
    • Shiga bacillus, Forms Shiga toxin
    • Causes HUS and HC
    • Catalase negative,
    • Mannitol non fermenter
  • Sh. dysenteriae Type 2 – Schmitzi – indole +ve
  • Sh. dysenteriae Type 3–7 – Large and Sachs group.
Pathogenicity
  • Shiga toxin – only by Sh. dysenteriae type – 1 (3 mechanisms)
    • Neurotoxin
    • Enterotoxin – ShET 1and2
    • Cytotoxin – acts on Vero cell lines
  • Endotoxin (LPS) – Diarrhea, ulcers
  • Invasiveness is due to – VMA (virulence marker antigen)
  • Infective dose- Low, 10 organisms.
Infective dose
  • Salmonella Typhi -103-106
  • Vibrio cholerae- 106–108
  • Low Infective dose- Shigella, EHEC, Entamoeba histolytica and Giardia.
Complications:
  • Reactive Arthritis, Toxic Neuritis, Intussusception, Hemorrhagic uremic syn- drome.
Diagnosis: (Common media for both Salmonella & Shigella)
  • Specimen- mucus flakes of stool
  • Transport media- Sach's buffered glycerol saline
  • Selective media → DCA (deoxycholate citrate agar), XLD (Xylose-lysine-deoxy- cholate agar), SS Agar
  • Enrichment Broth – Gram Negative broth, Selenite F broth, Tetrathionate broth.
Treatment:
  • Mild- no treatment required
  • Severe- Ampicillin, Cotrimoxazole, Norfloxacin.
 
SALMONELLA
  • All Salmonelle are Motile (by means of Peritrichous flagella)- except S. gallinarum pullorum.
Classification:
  • Older classification- Typhoidal (S.Typhi & S.Paratyphi) and Non Typhoidal Salmonella (Other Salmonella spp)
  • Molecular classification-DNA hybridization study 7 group (pathogenic is –group 1)
  • Antigenic classification: Kaufmann White Scheme Based on- O antigen, typed to > 2399 serotypes.
 
Antigens of S.Typhi
O, H, Vi Antigens
H antigen→ Flagellar antigen
  • Heat labile, alcohol labile.
  • Formaldehyde stable,
  • Stronger immunogenic,
  • H Antibody Appears late, goes late
  • 196Reacts to H Antibody- forms large loosed fluffy clumps
  • Exist in two alternative phases –Most of them are biphasic except, (monophasic - S.Typhi, S. Paratyphi A)
O Ag→ Somatic polysaccharide
  • Heat stable
  • Formaldehyde labile
  • Less immunogenic,
  • O antibody appears early, goes early – indicates recent infection
  • Reacts to O Antibody- forms granular chalky clumps
Vi Ag – Surface polysaccharide covering O Ag
  • Heat labile, when present it renders the bacterium in agglutinable by O antiserum
  • Possessed by S Typhi and S. Paratyphi-C, S.dublin, Citrobacter
  • Poorly immunogenic, but protective
  • Absence of Vi Antibody in a proven case of enteric fever - carries poor prognosis
  • Persistance of Vi Ab in convalescent stage – carrier state
  • Epidemiological typing of S.Typhi- by Vi specific bacteriophage
 
Enteric Fever
Pathogenesis:
  • Infective dose- 103–106, incubation period - 10–14 days (3–21 days)
  • Enter via M cells lining GIT mucosa, near Peyer's patches present in the anti mesenteric border
  • Salmonellae trigger formation of membrane ruffles of M cells
  • These ruffles enclose bacteria within large vesicles inside M cells, by a process called as bacteria-mediated endocytosis mediated by a type III secretion, → Trans- ported to basolateral pockets of M cells
  • Engulfed by the macrophages and induce an mononuclear cells Infiltration
  • Disseminate throughout the body in macrophages via the lymphatics and colonize reticulo-endothelial tissues (liver, spleen, lymph nodes, and bone marrow).
  • Risk factors- Low immunity, antacid therapy, prolonged oral antibiotics, virulent strain and heavy inoculums, poor sanitation, children >adult, Urban >rural people.
 
Clinical Feature
  • Characterized by Step ladder-fever, rose spots (30%), coated tongue, headache, al- tered mental status, hepatosplenomegaly, epistaxis, relative bradycardia and pea soup diarrhea > constipation.
  • GIT bleeding & perforation – May occur in 3rd /4th weeks of illness
  • S. paratyphi A – Less common (1:4 ratio), milder disease and mainly GIT symp- toms occur
  • Neuropsychiatric symptoms (described as “muttering delirium” or “coma vigil”).
 
Lab Diagnosis
  • Culture – Blood, faeces, urine, bone marrow, rose spots etc
    • Enrichment media – Tetrathionate broth, Selenite F broth, Gram Negative broth
    • Selective media→ DCA, XLD, SS agar, Wilson Blair media(jet black colony)
    • Blood culture bottle (Castenada's Biphasic media)- for blood culture
    • Need tryptophan as growth factor
  • Demonstration of Antibodies in serum- e.g. Widal test
  • Demonstration of circulating Ag.
Biochemical reaction
  • NLF (Non lactose fermenter)
  • Motile except Salmonella gallinarm-pullorum
  • 197H2S +ve except strains of S. paratyphi A
  • Citrate + except S. Typhi, S. paratyphi A
  • Gas +ve (aerogenic) except S.Typhi (anerogenic)
  • Indole, Urease –ve
  • Agglutination with specific antisera-
    • S.typhi- with O9 antisera
    • S.paratyphi A- with O2 antisera
    • S.paratyphi B- with O4 antisera
Detection of Antibodies (Widal test):
  • Four antigens are used:
    • O antigen of S.typhi and H antigen of S.typhi, S.paratyphi A, S.paratyphi B
    • Only one O Ag is used- As O Ag of all three species cross react (share a com- mon antigen, factor-12)
  • Strain used- S Typhi 901 strain, for S.parathyphi A & B- Lab maintained strains
  • Preparation of antigen-
    • Formalin added (for H Ag),
    • Cultured in phenol agar followed by alcohol added & heated (for O Ag)
  • Tubes used- Dreyer's conical bottom tube (for H), Felix round bottom tube (for O).
  • Procedure-Patient sera is diluted, antigens are added- Incubated at 37°c for over- night in water bath
Interpretation of Widal test:
  • Antibodies - appear by end of 1st week, peaks 3rd week then falls
  • 4 fold rise - between 1st and 3rd week is highly significant than single high titer
  • Anamnestic reaction- false +ve in unrelated infection
  • Significant titre-
    • Titer of 1: 100 of O agglutinins → significant,
    • 1: 200 of H agglutinins → significant
  • H agglutinins appears late, goes late
  • O agglutinins appears early, goes early, Rise in O antibody indicates recent infection
  • Patients on antibiotics – then gives false negative result may occur
  • If fimbrial antigens are not removed- then false positive result may occur
Widal test
Interpretation
Rise of S.Typhi O and H antibody
Enteric fever due to S.Typhi
Rise of S.Typhi O and S. Paratyphi- A H antibody
Enteric fever due to S. Paratyphi –A
Rise of S.Typhi O and S. Paratyphi -B H antibody
Enteric fever due to S. Paratyphi- B
Only Rise of S.Typhi O antibody
Recent infection (any type)
Rise of S.Typhi - H antibody
Convalescent stage
Rise of all three H antibody
Post TAB vaccination
Other antibody detection methods:
  • Typhidot- OMP antigen is used, detects both IgM & IgG
  • IDL Tubex- O9 antigen is used, detects only IgM
  • IgM dip stick test- LPS antigen is used, detects only IgM
  • Dot blot assay- Flagellar antigen is used, detects only IgG
Week wise diagnosis of choice:
  • 1st week – Gold standard – Duodenal content culture > BM culture>blood culture (best- combination of all three)
    • If patient is on antibiotic- Then BM culture is more sensitive than blood culture in 1st week
    • 198Blood culture sensitivity – 90% in 1st week, 75% in 2nd week, 60% in 3rd week
    • Clot culture – higher sensitive than blood culture
  • End of 1st week – Widal test
  • 2nd and 3rd week – Widal test
  • 3rd and 4th week – Stool and urine culture
  • Stool culture –
    • +ve in both case/carrier,
    • +ve even if after start of antibiotics
 
Carriers
  • Chronic carrier- >1 year, Common in women and infants.
  • Typhoid Marry gave rise to >1300 cases during her life time.
  • Prevalence- 3% in India (1–4%)
  • Resides mainly in biliary tract, gall bladder and rarely urinary tract
  • Fecal carriers is MC in India, but Urine carriers- more dangerous than fecal carriers and associated with urinary tract anomalies and Schistosoma infection
  • Carriers are detected by- Vi agglutinins (1:10) or fecal/urine culture
  • Technique followed- Sewer-swab technique and Milipore membrane filtration
Reference Centre:
  • National Salmonella phage typing centre- Lady Hardinge medical college, Delhi
  • National Salmonella Reference Centre – Central Research Institute, Kasuli
  • National Salmonella Reference Centre for animal origin- Izatnagar
Treatment:
Drug of choice (DOC)
Alternate drug
Empirical treatment
Ceftriaxone
Azithromycin
Fully Susceptible S.Typhi
Ciprofloxacin
Amoxicillin, Chloramphenicol Cotrimoxazole
Multidrug-Resistant S.Typhi
Resistant to - Ampicillin, cotrimoxazole, chloramphenicol
Ciprofloxacin
Ceftriaxone Azithromycin
Nalidixic Acid–Resistant S Typhi
Ceftriaxone
Azithromycin, Higher or longer doses of Ciprofloxacin
Carriers
Ampicillin or Amoxycilin plus probenecid for 6weeks
Cotrimoxazole or Ciprofloxacin
 
Vaccine
Parental vaccine:
  • Killed WC(whole cell)- 2 doses 6 wks apart
  • TAB vaccine contains S.Typhi, Paratyphi A,B
  • Purified Vi polysaccharide Purified Vi -Given >2 year, 1 dose, effective>7 days, Booster→2 year,
Oral vaccine (Typhoral):
  • Live attenuated S. typhi Ty 21a strain is used (Gal E mutant –i.e. lacks galactose epimerase)
  • Schedule- 3 doses given on alternate day(1/3/5) as capsules, Efficacy starts after 7 days, Booster dose is give after 5 years
  • Given >6 year, Given to travelers & HIV patients
 
199Non Typhoidal Salmonella
  • NTS is differentiated from typhoidal salmonella by:
    • Zoonotic infection
    • Neutrophilic infiltration
    • More drug resistant than typhoidal salmonella.
  • Transmission:
    • By-Consumption of animal food products-eggs, poultry, meat, dairy products,
    • Highest during the rainy season in tropical & during the warmer months in temperate.
  • Risk factor-Common in HIV infected people, neonates
  • Species:
    • S. Typhimurium, commonest species (30–40%)
    • Salmonella septicemia – typically caused by S. cholerasuis
    • Others are S.enteritidis, S.dublin, S.newport, S. heidelberg.
  • Clinical Manifestations:
    • Salmonella gastroenteritis
    • Bacteremia and Endovascular Infections
    • Localized Infections- CNS, Pulmonary, Urinary tract, Bone, Joint, and Soft Tissue Infections.
  • Treatment: Extended-spectrum cephalosporins and fluoroquinolones are the agents of choice
    • Preemptive Treatment (for high risk)- Ciprofloxacin
    • Severe Gastroenteritis- Ciprofloxacin
    • Localized Infection- Ceftriaxone.
 
YERSINIA PESTIS
Cultural character:
  • Safety pin appearance (Bipolar staining) with Wayson's / methylene blue stain
  • Pleomorphism, nonmotile, no growth at 22°c
  • Optimum temp-27°C but capsule grows at 37°C
  • In broth – stalactite growth
  • Production of pigmented colonies on medium containing haemin (blood agar).
Antigenic structure:
  • F1envelope Ag – inhibits phagocytosis
  • V and W proteins – inhibits phagocytosis
  • Pesticin, fibrinolysin, coagulase
  • Plague Toxin- Endotoxin and murine toxin (both exo and endotoxin activity)
  • Ability to synthesize purine.
 
Plague
  • Synonyms- Black death or Mahamari.
  • Reservoir of infection- Wild rodents (Tatera indica in India), important zoonotic disease
  • 3 types–bubonic septicemic, pneumonic Plague
    • Wild or Sylvatic plague–Wild rodents squirrels
    • Urban Plague- Domestic rodent–Flea-human cycle
    • Man →Human flea (Pullex irritans)→Man
  • Vector → Rat flea (Xenospsylla cheopsis in North India and X.astia in south India)
    • Incubation Period:
      • ✓ Bubonic and Septicemic (2–7 days),
      • ✓ Pneumonic (1–3 days)
  • Mode of Infection:
      • ✓ Bubonic- Rat flea bite> contact with rodent > Man →Human flea (Pullex irritans)→Man
      • 200✓ Pneumonic plague –(Man - Man by Inhalational)
  • Most common variety–Bubonic
  • Highly infectious and fatal–Pneumonic
  • Septicemic plague- following accidental lab inoculation or 2° to bubonic plague
  • Recent outbreaks of plague in India:
    • Karnataka in 1966
    • 1994- Bubonic plague in Beed– Latur district (Maharashtra), Pneumonic plague outbreak in Surat. Overall 4780 suspected cases, 167 confirmed cases, 53 deaths had occurred.
    • 2002- Pneumonic plague in Shimla (Hat koti village), 16 cases, 4 deaths had occurred.
    • 2004- Bubonic plague outbreak in Uttarakhand (Dangud village, Uttarakashi). 8 cases, 3 deaths.
  • Cheopis index (No. of X.cheopis/rat):- >1 is an indicator of potential explosiveness if outbreak occurs.
Treatment: Streptomycin (DOC for cases), Tetracycline (DOC for chemoprophylaxis)
Vaccine:
  • Killed (Sokhey's modification of Haffkine strain, prepared in Haffkine insti- tute in Mumbai)
  • Dosage:-1 dose-contains 2000 million killed organism/ml, 1 ml- male, 0.75 ml- female
  • Schedule-2 doses (subcutaneously) at 1 week gap
  • Booster doses → 6 months as Immunity lasts for 6 months.
 
Yersiniosis
  • Yesinia infection other than Y.pestis – i.e Y. pseudotuberculosis and Y.enterocolitica
  • Motile at 22°c, not at 37°c
  • Cold enrichment done for culture.
Y. pseudotuberculosis:
  • Urease+ve, Poor growth on MacConkey agar
  • Zoonosis- In guinea pig, multiple nodules in lungs, liver (resembling tuberculosis like lesions)
  • Clinical feature–Typhoid like illness, Mesenteric lymphadenitis, resembling appendicitis, Erythema nodosum.
Y.enterocolitica:
  • Causes acute enterocolitis and terminal ileitis, lymphadenitis.
  • Reactive arthritis in HLA B27 patients.
 
Pasteurella Multocida
  • Oxidase positive
  • Zoonotic
  • Causes wound infection
  • Respiratory tract infection.
 
Francisella Tularensis (Tularemia)
  • Mode of transmission- Direct contact with rodents> Tick bite>Inhalation and Ingestion
  • Capsulated, intracellular
  • Fastidious, filamentous, filterable
  • Divide by budding and binary fission
  • Media–Francis blood dextrose cysteine agar
  • Local ulceration with lymphadenitis (most common form), Typhoid like fever, flu like.
201MULTIPLE CHOICE QUESTIONS
 
ESCHERICHIA COLI
1. Which is not an important cause of neonatal sepsis? (AIIMS Nov 2013)
  1. E.coli
  2. Group B Streptococci
  3. Acinetobacter
  4. Staph. auerus
2. Enterobacteriaceae are: (PGI Dec 2006)
  1. Pseudomonas
  2. Thlebsiella
  3. V.cholerae
  4. Proteus
  5. E.coli
3. E.coli gets attached to a surface with the help of : (AI 2000)
  1. Fucose
  2. Concavalin
  3. Phytohemagglutinin
  4. Lectin
4. E.coli gives pink colour with: (PGI 1999)
  1. Chocolate agar
  2. L.J. medium
  3. MacConkey medium
  4. Saline broth
 
Diarrheagenic E.coli
5. Which of the following microbes has enterotoxigenic, enterohaemorrhagic, enteropathogenic and enteroinvasive strains ? (AIIMS Nov 2014)
  1. Shigella
  2. Salmonella
  3. Vibrio cholerae
  4. Escherichia coli
6. MC cause of diarrhea in children of developing country is: (NEET Pattern Based)
  1. EHEC
  2. ETEC
  3. EPEC
  4. EIEC
7. Traveller's diarrhea is caused by: (NEET Pattern Based)
  1. ETEC
  2. EHEC
  3. EPEC
  4. EIEC
8. In E. coli true is: (NEET Pattern Based)
  1. ETEC is invasive
  2. EPEC acts via cAMP
  3. Pilli present in uropathogenic type
  4. ETEC causes HUS
9. Which of the following is not true about EHEC? (AI 2012)
  1. It secretes shiga like toxin
  2. Affects children
  3. Causes HUS
  4. O157:H7 serotype is the less likely to cause HUS
10. Which of the following is not true about HUS? (AI 2012)
  1. May present with hemorrhagic colitis
  2. Shiga like toxin has no role in HUS
  3. Usually self-limited
  4. Fever is typically absent
11. About ETEC true is: (AIIMS Nov 2010)
  1. Doesn't cause traveller's diarrhea
  2. MC cause of diarrhea in children
  3. Invades mucosa
  4. Fomite borne
12. Culture media used for diagnosis of EHEC O157: H7 is: (AI 2009)
  1. O7 culture
  2. Sorbitol MacConkey media
  3. XLD agar
  4. Deoxycholate media
13. With reference to infections with Escherichia coli the following are true except: (AI 2005)
  1. Enteroaggregative E.coli is associated with persistent diarrhea
  2. Enterohemorrhagic E.coli produces a disease similar to salmonellosis
  3. Enterotoxigenic E.coli is a common cause of travelers’ diarrhea
  4. Enteroinvasive E.coli possess virulence marker antigen
14. A 20 year old man presented with hemorrhagic colitis. The stool sample grew Escherichia coli in pure culture. The following serotype of E.coli is likely to be causative agent: (AI 2004)
  1. O157:H7
  2. O159:H9
  3. O107:H7
  4. O55:H7
15. All are true about EHEC except: (AIIMS 2004)
  1. Sereny's test is positive
  2. Ferments sorbitol
  3. Causes HUS
  4. Elaborates shiga like exotoxin
16. A 20 year old male had pain in abdomen and mild fever followed by. The etiological agent responsible is most likely to be: (AIIMS May 2003)
  1. Enteroinvasive E.coli
  2. Enterotoxigenic E.coli
  3. Enteropathogenic E.coli
  4. Enteroaggregative E.coli
17. Which of these are true of E.coli: (PGI June 2002)
  1. The LT (labile toxin) in ETEC acts via cAMP
  2. In types causing UTI the organism attaches via pili
  3. The ST (stable-toxin) of ETEC is responsible for causing hemolytic-uremic syndrome (HUS)
  4. EIEC invasiveness is under plasmid control
  5. In EPEC the toxin helps in invasion by the bacteria
18. 202MC cause of HUS in developing country? (Recent Question 2013)
  1. EHEC
  2. Shigella dysentriae type 1
  3. ETEC
  4. Pneumococcus
19. Sereny test is employed for laboratory diagnosis of: (APPG 2014)
  1. Enterotoxigenic E. coli
  2. Enteropathogenic E. coil
  3. Enterohaemorrhagic E. coil
  4. Enteroinvasive E. coli
 
Uropathogenic E.coli
20. Regular drinking of which of the following can help prevent UTI? (AIIMS Nov 2011)
  1. Grape juice
  2. Raspberry juice
  3. Orange juice
  4. Cranberry juice
21. A young lady presents with fever, dysuria and pain abdomen. Uncomplicated acute cystitis was diagnosed. Which among the following is not true? (AI 2011)
  1. E.coli count was < 103/ml
  2. 1 pus cell per 7 fields
  3. 1 bacilli per field
  4. Nitrate test positive
 
KLEBSIELLA
22. A patient is kept on ceftriaxone and amikacin, ESBL Thlebsiella infection. What will you do next? (AIIMS Nov 2010)
  1. Continue with same antibiotic but in higher dose
  2. Change ceftriaxone and add ceftazidime
  3. Start imipenem in place of ceftriaxone
  4. Remove Amikacin
23. True about ESBL enzymes a/e: (AIIMS May 2011)
  1. Phenotypic classification based on molecular structure
  2. Plasmid mediated
  3. Carbapenam effective
  4. Ambler classification is based on structural characteristics
24. Drug against ESBL producing Pseudomonas: (AIIMS Nov 2014)
  1. Ceftriaxone + Piperacillin
  2. Ceftriaxone+ Tazobactam
  3. Piperacillin + Tazobactam
  4. Ceftriaxone + Piperacillin+ Tazobactam
25. About ESBL producing bacteria, true is: (Recent Question 2013)
  1. Only seen in gram positive bacteria
  2. Only seen in gram negative bacteria
  3. Plasmid mediated
  4. It is associated only with community acquired disease
 
PROTEUS
26. Dienes’ phenomena is seen with : (PGI 2002)
  1. Proteus mirabilis
  2. Klebsiella
  3. Proteus vulgaris
  4. Providentia
  5. Morganella
27. All of the following are true except : (AI 2001)
  1. E.coli is an aerobe and facultative anaerobe
  2. Proteus forms uric acid stones
  3. E.coli is motile by peritrichous flagella
  4. Proteus causes domination of phenylalanine to phenlpyruvic acid
 
SHIGELLA
28. Which of the following toxins acts by inhibiting protein synthesis: (AI 2004)
  1. Cholera toxin
  2. Shiga toxin
  3. Pertussis toxin
  4. LT of Enterotoxigenic E.coli
 
SALMONELLA
29. Steroid is used in the treatment of (AIIMS Nov 2013)
  1. Severe typhoid
  2. E.coli septicaemia
  3. Cerebral malaria
  4. Visceral leishmaniasis
30. True about Salmonellosis: (PGI June 2011)
  1. ed incidence in developed countries
  2. Antacid and prolonged antibiotic administration promote infection
  3. Always fatal
  4. Foodborn to man and animal
  5. Resist drying, salting and freezing for years
31. Salmonella Typhi is the causative agent of typhoid fever. The infective dose of S. Typhi is: (AI 2012, AIIMS Nov 2006)
  1. One bacillus
  2. 108 -1010 bacilli
  3. 102 – 105 bacilli
  4. 1 -10 bacilli
32. 15 year old girl had splenomegaly, leucopenia, fever and died in a few days. Longitudinal ulcers were found in intestine. What should be the probable diagnosis? (AI 2012)
  1. Typhoid
  2. Tuberculosis
  3. Amoebiasis
33. Regarding typhoid fever all are true except: (JIPMER 2010)
  1. Mostly affect 3–4th decades
  2. Man is the only reservoir
  3. Can cause perforation of intestine
  4. Rose spots seen in 2nd week of illness
34. All are correct regarding widal test, except: (AIIMS Nov 2009)
  1. Baseline titer differs depending on the endemicity of the disease
  2. High titer value is a single widal test is not confirmative
  3. O antibody last longer and hence is not indicative of recent infection
  4. O antibody cannot differentiate between types
35. All of the following statements about non typhoid salmonella are true, except: (AI 2009)
  1. Humans are the only reservoirs
  2. Transmission is most commonly associated with eggs, poultry and undercooked meat
  3. Common in immunocompromised individuals
  4. Resistance to fluoroquinolones has emerged
36. 203All of the following salmonellae are motile except: (SGPGI 2009)
  1. S. Typhi
  2. S. Enteridis
  3. S. Gallinarum pullorum
  4. S. Chester
37. Resistance of S. Typhi to the following anti-biotics is not reported as yet in India: (AI 2008)
  1. Chloramphenicol
  2. Ampicillin
  3. Ciprofloxacin
  4. Ceftriaxone
38. Drug commonly used against enteric fever are all except: (AI 2008)
  1. Amikacin
  2. Ciprofloxacin
  3. Ceftriaxone
  4. Azithromycin
39. Incubation period of salmonella Typhi: (PGI June 2005, AIIMS May 94)
  1. 2–5 days
  2. 3–21 days
  3. 14–25 days
  4. >60 days
40. Vi antigen found in: (PGI June 2005)
  1. Salmonella paratyphi ‘A’
  2. Salmonella paratyphi ‘C’
  3. Salmonella Dublin
  4. Thlebsiella pneumoniae
  5. Citrobacter freundii
41. There has been an outbreak of food born salmonella gastroenteritis in the community and the stool samples have been received in the laboratory. Which is the enrichment medium of choice: (AIIMS May 2003)
  1. Cary Blair medium
  2. V R medium
  3. Selenite “F” medium
  4. Thioglycollate medium
42. A 24 year – old cook in a hostel mess suffered from enteric fever 2 years back. The chronic carrier state in this patient can be diagnosed by: (AIIMS Nov 2002)
  1. Vi agglutination test
  2. Blood culture in Brain Heart infusion broth
  3. Widal test
  4. Bone marrow culture
43. In a patient with typhoid, diagnosis after 15 days of onset of fever is best done by: (AI 2001, 2002)
  1. Blood culture
  2. Widal
  3. Stool culture
  4. Urine culture
44. True about typhoid: (PGI June 2003)
  1. It is caused by food poisoning
  2. Water can transmit the disease
  3. Ty 21 a is an oral vaccine
  4. Chronic carrier is called when transmitted up to 6 month
  5. Widal test +ve in 1 week
45. Salmonella gastroenteritis is: (PGI 2002)
  1. Mainly diagnosed by serology
  2. Blood and mucus present in blood
  3. Caused by animal products
  4. Symptoms appear by 4–48 hour
  5. Features are due to exotoxin released
46. True about Salmonella Typhi infection in intestine is /are: (PGI 2001)
  1. Affects Peyer's patches
  2. Common in mesenteric border
  3. Erythrophagocytosis is characteristic
  4. Strictures are common
  5. Typhoid ulcer always bleed exsangiunantly
47. Pea – soup stool is characteristically seen in: (JIPMER 2000)
  1. Cholera
  2. Typhoid
  3. Botulism
  4. Polio
48. Typhoid carrier is diagnosed by? (AIIIMS 08)
  1. Detection of Core antigen
  2. Detection of Vi antigen
  3. Detection of Vi antibody
  4. Typhoid bacilli in stool
49. DOC for typhoid carrier? (DNB 01,02)
  1. Ampicillin
  2. Chloramphenicol
  3. Cotrimoxazole
  4. Clindamycin
50. True about typhoid? (DNB 08)
  1. Female carriers- less common
  2. Male carriers are less common but more dangerous
  3. Gall bladder Not involved in acrriers
  4. Tetracycline –DOC
51. In enteric fever, isolation is done till (UP 02, AP 06)
  1. Fever subsides
  2. Stool culture negative till three times
  3. Spleen subsides
  4. Blood negative till three times
52. Typhoral schedule: (UP 08)
  1. 1,3,5 days
  2. 1,2,3,days
  3. 1,2,4, days
  4. 1,7,14 days
53. H2S forming Salmonella: (PGI Nov 2012)
  1. S.Typhimurium
  2. S.Typhi
  3. S.Paratyphi A
  4. S.Paratyphi B
  5. S.Cholerasuis
 
YERSINIA
54. What is not true about yersiniosis: (NEET Pattern Based)
  1. Zoonosis
  2. Caused by Y.pestis
  3. By Yersinia enterocolitica
  4. By Yersinia pseudotuberculosis
55. Farmer presents with the features of high fever, painful inguinal lymphadenopathy, vomiting and diarrhea and hypotension. Which stain will help in the diagnosis? (AI 2011, AIIMS Nov 2012)
  1. Neisser stain
  2. Wayson's stain
  3. Albert's stain
  4. Mcfadyean's stain
56. Most common mode of transmission of Pasteurella multocida is: (AI 2009)
  1. Animal bites or scratches
  2. Aerosols or dust
  3. Contaminated tissue
  4. Human to human
57. 204A girl from Shimla presented to OPD with fever, hypotension, malaise and axillary and inguinal lymphadenopathy. Culture in glucose broth shows stalactite growth. Most likely causative organism is: (AIIMS Nov 2008)
  1. Yersinia pestis
  2. Francisella tularensis
  3. Brucella abortus
  4. Coxiella burnetii
58. A young boy had a flea bite while working in a wheat grain goes down. After 5 days he developed fever and had axillary lymphadenopathy. A smear was sent to the laboratory to perform a specific staining. Which one of the following staining method would help in the identification of the suspected pathogen: (AI 2006)
  1. Albert staining
  2. Ziehl – Neelsen staining
  3. McFadyean's staining
  4. Wayson staining
59. True about Y.pestis: (PGI Dec 2003, Dec 2006)
  1. Gram +ve
  2. Gram – ve
  3. Motile
  4. Non –motile
  5. It is coccobacilli
60. All of the following statement about plague are wrong except: (AIIMS 2004)
  1. Domestic rat is main reservoir
  2. Bubonic is the most common variety
  3. The causative bacillus can survive up to 10 years in the soil of rodent burrows
  4. Incubation period for pneumonic plague is one to two week
61. True statements about Y. Pestis are: (PGI June 2004)
  1. Gram positive
  2. Non – motile
  3. Benzyl penicillin is given in prophylaxis
  4. Patients are kept isolated till 48 hours of treatments
  5. Repeated blood culture is diagnostic
62. The drug of choice for chemoprophylaxis in contacts of a patient of pneumonic plague is : (AIIMS Nov 2002)
  1. Penicillin
  2. Rifampicin
  3. Erythromycin
  4. Tetracycline
63. Most effective way to break transmission chain plague? (AIIMS 02)
  1. Flea control
  2. Rodent control
  3. Early case detection and treatment
  4. Vaccination
64. Plague epidemic in Surat had occurred after a silence period of? (DPG 05)
  1. 18 yrs
  2. 20 yrs
  3. 28 yrs
  4. 30 yrs
65. Maximum explosiveness of plague is determined by? (AP 03, DPG 06)
  1. Total flea index
  2. Cheopis index
  3. Burrow index
  4. Specific percentage of flea
66. Highly infectious plague? (TN 03)
  1. Bubonic plague
  2. Pneumonic plague
  3. Septicemic plague
  4. Any of the above
67. Plague is transmitted by? (RJ 2001)
  1. Soft tick
  2. Hard tick
  3. Rat flea
  4. Louse
68. Which of the following is NOT a source of infection in plague? (MHPG 2014)
  1. Case of bubonic plague
  2. Case of pneumonic plague
  3. Infected rodents
  4. Infected rat fleas
205EXPLANATIONS
 
ESCHERICHIA COLI
1. Ans. (c) (Acinetobacter) Ref: Nelson Textbook of Pediatrics, 18th ed. Chapter 176.
Among the options, Acinetobacter is the best answer.
The infectious agents associated with sepsis in pediatric patients-
  • In the neonatal age group, group B streptococcus, Escherichia coli, Listeria monocytogenes, enteroviruses, and herpes simplex virus are the pathogens most commonly associated with sepsis.
  • In older children Streptococcus pneumoniae, Neisseria meningitidis, and Staphylococcus aureus are more common.
  • Toxic shock syndrome from group A streptococcus or S. aureus can also be seen in older children.
  • Infections with gram-negative bacteria (e.g., Escherichia coli, Pseudomonas, Acinetobacter, Klebsiella, Enterobacter, Serratia) and fungi (e.g., Candida, Aspergillus) most often occur in immunocompromised and hospitalized pa- tients colonized with these organisms.
  • Pseudobacteremia may be associated with contaminated heparin flush solutions, intravenous solutions, albu- min, cryoprecipitate, and infusion equipment. Contaminants include water-borne organisms such as Burkholde- ria cepacia, Pseudomonas aeruginosa, and Serratia.
2. Ans. (b), (d), (e) (Klebsiella, Proteus, E.coli) Ref: Koneman's Diagnostic Microbiology 5th/p229–234
  • Classification of Enterobacteriaceae – Refer chapter review.
3. Ans. (d) (Lectin) Ref: EXCLI Journal 2007; “E.coli K88 Interaction with IgA Oligosaccharides.”
  • “K88 fimbrial adhesins are surface filaments with lectin activity that recognize specific glycoconjugates and helps in adhesion to the surface of intestinal cells.”
  • About Other options
  • Phytoagglutinin- Protein that agglutinate RBC and WBC
  • Fucose –are hexose deoxy sugar present on mammalian cell surface that might inhibits adhesion of Shigella, Pseudomonas and few other organism
  • Concavalin A- mitogen, acts on T lymphocyte and stimulates thymocytes.
4. Ans. (c) (MacConkey medium) Ref: Ananthanarayan 9/e p275, 8/e p270
  • Lactose fermenter s among Enterobacteriaceae family- E.coli, Klebsiella, Citrobacter, Enterobacter
  • Detail- Refer chapter review.
5. Ans. (d) (Escherichia coli) Ref: Ananthnarayan 9th /p279
Diarrheagenic E.coli has six pathotypes. (Refer chapter review for detail)
6. Ans. (c) (EPEC) Ref: Ananthanarayan 9/e p 279, 8/e p277
  • Next to Rotavirus, EPEC is the MC cause of diarrhea in children of developing country.
7. Ans. (a) (ETEC) Ref: Ananthanarayan 9/e p 279, 8 /e p277
  • ETEC is the MC cause of traveler's diarrhea.
8. Ans. (c) (Pilli present in uropathogenic type) Ref: Ananthanarayan 9/e p279, 8/e p277
  • Uropathogenic E.coli strains characteristically possess fimbria such as mannose resistant fimbria and p fimbria.
9. Ans. (d) (O157:H7 serotype is the less likely to cause HUS.)
Ref: Ananthanarayan 9/e p279, 8/e p277, Harrison 18/e chapter 286/Vascular Injury to the Kidney, Chapter-149/p1251
  • STEC/EHEC(Shiga toxin producing or Entero hemorrhagic E.coli) produces verocytotoxin or shiga like toxin
  • VT can cause capillary microangiopathy that leads to can cause hemorrhagic colitis and the hemolytic-uremic syndrome (HUS).
  • 90% of HUS cases in children are caused by STEC/EHEC.
  • 206O157:H7 is the most prominent serotype, but other strains such as 0157/H7, 0111:H–, 026:H11/ H–, and 0145:H28 can also produce shiga toxin.
10. Ans. (b) (Shiga like toxin has no role in HUS)
Ref: Harrison 18/chapter 286/Vascular Injury to the Kidney, Chapter-149/p1251
Hemolytic Uremic Syndrome
Thrombotic Thrombocytopenic Purpura
Age
Children
Adult (40yr)
Pathogenicity
Hemorrhagic diarrhea due to- Shiga toxin mediated (S.dys-1)
Verocytotoxin mediated (EHEC)- 157/H7
Absence of metalloprotease ADAMTS13 specific for vWF
Neurologic symptoms
Less common
More pronounced
Fever
Typically absent
Present
Treatment
Usually self-limited
Plasma exchange -Not Required
Plasma exchange -Required for treatment
11. Ans. None or (d) Fomite borne Ref: Jawetz 24th /p257
  • ‘Shigella are transmitted by “food, fingers, feces, and flies and from person to person’
  • I confess that I could not get any reference for Fomite transmission of ETEC.
But all other options are wrong:
  • ETEC is the most common cause of traveller's diarrhea
  • ETEC can cause diarrhea in children, but the MC cause – Rotavirus and EPEC
  • ETEC doesn't invade mucosa
  • So I suggest You to go with ‘Fomite borne’
12. Ans. (b) (Sorbitol MacConkey media) Ref: Ananthanarayan 9/e p279, 8/e p277
  • EHEC can be differentiated from other E.coli as it is sorbitol non fermenter, hence it produces colorless colony on Sorbitol MacConkey media where as others can ferment sorbitol producing pink colony.
13. Ans. (b) Enterohemorrhagic E.coli produces a disease similar to Salmonellosis Ref: Ananthanarayan 9/e p279, 8/e p277
  • Enterohemorrhagic E.coli produces a disease similar to shigellosis (dysentery)
  • EHEC produces verocytotoxin which is similar to Shiga toxin produced by Sh.dysenteriae type1.
Other Options are correct statements:
  • Enteroaggregative and Enteropathogenic E.coli is associated with persistent diarrhea
  • Enterotoxigenic E.coli is a common cause of travelers’ diarrhea
  • Enteroinvasive E.coli possess virulence marker antigen
14. Ans. (a) (O157:H7) Ref: Ananthanarayan 9/e p279, 8/e p277
  • Enterohemorrhagic E.coli dysentery can lead to complications like hemorrhagic uremic syndrome and hemor- rhagic colitis.
  • Serotypes’ associated with EHEC are -O157:H7 and O26:H1
  • Similar illness also produced by Shigella dysenteriae type1.
15. Ans. (a) (Sereny test is positive) Ref: Ananthanarayan 9/e p279, 8/e p277, Harrison 17/e p 942 & 18/e p1252
– ‘Sereny test is positive for Enteroinvasive E.coli and Shigella. Instillation of fresh isolate into Guinea pig eye produces mucopurulent conjunctivitis and keratitis.”
About Other Options
  • EHEC causes Hemorrhagic Uremic Syndrome and hemorrhagic Colitis
  • EHEC produces Verocytotoxin (also known as Shiga like toxin)
  • It doesn't ferment sorbitol. So EHEC can be differentiated from other on Sorbitol MacConkey.
  • However, few strains of EHEC can ferment sorbitol, so they produce similar pink colony like other E.coli. Hence fermentation of Rhamnose is taken into consideration.
  • EHEC doesn't ferment rhamonose where as others ferment.
20716. Ans. (a) (Enteroinvasive E.coli) Ref: Ananthanarayan 9/e p279, 8/e p277
  • Gastroenteritis with presence of pus cells and RBCs in stool microscopy is suggestive of dysentery cause by EIEC.
  • Other options: causes diarrhea
17. Ans. (a), (b), (d) (The LT (labile toxin) in ETEC acts via cAMP, In types causing UTI the organism attaches via pili, EIEC invasiveness is under plasmid control) Ref: Ananthanarayan 9/e p276–77, 8/e p275–77
Option a: Heat labile toxin
  • Resembles cholera toxin in structure, Possess A, B5 subunits
  • A subunit- Causes ADP ribosylation of GTP binding protein → ↑ Adenyl cyclase → ↑ cAMP,
  • B subunit-binds GM1 ganglioside
Option b:
  • Mannose resistant Pili and P Pili are important virulence factors of uropathogenic E.coli.
  • They bind to uroepithelial cells.
Option c:
  • ST (heat stable-toxin) is expressed by ETEC where as hemolytic-uremic syndrome (HUS) is a complication of EHEC and Shigella dys-1 mediated by verocytotoxin or shiga toxin.
  • ST acts by ↑ cGMP
Option d:
  • Invasiveness of EIEC is due to Virulence marker antigen which is under plasmid control
Option e:
  • EPEC doesn't produce any toxin
18. Ans. (b) (Shigella dysentriae type 1) Ref: Ananthanarayan 9/e p287
MC cause of HUS in developing country is Shigella dysentriae type 1; where as in developed country, it is EHEC O157:H7.
19. Ans. (d) (Enteroinvasive E. coli) Ref: Ananthnarayan 8th /p277
Sereny's test is an animal pathogenicity test employed for laboratory diagnosis of Enteroinvasive E. coli and Shigella. Instillation of freshly isolated suspension of bacteria into the eyes of guinea pigs leads to mucopurulent conjunctivitis and severe keratitis.
 
Uropathogenic E.coli
20. Ans. (d) (Cranberry juice)
Ref: Journal- “Inhibitory activity of cranberry extract on the bacterial adhesiveness in the urine of women:
An ex-vivo study”. Int. journal of immunopathology and pharmacology, Tempera et al 2010–23 (2): 611–8.
  • Cranberry juice may help prevent and relieve the symptoms of urinary tract infections
  • Mainly by 3 mechanisms:
    • ✓ Directly by altering the molecular structure of the fimbriae on the pathogenic strains of the bacteria that cause the infections.
    • ✓ Proanthocyanidins in cranberries prevents the bacteria adherence to the bladder and urinary tract.
    • ✓ Indirectly on the bacteria by reducing the intravesical pH
21. Ans. (a) (E.coli count was < 103/ml) Ref: Bailey and Scott Diagnostic Microbiology 12/e p848
  • UTI is diagnosed when – The bacterial count exceeds 105/ml of urine(significant Bacteriuria),
About other options
Option (b) - “Under appropriate condition, finding of 1 leukocyte per 7 high power field correlates with 104 leucocytes /ml and which implies significant”. …..Mackie and McCartney Practical Microb.14th /p86
Option (c) - Gram staining is not a reliable method to estimate the significance of UTI.
Using >1/Oil immersion field, the sensitivity is 91% which correlates with significant Bacteriuria.
…………. Baily and Scott Diagnostic Microbiology 12/p848
Option (d) - The most common organisms causing UTI are E.coli (70%), Klebsiella, Proteus. All these belong to family Enterobacteriaceae and all can reduce nitrate to nitrite.208
 
KLEBSIELLA
22. Ans. (c) (Start imipenem in place of ceftriaxone)
Ref: Journal –Extended –Spectrum- β - Lactamases: clinical update, Clinical Microbiology Reviews, Oct.2 005, p. 657–686
  • ESBL (Extended spectrum beta lactamases) are resistant to all Penicillin and1st/2nd/3rd cephalosporin and monobactam
  • Which can be overcome by addition of b lactamase inhibitor like clavulinic acid
  • Other alternate which can be given are-
    • ✓ Carbapenams like Imipenam and meropenam
    • ✓ Cephamycins (like cefoxitin and cefotetan)
    • ✓ Different class of antibiotics like aminoglycoside
Beta lactamase can be classified by:
  • Ambler classification- Structural or molecular classification
  • Bush Jacoby Medeiros classification- Functional (phenotypic) classification
Ambler Classification of Beta Lactamases
Class A-ESBL (Extended spectrum beta lactamases)
Resistant to all Penicillin + 1st/2nd /3rd cephalosporin + Monobactam
But still sensitive to Carbapenams and cephamycins
Can be Overcome by β lactam + β lactamase inhibitor combination
Detected by- Combination disk test (Ceftazidime and cetftazidime+clavulinic acid), Three dimensional test (best method)
Class B-MBL (Metallo betalactamase)
Resistant to same as AMPc spectrum + Carbapenams
Cannot be Overcome by β lactam + β lactamase inhibitor combination
Detected by EDTA disk synergy test, modified Hodge test
Class C- AMP C β lactamase
Resistant to same as ESBL spectrum + resistant to cephamycins (like cefoxitin and cefotetan)
But still sensitive to imipenem
Class D- oxacillinase
Can be Overcome by β lactam + β lactamase inhibitor combination
23. Ans. (a) (Phenotypic classification based on molecular structure)
Ref: Review article- IJMM /Year: 2004, volume: 22, issue: 2, page: 75–80
(Extended spectrum -lactamases (ESBL) - an emerging threat to clinical therapeutics)
  • Ambler classification of beta lactamase – based on Structural or molecular classification
  • Bush Jacoby Medeiros (or Functional) classification of beta lactamase– Based on phenotypic properties
  • Beta lactamases are plasmid mediated
  • ESBL are sensitive to Carbapenems
24. Ans. (c) (Piperacillin + Tazobactam) Ref: Journal–ESBL:Clinical update, Clinical Microbiology Reviews, Oct.2005, p. 657–686
Extended Spectrum-β-Lactamases (ESBL) producing Pseudomonascan be treated with an antiPseudomonial β lactam (e.g. piperacillin) plus β lactamase inhibitor such as tazobactam combination therapy.
25. Ans. (c) (Plasmid mediated) Ref: Mackie McCartney 14/e p169
  • Extended spectrum beta lactamases (ESBL) are plasmid coded, produced by both gram positive and negative bacteria, mainly expressed by hospital strains and sometimes also by community strains.
 
PROTEUS
26. Ans. (a) and (c) (P. mirabilis, P. vulgaris) Ref: Mackie and McCartney Practical Microb.14/e p373
Diene's phenomenon – to know the relatedness between different strains of Proteus
  • When two Proteus strains are streaked at two ends of a blood agar, they start swarming and join to each other.
  • 209If both the strains are related, there will not be any line of separation and if both the strains are not related, they will be separated by a line of demarcation.
27. Ans. (b) (Proteus forms uric acid stones) Ref: Ananthanarayan 9/e p2782, 8/e p280 Proteus forms struvite stone in bladder in alkaline urine
About other options
  • E.coli is an aerobe and facultative anaerobe
  • E.coli is motile by peritrichous flagella
  • Proteus produces enzyme phenylalanine deamidase which converts phenylalanine to phenlpyruvic acid
 
SHIGELLA
28. Ans. (b) (Shiga toxin) Ref: Ananthanarayan 9/e p286, 8/e p284 Toxin which acts by inhibiting protein synthesis-
  • Diphtheria toxin and Pseudomonas Exotoxin A – inhibiting elongation factor 2
  • Shiga toxin and Verotoxin (Shiga like toxin) of EHEC- inhibiting 60s ribosome
 
SALMONELLA
29. Ans. (a) (Severe typhoid) Ref: Harrison 18/e p1277, Goodman & Gilman/11th Ed. (chapter 59)
Severe enteric fever remains one of the few indications for glucocorticoid treatment of an acute bacterial infection
  • In a randomized, prospective, double-blind study of critically ill patients with enteric fever (i.e., those with shock and obtundation) in Indonesia in the early 1980s, the administration of dexamethasone with chloram- phenicol was associated with a substantially lower mortality rate than was treatment with chloramphenicol alone. Although this study has not been repeated in the “post-chloramphenicol era, Severe enteric fever remains one of the few indications for glucocorticoid treatment of an acute bacterial infection
  • Although the use of immunosuppressive glucocorticoids in infectious diseases may seem paradoxical, there are a limited number of settings in which they are indicated in the therapy of specific infectious pathogens
  • Example of such beneficial effects is seen in-
    • ✓ AIDS patients with Pneumocystis carinii pneumonia and moderate to severe hypoxia; addition of glucocor- ticoids to the antibiotic regimen increases oxygenation and lowers the incidence of respiratory failure and mortality.
    • ✓ Similarly, glucocorticoids clearly decrease the incidence of long-term neurological impairment associated with Haemophilus influenzae type b meningitis in infants and children 2 months of age or older.
    • ✓ Severe enteric fever
30. Ans. (a) (b) (d) (↓ed incidence in developed countries, Antacid and prolonged antibiotic administration promote infection, Foodborn to man and animal)
Ref: Harrison 17/e p956, 18/e p1274–76
  • Option a- Typhoid fever is virtually eliminated from advanced countries but continues to be a major problem in developing nations due to poor sanitation.
    …………… Ananthanarayan 8/e p294
  • Option b- Risk factor- Low immunity, antacid therapy, prolonged oral antibiotics, virulent strain and heavy inoculums, poor sanitation, children >adult, Urban >rural people… Harrison 18/e p1274
  • Option c- Rarely Salmonella is fatal. Prompt administration of appropriate antibiotic therapy prevents severe complications of enteric fever and results in a case-fatality rate of <1%.... Harrison,18/e p1276
  • Option d- Most commonly, Salmonella is transmitted by contaminated food or water. Non typhoidal salmonella (NTS) can be acquired from multiple animal reservoirs … Harrison 18/e p1274
  • Option e- Most of the microorganisms don't survive drying except moulds and some spore bearers. Few organ- isms like halophilic Vibrio and Staphylococcus can survive in increased salt concentration.
    …….Murray's Manual of Clinical Microbiology 9th /p56
  • So these lines indicate that Salmonella cannot survive by drying and salting.
31. Ans. (c) (102 – 105 bacilli) Ref: Harrison 18/e p1274
  • All Salmonella infections begin with ingestion of organisms, most commonly in contaminated food or water. The infectious dose is 103–106 colony-forming units.
32. Ans. (a) (Typhoid) Ref: Harrison18/e p1276
  • 210Splenomegaly, leucopenia, fever and Longitudinal ulcers found in intestine are characteristic features of enteric fever.
33. Ans. (a) (Mostly affect 3–4 th decades) Ref: Ananthanarayan 9/e p295–96,8/e p294, Harrison 18/e p1276
  • Mostly affect 5–20 years (Though can affect any age)
  • Man is the only reservoir (Animal reservoir are important for non typhoidal Salmonella)
  • Can cause perforation of intestine in 3rd -4th week of infection
  • Rose spots -Faint, salmon-colored, blanching, maculopapular rash on the trunk and chest. The rash is evident in 30% of patients at the end of the first week and resolves without a trace after 2–5 days
34. Ans. (c) (O antibody last longer and hence is not indicative of recent infection)
Ref: Ananthanarayan 9/e p298, 8/e p296–97
  • Antibody appears early and goes early and rise of O Antibody without rise of H Antibody indicates recent infection.
  • In widal test, 4 antigens are used to detect respective antibodies i.e. antigen of S.Typhi and H antigen of S.Typhi, S. Paratyphi A,B
  • Only one O antigen is used- as O antigen of all 3 cross reacts with each other due to sharing of factor 12. Hence use of single O antigen can detect O antibody against any of the type where as H antigen is type specific.
  • Baseline titer occurs in the community depending on the endemicity of the disease. Low titers are considered as base line titre in endemic population. There is cut off, above which the titre is considered as significant.
    • ✓ Titer of 1: 100 of O agglutinins → significant,
    • ✓ 1: 200 of H agglutinins → significant
  • 4 fold rise - between 1st and 3rd week is highly significant than single high titer because of Anamnestic reaction (i.e. false +ve in unrelated infection). After one week interval,
    • ✓ If the titre increases- indicates true infection
    • ✓ If the titre decreases- indicates anamnestic reaction
35. Ans. (a) (Humans are the only reservoirs) Ref: Harrison 17/e p 959–960 & 18/e p1278
“Unlike S. Typhi and S. Paratyphi, whose only reservoir is humans; Non Typhoid Salmonella can be acquired from multiple animal reservoirs.”
About other options
  • Option b: Transmission is most commonly associated with animal food products, especially eggs, poultry, under cooked ground meat.
  • Option c: Mortality associated with NTS are highest among the elderly, infants, and immuno-compromised individuals, including those with hemoglobinopathies, HIV infection, or infections that cause blockade of the reticuloendothelial system.
  • Option d: Resistance to Nalidixic acid and fluoroquinolones also has begun to emerge and is most commonly associated with point mutations in the DNA gyrase genes gyrA and gyrB.
36. Ans. (c) (S. gallinarum pullorum) Ref: Ananthanarayan 9/e p 2990, 8/e p289
Salmonelle are motile except S. gallinarum pullorum which are nonmotile (aphasic)
37. Ans. None
Ref: Harrison 17/e p959 & 18/e p1277, Journal: Antimicrobial resistance in typhoidal salmonellae, BN Harish, GA Menezes et al. IJMM: Year : 2011 Volume : 29 Issue : 3 Page : 223–229.
  • ✓ Resistance of S.Typhi to quinolones, ampicillin, cotrimoxazoles and chloramphenicol is reported since many years.
  • ✓ Resistance to ceftriaxone has been reported recently…IJMM: Year : 2011, Volume- 29,Issue : 3
Emergence of resistance to third-generation cephalosporins to S.typhi:
  • There are sporadic reports of high-level resistance to ceftriaxone in typhoidal salmonellae where CTX-M-15 and SHV-12 extended spectrum β- lactamases (ESBLs) have recently been reported
  • Recently, for the first time ACC-1 AmpC β- lactamase producing S. Typhi has been reported.
  • Spread of broad-spectrum β-lactamases would greatly limit therapeutic options and leave only carbapenems and tigecycline as secondary antimicrobial drugs.
21138. Ans. (a) (Amikacin) Ref: Harrison 17/ e p959 & 18/e p1277
  • Aminoglycosides are not given for enteric fever
  • Treatment for Enteric fever- Refer Chapter Review
39. Ans. (b) (3–21 days) Ref: Harrison 17/e p957 & 18/e p1275
  • “The incubation period for S. Typhi averages 10–14 days but ranges from 3 to 21 days, with the duration likely reflecting the inoculum size and the host's health and immune status.”
40. Ans. (b), (c), (e) (S. Paratyphi C, S. Dublin, Citrobacter) Ref: Ananthanarayan 9/e p293, 8/e p290
Vi Antigen is possessed by S Typhi and S. Paratyphi C, S. Dublin, Citrobacter (Ballerup-Bethesda group). For detailed explanation refer chapter review.
41. Ans. (c) (Selenite “F” medium) Ref: Mackie Mc Cartney Practical Microbiology 14th /p387
Outbreak of food born salmonella gastroenteritis
For the stool culture, 1st it has to be processed in an enrichment medium followed by streaking in a selective media.
Enrichment medium for Salmonella- Tetrathionate broth, Selenite F broth, Gram negative broth
42. Ans. (a) (Vi agglutination test) Ref: Ananthanarayan 9/e p299, 8/e p297
  • Vi agglutinins of titer of > 1:10 is taken significant for diagnosis of Typhoidal carriers.
43. Ans. (b) (Widal) Ref: Ananthanarayan 9/e p298, 8/e p297
  • “Agglutinins in Widal test start appearing at the end of 1st week and increases steadily till 3rd/4th week then fall.”
  • Week wise diagnosis of choice of Enteric fever- Refer chapter review.
44. Ans. (b), (c) (Water can transmit the disease, Ty2 1 a is an oral vaccine)
Ref: Ananthanarayan 9/e p295–96, 8/e p298–299, Harrison 17/p958 & 18/e p1278
  • ‘All Salmonella infections begin with ingestion of organisms in contaminated food or water’
Ty2 1a vaccine:
  • It lacks the enzyme UDP galactose epimerase. So, on ingestion, it initiates infection but self- destructs stop multiplying after 4–5 cycle.
  • Vaccine is available as enteric coated tablets 3 doses given orally on day 1, 3, 5.
About other options:
  • Salmonella food poisoning can be caused by any spp other than S.Typhi
  • Chronic carrier is called when transmitted up to 1 year (HBV chronic carrier is up to 6months)
  • Blood culture is usually done in 1st week. Widal test becomes +ve at the end of 1week.
45. Ans. (c), (d) (Caused by animal products, Symptoms appear by 4–48 hour) Ref: Harrison 17/ e p 957 & 18/e p 1278–79
  • Option c- ‘Non typhoidal Salmonella causing gastroenteritis is most commonly associated with animal food products, especially eggs, poultry, undercooked ground meat’
  • Option d- ‘Nausea, vomiting, and diarrhea occur 6–48 hour after the ingestion of contaminated food or water.’
About Other Options:
  • Option a- ‘The diagnosis of NTS infection is based on the isolation of the organism. Serology is not useful.’
  • Option b- ‘Diarrheal stools are usually loose, non bloody, and of moderate volume. However, large-volume watery stools, bloody stools, or symptoms of dysentery may occur’.
  • Since bloody diarrhea is unusual feature, so it should not be taken into account.
  • Option e- Pathogenesis is due to invasion, endotoxin and Pathogenicity Island. But it doesn't produce any Exotoxin.
46. Ans. (a) (Affects Peyer's patches) Ref: Harrison 17/e p 956 & 18/p1274
  • In small intestine, Salmonella penetrate through phagocytic microfold (M) cells within Peyer's patches.
  • Peyer's patches are present in the anti mesenteric border
  • Erythrophagocytosis, stricture and GI hemorrhage (Typhoid ulcer bleeding) are not common.
21247. Ans. (b) (Typhoid) Ref: Farlex's Free Online Medical dictionary
  • “Pea soup stool is used for the khaki-green, slimy stools typically occurs in the 3rd week of typhoid fever, at which point the Patients are in a ‘toxic state’ and at greatest risk for intestinal perforation and hemorrhage; Similar stools occur in enteropathic E coli infections of infants”
48. Ans. (d) > (c) (Typhoid bacilli in stool > Detection of Vi antibody) Ref: Ananthanarayan 9/e p295–96, 8/e p298–299
Typhoid carrier is diagnosed by-
  • Detection of Vi antibody
  • Culture in stool and urine (Best method)
49. Ans. (a) (Ampicillin) Ref: Park 22/p216
DOC for typhoid carrier- Ampicillin or Amoxycilin plus probenecid for 6 weeks.
50. Ans. (b) (Male carriers are less common but more dangerous) Ref: Park 22/e p213–16
  • Male carriers are less common but more dangerous
  • MC site of for typhoid carrier - Gall bladder and billiary tract and also urinary tract
  • DOC for typhoid carrier - Ampicillin or Amoxycilin plus probenecid for 6 weeks.
51. Ans. (b) (Stool culture negative till three times) Ref: Park 22/p215
  • In enteric fever, isolation is done till -Stool culture negative till three times.
52. Ans. (a) (1,3,5) Ref: Ananthanarayan 9/e p295–96,8/e p298–299, Harrison 17/p 958 & 18th/p1278
  • Typhoral schedule: 3 doses on alt day (1/3/5) in capsules, Effective after 7 days, Booster needed @5yr.
53. Ans. (a),(b),(d),(e) (S.Typhimurium, S.Typhi, S.Paratyphi B, S.Cholerasuis) Koneman's Diagnostic Microbiology 6th/p242
H2S forming Salmonella:
  • Salmonella Paratyphi -A : Absent H2S
  • Salmonella Paratyphi –B : Big H2S (Produce large quantity of H2S)
  • Salmonella Typhi &S.Cholerasuis- Tiny H2S (i.e small amount of H2S)
 
YERSINSIA
54. Ans. (b) (Caused by Y.pestis) Ref: Ananthanarayan 9/e p324, 8/e p325
  • The term Yersiniosis denotes infection with Yersiniae other than Y.pestis. These include zoonotic infections by Y.pseudotuberculosis and Y.enterocolitica.
55. Ans. (b) (Wayson stain) Ref: Ananthanarayan 9/e p323
This is a case of bubonic plague
Points in favor:
  • Farmer – High risk of exposure to rodents
  • Presented with high fever, painful inguinal lymphadenopathy,
    So, now we have to use a stain that will stain the causative agent Y.pestis.
  • ‘When stained with a polychromatic stain (e.g., Wayson or Giemsa), Y. pestis isolated from clinical specimens exhibits a characteristic bipolar appearance, resembling closed safety pins’.
Other stains given in the question:
  • Neisser stain and Albert's stain- Used to demonstrate metachromatic granules
  • Mcfadyean's stain- Used to demonstrate capsule of B.anthracis.
56. Ans. (a) (Animal bites or scratches) Ref: Ananthanarayan 9/e p325, 8/e p326
  • Pasteurella multocida causes zoonotic infection affecting birds and various animal.
  • Human infection is rare but occurs following animal bite or trauma.
  • Clinically manifested as local suppuration following animal bite (wound infection, cellulitis, abscess, Osteomyelitis), meningitis and respiratory infection.
  • Characterized by –Oxidase positive, Fails to grow on MacConkey, Indole positive
21357. Ans. (a) (Yersinia pestis) Ref: Ananthanarayan 9/321–22, 8/e p321
Points in favor:
Girl from Endemic area of plague (Shimla) presenting with lymphadenopathy. Culture showing stalactite growth
58. Ans. (d) (Wayson staining) Ref: Ananthanarayan 9/e p323, Harrison 18/p1308, 17/e p980
Refer previous explanation.
59. Ans. (b), (d), (e) (Gram-negative, Nonmotile, Coccobacilli)
Ref: Ananthanarayan 9/e p320, Harrison 18/p1305, 17/e p980
Yersinia pestis is:
  • Gram-negative
  • Coccobacilli
  • Belong to family Enterobacteriaceae
  • Microaerophilic,
  • Nonmotile
  • Biochemically unreactive.
60. Ans. (b) (Bubonic is the most common variety) Ref: Park 22/e p 267–68, 20/e p256, 57
This is a type of dangerous question that has to be attempted very carefully.
“All are wrong except – means which is a right statement”
  • Among the 3 variety, Bubonic plague is the MC variety where as Pneumonic plague is the most fatal and infectious variety.
About Other Options:
  • Option a: ‘Wild rodents (field mice, gerbil, skunks and other small animal) are the natural reservoir
  • In India, wild rodents- Tatera indica is the main reservoir’ --- Park 22/e p267, 68
  • Option c: Rat flea can survive in the rat burrow for as long as 4 years is the most likely the main mechanism of maintaining the Plague Bacilli in the nature. Park 22/e p267, 68
  • (10 year is Too Long… So it should be a wrong statement)
  • Option d: Incubation Period - Bubonic and septicemic (2–7day), Pneumonic Plague (1–3day).
61. Ans. (b), (d), (e) (Non – motile, Patients are kept isolated till 48 hours of treatments, Repeated blood culture is diagnostic)
Ref: Ananthanarayan 9/e p323–24, Harrison 18/e p1305–09, 17/e p 983, 84
  • Option a, b: Y.pestis is Non Motile Gram negative Coccobacilli
  • Option c: Treatment- Streptomycin (DOC) …………Harrison 17/e p984
  • Others- Gentamicin, Tetracycline, Doxycycline and Chloramphenicol
  • Option d:
  • Patients in whom respiratory plague is suspected should be managed under isolation, until pneumonia has been ruled out
    or until 48 hour of antimicrobial therapy ……Harrison 17/e p984
  • Option e:
    Cultures of three blood samples taken over a 45-min period before treatment usually results in isolation of the bacterium. Y. pestis’ …………Harrison 17/e p 983.
62. Ans. (d) (Tetracycline) Ref: Ananthanarayan 9/e p324, Harrison 18/e p1309, 10, 17/e p984
DOC for chemoprophylaxis of plague- Tetracycline
(Others- Doxy, Cotrimoxazole, Ciprofloxacin)
DOC for treatment of plague – Streptomycin
(Others- Gentamicin, Tetracycline, Doxycycline and Chloramphenicol)
63. Ans. (a) (Flea control) Ref: Park 22/270
  • Most effective way to break transmission chain plague – Rat flea control
64. Ans. (c) (28 years) Ref: Park 22/270
  • Plague epidemic in Surat in 1994 had occurred after 28 years of last outbreak that had occurred in Karnataka in 1966.
21465. Ans. (b) (Cheopis index) Ref: Park 22/269
  • Cheopis index- Average no. of X.cheopis per rat
  • It is the most efficient indicator to measure the explosiveness of plague.
66. Ans. (b) (Pneumonic plague) Ref: Park 22/269
  • MC form of plague- Bubonic plague
  • Highly infectious and highly fatal- Pneumonic plague.
67. Ans. (c) (Rat flea) Ref: Park 22/267
  • Plague is transmitted by bite of an infected Rat flea such as X.cheopis.
68. Ans. (a) (Case of bubonic plague) Ref: Ananthnarayan 9th /p320–22
  • MC source for bubonic plague is- Infected rat fleas followed by Infected rodents
  • Source for pneumonic plague is- Case of pneumonic plague

VibrioCHAPTER 3.8

 
GENERAL PROPERTIES
  • Gram negative curved rods (comma shaped)
  • Fish in stream appearance- in gram stain
  • Oxidase positive, Non/late lactose fermenter
  • Indole +ve and Nitrate+ve (Cholera red reaction)
  • String test+ve with Na deoxycholate
  • Motility- Darting or shooting star motility or swarm of gnats – Also shown by Campylobacter
  • All are halophilic except V.cholerae, V.mimicus
:
 
CULTURE MEDIA (ALKALINE PH)
Transport Media
  • Venkatraman Ramakrishnan (VR) medium
  • Cary Blair medium
  • Autoclaved sea water
Enrichment:
  • APW (alkaline peptone water)
  • Mansur's taurocholate tellurite peptone water
Selective:
  • Bile salt agar (BSA)
  • Mansour's gelatin taurocholate trypticase tellurite agar (GTTA)
  • Thiosulphate citrate bile sucrose (TCBS) agar
    • Yellow colony (V.cholerae), Green colony (V parahemolyticus)216
Biotypes of V.cholerae O1
Classical strain
El Tor
El Tor Variant
El Tor Hybrid
Sheep RBC Hemolysis
-ve
+ve
-ve
+/-
Chick erythrocyte agglutination
-
+ve
+ve
+/-
Polymyxin B (50 iu)
Sensitive
Resistant
Resistant
Sensitive/Resistant
Group IV phage susceptibility
Susceptible
Resistant
Resistant
Susceptible/Resistant
ElTor Phage V susceptibility
Resistant
Susceptible
Susceptible
Susceptible/Resistant
VP test
-ve
+ve
+ve
+/-
CAMP test
-ve
+ve
+ve
+/-
Cholera toxin (CT) type
CT-1
CT-2
CT-1
CT 1 or 2
 
Epidemiology of Cholera
  • 1st 6 pandemics- started in Indian subcontinent and mediated by classical Vibrio
  • 7th pandemic- started in Indonesia, due to ElT or Vibrio, characterized by less severe illness and more carriers
  • ElTor Vs Classical Vibrio-
    • ElTor is much hardier/resistant than classical,
    • ElTor produces less severe cholera,
    • ElTor infection is associated with more carrier than cases
  • MC Serotype - Ogawa >Inaba> Hikojima (agglutinated by both antisera)
 
Factors Contributing to Pathogenicity
  • Occurs only in man (rice water stools)
  • No known animal reservoir
  • Acid labile (alkali stable), so require high infective dose of 106 organisms is required to bypass the gastric barrier
  • Heat labile, but can resist refrigeration
  • Blood group O-High risk, AB- lowest risk
  • Maintained-
    • Inter epidemics- in sea water
    • During epidemic- In carriers
  • Age-
    • Inter epidemics – all age equally
    • During epidemic- More children
 
Pathogenicity of Cholera
  • TCP- toxin co-regulatory pilus- adhesion
  • Cholera Toxin (CT)- resembles heat labile toxin of E.coli (LT)
  • LPS- has no role in pathogenesis
  • Mechanism of Cholera toxin-
    • A unit – ADP ribosylation of GTP binding protein→Adenyl cyclase →cAMP,
    • B unit – Binds to ganglioside receptors
 
Clinical Features of Cholera
  • Sudden onset of painless voluminous watery secretory diarrhea
  • The stool is- non bilious, gray, slightly cloudy fluid with flecks of mucus, no blood, and sweet, inoffensive odor («rice-water» stool)
  • Non invasive diarrhea (So no pus cells in stool)
  • Vomiting but Fever is usually absent.
  • Muscle cramps due to electrolyte disturbances
  • 217Postural hypotension, tachycardia
  • Weakness, Decreased skin turgor
  • Severe case:
    • Oliguria, Weak pulses,
    • Sunken eyes wrinkled (“washerwoman”)
    • Hypovolemic shock and coma.
 
Biochemical Parameter
  • Mild neutrophilic Leucocytosis
  • Elevated levels of blood urea nitrogen and creatinine
  • Reduced bicarbonate level (<15 mmol/L);
  • Elevated anion gap
  • Low Arterial pH
 
O139 Vibrio- (Bengal Strains) - 1st in 1992 from Chennai
  • Capsulated, might be invasive
  • Arise from ElTor by horizontal transfer
  • Clinically and epidemiologically indistinguishable from O1
  • Not neutralized by O1 antisera
  • Currently – both O1 ElTor and O139 coexist. Classical Vibrio still reported in Bang- ladesh.
 
Treatment of Cholera
  • DOC in adult- Doxycycline (if resistant, then ciprofloxacin)
  • DOC in children- cotrimoxazole
  • DOC in pregnancy- Furazolidone
  • DOC for chemoprophylaxis- Tetracycline
 
Vaccines
  • Parenteral vaccine:
    • Contains- Classical (6000 million of Ogawa & Inaba)
    • Efficacy- 50%, lasts for 3–6 months, efficacy is better in adult > children
    • No cross protection b/t ogawa & inaba
    • Cross protection b/t ElTor & Classical
  • Oral vaccine-
    • WC/rBS- 2 dose, 50–60% protection for 3 year,
    • CVD 103-HgR- 1dose, 80% protection
 
Non-O1/O139 (NAG-Non Agglutinable V.cholerae)
  • Biochemically similar to V.cholerae but do not agglutinate in O1 or O139 antiserum
  • Cause gastroenteritis, sporadic cases of otitis media, wound infection, and bacteremia
  • Do not cause epidemics of cholera
  • Widely distributed in marine environments (raw oysters)
 
HALOPHILIC VIBRIO
  • ↑NaCl tolerance-
    • V. parahemolyticus (7%), V. alginolyticus (10%), V. vulnificus (8%)
V. parahemolyticus:
  • Shows kanagawa phenomenon on wagatsuma agar
  • Capsulated, bipolar staining, Peritrichous flagella, green colony on TCBS (sucrose non fermenter)
  • Causes Gastroenteritis (watery diarrhea > dysentery) and Wound infection (Shell- fish, seawater),
218V. vulnificus:
  • Lactose fermenter
  • Causes-Wound infection, secondary cellulitis, gastroenteritis (sea water exposure)
  • Also causes- Sepsis (in low immunity and underlying liver disease like cirrhosis or hemochromatosis) (Shellfish)
V. alginolyticus:
  • Most salt-tolerant (>10%)
  • Occasionally causes eye(conjunctivitis), ear (otitis media), and wound infections
  • Also causes Sepsis – in patients with burned, other immunosuppressed condition
  • Seawater
  • Tetracycline- DOC
V.metschnikovii- Oxidase negative
219MULTIPLE CHOICE QUESTIONS
1. Which of the following statement(s) is/are wrong about the 8th pandemic of cholera? (PGI Nov 2014)
  1. Caused by Vibrio cholerae O139
  2. Started in1992
  3. Started in Bangladesh
  4. Mostly confined to Bangladesh & India
  5. Capsulated
2. True about mechanism of bacterial toxins: (PGI May 2013)
  1. Cholera toxin acts by inhibition of guanylcyclase
  2. Botulinum toxin inhibits Ach release
  3. Shiga toxin of Shigelladysenteriae act by inhibiting protein synthesis
  4. Diphtheria toxin act by inhibiting protein synthesis
3. Invasive infections caused by all except: (NEET Pattern Based)
  1. Shigella
  2. Salmonella
  3. V. cholera
  4. Yersinia
4. True about cholera: (NEET Pattern Based)
  1. Gram negative rod
  2. Associated with fever
  3. Causes painful watery diarrhea
  4. It is an achlorhydria which renders an individual susceptible to disease
5. The endotoxin of the following gram negative bacteria doesn't play any part in the pathogenesis of the natural disease: (AI 2012, AIIMS 2006, AIIMS Nov 2012)
  1. E. coli
  2. Thlebsiella
  3. Vibrio cholerae
  4. Pseudomonas
6. Tetracycline is used in prophylaxis of: (AI 2011)
  1. Shigellosis
  2. Cholera
  3. Brucellosis
  4. Leptospirosis
7. Not true about ElTor Vibrio 01: (AI 2010)
  1. Animals are the only reservoir
  2. Epidemiologically indistinguishable from V. cholerae 0139
  3. Human acts as vehicle for spread
  4. The efficacy of vaccine against E1tor Vibrio is great
8. Cholera is caused by: (DNB June 2009, PGI June 2009)
  1. Vibrio cholerae – 01
  2. Vibrio cholerae – 0139
  3. Vibrio parahemolyticus
  4. NAG Vibrio
9. True about Vibrio cholerae is: (SGPGI 2009)
  1. Disease more common in woman
  2. Classical Vibrio protect against development of O-139
  3. E1 – Tor is milder than classical
  4. Erythromycin is used in treatment
10. Darting motility which occur in V.cholerae, also found in: (PGI Dec 2008)
  1. Shigella
  2. Campylobacter jejuni
  3. Pneumococcus
  4. Bacillus anthrax
  5. Aeromonas
11. Selective media for Vibrio: (AIIMS May 2008, Nov 2007)
  1. TCBS
  2. Stuart
  3. Skirrow's
  4. MYPA
12. Which of the following bacteria act by increasing c-AMP: (AI 07, 2012, AIIMS Nov 2006)
  1. Vibrio cholerae
  2. Staphylococcus aureus
  3. E.coli, heat stable toxin
  4. Salmonella
13. Which is not true about Vibrio cholera: (AI 2007)
  1. It is non halophilic
  2. Grows on simple media
  3. Man is the only natural host
  4. Cannot survive in extracellular environment
14. True about V cholera 0139 all except: (AI 2007, 2008)
  1. Clinical manifestations are similar 01 ElTor strain
  2. First discovered in Chennai
  3. Epidemiologically indistinguishable from 01 ElTor strain
  4. Produces O1 Lipopolysaccharide
15. Cholera transmission by: (PGI Dec 2006)
  1. Food
  2. Vaccination gives 90% efficiency
  3. Healthy carrier
  4. Chlorination is not effective
16. V. cholerae is able to stay in GIT because of: (PGI Dec 2006)
  1. Acid resistance
  2. Bile resistance
  3. Motility
  4. Binds to specific receptors
  5. Anaerobic potential
17. Cholera toxin : (AI 2005, AIIMS May 2006)
  1. Increases the levels of intracellular cGMP
  2. Acts through the receptor the opiates
  3. Causes continued activation of Adenylate cyclase
  4. Inhibits the enzyme phosphodiesterase
18. Antibiotic treatment of choice for cholera in an adult is a single dose of: (AI 2005)
  1. Tetracycline
  2. Cotrimoxazole
  3. Doxycycline
  4. Furazolidone
19. 220The drug of choice for treating cholera in pregnant women is: (AIIMS Nov 2005)
  1. Tetracycline
  2. Doxycycline
  3. Furazolidone
  4. Cotrimoxazole
20. Which of the following is the drug of choice of chemopro- phylaxis of cholera: (AI 2011, TNPG 2014) (AIIMS Nov 2005, UP 2004)
  1. Tetracycline
  2. Doxycycline
  3. Furazolidone
  4. Cotrimoxazole
21. DOC of cholera in children: (AIIMS Nov 2005)
  1. Tetracycline
  2. Doxycycline
  3. Furazolidone
  4. Cotrimoxazole
22. For cholera control: all recommended except: (TNPG 2014, AI 2011, AIIMS May 2005, AIIMS Nov 2005, UP 2004)
  1. Mass chemoprophylaxis
  2. Early case detection
  3. Chlorination of water
  4. Proper disposal of excreta
23. A convalescent case of cholera remains infective for: (DPG 05)
  1. <7 days
  2. 7–14 days
  3. 14–21 days
  4. 21–28 days
24. Best disinfectant for cholera stool? (MP 2000)
  1. Bleaching powder
  2. Cresol
  3. Formalin
  4. Alcohol
25. True about epidemiology of cholera: (PGI 2003)
  1. Chemoprophylaxis is not effective
  2. Boiling of water cannot destroy the organism
  3. Food can transmit the disease
  4. Vaccination give 90% protection
26. Which of the following is true of cholera: (PGI 2002)
  1. Recent epidemic was due to classical type
  2. Causes secretory diarrhea
  3. Caused by endotoxin
  4. Vibriocidal antibodies correspond to susceptibility
27. True about V.cholerae is: (AIIMS May 2002)
  1. One attack of V. Cholerae gives live-long immunity
  2. Affects adults and children with equal propensity in non epidemic regions.
  3. In between epidemics, carrier states maintain the organism
  4. Pathogenicity of 0–139 Vibrio is due to O antigen
28. In a patient presenting with diarrhea due to Vibrio cholerae, which of the following will be present: (PGI Dec 2001)
  1. Abdominal pain
  2. Presence of leukocytes in stool
  3. Fever
  4. Neutrophilia
29. Red leg disease is caused by: (PGI 2001)
  1. Pseudomonas
  2. Mouldy sugar cane fiber
  3. Conidiosporium
  4. Aeromonas
30. A stool examination was carried out which showed organism with darting motility. Which of the following organism may be present in stool: (PGI May 2013)
  1. V.cholerae
  2. Shigella
  3. Salmonella
  4. Camplyobacterjejuni
  5. E.coli
31. Type of diarrhoea in Vibrio cholerae: (Recent Question 2013)
  1. Osmotic
  2. Secretory
  3. Colloidal
  4. Bloody
32. The best way to classify organism associated with epidemics of watery diarrhoea in Southeast Asia? (Recent Question 2013)
  1. Phage typing
  2. Biotyping
  3. Colicin typing
  4. Antibiogram typing
 
HALOPHILIC VIBRIO
33. A Gram-negative marine bacterium that causes ear infection (AI 2012)
  1. Vibrio cholera
  2. Vibrio parahemolyticus
  3. Vibrio alginolyticus
  4. Vibrio mimicus
34. Vibrio parahemolyticus is seen in undercooked: (DNB Dec 2009)
  1. Crab
  2. Shell fish
  3. Prawn
  4. Fish
35. All of the following Vibrio sp. are halophilic, except:(AI 2005)
  1. V. cholerae
  2. V. parahaemolyticus
  3. V. alginolyticus
  4. V. fluvialis
36. 32 year old male, Thallu, who recently visited a sea coast presented with ulcer over left leg. The probable cause is: (AIIMS May 2001)
  1. Pasteurella multocida
  2. Micrococcus halophillus
  3. Vibrio vulnificus
  4. Neisseria gonorrhea
37. True about Vibrio alginolyticus: (PGI May 2013)
  1. Non-halophilic
  2. VogesProskauer (VP) positive
  3. Swarming
  4. Causes sea borne auricular infections
  5. Does not grow in 10% NaCI
221EXPLANATIONS
1. Ans. (c) (Started in Bangladesh) Ref: Ananthnarayan 9th/p308
O139 (The Bengal strain) was isolated first from Chennai in 1992 and spread rapidly along the costal region of way of Bengal up to West Bengal, then to the adjacent areas of Bangladesh.
  • O139 appears to be a derivative of O1 El Tor but differs from the later having a distinct LPS and being capsulated. Thus it is invasive and can cause bacteraemia and extraintestinal manifestations also.
  • There is no cross protection between O1 and O139.
  • It had caused large-scale outbreaks of clinical cholera and spread rapidly across almost 11 Asian countries and became a threat to cause the next 8th pandemic.
  • However, by 1994 the fear had come down and once again the O1 El Tor became dominant & largely replaced O139.
  • Currently, O139 still causes a minority of cases in India and Bangladesh.
2. Ans. (b) (c) (d) (Botulinum…,Shiga toxin….,Diphtheria toxin…..) Ref: Ananthnarayan 9th /p307,286,264,237
  • Cholera toxin acts by activating cAMP
  • Botulinum toxin inhibits Acetyl choline release at NM junction→ flaccid paralysis
  • Shiga toxin of Shigella dysenteriae acts by inhibiting protein synthesis by inhibiting ribosome
  • Diphtheria toxin act by inhibiting protein synthesis by inhibiting elongation factor-2
3. Ans. (c) (Vibrio cholerae) Ref: Ananthanarayan 9/e p308, 8/e p307
  • V.cholerae is non invasive. The pathogenesis of V.cholerae is due to exotoxin.
4. Ans. (a) (Gram negative rod) Ref: Ananthanarayan 9/e p308, 8/e p307
  • V.cholerae is comma shaped gram negative rod.
  • Cholera is characterized by painless watery diarrhea without fever.
  • Due to high infective dose, V.cholerae can cross the gastric barrier. However, HCl secretion is normal and there is no achlorohydria.
5. Ans. (c) (Vibrio cholerae) Ref: Ananthanarayan 9/e p308, 8/e p307, Harrison 18/e p1291
  • Cholera Vibrios also possess lipopolysaccharide O antigen (LPS, endotoxin) as in other gram negative intestinal bacilli. But unlike other gram negative intestinal bacilli like E.coli, Klebsiella and Pseudomonas etc. (where endotoxin is the principle virulence factor), LPS apparently plays no role in pathogenesis of cholera but is responsible for the immunity induced by killed vaccine.
  • For Vibrio cholerae the principle virulence factor is the exotoxin i.e. cholera toxin.
6. Ans. (b) (Cholera) Ref: Harrison 17/p970 & 18/e p1293
  • DOC for che.moprophylaxis for cholera- Tetracycline
7. Ans. (a) > (d) (Animals are the only reservoir, The efficacy of vaccine against E1 tor Vibrio is great)
Ref: Harrison 17/p968–71 & 18/e p1289–93
  • Option a- ‘The natural habitat of V. cholerae is coastal salt water and brackish estuaries, where the organism lives in close relation to plankton. There is no known animal reservoir’ ….Harrison 17th /e p968
  • Option d- Classical and ElTor are biotypes, i.e. they are biochemically different but antigencically same. Hence, the efficacy of both Classical and ElTor should be same.
  • Option b- ‘The clinical manifestations and epidemiologic features of the disease caused by V. cholerae O139 Bengal are indistinguishable from those of O1 cholera’. ….Harrison 17th /e p969
  • Option c- ‘Humans become infected incidentally but, once infected, can act as vehicles for spread.’ Harrison 17th /e p968
8. Ans. (a), (b), (d) (Vibrio cholerae – 01,Vibrio cholerae – 0139, NAG Vibrio)
Ref: Jawetz 24/p271 & 25/p235, Harrison 17 /p968–71 & 18/e p1289–93
  • Epidemic and pandemic Cholera is caused by both Vibrio cholerae O1 and O139 strains.
  • 222However, Cholera like illness can also be caused by NAG (Non agglutinable Vibrio) i.e V cholerae serogroups non-O1/non-O139.
Organism
Human Disease ……Jawetz 24th /271
V.cholerae serogroups O1 and O139
Epidemic and pandemic cholera
V.cholerae serogroups non-O1/non-O139
Cholera-like diarrhea; mild diarrhea; rarely, extra intestinal infection
9. Ans. (c) (El – Tor is milder than classical) Ref: Park 22/p209–10, 20 /p202–207 & 21/p208
  • ‘ElTor is much hardier and resistant than classical, But ElTor produces less severe cholera,
  • Hence, ElTor infection leads to more carrier than cases’
About Other Options-
  • Classical Vibrio belongs to serovar O1, which is antigenically different from Serovar O139, it cannot protect against it.
  • But Classical Vibrio protect against development of ElTor.
  • Cholera Vibrio affects all ages and sexes but in epidemic area it affects more children. Park 22/p209
10. Ans. (b) (Campylobacter jejuni) Ref: Ananthanarayan 9/e p399, 8/e p398
  • Campylobacter jejuni possess single polar flagella and, they exhibit darting motility appreciated under dark field or phase contrast microscopy.
11. Ans. (a) (TCBS) Ref: Ananthanarayan 9/e p304, 8/e p303
  • TCBS (Thiosulfate Citrate Bile Sucrose Agar) is a Selective media for Vibrio.
  • V. Cholerae can ferment sucrose and it produces Yellow colonies on TCBS.
  • Whereas V.parahemolyticus produces green colonies on TCBS
Culture medias for Vibrio: Refer text (chapter review).
12. Ans. (a) (Vibrio cholerae) Ref: Ananthanarayan 9/e p308,8/e p307
  • Mechanism of action of cholera toxin – refer text (chapter review).
  • Toxin acts by cAMP are : LT of E.coli, Cholera toxin, Pertussis toxin, Anthrax toxin (edema factor)
13. Ans. (d) (Cannot survive in extracellular environment)
Ref: Park 22/p208–09 & 21/p207, Ananthanarayan 9/304,8/e p303
  • V.cholerae is not an intracellular organism.
  • It adheres to the intestinal epithelium and acts by producing toxin.
About Other Options
  • V.cholerae is non halophilic. It cannot withstand salt concentration of >6%.
  • However 0.5%-1% salt is optimally required for its growth.
  • Man is the only known reservoir. There is no animal reservoir. However bacilli can survive coastal salt water and brackish estuaries.
  • It grows on simple media like Nutrient agar.
14. Ans. (d) (Produces 01 Lipopolysaccharide) Ref: Ananthanarayan 9/e p308, 8/e p308
  • Serovar O1 and O139 are antigenically different So, they cannot be neutralized by each other's antisera. Detail of O139 Vibrio – Refer text (chapter review)
15. Ans. (a), (c) (Food, Healthy carrier) Ref: 22/p208–09,205 & 21/p207–10
  • Cholera is transmitted by-Contaminated water & food, Direct contact through finger from Carriers
Carriers -
  • Healthy carrier - has subclinical infection. Usually it sheds the bacilli for <10 days. Though it is not that important but still cannot be ignored as a source of infection.
  • Incubatory carrier- shed for 1–5 week during incubatory period.
  • Convalescent carrier – shed for 2–3 weeks
  • Chronic carrier- months to year (longest recorded is 10 year)
223About Other Options-
Option b- Vaccination gives protection rate of 50–60% ……………….Ananthanarayan 9/e p311
Option d- Chlorination is effective. ‘In rural area, water can be made safe from cholera bacilli by boiling or chlorination’ Park 22nd /p209.
16. Ans. (b), (c), (d) (Bile resistance, Motility, Binds to specific receptors) Ref: Ananthanarayan 9/e p305, 8/e p304
  • V.cholerae is acid labile that is why high infective dose of 106 bacilli is required to cross the gastric juice barrier. (and achlorohydria facilitates transmission)
  • V.cholerae is bile resistant, hence can survive in the intestine. (bile resistant property helps to grow on TCBS -Thiosulfate Citrate Bile Sucrose Agar)
  • V.cholerae is strongly aerobic but still grows slowly and scanty anaerobically.
Other reasons include:
  • Factors that help to cross the protective mucus layer in small intestine are - Motility, chemotaxis, mucinase, and other proteolytic enzymes.
  • Other factors- Siderophore, neuraminidase, accessory colonizing factor
  • Adhere to epithelium by – TCP (Toxin Co-regulatory Pilus)
  • Cholera Toxin - Principle virulence factor- binds to specific GM1 receptor on intestinal epithelium.
17. Ans. (c) (Causes continued activation of Adenylate cyclase) Ref: Ananthanarayan 8/e p307 Cholera Toxin (CT): causes ADP ribosylation of G protein→Adenyl cyclase →cAMP,
18. Ans. (c) (Doxycycline) Ref: Harrison 17/e p970 & 18/e p1293
DOC in adult- Doxycycline (if resistant, then next DOC is ciprofloxacin)
19. Ans. (c) (Furazolidone) Ref: Harrison 17/e p970 & 18/e p1293
  • DOC in pregnancy- Furazolidone
20. Ans. (a) (Tetracycline) Ref: Harrison 17/e p970 & 18/e p1293
  • DOC for chemoprophylaxis- Tetracycline
21. Ans. (d) (Cotrimoxazole) Ref: Harrison 17/e p970 & 18/e p1293
  • DOC of cholera in children -Cotrimoxazole
22. Ans. (a) Mass chemoprophylaxis Ref: Park 22/p211
  • Mass chemoprophylaxis is not recommended for the control of cholera
  • Measures for cholera control includes-
    • ✓ Early case detection
    • ✓ Water control by Chlorination of water
    • ✓ Proper disposal of excreta
23. Ans. (c) (14–21 days) Ref: Park 22/e p209
  • Preclinical (incubatory carrier)- Excretes for 1–5days
  • Convalescent carrier- Excretes for 2–3weeks
  • Contact/healthy carrier- Excretes for <10days
  • Chronic carrier- Excretes for up to 10 years
24. Ans. (b) Cresol Ref: Park 22/e p211
  • The most effective disinfectant for cholera stool is a coal tar disinfectant with Rideal walker coefficient of >10 such as cresol.
25. Ans. (c) (Food can transmit the disease) Ref: Park 22nd /p208–09 & 21st/p207
  • Cholera is transmitted by Contaminated food like bottle feeding, vegetable and fruits which is more commonly seen in ElTor Vibrio
About Other options:
  • Vibrio is susceptible to heat but can resists cold and survives unto 1 week in refrigerator.
  • Chemoprophylaxis (Tetracycline) is affective and indicated in epidemics
  • Vaccination gives protection rate of 50–60%….Ananthanarayan 9/e p311, Park 22nd /p212& 21st/p207
22426. Ans. (b) (Causes secretory diarrhea) Ref: Harrison 17/e p970 & 18/e p1289–93, Jawetz 25/e p237
  • Vibrio causes secretory diarrhea.
About Other options:
  • Recent epidemics are due to O1 ElTor and also O139 serovar. (Classical still exists and few cases were reported from Bangladesh)
  • Vibriocidal antibodies in serum (titer 1:20) have been associated with protection against colonization and disease. ……………Jawetz 25/e p237
  • Endotoxin (LPS) doesn't play any role in pathogenicity of cholera but is responsible for immunity developed by killed vaccine
27. Ans. (b) (Affects adults and children with equal propensity in non epidemic regions)
Ref: Harrison 17/e p968 & 18/e p1290
“Cholera is predominantly a pediatric disease in endemic areas, but it affects adults and children equally when newly introduced into a population.” ……………….Harrison 17/e p968
About Other options-
  • Vaccination gives protection rate of 50–60% for 3–6 months …………Ananthanarayan 9/e p311
  • In between epidemics, it is maintained in sea water. During the epidemics, carrier states maintain the organism
  • Pathogenicity of 0–139 Vibrio is due to cholera toxin and capsule.
  • LPS has no role in pathogenesis of O139, however since it is different from LPS of O1, so it is not neutralized by antisera to O1. …………..Harrison18/e p1290
28. Ans. (d) (Neutrophilia) Ref: Harrison 17/e p970 & 18/e p1291
Clinical feature of cholera (detail- refer chapter review)-
  • Sudden onset of painless voluminous watery diarrhea
  • Fever is usually absent.
  • Mild neutrophilic Leucocytosis
  • Non invasive diarrhea (So no pus cells in stool)
29. Ans. (d) (Aeromonas) Ref: Wikipedia, Ananthanarayan 9/e p312, 8/e p313
  • Red Leg Disease is a severe, acute, bacterial infection of amphibians.
  • It is termed «Red Leg Disease» because it causes hemorrhages of the leg (often the inner thigh) as a result of septicemia.
  • Aeromonas is the most common bacteria involved,
30. Ans. (a) (d) (V.cholerae, Camplyobacter jejuni) Ref: Ananthnarayan 9th /p304,399
Darting or shooting star motility is exhibited by V.cholerae and Camplyobacter jejuni.
31. Ans. (b) (Secretory) Ref: Ananthanarayan 9/e p307
  • Cholera stool is painless voluminous watery secretory diarrhoea with non bilious, gray, slightly cloudy fluid with flecks of mucus, no blood, and sweet, inoffensive, non invasive diarrhoea (So no pus cells in stool).
32. Ans. (b) (Biotyping) Ref: Ananthanarayan 9/e p306
  • Organism associated with epidemics of watery diarrhoea in Southeast Asia- suggestive of V.cholerae.
  • V.cholerae is commonly typed by serotype (O1 to O139) and O1 is further biotypes to classical and Eltor.
 
Halophilic Vibrio
33. Ans. (c) (Vibrio alginolyticus) Ref: Ananthanarayan 9/e p311–12, 8/e p313, Harrison 18/e p1295
  • V. alginolyticus occasionally causes eye, ear, and wound infections. …….. Harrison 18/e p1295
  • V. alginolyticus is associated with infection of ear, eye and wounds in human beings exposed to sea water. ………………….. Ananthanarayan 9/e p312
34. Ans. (b) (Shell fish) Ref: Ananthanarayan 9/e p311, 8/e p313, Harrison 18/e p1295
  • Refer chapter review
22535. Ans. (a) (V. cholerae) Ref: Ananthanarayan 9/e p311,8/e p312
Examples of Halophilic Vibrio –V. alginolyticus, V. parahaemolyticus,V.vulnificus & V. fluvialis Examples of Non Halophilic Vibrio – V.cholerae and V. mimicus
36. Ans. (c) (V. vulnificus) Ref: Ananthanarayan 9/e p312, 8/e p312
Points in Favor- Person belongs to Sea coast presented with ulcer over the left leg
“Halophilic Vibrio usually cause infection in sea coastal region following ingestion of seafood like Oysters etc.”
37. Ans. (b) (d) (VogesProskauer (VP) positive, Causes sea borne auricular infections) Ref: Ananthnarayan 9th /p313
  • V.alginolyticus can occasionally cause eye, ear, and wound infections following sea water exposure.
  • Few cases of otitis externa, otitis media, and conjunctivitis have been reported.
  • Halophilic - V.alginolyticus is the most salt-tolerant vibrio and can grow in salt concentrations of >10%.
  • VogesProskauer (VP) test differentiates V.parahaemolyticus (positive) from V.alginolyticus (negative)
  • V.parahaemolyticus can swarm on blood agar, but not V.alginolyticus
    (In Ananthnarayan, it is wrongly mentioned as V.alginolyticus can swarm. Topley Wilson clearly states that swarming is exhibited by Proteus, V.parahaemolyticus, Serratia marcescens and sometime by Pseudomonas aeruginosa.

Pseudomonas & Other Non Fermenters and Haemophilus, Bordetella, Brucella (HBB)CHAPTER 3.9

 
NON FERMENTER
 
Pseudomonas Aeruginosa
MC nosocomial pathogen because:
  • Ubiquitous water and soil organism
  • Resists wide temp range 5°C to 42°C
  • Produces biofilm
  • Resistant to disinfectant, antiseptics and multi-drug resistance (d/t resistant plasmids)
Virulence factor:
  • Exotoxin A- inhibiting protein synthesis (like diphtheria toxin)
  • Exotoxin S- Ribosylation of GTP binding protein
  • Others- Capsule/slime layer, Exotoxin S, U,Y, protease, elastase, Hemolysin, Endotoxin
Distinguishing Characteristics
  • Oxidase – positive, non – fermenting
  • Motile with polar flagella
  • Pigments: Pyocyanin (blue – green) by P.aeruginosa and Fluorescein (greenish yellow- by all spp.)
  • Pigmentation is Produced in King’s media
  • Selective media- cetrimide agar
  • Grape – like fruity odor
  • Produces slime layer (alginate)
  • Non – lactose fermenting colonies on MacConkey agar.
Infections:
  • Burn patients infection
  • Malignant otitis media
  • Shanghai fever
  • Cellulitis (blue – green pus)
  • Neutropenic Patients –causes Pneumonia and septicemia
  • Chronic Granulomatous Disease (CGD)
  • Ecthyma gangrenosum
  • UTI in Catheterized patients
  • Cystic fibrosis - by slime producing strains (alginate)
Anti Pseudomonal drugs:
Penicillin- Piperacillin, Mezlocillin, Ticarcillin
Cephalosporin – Ceftazidime, cefoperazone, cefipime
Carbapenems- imipenem, meropenem
Monobactam- Aztreonam
Aminoglycoside-
Tobramycin, Gentamicin, Amikacin
Quinolone- Ciprofloxacin, Levofloxacin
Other- PolymyxinB, Colistin
 
Burkholderia
  • Gram –ve Bacilli, Non Fermenter, Oxidase +ve,
  • Resistant to polymyxin B,
  • 227Bipolar stained
  • Zoonotic, agent of bioterrorism,
  • Occupational spread,
  • Treatment- ceftazidime/Carbapenems
 
B. PSEUDOMALLEI
  • Causative agent of melioidosis
  • Motile
  • Known as Vietnam Time-Bomb disease
  • Zoonotic - rodents
  • Humans - Pulmonary infections like TB (MC), Multiple abscesses, LN↑
 
BURKHOLDERIA MALLEI
  • Non-motile,
  • Zoonotic - horse
  • Mallein test- like tuberculin test
  • Glanders: nodules in Respiratory system
  • Farcy: involve skin with prominent lymphatics underneath
  • Strauss reaction: intraperitoneal injection in male guinea pig causes - testicular swelling
 
HAEMOPHILUS
 
H. Influenzae
  • Blood loving,
  • Known as Pfeiffer's bacillus
  • Fastidious, Require X and V factor for growth –
  • X (haemin in blood) and
  • V factor (NADP inside RBC)
  • Shows Pleomorphism
  • Oxidase +, Catalase +
  • Killed by refrigeration (i.e. why CSF sample should never be refrigerated)
  • Staellitism on blood agar – H.influenzae usually doesn't grow on blood agar but can grow around Staph streak which supplies V factor
  • Culture Media
    • Levinthal's agar
    • Field's agar,
    • Chocolate agar
  • Antigen
    • Capsular polysaccharide- serotyping is based.
    • OMP (outer membrane protein)
    • LOS (lipo-oligosaccharide)
  • Typed to 6 serotypes- Based on Capsular polysaccharide (a-f)
    • MC is type b (capsule is made up PRP –poly ribosyl ribitol phosphate)
    • H.influenzae and H.pylori- complete genome is mapped
Pathogenicity
Type b Strains
Non Typeable Strains
Capsulated-Poly Ribosyl-ribitol phosphate (PRP)
Non Capsulated
Invasive- causes meningitis, epiglottitis, pneumonia, bacteremia, endocarditis
Non-invasive – causes otitis media (MC in adult) and LRTI (adult)
Affect Children
Affect Adult
Hematogenous spread
Contagious spread
Vaccine available for type b only
Not available
228Hib PRP Vaccine
  • Available for type b only
  • Not effective in <2 years of age,
  • In quadruple DPT+Hib vaccine- Immunogenicity of Hib by increased by DT and TT
  • No vaccine for other H.influenza types
Chemoprophylaxis: Rifampicin
 
H. Aegyptius
  • Koch's –Week's bacillus
  • Pink eye syndrome (Egyptian ophthalmia)
  • Brazilian purpuric fever
 
H. Ducreyi
  • Chancroid/soft sore: Painful lymph node, tender nonindurated genital ulcer
  • Bipolar staining
  • School of fish” or “rail road track” appearance,
  • Antigenically homogenous
  • Culture Medium used-
    • Chocolate agar with 1% isovitalex, Vancomycin,
    • Chorioallantoic membrane (CAM)
Growth factor requirement of Haemophilus species
Haemophilus that need both V and X factors-
H.influenzae, H.aegypticus and H.haemolyticus
Haemophilus that need only X factors-
H.ducreyi and H.aprophilus
Haemophilus that need only V factors-
All para i.e. H. parainfluenzae, H.parahaemolyticus and H. paraprophilus
Haemophilus that need CO2-
H.aprophilus and H. paraprophilus
Haemophilus that produces hemolysis-
H.haemolyticus and H.parahaemolyticus
So Satelitism is shown by all that need V factor i.e. Satelitism is NOT shown by H.ducreyi and H.aprophilus
 
HACEK Group of Organisms
  • Fastidious, slow growing bacteria,
  • Present normally in oral cavity, Can cause endocarditis
  • Agents include-
    • Haemophillus species- H.parainfluenzae, H.aprophillus, H.paraaprophilus
    • Actinobacillus actinomycetemcomitans.
    • Cardiobacterium hominis,
    • Eikenella corrodens- corrodes on blood agar
    • Kingella kingae
 
BORDETELLA
  • Gram negative coccobacilli, oxidase +ve
  • Thumb print appearance in gram staining
  • Culture-
    • Bordet Gengou (glycerol, potato extract) – Bisected pearls or mercury drop colonies
    • Regan Low medium
    • Lacey DFP medium (diamidine fluoride and penicillin)
Virulence factors:
1. Pertussis toxin-↑cAMP (only by B.pertussis)
2. Agglutinogen
3. Filamentous hemagglutinin
4. Heat labile toxin
5. Tracheal Cytotoxin
6. Adenylate cyclase
7. LPS
8. Pertactin (OMP)
 
229Pathogenesis
  • Source of infection - patient in early stage
  • Patient is infectious- 1 week after exposure to 3 week after Paroxysmal stage
  • No healthy carrier
  • Secondary Attack rate of more than 90%, cyclical outbreaks occur every 3–5 years
  • Whooping cough or 100 days cough
  • Incubation period averaging 7–10 days
  • 3 stages:
    • Catarrhal (10 d)- nonspecific symptom, highly infectious, culture and smear+ve
    • Paroxysmal (2–4 week) – whooping cough, posttussive vomiting, usually culture and smear –ve, less infectious
    • Convalescent stage (1–2 week)
  • Whooping cough like symptoms- also by Mycoplasma, Adenovirus, Parainfluenza, B.parapertussis
  • Pertussis causes more severe infection than parapertussis.
 
Diagnosis
  • Best Specimen –nasopharyngeal aspirate
  • Plated immediately or transported through Dacron/rayon swab
  • Gold standard method- nasopharyngeal aspirate culture
  • PCR- Most sensitive, gives quicker results, but yet to be standardized properly.
  • Culture sensitivity- <60% (only in 1st 3 weeks) & better in young children
  • Direct fluorescent antibody tests of nasopharyngeal secretions
  • Enzyme immunoassays detecting IgA and IgG antibodies to pertussis toxin, filamentous hemagglutinin,
  • Lymphocytosis in young children
 
Vaccine
Whole cell pertussis:
  • Efficacy 85%
  • Side effect- febrile seizure, encephalitis and Hypotonic hypo responsive syndrome
Acellular pertussis contains
  • Pertussis toxoid – (Prevent disease but not infection)
  • Filamentous hemagglutinin
  • Agglutinogen 1, 2, 3
  • Pertactin (in some preparations)
  • Efficacy is same but Not encephalitogenic (can be given > 5 years)
  • Treatment-
DOC- Macrolide (Erythromycin), alternate (cotrimoxazole)
 
BRUCELLA
  • Gram -ve coccobacilli, Strict Aerobe
  • Zoonosis- different species affect different animals
    • B. melitensis – Sheep, Goat
    • B. abortus – Cattle
    • B. suis – Pig
    • B. canis – Dog
 
Brucellosis or Malta/Undulant Fever
  • Intracellular affecting RES
  • Transmitted by:
    • Raw milk intake (MC)
    • Contact with animal feces/urine
    • No person to person spread
  • 230Typhoid like illness but less severe & less acute + musculoskeletal (arthritis/ osteomyelitis) + undulating nature of fever
  • Complication- Hepatosplenomegaly, lymphadenopathy, CNS- depression, Meningoencephalitis, CVS-endocarditis
  • Possess two antigens - A and M
  • B melitensis- most pathogenic (M Antigen > A Antigen)
  • B abortus- A Antigen > M Antigen
 
Diagnosis
  • Culture → Blood (Castaneda method) and Bone Marrow culture (↑Sensitive)
  • Erythritol- improves growth
  • Tbilisi phage typing is done
  • Catalase +ve, Oxidase +ve, Urease+ve
Serology:
Standard Agglutination Test (SAT)
  • For IgM detected using B. abortus Antigen, useful for acute infection.
  • NOT useful for chronic brucellosis
  • False –ve:-i) due to blocking antibody (removed by prior heating/4%Nacl/ coomb test), ii)Also due to prozone phenomena
  • False +ve: cross reacting antibody in Yesinia. cholera, tularemia (removed by agglutination absorption test)
  • CFT and ELISA – useful for chronic infection
Animal Brucellosis Diagnosis:
  • Rose Bengal Card test,
  • Milk Ring Test,
  • Whey agglutination test
 
Treatment- WHO Regimen
  • Gold standard-IM streptomycin (2–3 wk)+ Doxycycline (6 wk)
  • WHO regimen - Rifampicin (6 wk) + Doxycycline (6 wk)
  • Relapse- 10–20%
  • CNS involvement-Ceftriaxone is added to the above regimen and duration is extended to 3–6 months.
231MULTIPLE CHOICE QUESTIONS
 
PSEUDOMONAS
1. Drug against ESBL producing Pseudomonas: (AIIMS Nov 2014)
  1. Ceftriaxone + Piperacillin
  2. Ceftriaxone+ Tazobactam
  3. Piperacillin + Tazobactam
  4. Ceftriaxone + Piperacillin+ Tazobactam
2. A cutaneous lesion is seen almost exclusively in neutropenic patients with sepsis is ecthymagangrenosum and it is caused by: (APPG 2014)
  1. N.meningitidis
  2. P. aeruginosa
  3. V. vulnificus bacteraemia
  4. Staph. aureus
3. Significance of mucoid strain Pseudomonas aeruginosa: AIIMS Nov 2014)
  1. Exhibit more drug resistant
  2. Produces biofilms
  3. Secrete more toxins
  4. Produce more bacteriocin
4. Ecthyma gangrenosum is caused by: (NEET Pattern Based)
  1. Pseudomonas
  2. Streptococcus
  3. Staphylcoccus
  4. H. influenzae
5. Drugs used in pseudomonas treatment: (PGI June 2011)
  1. Cefixime
  2. Ceftazidime
  3. Ceftriaxone
  4. Colistin
  5. Ampicillin
6. Which one of the following drugs is an Anti pseudomonal penicillin? (AI 2006)
  1. Cephalexin
  2. Cloxacillin
  3. Piperacillin
  4. Dicloxacillin
7. An organism grown on agar shows green colored colonies, likely organism is: (AI 2001)
  1. Staphylococcus
  2. Streptococcus
  3. Pseudomonas
  4. E.coli
 
BURKHOLDERIA
8. Burkhaolderia is infrequently seen in: (DNB June 2011)
  1. Pools
  2. Plants
  3. Soil
  4. Air
9. Cause of melioidosis is: (DNB Dec 2011)
  1. Burkholderia mallei
  2. Burkholderia pseudomallei
  3. Burkholderia cepacia
  4. None
10. Cause of chest infection in a child with cystic fibrosis: (AIIMS May 2011)
  1. Burkholderia
  2. Pseudomonas (non-mucoid strain)
  3. Thlebsiella
  4. Streptococcus pneumoniae
11. The following statements are true regarding melioidosis except: (AI 2005)
  1. It is caused by Burkholderia mallei
  2. The agent is a gram negative aerobic bacteria
  3. Bipolar staining of the etiological agent is seen with methylene blue stain
  4. The most common form of melidiosis is pulmonary infection
12. A 50 year old chronic alcoholic male agricultural worker presented with high grade fever of one week duration with spells of chills and rigor. Examination of the respiratory system revealed bilateral crepitation with scattered rhonchi. Multiple subcutaneous nodules were found on the extensor surface of the left forearm, arm and left leg. Direct microscopy of the pus aspirated from the skin nodule revealed plenty of Gram negative bacilli with bipolar staining. Culture revealed distinct rough corrugated grey-white colonies on Blood agar. The organisms were motile and oxidase positive. The most likely diagnosis is: (AIIMS Nov 2003)
  1. Plague
  2. Melioidosis
  3. Bartonellosis
  4. Actinomycosis
13. True about Burkholderiamallei: (PGI Nov 2012)
  1. Causes glanders disease
  2. Affects horses
  3. Non-motile
  4. Cause melioidosis
 
HAEMOPHILLUS
14. True about H influenza: (NEET Pattern Based)
  1. Grown on sheep blood agar & CO2
  2. It is not capsulated
  3. Invasive strain is most common
  4. Gram positive
15. Which of these need both V and X factors: (PGI June 2011)
  1. Hemophilus influenzae
  2. H.ducrei
  3. 232H.paraphrophilus
  4. H.aegyptius
  5. H.haemolyticus
16. Capsular polysaccharide constitutes an important virulence factor responsible for protective antibody response in infections caused by all the following bacteria except: (AI 2011)
  1. Haemophilus influenza
  2. Streptococcus pneumoniae
  3. Neisseria meningitides
  4. Bordetella pertussis
17. A patient with history of discharge from right ear for past 1 week presented with severe ear ache. The discharge was cultured on chocolate agar and the organism was found to be gram negative coccobacilli. The most likely cause is: (AIIMS Nov 2010)
  1. Pseudomonas
  2. Streptococcus pneumoniae
  3. Staphylococcus
  4. Haemophilus influenzae
18. Disease caused by Haemophillus: (PGI June 2009)
  1. Chancroid
  2. Influenza
  3. Acute epiglottitis
  4. Brain abscess
  5. Brazilian purpuric fever
19. HACETh group includes all except: (AIIMS Nov 2007 May 2008)
  1. Haemophilus aprophillus
  2. Acinetobacter baumanii
  3. Eikenella corrodens
  4. Cardiobacterium hominis
20. Chancroid is caused by: (NEET Pattern Based)
  1. H. ducreyi
  2. T. pallidum
  3. Gonococcus
  4. HSV
21. A 20 year old male patient present to the STD clinic with a painful genital ulcer. The gram stain of the smear from ulcer shows gram negative coccobacilli. The most appropriate media for culture would be: (AI 2004)
  1. Thayer-Martin Medium
  2. Blood agar with X and V factors
  3. Chocolate agar with isovitalex X
  4. Tellurite blood agar
22. All of the following are true regarding Haemophilus influenzae except: (AIIMS Nov 2003)
  1. It can be a part of the normal flora in some persons
  2. The serotyping is based on the bacterial outer membrane proteins
  3. It requires Haemin and NAD for growth in culture medium
  4. Type b is responsible for invasive disease
23. A 2 years old child is brought to the emergency with history of fever and vomiting. On examination he has neck rigidity. The CSF examination shows polymorphs more that 2000/ml protein 100mg /dl and glucose 10mg/dl. The gram stain shows the presence of Gram negative coccobacilli. The culture shows growth of bacteria only on chocolate agar and not on blood agar. The causative agent is: (AIIMS Nov 2002)
  1. Neisseria meningitides
  2. Haemophilus influenzae
  3. Branhamella cartarrhalis
  4. Legionella pneumophila
24. Non-typableH.influenzae can cause all except: (AIIMS Nov 2014)
  1. Meningitis
  2. Otitis media
  3. Sinusitis in adults
  4. Puerperal infection
25. True about H.influenzae: (PGI May 2013)
  1. Also called as Pfeiffer's bacilli
  2. In acute infections capsulated strains are often isolated
  3. Gram negative motile bacilli
  4. Easily cultured
  5. VP Test positive
26. Which of the following about H.influenzae is not true? (Recent Question 2013)
  1. Type-b is more pathogenic
  2. Its complete genome has been studied
  3. It grows fastidiously in blood agar
27. A patient presented to a STD clinic with painful genital ulcer and painful, soft non indurated enlarged inguinal lymph node. Identify the pathogen? (Recent Question 2013)
  1. H.ducreyi
  2. T.pallidum
  3. Calymmatobacter
  4. Herpes
 
BORDETELLA
28. Which does not has a known animal reservoir: (NEET Pattern Based)
  1. Bordetella pertussis
  2. Francisella tularensis
  3. Brucella melitensis
  4. Pasturella multocida
29. All are true statement regarding pertussis except: (PGI May 2012)
  1. Secondary attack rate averages 90% in unimmunized contacts
  2. Incubation period is around 14 days
  3. Erythromycin is during of choice
  4. Can affect people of any age
  5. Main source of infection is chronic carriers
30. Components of acellular pertussis vaccine: (AIIMS May 2011)
  1. PT + FHA + fimbriae
  2. PT + cytotoxin + fimbriae
  3. Endotoxin +fimbriae +pili
  4. Pertactin +FHA +outer membrane protein+ fimbriae
31. A 7 month old infant with the history of incomplete childhood vaccination presents with bouts of spasmodic cough with cyanosis and a typical inspiratory whoop. Which is most appropriate clinical specimen to be collected for the isolation of pathogen? (AI 2011)
  1. “Cough plate culture”
  2. Per oral swab
  3. Nasopharyngeal swab
  4. Endotracheal aspirate
32. Pertussis toxin acts by all of the following mechanisms except? (AIIMS Nov 2008)
  1. ADP ribosylation of proteins associated with receptors
  2. Increase cyclic AMP
  3. Increased calcium release from sarcoplasmic reticulum
  4. 233Acts through G alpha subunit
33. The usual incubation period of pertussis is: (AIIMS Nov 1996 & 2005, AIIMS June 2000)
  1. 7–14 days
  2. 3–5 days
  3. 21–25 days
  4. Less than 3 days
34. The following are true for Bordetella pertussis except: (AI 2003)
  1. It is a strict human pathogen
  2. It can be cultured from the patient during catarrhal stage
  3. It leads to invasion of the respiratory mucosa
  4. Infection can be prevented by acellular vaccine
35. Treatment for pertussis contacts of children: (UP 2000)
  1. Prophylactic antibiotic for 10 days
  2. Prophylactic antibiotic for 14 days
  3. Prophylactic antibiotic for 20 days
  4. Prophylactic antibiotic for 12 days
36. A child with pertussis should be isolated for: (Tholkatta 2003)
  1. 1–2 weeks
  2. 2–4 weeks
  3. 3–4 weeks
  4. 4–6 weeks
 
BRUCELLA
37. Milk ring test is seen in: (NEET Pattern Based)
  1. Brucellosis
  2. Tuberculosis
  3. Bacteroides
  4. Salmonellosis
38. Brucella melitensis is commonly found in (animal) : (PGI June 2011)
  1. Pig
  2. Camel
  3. Sheep
  4. Goat
  5. Reindeer
39. Malta fever is caused by: (PGI June 2002)
  1. Treponema pallidum
  2. Borrelia burgdorferi
  3. Brucella melitensis
  4. Pseudomonas aeruginosa
40. Treatment of Brucellosis: (PGI June 2008, JIPEMR 2010)
  1. Doxycycline
  2. Streptomycin
  3. Erythromycin
  4. Penicillin
  5. Rifampicin
41. Brucellosis can be transmitted by all of the following, except: (AIIMS May 2007, PGI June 2002) (AIIMS Nov 2006, AI 2007)
  1. Contact with infected placenta
  2. Ingestion of raw vegetables from infected forms
  3. Person to person transmission
  4. Inhalation of infected dust or aerosol
42. A farmer presenting with fever off-and on for the past 4 years was diagnosed to be suffering from chronic brucellosis. All of the following serological tests would be helpful in the diagnosis at this state except: (AI 2004)
  1. Standard Agglutination test
  2. 2 Mercaptoethanol test
  3. Complement fixation test
  4. Coomb's test
43. A farmer rearing sheep, presented with complaints of fever and weakness for the last one month. There is generalized lymphadenopathy. There was also associated hepatomegaly. Biopsy of liver showed non caseating granuloma. These are most likely due to infection with: (AIIMS Nov 2000)
  1. Yesinia pestis
  2. Brucella canis
  3. Francisella tularensis
  4. Brucella melitensis
234EXPLANATIONS
 
PSEUDOMONAS
1. Ans. (c) (Piperacillin + Tazobactam) Ref: Journal–ESBL: Clinical update, Clinical Microbiology Reviews, Oct.2005, p. 657–686
Extended Spectrum-β-Lactamases (ESBL) producing Pseudomonas can be treated with an antipseudomonial β lactam (e.g. piperacillin) plus β lactamase inhibitor such as tazobactam.
2. Ans. (b) (P. aeruginosa) Ref: Ananthnarayan 9 e /p315
Ecthyma gangrenosum in a neutropenic patients with sepsis is suggestive of Pseudomonas aeruginosa infection.
3. Ans. (b) (Produces biofilms) Ref: Harrison 18th/p1267–73
Mucoid strain of Pseudomonas aeruginosa are able to produce biofilm(or alginate layer), hence can cause infection in patient with cystic fibrosis.
4. Ans. (a) (Pseudomonas) Ref: Ananthanarayan 9/e p315, 8/e p 315
  • Ecthyma gangrenosum is caused by –Pseudomonas aeruginosa
5. Ans. (b) (d) (Ceftazidime, Colistin)
Ref: Harrison 18/e p1269, 17/e p952, KD Tripathy 6/e p688–692
  • Anti Pseudomonal drugs: Refer text (chapter review).
6. Ans. (c) (Piperacillin) Ref: Harrison 18/e p1269,17/e p952
  • Anti Pseudomonal drugs: Refer text (chapter review).
7. Ans. (c) (Pseudomonas) Ref: Ananthanarayan 9/e p315, 8th /315
Pigment produced by Pseudomonas-
  • Pyocyanin (blue – green) (by P.aeruginosa)
  • Fluorescein (greenish yellow- by all spp)
  • Pyorubin
  • Pyomelanin
 
BURKHOLDERIA
8. Ans. (b) (plants) Ref: Harrison 18/e p1273
  • B. pseudomallei is found in soil and water (pools). Humans and animals are infected by inoculation, inhalation (from air), or ingestion; rarely from person to person
9. Ans. (b) (Burkholderia pseudomallei) Ref: Ananthanarayan 9/e p317, 18, 8/e p318
  • B. pseudomallei is the causative agent of melidiosis (Vietnam time bomb disease)
10. Ans. (a) (Burkholderia) Ref: Harrison 18/e p2147, 48, Ananthanarayan 9/e p316
“Mucoid strain of Pseudomonas is often found in sputum of cystic fibrosis patients. Burkholderia is also recovered from CF sputum and is pathogenic.”
Cystic fibrosis patients exhibit characteristic sputum microbiology profile-
  • Haemophilus influenzae and S. aureus - first organisms recovered in newly diagnosed patients with CF.
  • Mucoid P. aeruginosa (biofilm producing, antibiotic-resistant)– MC organism recovered from CF.
  • Burkholderia – 2nd most common (next to mucoid Pseudomonas)
  • Other gram-negative rods recovered from CF sputum include –
    • Alcaligenes xylosoxidans, occasionally mucoid forms of Proteus, Escherichia coli, and Klebsiella.
  • Mycobacterium tuberculosis is rare in patients with CF.
  • 235However, 10–20% of adult patients have non tuberculous Mycobacteria
  • MC fungus isolated- Aspergillus fumigatus (allergic bronchopulmonary Aspergillosis)
11. Ans. (a) (It is caused by Burkholderia mallei) Ref: Ananthanarayan 9/e p317, 8/e p318
  • B.mallei causes glanders and farcy
  • B. pseudomallei- Causative agent of melidiosis (Glanders like disease)
    • Motile and Zoonotic - rodents
    • ✓ Humans –Produces pulmonary infections like TB (MC), Multiple abscesses, LN↑
12. Ans. (b) (Melioidosis) Ref: Ananthanarayan 9/e p317, 8/e p318
Explained earlier
Points in favor:
  • Agricultural worker
  • Fever, respiratory symptoms, Multiple subcutaneous nodules
  • Gram negative bacilli with bipolar staining.
  • Culture - corrugated grey-white colonies on Blood agar.
  • Motile and oxidase positive
13. Ans.(a), (b),(c) (Causes glanders disease, Affects horses, Non-motile) Ref: Ananthnarayan 9/e p317
Refer chapter review for detail.
 
HAEMOPHILUS
14. Ans. (c) (Invasive strain is most common) Ref: Ananthanarayan 9/e p328, 8/e p330
  • H. influenzae-
    • ✓ Most of strains are capsulated and are invasive.
    • ✓ It can grow on sheep blood agar only adjacent to Staph.aureus streak line
    • ✓ Gram Negative coccobacilli
15. Ans. (a) (d) (e) (H.influenzae, H.aegypticus and H.haemolyticus) Ref: Ananthanarayan 9/e p328, 8/e p330
  • Haemophilus that need both X and V factors- H.influenzae, H.aegypticus and H.haemolyticus
16. Ans. (d) (B.pertussis) Ref: Ananthanarayan 9/e p332, 8/e p332
  • H.influenzae, N.meningitidis and Pneumococcus are capsulated and capsular polysaccharide vaccine is available against these organism.
17. Ans. (d) (H.influenzae) Ref: Ananthanarayan 9/e p328, 8/e p329
  • Gram negative coccobacilli isolated from ear discharge that grows on chocolate agar
  • Among the options, in gram staining, H.influenzae appears as Gram negative coccobacilli.
18. Ans. (a), (c), (d), (e) (Chancroid, Acute epiglottitis, Brain abscess, Brazilian purpuric fever)
Ref: Ananthanarayan 9/e p329, 331, 8/e p 329, 331
  • Influenza is caused by influenza virus.
  • Pfeiffer observed Haemophilus in sputum of Influenza pandemic patients and wrongly named as H.influenzae
  • Brazilian Purpuric fever-H.aegypticus
  • Chancroid (Soft Chancre)- H.ducreyi
  • Laryngoepiglottitis (Croup), Meningitis Brain abscess- H.influenzae
19. Ans. (b) (Acinetobacter baumanii) Ref: Ananthanarayan9/e p331, 8/e p333
HACEK group of organisms- refer text (chapter review).
20. Ans. (a) (H. ducreyi) Ref: Ananthanarayan 9/e p330, 8/e p333
  • ✓ Chancroid is caused by – H. ducreyi
23621. Ans. (c) (Chocolate agar with isovitalex X) Ref: Ananthanarayan 9/e p330, 8/e p333
  • Causes of Genital ulcer includes:
    • ✓ Chancroid or Herpes- painful ulcer
    • ✓ Donovanosis or primary syphilis or LGV – Painless ulcer
  • Painful Genital ulcer showing gram negative coccobacilli – Indicates Chancroid
  • Chocolate agar with isovitate X and vancomycin is used as a medium for H.ducreyi, the causative agent of Chancroid.
About Other Options
  • ✓ Thayer-Martin Medium- for Gonococcus
  • ✓ Blood agar with X and V factors – for H.influenzae
  • ✓ Tellurite blood agar – for C.diphtheriae
22. Ans. (b) (The serotyping is based on the bacterial outer membrane proteins) Ref: Ananthanarayan 9/e p328, 8/e p333
  • Serotyping of H.influenzae is based on the capsular polysaccharide
About Other Options:
  • H.influenzae can be typed to Six serotypes (a-f), out of which type b is responsible for most of the invasive infection.
  • Type b and nontypable strains are the most relevant strains clinically, responsible for invasive disease
  • Haemin (X factor) and NAD (V factor) are essential for growth in culture medium
  • Can be a part of the normal flora in some persons
  • Nontypable strains colonize the upper respiratory tract of up to three-fourths of healthy adults’
  • Further details refer chapter review.
23. Ans. (b) (Haemophilus influenzae) Ref: Ananthanarayan 9/e p328, 30, 8/e p331, 33
Points in favor- Case of meningitis, gram stain shows the presence of Gram negative coccobacilli. The culture shows growth of bacteria only on chocolate agar and not on blood agar.
  • H. influenzae requires X and V factor for its growth.
  • In blood agar, X factor is available but V factor is not free as it is inside RBC. Where as chocolate agar is a heated blood agar, So RBC gets lysed to release V factor in medium. Hence, H.influenzae can grow in chocolate agar but not in blood agar.
  • However, if S.aureus is streaked on blood agar, it provides V factor, hence H.influenzae can grow in blood agar adjacent to the S.aureus steak line (called as Staellitism)
Media for H. influenzae:
  • Levinthal's agar
  • Field's agar,
  • Chocolate agar
  • Blood agar with S.aureus streak line
24. Ans. (a) (Meningitis) Ref: Harrison 18th/1229–33
Non-typableH.influenzae
Next to H.influenzae-b (Hib), Non typable strains are the commonest group of Haemophilus influenzae clinically. They are non-invasive, spread by contagious spread. They cause the following infections -
  • Childhood otitis media
  • Exacerbations of COPD (chronic obstructive pulmonary disease)- They are the most common bacterial cause.
  • Pneumonia in adults among patients with COPD or AIDS
  • Puerperal sepsis and neonatal bacteraemia- Especially by non-typable strains of biotype IV that colonizes the female genital tract.
  • Sinusitis in adults and children
  • 237Rarely causes invasive infections such as empyema, adult epiglottitis, pericarditis, cellulitis, septic arthritis, osteomyelitis in countries where Hib vaccines are used widely.
25. Ans. (a) (b) (Also called as Pfeiffer's bacilli, In acute infections capsulated strains are often isolated)
Ref: Ananthnarayan 9/e p327
  • H.influenzae or the Pfeiffer's bacillus is non motile, Gram negative pleomorphic bacilli. VP is negative.
  • Capsulated strains are isolated in invasive acute infections whereas the non-typable strains are often responsible for non- invasive conditions such as otitis media.
26. Ans. (c) (It grows fastidiously in blood agar) Ref: Ananthanarayan 8/e p330 & 9/e p328
  • H.influenzae needs both factors X and V for growth. Blood agar has free Factor X but factor V is entrapped inside RBC, not free. Hence, it cannot grow on blood agar.
27. Ans. (a) (Haemophilus ducreyi) Ref: Ananthanarayan 9/e p331
  • Painful genital ulcer and painful, soft non indurated enlarged inguinal lymph node. Suggestive of soft sore / chancroid which is caused by Haemophilus ducreyi.
 
BORDETELLA
28. Ans. (a) (Bordetella pertussis) Ref: Park 22/e p155, 21/e p153
  • Bordetella pertussis doesn't have a known animal reservoir.
29. Ans. (e) (Main source of infection is chronic carriers) Ref: Park 22/e p155, 21/e p153, Harrison 18/e p1241, 43
Bordetella pertussis:
  • Secondary attack rate averages 90% (80–100%) in unimmunized contacts
  • Incubation period is averaging 7–14 days (not more than 3 weeks)
  • Macrolide like Erythromycin estolate is during of choice
  • Although thought of as a disease of childhood, pertussis can affect people of all ages and is increasingly in adolescents and adults.
  • There is no carriers, Source of infection -patient in early stage
  • Patient is infectious- 1 week aft exposure to 3 week aft Paroxysmal stage
30. Ans. (a) (PT + FHA + fimbriae) Ref: Harrison 17/e p936, 18/e p1245
  • “Although a wide variety of acellular pertussis vaccines were developed, only a few are still widely marketed; all contain pertussis toxoid and filamentous hemagglutinin.
Acellular pertussis contains:Pertussis toxoid
  • ✓ Filamentous hemagglutinin
  • ✓ Agglutinogen 1,2,3
  • ✓ Pertactin (in some vaccine preparations)
31. Ans. (c) (Nasopharyngeal swab) Ref: Ananthanarayan 9/e p336, 8/e p 938, Harrison 17/e p935 & 18/e p1244
  • History of Incomplete childhood vaccination with bouts of spasmodic cough with cyanosis and a typical inspiratory whoop is suggestive of Whooping cough due to Bordetella pertussis.
  • “The best specimen is collected by nasopharyngeal aspiration, by fine flexible plastic catheter.
  • Is attached to a 10-ml syringe is passed into the nasopharynx and withdrawn while gentle suction is applied.
  • Since B. pertussis is highly sensitive to drying, secretions for culture should be inoculated without delay onto appropriate medium”
    ………….Harrison 17/e p935& 18/e p1244
About Other Options
  • Cough plate method though offers advantage like direct bedside plating without delay, but chance of contamination is more.
  • Per oral swab is used to collect secretions from posterior pharyngeal wall.
  • Endotracheal swab is not useful as the secretion will be predominantly in nasopharynx.
23832. Ans. (c) (Increased calcium release from sarcoplasmic reticulum) Ref: Jawetz 24/e p283 & 25/e p248
  • Increased calcium release from sarcoplasmic reticulum is not a mechanism of Pertussis toxin
  • Pertussis toxin-
  • Promotes lymphocytosis
  • Sensitization to histamine
  • Enhances insulin secretion
  • ADP-ribosylating activity, with an A/B structure and mechanism of action similar to that of cholera toxin.
  • i.e ADP ribosylation of GTP binding protein → activation of Adenyl cyclase → activation of cAMP
33. Ans. (a) (7–14 days) Ref: Harrison 17/e p 934 & 18/e p1243, Park 22/e p155, 21/e p153 Incubation period varies averaging- 7–10 days (Harrison), 7–14 days (Park)
34. Ans. (d) (Infection can be prevented by acellular vaccine) Ref: Harrison 17/e p933–34 & 18/e p1243
  • Acellular pertussis vaccine contains pertussis toxoid and filamentous hemagglutinin.
  • Since it contains Toxoid, so antitoxin will be developed in the body which will gives protection from the toxin but has no role in colonization and infection.
About Other Options:
  • B. pertussis infects only humans where as B. bronchiseptica is an important pathogen of domestic animals. …………… Harrison 17/e p933 & 18/e p 1244
  • B. pertussis is found abundantly during catarrhal stage, so both microscopy and culture are more sensitive during catarrhal stage. In Paroxysmal stage, the bacilli are scanty and in conventional stage they are not demonstrable. …….Ananthanarayan 9/e p335, 8/e p338
  • There is local cellular invasion, with intracellular bacterial persistence; however, systemic dissemination does not occur. Systemic manifestations result from the effects of the toxins. …..Harrison 18/e p1244
35. Ans. (a) (Prophylactic antibiotic for 10 days) Ref Park 22/p156
  • Treatment for pertussis contacts of children- Prophylactic antibiotic like erythromycin for 10 days
36. Ans. (c) (3–4 weeks) Ref: Park 22/e p156
  • A child with pertussis should be isolated till he is considered non infectious, probably for 3–4weeks
 
BRUCELLA
37. Ans. (a) (Brucellosis) Ref: Ananthanarayan 9/e p339, 8/e p343
  • Milk ring test is done for diagnosis of Brucellosis in animals.
38. Ans. (b) (c) (d) (Sheep, Camel, Goat) Ref: Ananthanarayan 9/e p339, 8/e p343, Harrison 18/e p1296, 17/e p952
  • The nomen system recognizes B. melitensis, which is the most common cause of symptomatic disease in humans and for which the main sources are sheep, goats, and camels …Harrison 18/e p1296,
The nomen system of classification of Brucella:
(Based on differences in chromosomal structure and host preference)-Brucella species show a high degree of host specificity-B. melitensis- infects goats, sheep and camel
  • B. abortus - infects cattle;
  • B. suis - infects Pigs (and also enzootic in reindeer, caribou and in rodent)
  • B. canis - infects dogs and foxes.
  • B.neotome- infects Wood Rat
  • B.ovis- infects Sheep
39. Ans. (c) (Brucella melitensis) Ref: Ananthanarayan 9/e p339, 8/e p341
“Brucellosis is also known as Malta or Undulant fever or Mediterranean fever”
40. Ans. (a), (b), (e) (Doxycycline, Streptomycin, Rifampicin) Ref: Harrison 17/e p975 & 18/e p1296
Treatment of brucellosis- refer chapter rivew
23941. Ans. (c) (Person to person transmission)
Ref: Ananthanarayan 9/e p342,8/e p343, Harrison17/e p973 & 18/e p1296
‘Person-to-person transmission is extremely rare’
Mode of Transmitted by:
  • Intake of Raw milk (MC), or milk products /meat contaminated with infected animal feces/urine
  • Contact with animal feces/urine
  • Inhalation of dried material of animal origin
42. Ans. (a) (Standard Agglutination test) Ref: Ananthanarayan 9/e p343, 8/e p345
  • Standard Agglutination test identifies mainly IgM antibody, hence they are useful in acute but not in chronic brucellosis.
  • In acute brucellosis- Following 7–10 days of infection, both IgM & IgG appear, however the IgM antibodies gradually fall and only IgG antibodies persist in chronic brucellosis
Standard Agglutination test:
  • May give false negative results due to- Presence of IgG antibodies (which act as blocking antibody).
    • ✓ Blocking IgG antibody can be made obviate by adding antiglobulin (Coomb's sera).
  • May give false positive results due to presence of cross reacting antibodies in cholera, yersinia and tularemia infection.
  • Cholera induced antibodies are removed by- Prior treatment with 2- Mercaptoethanol
  • Prior treatment with 2- Mercaptoethanol also removes IgM antibodies, so that remaining IgG antibody can be separated from a mixture of IgM & IgG.
  • Chronic brucellosis can be diagnosed by: CFT and ELISA
43. Ans. (d) (Brucella melitensis) Ref: Harrison 17/e p974 &18/e p1297
Points in favor:
  • Presented with fever and weakness, generalized lymphadenopathy and hepatomegaly and Biopsy of liver
  • Revealing non caseating granuloma – Suggestive of Brucellosis
  • Contacts with sheep – indicates the agent is Brucella melitensis.

SpirochaeteCHAPTER 3.10

 
GENERAL PROPERTIES
  • Spirally coiled and hair like extremely thin
  • Cannot be visualized by light microscope
  • Visualized by dark field, fluorescent microscope or sliver staining methods (to in- crease the thickness)
  • Pathogenic spirochetes- Treponema, Borrelia, Leptospira Motile but Possess- endoflagella
  • Motility- Cork screw, lashing, flexion-extension, rotatory
 
TREPONEMA
T. pallidum
: Syphilis
T. pertenue
: Yaws
T. carateum
: Pinta
T. endemicum
: Endemic Syphilis (Bejel)
 
SYPHILIS
  • Incubation Period -9–90 days
  • Primary syphilis:
    • Painless LN + painless indurated genital ulcer (Hunterian or hard chancre)
    • MC site of ulcer is genitalia > mouth, nipple,
    • If transmitted by non veneral route – ulcer may be extra-genital (fingers)
    • If transmitted by blood transfusion- primary stage may be absent
  • Secondary syphilis:
    • Skin rashes + condyloma lata at Mucocutaneous junction + mucosal patches
    • Highly infectious
    • All the serological tests are 100% positive
  • Latent syphilis: No symptoms, diagnosis by serology only.
  • Tertiary syphilis:
    • CVS (aortic regurgitation, aneurysm) +
    • Gummata
    • Meningovascular (General paralysis of insane and tabes dorsalis – quaternary)
 
Diagnosis
  1. Microscopy:
    • Sample-Ulcer lesion cleaned with saline, margin gently scraped to remove epithelium, gentle pressure applied to the base of the lesion, exudates is col
    • Dark ground- detection limit 104/ml
    • DFA (direct fluorescent antibody) - ↑Sensitive (uses monoclonal Antibody to T. pallidum) and ↑ Specific
    • Light microscopy- not useful as they are extremely thin
    • Staining: Silver Impregnation such as Levaditi stain (tissue), Fontana stain (smear)
  2. Culture:
    • Pathogenic Treponema species cannot be cultivated on artificial media (doesn't follow Koch's postulates)
      241
    • Maintained in rabbit testes (Nichol's strain)
    • Non pathogenic spp (Reiter Treponema) culture tried on - Smith Noguchi medium
  3. Serology:
    • (A) Non Treponemal or non specific test (STS-Standard tests for syphilis):
      • Uses-Cardiolipin (Diphosphatidyl Glycerol) antigen
      • Detects reagin antibodies
      • Older Non Treponemal test- Wasserman (CFT), and Kahn Test (Tube Flocculation)
      • Newer Non Treponemal Test –
        • ✓ VDRL,
        • ✓ RPR (rapid plasma regain),
        • ✓ USR (Unheated serum regain test)
        • ✓ TRUST(toludine red unheated serum test)
    • (B) Specific/ Treponemal test:
      • TPI (T.pallidum immobilization test)--using live T.pallidum
      • FTA-ABS (Fluorescent treponemal antibody absorption test)- Uses Killed T.pallidum
      • TPHA (T.pallidum hemagglutination test)- uses antigenic extract of T.pallidum
      • Western blot & Enzyme immunoassay (EIA)
    • (C) Group specific Using Reiter strain: RP CFT (Reiter protein Complement fixation test)
A. Non Treponemal or non specific test
 
VDRL
  • Devised at-Veneral disease research laboratory- located in Newyork
  • E.g of Slide Flocculation test (precipitation reaction)
  • Antigen used- Cardiolipin + Lecithin+ Cholesterol, prepared by Pangborn
  • Procedure-1) reconstitution of VDRL antigen with buffer, 2) preheating of patient's sera at 56°c for 30min to remove non specific inhibitors, 3) mixing a drop of sera and antigen on a VDRL slide, rotating for 4 min at 180 rpm, 4) observed under microscope for antigen antibody complexes as clumps.
  • Biological false positive (1%) seen in unrelated infections like-
    • Acute BFP- Seen in injuries, acute infection and acute inflammatory conditions, technical error
    • Chronic BFP (after 6m)- Hepatitis, malaria, chickenpox, infectious mononul- ceosis, measles, pregnancy, in SLE & other collagen vascular disorder, lepro- matous leprosy, relapsing fever, tropical pulmonary eosinophilia, HIV, and IV drug abusers, ageing, malignancy
    • VDRL antibodies are IgG where as BFP antibodies are IgM.
  • False negative VDRL occurs – due to Prozone phenomena (excess antibody)
  • Reagin antibodies appear 7–10 days after primary syphilis (or 3–5 weeks after infection)
VDRL
RPR
Result-read microscopically
Read Macroscopically- carbon particle added to Cardiolipin Antigen
Once reconstituted, should be used in 24 hour
Can be stored – EDTA used as stabilizer
Preheating of serum is required- to remove inhibitors
Preheating of serum not required- as choline chloride used to remove inhibitors
Sample- blood, plasma, serum, CSF
Sample- blood, plasma, serum, but not CSF
Rotation for 4 min
Rotation for 8 min
78% Sensitive in primary syphilis
86% Sensitive in primary syphilis
B. Specific/ Treponemal test:
  • TPI (T.pallidum immobilization test):
    • {Live T.pallidum (Nichol's strain) + patient's sera + complement}- incubated
    • If sera contain antibodies- then antibodies bind to T.pallidum surface anti- gens and complement mediated immobilization takes place.
    • Immobilization- >50% indicates positive reaction, <20%- negative, 20–50%- equivocal242
  • TPHA –T.pallidum hemagglutination test
    • Tanned sheep RBCs coated with sonicated antigenic extract of T.pallidum + patient's sera (preabosrbed with Reiter treponema to remove the group specific antibodies)
    • E.g. of passive agglutination test
  • TPPA- T. pallidum particle agglutination:
    • Uses gelatin particle to coat more purified T.palludim antigens.
    • No need for absorption of patient's sera with Reiter treponema
  • FTA-ABS (Fluorescent treponemal antibody absorption test)-
    • Slides smeared with Nichol's strain + Patient's serum (pre absorbed with Reiter treponema to remove the group specific antibodies) + fluorescent tagged anti gamma globulin
    • Currently- the standard reference test accepted worldwide
Points to remember:
Test
Sensitivity
Specificity (%)
Treponemal
Primary (%)
Secondary (%)
Latent (%)
Late (%)
Over all
FTA ABS
84 (70–100)
100
100
96
97 (94–100)
TPPA
88
100
100
98 (95–100)
TPHA
76 (69–90)
100
97 (97–100)
94
99 (98–100)
EIA
90
100
100
99
Western blot
90
100
100
98
Non treponemal
VDRL
78 (74–87)
100
95 (88–100)
71 (37–94)
98 (96–99)
RPR
86 (77–100)
100
98 (95–100)
73
98 (93–99)
USR
80 (72–88)
100
95 (88–100)
99
TRUST
85 (77–86)
100
98 (95–100)
99 (98–99)
 
Congenital Syphilis
  • Transmission across the placenta - may occur at any stage of pregnancy,
  • Fetal lesions appear after the fourth month of gestation when the fetal immune response starts appearing.
  • High Risk - Untreated early maternal syphilis
Manifestations of congenital syphilis-
Earliest manifestations (<2yr, are infectious)- Rhinitis, or “snuffles”, mucocutaneous lesions, bone changes hepatosplenomegaly, lymphadenopathy, anemia, jaundice, thrombocytopenia; and leukocytosis
Late congenital syphilis – (>2 years and are noninfectious)-subclinical (MC form), interstitial keratitis, eighth-nerve deafness, arthropathy, bilateral knee effusions (Clutton's joints),asymptomatic neurosyphilis
Residual stigmata- Hutchinson's teeth (centrally notched, wide, peg-shaped upper central incisors), “mulberry” molars (sixth-year molars with multiple, poorly developed cusps), saddle nose, and saber shins.
Diagnosis Congenital syphilis
  • 243Definitive diagnosis:
    • Direct demonstration of T. pallidum from umbilical cord, placenta, nasal dis- charge, or skin lesion by dark field microscopy
  • Presumptive diagnosis:
    • Mother with untreated or inadequately treated syphilis at delivery, regard- less of findings in the infant, or any infant who has a reactive treponemal test result and any clinical sign or symptoms of congenital syphilis
    • or an abnormal CSF finding without other cause,
    • or a reactive VDRL-CSF test result,
    • Detecting infant's serum antibodies:
      • ✓ IgM based Assay-19s IgM FTA-ABS and ELISA and western blot
      • VDRL is not useful (as it detects IgG)….. However, VDRL is used for screening in India and the interpretation should be done as follows…
Screening based on VDRL test:
  • Serial IgG in baby (VDRL)-
    • If maternal transfer→ titer falls in 3 months,
    • If Congenital infection → titer rises in 3 months
  • Simultaneous VDRL test of mother and baby-
    • If mother's titer > baby's titer→ Indicates maternal transfer
    • If baby's titer > mother's titer → I Indicates congenital infection
  • Sample : Baby serum (Most appropriate) > cord blood > Maternal serum
Infant's result (IgM test)
Clinical feature in infant
Mother's treatment history
Serodiagnosis
Non reactive
Present
Untreated /inadequately treated
Congenital syphilis
Non reactive
Absent
Untreated /inadequately treated early syphilis
Possibly incubating Congenital syphilis
Non reactive
Absent
Untreated /inadequately treated latent syphilis
Unknown risk of Congenital syphilis
Reactive
Present
Untreated /inadequately treated
Congenital syphilis
Reactive
Absent
Untreated /inadequately treated
Suggestive of Congenital syphilis
Treatment of syphilis:
Stage
DOC
Alternate
Primary, secondary, or early latent
Penicillin G
(single dose 2.4 mU IM)
Tetracycline (2 wk)
Late, latent CVS or benign tertiary
Penicillin G benzathine
(weekly for 3 weeks)
Tetracycline (4 wk)
Neurosyphilis or Pregnancy or HIV
Aqueous crystalline or Procaine penicillin G (10–14d)
Desensitization and Penicillin
244
Feature
Venereal Syphilis
Yaws
Endemic Syphilis
Pinta
Organism
T. pallidum
T. pertenue
T. endimicum
T. carateum
Mode of transmission
Sexual, transplacental
Skin-to-skin
Household contacts: kissing, sharing utensils or insect vector
Skin-to-Skin
Age
Adulthood
Early childhood
Early childhood
Late childhood
Primary lesion
Cutaneous ulcer (Chancre)
Papilloma, often ulcerative
Rarely seen
Non ulcerating papule with satellites, pruritic
Site of 1° lesion
Genital, oral, anal
Extremities
Oral
Extremities, face
Secondary lesions
Mucocutaneous lesions condylomata lata
Cutaneous papulosquamous lesions; osteoperiostitis
Florid mucocutaneous lesions (mucous patch, spilt papule; condyloma lata); osteoperiostitis
Pintides, pigmented. pruritic
Infectious relapses
~25%
Common
Unknown
None
Late complications
Gummata, CVS and CNS lesion
Destructive gummata of skin, bone, cartilage
Destruction of the nose, maxilla, palate, and pharynx is termed gangosa
No CNS or CVS lesion
Non destructive,
Dyschromic macules
No CNS or CVS lesion
Diagnosis-
Dark-field microscopy and serologic testing (same tests that are used for syphilis)
Treatment-
Benzathine penicillin – for patients and their contacts
Syphilis
Chancroid
LGV
Donovanosis
Herpes
Ulcer
Single Painless indurated
Painful non indurated irregular
Painless
Painless
Multiple painful vesicular, bilateral
LN
Painless
Painful
Painful
Not enlarged
Not enlarged
IP
9–90days
<1 week
3 days to 6 weeks
1–4 week
<1week (2–3days)
 
BORRELIA
 
Relapsing Fever
Characters
Epidemic or relapsing fever
Endemic relapsing fever
Agent
Caused by B. recurrentis
Caused by B. duttoni, B. hermsii
Host
Exclusively human disease
Natural host is rodent
Transmitted by
Louse (Pediculus humanus) By crushing or rubbing
Tick (Ornithodoros tick), By bite
Distribution
Worldwide (Asia & Africa)
North America,
Incubation period
8 days
7 days
1st Febrile episode
5.5 days
3 days
Afebrile period
9 days
7 days
Subsequent Febrile episode
Shorter
Shorter
Treatment
Doxycycline and erythromycin, Single-dose
Doxycycline and erythromycin, for 1 week
  • 245So named because :Alternate periods of febrile and afebrile episodes
  • B. recurrentis shows antigenic variation- reason for relapse
Lab diagnosis
  • Microscopic examination of Blood –
    • ✓ Wet film under dark ground or phase contrast microscope
    • ✓ Giemsa and Leishman staining or QBC
  • PCR- More sensitive and speciation possible
  • Antibody detection- ELISA, IFA- Not reliable, gives false positive
  • Animal inoculation into mice – Causes intra peritoneal infection
 
Lyme's Disease
  • B. burgdorferi – Tick borne (Ixodes ricinus)
  • Clinical Triad:
    • Stage 1:Erythema migrans (annular lesions),
    • Stage 2:Disseminated infection: skin lesions, meningitis, neuritis, nodal block, or joint pain
    • Stage 3:Persistent infection: arthritis, encephalopathy or acrodermatitis
  • Lab Diagnosis:
    • Diagnosis mainly on clinical ground.
    • Isolation from skin lesions or blood- modified Kelley's medium
    • Microscopic detection – Dark ground, phase contrasts, Immunfluroscence, silver staining
    • Antigen in urine
    • PMNs in joint fluid but lymphocytosis in CSF
    • In Europe and Asia, first radicular pain occurs followed by CSF pleocytosis (called meningopolyneuritis, or Bannwarth's syndrome), meningeal or en- cephalitic signs are absent.
  • Treatment:
    • DOC for all cases (except CNS or AV block) - Oral doxycycline > amoxicillin,
    • DOC for CNS or AV block- IV Ceftriaxone > Cefotaxime
 
Borrelia Vincentii
  • Commensal of mouth
  • Vincent's angina- ulcerative gingivostomatitis (predisposing conditions such as malnutrition, viral infection)
  • Often associated with fusiform bacillus (Fusobacterium fusiformis)
 
LEPTOSPIRA
General feature:
  • Obligate aerobes
  • L. interrogans – parasitic strains - 26 serogroup and >250 serovars
  • L. biflexa – free living saprophytic strains
  • Reservoir-Zoonotic (domestic & wild rat)
  • Transmission- direct contact with animals (occupational), indirect contact with animal urine
  • Human-to-human transmission does not occur
  • India- MC in Orissa, Gujarat, Maharashtra, TN, Andaman, Kerala
Clinical Stage
  • Incubation period averages 5–14 days
  • Anicteric (90%) stage- Biphasic-
    • Septicemic or acute stage(1st week)
    • Immune stage (meningitis, uveitis, rash)- after 2nd week, excreted in urine
  • 246Icteric-(10%)- (Weil's/hepatorenal hemorrhagic syndrome)-
    • Hepatomegaly & jaundice
    • Acute tubular injury(oliguric, with serum electrolyte abnormalities)
    • Hemorrhages (MC –pulmonary)
    • Hypotension
Diagnosis
  • Sample- Septicemic stage (Blood > CSF), immune stage (urine)
  • Microscopy-
    • Dark field- detection limit 104 /ml, phase contrast, silver impregnation
    • More no. of spirals with hooked end
  • Culture-
    • EMJH [Ellinghausen–McCullough–Johnson–Harris], Fletcher's, Korthoff's medium
    • Incubation at 28–30°C for 13 week, tedious, Sensitivity is less than serology
  • Serology-
    • Genus specific – CFT, ELISA
    • Microscopic agglutination test (MAT)- Gold standard (Sero group and serovar specific)
Treatment
  • Harrison says-
    • Mild leptospirosis- Doxycycline > Amoxicillin for 7 days.
    • Moderate/severe leptospirosis- Penicillin > Ceftriaxone for 7 days.
    • Chemoprophylaxis- Doxycycline > or Azithromycin for 7 days
  • Park says- Penicillin is DOC, alternate- tetracycline or doxycycline
247MULTIPLE CHOICE QUESTIONS
 
SYPHILIS
1. Which of the following is/are NOT spirochete(s): (PGI Nov 2014)
  1. Borrelia
  2. Fusobacterium
  3. Lactobacillus
  4. Leptospira
  5. Leptotrichia
2. Syphilis was first identified by:
  1. Fraenkel
  2. Nicolaicu
  3. Schaudinn&Hoffman
  4. Ogston
3. Tabes dorsalis is seen in: (NEET Pattern Based)
  1. Primary syphilis
  2. Secondary syphilis
  3. Tertiary syphilis
  4. Latent syphilis
4. In a syphilis patient, site which does not help in isolation of organism: (DNB DEC 2010, JIPMER 2002, NEET Pattern Based)
  1. Gumma
  2. Primary chancre
  3. Mucosal patch
  4. Maculopapular rash
5. Stain for treponema: (NEET Pattern Based)
  1. Fontana's
  2. Acid-fast
  3. Methenamine-silver
  4. PAS
6. Painless genital ulcer in male with everted margin is seen in: (NEET Pattern Based)
  1. Syphilis
  2. Chancroid
  3. Herpes
  4. LGV
7. In a pregnant lady of 8 weeks, allergic to penicillin, VDRL was done which found to be positive. The drug of choice for treatment is: (AI 2012)
  1. Erythromycin
  2. Penicillin
  3. Tetracycline
  4. desensitization
8. False + ve VDRL test is/are seen in: (PGI DEC 2008, JIPMER 2011)
  1. Leprosy
  2. Malaria
  3. Relapsing fever
  4. I.V. drug user
  5. HIV infection
9. Correct combination of incubation period is: (PGI June 2011)
  1. Syphilis: 9 – 90 days
  2. Herpes genitalis : 4 – 5 week
  3. LGV : 3 d – 6 week
  4. Donovanosis: 1 – 4 week
  5. Chancroid: 2 – 3 week
10. A new born premature baby presented with bullous lesions on skin and a shin on knee. X-ray shows periostits. Best investigation for diagnosis is: (AIIMS Nov 2011)
  1. VDRL from mother & baby
  2. PCR for tuberculosis
  3. HBsAg detection
  4. ELISA for HIV
11. In India, syndromic management is applicable for: (AIIMS Nov 2011)
  1. Chancroid and chancre
  2. Chancroid and herpes genitalis
  3. Chancroid, chancre, herpes genitalis
  4. Chancre and herpes genitalis
12. A patient had penile ulcer 2 months back. Now presents with multiple maculopapular rash all over the body and oral ulcers. Which of the following is causative organism? (AI 2011)
  1. Treponema pallidum
  2. LGV
  3. Herpes simplex II
  4. Haemophilus ducreyi
13. VDRL is most sensitive in the diagnosis of which stage of syphilis? (JIPMER 2011)
  1. Primary
  2. Secondary
  3. Tertiary
  4. Reactivation
14. 25 year old laborer 3 years back presented with penile ulcer not treated. Later he presented with neurological symptoms for which he got treated. Test to monitor response to treatment is: (AIIMS Nov 2009)
  1. VDRL
  2. FTA-ABS
  3. TPI
  4. RPR
15. True about primary chancre: (PGI Dec 2008)
  1. Painless ulcer
  2. Painless lymphadenopathy
  3. Covered with exudates
  4. Indurated lesion
  5. Organism can be cultured from exudative fluid
16. A VDRL reactive mother gave birth to an infant. All of the following would help in determining the risk of transmission to the infant except: (AI 2006)
  1. TPHA test on the serum sample of the mother
  2. TPHA test on the serum sample of the infant
  3. VDRL on the paired serum sample of the infant and mother
  4. Time interval between the treatment of the mother and her delivery
17. Spirochetes among following are: (PGI Dec 2006)
  1. Syphilis
  2. Leptospira
  3. Mycoplasma
  4. Brucella
  5. Borrelia
18. 248A 23 year old male had unprotected sexual intercourse with a commercial sex worker. Two weeks later, he developed a painless, indurated ulcer on the glans that exuded clear serum on pressure. Inguinal lymph nodes in both groins were enlarged and not tender. The most appropriate diagnostic test is : (AIIMS May 2004)
  1. Gram's stain of ulcer discharge
  2. Dark field microscopy of ulcer discharge
  3. Giemsa stain of lymph node aspirate
  4. ELISA for HIV infection
19. True about primary chancre: (PGI Dec 2004)
  1. Multiple ulcers
  2. Painless solitary ulcer
  3. Most infective state
  4. Heals automatically in few months without treatment
  5. Penicillin is the drug of choice
20. Investigation of choice for detection of syphilis in a patient after 2 course of complete therapy: (AI 2002)
  1. FTA ABS
  2. VDRL
  3. TPI
  4. Dark ground microscopy
21. True about VDRL test: (AIIMS 2001)
  1. Non –specific
  2. Slide flocculation test
  3. Best followed for drug therapy
  4. All
22. Congenital syphilis can be best diagnosed by: (AI 2001)
  1. IgM FTA – ABS
  2. IgG FTA – ABS
  3. VDRL
  4. TPI
23. All are true about FTA – ABS in Syphilis, except: (AI 2000)
  1. FTA –ABS becomes negative after treatment
  2. Present in secondary syphilis
  3. It is a specific test
  4. May be positive in Lyme's disease
24. Hutchinson's triad a feature of: (TNPG 2014)
  1. Primary Syphilis
  2. Secondary Syphilis
  3. Tertiary Syphilis
  4. Late Congenital Syphilis
 
NON VENEREAL TREPONEMA
25. Bejel is caused by: (NEET Pattern Based)
  1. Trepnema pertenue
  2. Treponema caratenum
  3. Treponema pallidum
  4. Treponema endemicum
26. Causative agent of Yaws is: (NEET Pattern Based) (DNB June 2011)
  1. Trepnema pertenue
  2. Treponema caratenum
  3. Treponema pallidum
  4. Treponema endemicum
27. About yaws all are true except (AI 2008, 2011)
  1. Caused by Treponema pertenue
  2. Transmitted non-venereal
  3. Secondary yaws can involve bones
  4. Last stages involve heart and nerves
28. Non venereal treponemes are: (PGI June 2004)
  1. T. pertenue
  2. T. Carateum
  3. T. pallidum
  4. T. cuniculi
29. True about Yaws: (PGI May 2013)
  1. Sexually transmitted disease
  2. Transmitted by fomites
  3. Mother-child transmission
  4. Periostitis occurs
  5. Caused by T. pallidum subspecies endemicum
 
BORRELIA
30. True about B. recurrentis: (NEET Pattern Based)
  1. Causes leptospirosis
  2. Water borne disease
  3. Transmitted by louse
  4. Transmitted by flea
31. True about Lyme disease except: (AIIMS May 2011)
  1. Borrelia burgdorferi multiplies at the local site and produces pro inflammatory cytokines
  2. Persistent infections occurs despite good Humoral immune response
  3. Polymorphonuclear lymphocytes increase in CSF indicates meningeal involvement
  4. Specific IgA produces intrathecally is diagnostic of Meningitis
32. Lyme's disease is caused by: (DNB Dec 2009, PGI June 2001)
  1. Borrelia parkeri
  2. Borellia burgdoferi
  3. Borrelia recurrentis
  4. Borrelia hermsi
33. All of the following species of Borrelia are associated with tick borne relapsing fever, except: (AI 2009, PGI June 2008)
  1. Borrelia recurrentis
  2. Borrelia hermsii
  3. Borrelia turicatae
  4. Borrelia duttani
34. Which one of the following microorganisms uses antigenic variation as a major means of evading host: (AIIMS Nov 2004)
  1. Streptococcus pneumoniae
  2. Borrelia recurrentis
  3. Mycobacterium tuberculosis
  4. Listeria monocytogenes
35. The following are true regarding Lyme's Disease, except: (AI 2003)
  1. It is transmitted by ixodid tick
  2. Erythema chronicum migrans may be a clinical feature
  3. Borrelia recurrentis is the etiological agent
  4. Rodents act as natural hosts
 
LEPTOSPIRA
36. Leptospirosis is transmitted by: (NEET Pattern Based, DNB June 2009)
  1. Rat
  2. Cat
  3. Dog
  4. Fish
37. 249Weil's disease is caused by? (DNB June 2011)
  1. Leptospira
  2. Plague
  3. Yersinia
  4. Rickettsial Fever
38. A sewer worker comes with high grade fever, neck rigidity and signs of meningismus. Lab findings suggestive of renal failure and elevated liver enzymes. Most appropriate drug? (AI 2011)
  1. Benzyl pencillin
  2. Ciprofloxacin
  3. Doxycycline
  4. Cotrimoxazole
39. Which one of these is true regarding Leptospirosis? (AI 2011)
  1. Rat is the principal animal reservoir.
  2. Transmission is oro-faecal.
  3. Renal and hepatic involvement is seen in half of the affected children.
  4. Fluoroquinolones are the drugs of choice.
40. Which of the following is not used to diagnose Leptospirosis? (AIIMS May 2010)
  1. Microscopic agglutination test
  2. Dark field microscopy
  3. Macroscopic agglutination test
  4. Weil-felix reaction
41. All of the following statements about Leptospirosis are true except: (AI 2009, PGI 2002)
  1. Infection acquired by direct contact with infected urine
  2. Mortality is 5–15% in severe cases
  3. Antibodies are usually detectable in the first week
  4. In penicillin is recommended for treatment of severe cases
42. Culture media of Leptospira: (AIIMS May 2009)
  1. Thorthoff
  2. Perkin
  3. Tinsdale
  4. Baker ‘s
43. Which human infection spreads through urine: (AI 2002, PGI Dec 2006)
  1. Leptospira
  2. Legionella
  3. Plague
  4. Diphtheria
44. A bacterial disease that has been associated with the 3 “R” i.e rats, rice fields, and rainfall is: (AI 2005)
  1. Leptospirosis
  2. Plague
  3. Melioidosis
  4. Rodent – bite fever
45. A 25 year old farmer presented with history of high grade fever for 7 days and altered sensorium for 2days. On examination, he was comatosed and had conjunctival hemorrhage. Urgent investigations showed hemoglobin of 11gm/dl, serum bilirubin 8 mg/dl and urea 78mg/dl. Peripheral blood smear was negative for malarial parasite. What is the most likely diagnosis: (AIIMS Nov 2005)
  1. Brucellosis
  2. Weil's disease
  3. Acute viral hepatitis
  4. Q fever
46. A fourteen year old body is admitted with history of fever, icterus, conjunctival suffusion and haematuria for twenty days. Which of the following serological test can be of diagnostic utility: (AIIMS Nov 2004)
  1. Widal test
  2. Microscopic Agglutination Test
  3. Paul Bunuel test
  4. Weil felix reaction
47. A laborer involved with repair- work of sewers was admitted with fever, jaundice and renal failure. The most appropriate test to diagnose the infection in this patient is: (AI 2003)
  1. Weil Felix test
  2. Paul Bunnel test
  3. Microscopic Agglutination Test
  4. Microimmunofluorescence test
48. Leptospira is a: (Recent Question 2013)
  1. Rickettsia
  2. Chlamydia
  3. Spirochete
49. Which of the following is not a features of Weil's disease: (APPG 2014)
  1. Fever & jaundice
  2. Hepatic encephalitis
  3. Renal failure
  4. Conjunctival hyperaemia and Skin purpura
50. Which of the following infection is transmitted by rat's urine: (TNPG 2014)
  1. Leptospirosis
  2. Brucella
  3. Anthrax
  4. Plague
250EXPLANATIONS
 
SYPHILIS
1. Ans. (b),(c),(e) (Fusobacterium, Lactobacillus, Leptotrichia) Ref: Ananthnarayan 9th/p370
  • Spirochaetes include Treponema, Borrelia and Leptospira
2. Ans. (c) (Schaudinn & Hoffman) Ref: Ananthanarayan 8/e p5
  • Schaudinn & Hoffman were the first to identify Treponema pallidum.
3. Ans. (c) (Tertiary syphilis) Ref: Ananthanarayan 9/e p373, 8/e p374
  • Tabes dorsalis is seen in Tertiary syphilis.
4. Ans. (a) (Gumma) Ref: Ananthanarayan 9/e p372, 8/e p377
  • Demonstration of T.pallidum in gumma is very rare.
5. Ans. (a) (Fontana's) Ref: Ananthanarayan 9/e p372, 8/e p377
  • Sliver impregnation stains like Fontana & Levaditi stains are used for T.pallidum.
6. Ans. (a) (Syphilis) Ref: Ananthanarayan 9/e p372, 8/e p377
  • Primary syphilis is characterized by painless & indurated genital ulcer and hard & indurated lymphnode.
7. Ans. (d) (desensitization) Ref: Harrison 18/e p1108
  • DOC of syphilis in pregnancy- Penicillin
  • DOC of syphilis in pregnancy allergic to Penicillin- desensitization followed by penicillin
  • For detail- refer chapter review
8. Ans. (a) (b) (c) (d) (e) (Leprosy, Malaria, Relapsing fever, I.V. drug user and HIV infection)
Ref: Ananthanarayan 9/e p374, 8/e p375, Jawetz 25/e p303, 24/e p334, Internet sources
  • Biological false positive (BPF) is defined as positive reaction obtained in non-treponemal test with negative treponemal test in absence of past or present treponemal infection.
For detail- refer chapter review
9. Ans. (a) (c) (d) (Syphilis: 9 – 90 days, LGV : 3 d – 6 week, Donovanosis: 1– 4 week)
Ref: Ananthanarayan 9/e p372, 421, 331, 468 & 8/e p333, 373, 396, 420, 470, Harrison 18/e p1108
Feature
Syphilis
Herpes
Chancroid
LGV
Donovanosis
Incubation period
9–90 days
2–7 days
1–14 days
3 days–6 weeks
1–4 weeks (up to 6 months)
10. Ans. (a) (VDRL from mother and baby)
Ref: Review article- Laboratory Diagnosis and Interpretation of Tests for Syphilis, Clin. Microbiol. Rev, Larsen et al. 1995 V8/no1, p/14
  • Clinical clue-Newborn with bullous lesions on skin and a shin on knee with periostits on X ray is pointing towards congenital syphilis.
  • Among the options- only option –a is lab method used for of syphilis. (Other option can easily be ruled out).
  • However, even “VDRL from mother and baby” is not a satisfactory answer. Because- VDRL detects IgG antibodies, hence cannot differentiate between congenital syphilis and maternal transfer. However, there are certain guideline to for screening of congenital syphilis based on VDRL test
  • Screening based on VDRL test-
    • Serial IgG in baby (VDRL)-
      • ✓ If maternal transfer→ titer falls in 3 months,
      • ✓ If Congenital infection → titer rises in 3 months
    • Simultaneous VDRL test of mother and baby-
      • ✓ If mother's titer > baby's titer → Indicates maternal transfer
      • ✓ If baby's titer > mother's titer → I Indicates congenital infection
25111. Ans. (c) (Chancroid, chancre, herpes genitalis) Ref: Harrison 18/e p1108–09
  • Immediate syndrome-based treatment is recommended for the usual causes of acute genital ulcerations like syphilis, chancroid and herpes.
Immediate syndrome-based treatment for acute genital ulcerations at the first visit is often appropriate before all test results become available, because –
  • Early treatment cures early and prevents complication.
  • Early treatment decreases further transmission;
  • Many patients do not return for test results and treatment.
Initial Management of Genital or Perianal Ulcer (Harrison 18/e p1109, Table 130–8)
Usual causes
  • Herpes simplex virus (HSV)
  • Treponema pallidum (primary syphilis)
  • Haemophilus ducreyi (chancroid)
Usual initial laboratory evaluation
  • For syphilis- Dark-field exam Direct FA, or PCR for T. pallidum; RPR, VDRL, or EIA test
  • For HSV- culture, direct FA, ELISA, or PCR; consider HSV-2-specific serology.
  • In chancroid endemic area: PCR or culture for H. ducreyi
Initial Treatment
Herpes confirmed or suspected (history or sign of vesicles):
  • Treat for genital herpes with acyclovir, valacyclovir, or famciclovir
Syphilis confirmed (dark-field, FA, or PCR showing T. pallidum, or RPR reactive):
  • Benzathine penicillin [2.4 million units IM] once to – Patient and partners
12. Ans. (a) (T.pallidum) Ref: Ananthanarayan 9/e p372, 8/e p377
  • History of penile ulcer 2 months back and currently presents with multiple maculopapular rash all over the body and oral ulcers- suggestive of secondary syphilis.
13. Ans. (b) (Secondary syphilis) Ref: Topley Wilson 10th, Ananthanarayan 9/e p375,8/e p377
  • VDRL is 100% sensitive in Secondary syphilis.
14. Ans. (a) (VDRL) Ref: Topley Wilson Ananthanarayan 9/e p374–75,8/e p377
  • History of “present neurological symptoms with penile ulcer 3 years back” is suggestive of tertiary Syphilis.
  • Test to monitor response to treatment for Syphilis – VDRL followed by RPR.
15. Ans. (a), (b), (c), (d) (Painless ulcer, Painless lymphadenopathy, Covered with exudates, Indurated lesion)
Ref: Ananthanarayan 8/e p373, Jawetz 24/e p333 & 25/e p302
  • Clinical features of Syphilis- Refer chapter review.
16. Ans. (a) (TPHA test on the serum sample of the mother)
Ref: Review article- Laboratory Diagnosis and Interpretation of Tests for Syphilis, CMR 1995/V8/no 1, Harrison 18/e p1384
TPHA test on the serum sample of the mother:
  • Indicates mother has syphilis. But it doesn't say about the congenital syphilis. Moreover, TPHA remains elevated even after treatment. So, an adequately treated mother before 4th month of gestation has a very less risk of transmission but can still have an elevated TPHA.
About Other Options
  • Since all the non Treponemal tests like VDRL mainly detects IgG, so they cannot be used to detect congenital syphilis as maternally transferred IgG will be there till 12–18 months of birth. However, VDRL is used for screening in India, and the interpretation should be done as follows…
  • Simultaneous VDRL test of mother and baby-
    • ✓ If mother's titer > baby's titer→ Indicates maternal transfer
    • ✓ If baby's titer > mother's titer →I Indicates congenital infection
  • TPHA test on the serum sample of the infant - suggestive of established congenital syphilis
  • Adequate treatment of the mother before the 16th week of pregnancy should prevent fetal damage.
17. Ans. (a), (b), (e) (Syphilis, Leptospira, Borrelia) Ref: Harrison 17/e p1038 & 18/e p1380 The Spirochaetales include three genera that are pathogenic for humans
  • Leptospira, Borrelia and Treponema.
25218. Ans. (b) (Dark field microscopy of ulcer discharge) Ref: Ananthanarayan 9/e p373, 8/e p374
  • History of unprotected sexual intercourse with a commercial sex worker followed by-
    • ✓ Painless, indurated ulcer on the glans
    • ✓ Enlarged and not tender Inguinal lymph nodes
  • Suggestive of primary Syphilis.
  • Diagnosis of primary Syphilis can be done by Dark field microscopy of ulcer discharge.
  • For further details about laboratory diagnosis of syphilis refer chapter review.
19. Ans. (b), (d), (e) (Painless solitary ulcer, Heals automatically in few months without treatment, Penicillin is the drug of choice) Ref: Harrison 17/e p1044 & 18/e p1387
  • Most infective stage of Syphilis- Secondary syphilis
  • Primary Syphilis is characterized by Single Painless indurated ulcer and hard painless LN
  • Treatment of Syphilis- Refer Chapter review
20. Ans. (b) (VDRL) Ref: Ananthanarayan 9/e p375, 8/e p377
  • For monitoring treatment- VDRL is best followed by RPR
  • VDRL Sero- reversal occurs in primary syphilis in 4 months, in secondary and early latent syphilis in 12–18 months. In late syphilis it might take 5 years.
21. Ans. (d) (All) Ref: Ananthanarayan 9/e p374,8/e p375
VDRL:
  • Slide Flocculation test-
  • Antigen used- Cardiolipin + Lecithin+ Cholesterol
  • Biological false +ve can be seen
  • Prozone phenomena seen
  • Best test to monitor response to treatment
22. Ans. (a)(IgM FTA – ABS)
Ref: Review article- Laboratory Diagnosis and Interpretation of Tests for Syphilis, CMR 1995/V8/no1
  • Explained earlier
  • Non treponemal test (VDRL) detects mainly IgG, So cannot differentiate from passive maternal transfer Method to diagnose congenital syphilis is -A reactive IgM test specific for syphilis
  • 19s IgM FTA-ABS, Captia syphilis M test (IgM ELISA) and Western blot
23. Ans. (a) (FTA –ABS becomes negative after treatment)
Ref: Review article- Laboratory Diagnosis and Interpretation of Tests for Syphilis, CMR 1995/V8/no1
  • Specific Treponemal test for syphilis remain elevated even after treatment, so cannot be used to monitor re- sponse to treatment.
24. Ans. (d) (Late Congenital Syphilis) Ref: Harrison 18th/Chapter 169
  • Hutchinson's triad, which consists of Hutchinson's teeth (notched incisors), keratitis and deafness and occurs in 63% of cases of late congenital syphilis.
25. Ans. (d) (Treponema endemicum) Ref: Ananthanarayan 9/e p377, 8/e p379
  • Bejel or endemic syphilis is caused by Treponema endemicum
26. Ans. (a) (Trepnema pertenue) Ref: Ananthanarayan 9/e p377, 8/e p379, Harrison 18/e p1389, 90
  • Trepnema pertenue is the causative agent of yaws.
27. Ans. (d) (Last stages involve heart and nerves) Ref: Harrison 17/e p1046, 47 & 18/e p1389, 90
  • No CNS/CVS involvement occurs in yaws.
28. Ans. (a), (b), (T. Pertenue, T. Carateum) Ref: Ananthanarayan 9/e p377, 8/e p379
T.cuniculi is pathogenic to rabbit but mostly nonpathogenic to human.
29. Ans. (b) (d) (Transmitted by fomites, Periostitis occurs) Ref: Ananthnarayan 9th /p377
Yaws is caused by T.pertenue, transmitted by non-sexual mode such as direct or indirect contact through fomites/files feeding on to lesions.
 
BORRELIA
25330. Ans. (c) (Transmitted by louse) Ref: Ananthanarayan 9/e p378, 8/e p379
  • Epidemic relapsing fever is transmitted by louse, endemic relapsing fever is transmitted by tick.
31. Ans. (c) (Polymorphonuclear lymphocytes increase in CSF indicates meningeal involvement) Ref: Harrison 18/e p1403
  • Chemokines released by Borrelia burgdorferi infected constituent cells in the skin lead to the recruitment of neutrophils and macrophages
  • Erythema migrans occurs in 1st stage →dissemination (2nd stage) →Persistent infection (3rd stage).
  • Months after the onset of infection, ~60% of untreated patients develop Persistent Infection characterized by oligoarticular arthritis in large joints.
  • White cell counts in joint fluid range from 500 to 110,000/µL (average, 25,000/µL); most of these cells are polymorphonuclear leukocytes
  • However, CNS involvement- CSF shows lymphocytic pleocytosis. Lyme's meningitis:
  • Lymphocytic pleocytosis in CSF
  • Protein increased
  • Slightly decrease in glucose
  • Specific IgA, IgM or IgG appear in CSF and may indicate intratheal antibody synthesis
32. Ans. (b) (Borellia burgdoferi) Ref: Ananthanarayan 9/e p378,8/e p380
  • Lyme's disease is caused by Borellia burgdoferi
33. Ans. (a) (Borrelia recurrentis) Ref: Ananthanarayan 9/e p378, 8/e p379
Refer text (chapter review).
34. Ans. (b) (Borrelia recurrentis) Ref: Ananthanarayan 9/e p378, 8/e p379
  • Borrelia recurrentis shows antigenic variation due to DNA rearrangement in the linear plasmid of Borrelia.
  • This explains the reason for relapse of fever in relapsing fever.
35. Ans. (c) (Borrelia recurrentis is the etiological agent) Ref: Ananthanarayan 9/e p378,8/e p381
  • Borrelia burgdorferi is the causative agent of Lyme's disease
  • Lyme's disease is transmitted by ixodid tick
  • Erythema migrans (annular skin lesions) occurs in 1st stage
  • Rodents act as natural hosts
 
LEPTOSPIRA
36. Ans. (a) (Rat) Ref: Ananthanarayan 9/e p381, 8/e p384, 85
  • Leptospira is transmitted by direct contact with the rodents
37. Ans. (a) (Leptospira) Ref: Ananthanarayan 9/e p381, 8/e p384, 85
  • Weil's disease is caused by- Leptospira
38. Ans. (a) (Benzyl penicillin) Ref: Ananthanarayan 9/e p384, 8/e p384, 85
The history is suggestive of Weil's disease
Points in favor:
  • Sewer worker
  • High grade fever, neck rigidity and signs of meningismus.
  • Lab findings suggestive of renal failure and elevated liver enzymes.
  • DOC for Weil's disease - IV Penicillin
39. Ans. (a) (Rat is the principal animal reservoir) Ref: Ananthanarayan 9/e p383, 8/e p384, 85
  • Transmission is – contact with animal
  • Renal and hepatic involvement is seen in 10% of the affected cases
  • Penicillin- DOC in severe cases, doxycycline is DOC in milder cases.
25440. Ans. (d) (Weil felix test) Ref: Harrison 17/e p1050 & 18/e p1395
  • Weil felix test is used for diagnosis of rickettsiosis.
Diagnosis of Leptospirosis- Refer chapter review.
41. Ans. (c) (Antibodies are usually detectable in the first week) Ref: Harrison 17/e p1048, 51 & 18/e p1392, 96
  • Antibodies generally do not reach detectable levels until the second week of illness.
About Other Options
  • Transmission of Leptospira to humans may follow direct contact with urine, blood, or tissue from an infected animal or exposure to a contaminated environment.
  • Mortality rates in Anicteric Leptospirosis are low but in Weil's syndrome, the most severe form of Leptospirosis, the mortality rates is 5–15%
  • Mild Leptospirosis, Oral- Doxycycline is given, in severe cases, Penicillin is indicated.
42. Ans. (a) (Korthoff) Ref: Harrison 17/e p1050 & 18/e p1395
Culture media for Leptospira- EMJH (Ellinghausen-McCullough-Johnson-Harris), Korthoff and Fletcher media
43. Ans. (a) (Leptospira) Ref: Harrison 17/p1048 & 18/e p1395
  • Transmission of Leptospira to humans may follow direct contact with urine, blood, or tissue from an infected animal or exposure to a contaminated environment;
  • Human-to-human transmission is rare.
  • Legionella antigen detection test is done in urine sample but it spreads through inhalational route
44. Ans. (a) (Leptospirosis) Ref: Harrison 17/e p1049 & 18/e p1394 Leptospirosis
  • More common in tropics
  • During the summer (Western) and during the rainy season in the tropics
  • Rodents, especially rats, are the most important reservoir
  • High risk occupational exposures include- veterinarians, agricultural workers, sewage workers, slaughterhouse employees, and workers in the fishing industry
45. Ans. (b) (Weil's disease) Ref: Harrison 17/e p1049, 50 & 18/e p1394, 94
  • Farmer with high grade fever, altered sensorium and conjunctival hemorrhage with hepatic (raised bilirubin) and renal (raised creatinine) involvement is suggestive of Leptospirosis with hepatorenal syndrome (Weil's disease)
  • Clinical Stage of Leptospirosis- Refer chapter review.
46. Ans. (b) (Microscopic Agglutination Test) Ref: Harrison 17/e p1050 & 18/e p1395
  • History of fever, icterus, conjunctival suffusion and haematuria is suggestive of Leptospirosis with hepatorenal syndrome (Weil's disease)
  • The Gold Standard test for Leptospirosis - Microscopic Agglutination Test
47. Ans. (c) (Microscopic Agglutination Test) Ref: Harrison 17/e p1050 & 18/e p1395
  • Sewer worker with fever, jaundice and renal failure is suggestive of Leptospirosis with hepatorenal syndrome (Weil's disease)
  • The Gold Standard test for Leptospirosis - Microscopic Agglutination Test
48. Ans. (c) (Spirochete) Ref: Ananthanarayan 9/e p370 Leptospira, Borrelia and Treponema are e.g. of spirochetes.
49. Ans. (b) (Hepatic encephalitis) Ref: Harrison 18th /p1393–96
  • Weil's disease is characterized by - jaundice, acute kidney injury, hypotension, and haemorrhage (MC site- lungs).
  • Other syndromes include aseptic meningitis with ↑PMNs, uveitis, cholecystitis, acute abdomen, pancreatitis (with hypo- or hyperglycemia), hypotension, hepato-splenomegaly and electrolyte abnormalities (Hypokalemia, hypomagnesemia and hypomagnesemia).
50. Ans. (a) (Leptospirosis) Ref: Ananthnarayan 9th/383 Refer chapter review for detail.

Rickettsia, Chlamydia and MycoplasmaCHAPTER 3.11

 
RICKETTSIA
  • Obligate intracellular pathogen
  • Transmitted by arthropod – except Coxiella
  • Not cultivable in cell free media- except Bartonella
Species
Disease
Vector
Distribution
Rash
Eschar
LN
Weil Felix
R. prowazekii
Epidemic Typhus
Brill Zinsser
Louse (Rubbing) Code-(LET)
World wide
80%
-
-
Ox19 ++++
Ox2 +/-
-ve or weak +ve
R. typhi
Endemic Typhus
Flea Code- (FEN)
World wide
80%
-
-
Ox19 ++++
Ox2 +/-
R. rickettsii
Rocky mountain spotted fever (RMS)
Tick
Code- (TRAI)
America
90%
<1%
+
Ox19 ++
Ox2++
R.conori
Indian tick typhus (ITT)
Tick
Europe, Asia
97%
50%
+
R.africae
African Tick bite fever
Tick
Sub-Saharan Africa
50%
(Vesicular)
90%
++++
R. akari
Rickettsial Pox
Mite (gamasid)
Code- (PSM)
USA, Ukraine, Turkey, Mexico,
100%
(Vesicular)
90%
+++
All -ve
Orientia tsutsugumashi
Scrub typhus
Mite (trombiculid)
Asia, Australia,
50%
35%
+++
OxK+++
 
Points to Remember
  • Most severe form- with systemic involvement- Rocky mountain spotted fever
  • Mildest form- R pox
  • Vesicular /vericelliform rash seen- Rickettsial pox
  • Transovarial transmission occurs only in tick and mite
  • Chronic form- seen in Q fever (endocarditis)
  • R.prowazekii- recrudescent illness (Brill-Zinsser disease)
Distribution of Rash
  • Epidemic (all over except palm and sole)
  • Endemic (trunk followed by extremities)
  • RMS, ITT (palm and sole),
  • No rash- Q fever
 
Lab Diagnosis
  • Isolation – Mice inoculation, Egg inoculation (yolk sac) and cell cultures
  • Direct detection of the organisms and their antigens - Giminez, Machiavello and Giemsa stain
  • Neil mooser Reaction -Male guinea pig intraperitoneal inoculation leads to-
    • Rocky mountain spotted fever- Produces scrotal necrosis
    • Epidemic typhus- Produces only fever, but negative tunica reaction
    • R.typhi/conori/akari (CAT) – Produces fever and positive tunica reaction (testicular inflammation)
  • Serology - Weil Felix reaction
    • Cross reacting alkali stable polysaccharide Ag in nonmotile strains of P.vulgaris (OX2, X19) and P.mirabilis (OXK)
    • Rise in titer is more meaningful
    • False +ve if Proteus infection present and false negative due to prozone phenomena
  • 256Indirect immunofluorescence assay-Most common serologic test for confirmation of the diagnosis.
Weil Felix
OX2
OX19
OXK
Epidemic typhus
+
++++
-
Endemic Typhus
+
++++
-
RMS Fever
+ to ++++
++++
-
Scrub typhus
-
-
+++
 
Scrub Typhus
  • Agent- Orientia tsutsugamushi (it lacks LPS)
  • Transmitted by- Trombiculid Mite
    • Leptotrombidium akamushi in Japan
    • Leptotrombidium deliensis in India
  • Larval mites (chiggers)- the only stage that feeds man
  • Transovarial transmission seen
  • Affect rats in deserts, rain forest
  • Eschar at site, rash, LN↑
  • ↑OXK titer in Weil Felix
  • Three antigenic type-Karp, Gilliam, Kato
 
Coxiella Burnetii
  • Agent of Q fever (Query fever)
  • Transmitted without arthropod vector (respiratory mode) and also Zoonotic (wild animal)
  • It survives holders method of pasteurization of milk but killed by flash back method
  • No skin rash in Q fever.
  • Survives in macrophages by-
    • Inhibiting the final phagosome maturation step (cathepsin fusion)
    • Has adapted to the acidic phagolysosome by producing superoxide dismutase.
Clinical feature
  • Acute Q fever-
    • Hepatitis
    • Interstitial pneumonia, shows latency
    • Isolated fever (14%),
    • CNS involvement and pericarditis or myocarditis
  • Post–Q Fever Fatigue Syndrome
  • Chronic Q Fever- endocarditis
    • In patients with previous valvular heart disease, immuno-suppression
    • Endothelium-covered nodules on the valves
 
Ehrlichia
Ehrlichiosis
E. sennetsu
E. chaffeensis
E.phagocytophila
Causes-
Human Lymphocytic Ehrlichiosis
Human Monocytic Ehrlichiosis
Human Granulocytic Ehrlichiosis
Feature
Atypical lymphocytosis Lymphoid hyperplasia
Leucopenia Thrombocytopenia
Leucopenia Thrombocytopenia
Transmitted by
Ingestion of fish carrying infected flukes
Amblyomma Tick
Ixodes Tick
Common to all three Species-
Obligate intracellular parasite, Cannot be cultivated in artificial media
Grow in cluster inside the phagosome as mulberry like inclusions called as MORULA DOC- Doxycycline
257Treatment of rickettsial diseases:
  • Doxycycline is the drug of choice
  • Addition of hydroxychloroquine (to alkalinize the phagolysosome) renders doxy- cycline bactericidal against C. burnetii
Bartonella (infect RBC):
  • B.bacilliformis:
    • Causes bartonellosis (Carrión's Disease) transmitted by sandfly Lutzomyia verrucarum
    • Two stages- Oroya fever (early, bacteremic) and Verruga peruana (late),
  • B. quintana – Louse borne:
    • Trench fever or 5 day fever
    • Bacillary angiomatosis (epithelioid angiomatosis) in HIV patients- subcuta- neous and lytic bone lesions
    • Bartonella Endocarditis
  • B. hensalae:
    • Cat scratch disease (rarely caused by Afipia felis or B.quintana):
    • Typical from (90%)-Lymphadenopathy (MC - axillary/epitrochlear nodes) & painless pustule
    • Atypical from (10%)- Extra nodal manifestations involving various organs
    • Bacillary angiomatosis in HIV patients- cutaneous lesions; hepatosplenic lesions
    • Bacillary peliosis -Angioproliferative disorder of liver > spleen and lymph nodes
Treatment:
  • Typical Cat scratch diseases-Azithromycin
  • Atypical Cat scratch diseases-Doxycycline
  • Trench fever-Gentamicin + Doxycycline
  • Bacillary angiomatosis peliosis- Erythromycin
  • Oroya fever- Chloramphenicol or Ciprofloxacin
  • Verruga peruana- Rifampin or Streptomycin
 
CHLAMYDIA
General properties:
  • Obligate intracellular gram negative bacteria.
  • Possess modified peptidoglycans
  • Posses both RNA and DNA.
  • Can't produce their own ATP (Energy parasite) Shows tropism for squamous epithelium and LN
Species
Character
Serotype
Disease
C.trachomatis
  • Form compact inclusions
  • with glycogen matrix
  • Sensitive to sulfonamide
  • Natural parasite of human
  • Leave the host cell with a scar
Biovar TRIC -A, B, Ba, C
  • Trachoma
Biovar TRIC - D-K
  • Inclusion conjunctivitis
  • infant pneumonia
  • Genital chlamydiasis
Biovar LGV - L1,L2,L3
  • LGV(lymphogranuloma venerum)
C. psittaci
  • Form diffuse vacuolated inclusions without glycogen matrix
  • Resistant to sulfonamide
  • Natural parasite of birds
  • Leave host cell by lysis
Many
  • Psittacosis (Atypical Pneumonia)
  • Inhalational route from birds
  • Zoonotic- from pet birds (parrots) and poultry (turkeys and ducks)
  • No man to man transmission
  • Diagnosis- By CFT and Micro- IF (gold standard serologic tests)
258
C.pneumoniae TWAR agent
  • Exclusively human pathogen
  • inclusions without glycogen matrix
  • Resistant to sulfonamide
Only 1
  • Community-acquired atypical pneumonia
  • Associated with
    • ✓ Atherosclerosis,
    • ✓ Asthma,
    • ✓ Sarcoidosis
  • Diagnosis- serology, culture (Hep2 cell line),PCR
 
Life Cycle
  • Divide by binary fission without eclipse phase
  • Takes 24–48 hr
Elementary body
Reticulate body
Extracellular form
Intracellular form
Infectious form
Replicating form
Metabolically inactive
Metabolically active
Rigid cell wall
fragile cell wall
Small size
Large size
Electron dense nucleoid,
No electron dense nucleoid
DNA=RNA
RNA> DNA
Antigen:
  • LPS- genus specific, used for CFT
  • Envelop surface Antigen- species specific,
  • Major OMP- Serovar serotype specific- used for micro-IF test
Inclusion body:
  • LCL body (Levinthal-Cole-Lillie) body- Psittacosis
  • Miyagawa corpuscle- LGV
  • HP (Halberstaedter – Prowazek) body- trachoma
 
Diagnosis of Chlamydial Infection
Microscopy:
  • Stained- by Castaneda, Machiavello, Giminez stains
  • Lugol's I2- used only for C.trachomatis (stains the glycogen inclusion body)
  • IF using monoclonal Antibody- ↑Sensitivity, detect both inclusion and elementary body
Culture:
  • Mice (infective by only C.psittaci and LGV biovars)
  • Yolk sac, cell culture (McCoy Cells and HeLa)
  • Hep2 for C.pneumoniae
Serology:
  • Micro-IF (serovar specific)- Serology of choice
  • CFT (genus specific)
  • High titer seen in – LGV, infant pneumonia, salpingitis
Nucleicacidamplificationtests(NAATs)-PCR:Diagnosisofchoice, moreSensitiveandspecific
 
Treatment of Chlamydial Infections
  • C. trachomatis- Azithromycin single dose > Doxycycline for 7 days
  • C. psittaci- tetracycline > Erythromycin for 714 days
  • C. pneumoniae- tetracycline or erythromycin for 14 days
 
259C. Trachomatis
  • MC cause of STD, nongonococcal urethritis (NGU) and post-gonococcal urethritis (PGU),
  • MC cause of Pelvic Inflammatory Disease(PID) and acute epididymitis
  • Ocular manifestation:-Swimming pool conjunctivitis and ophthalmic neonatorum
  • Complications-
    • Reiter syndrome-
      • CUP-(conjunctivitis +urethritis+ polyarthritis) and mucocutaneous lesions
      • ✓ Initiate an aberrant hyperactive immune response that produces inflam- mation of the involved target organs in these genetically predisposed individuals (HLAB27 +ve)
      • ✓ Associated with gastroenteritis (Salmonella, Shigella, Yersinia, Campy- lobacter), N.gonorrhoeae
      • Fitz Hugh Curtis syndrome- perihepatitis in, sexually active women
    • Urethral Syndrome in Women- dysuria and frequency, urethritis, pyuria, and no bacteriuria
Inclusion conjunctivitis – neonate (inclusion blenorrhea), adult (swimming pool conjunctivitis)
 
Trachoma
  • Chronic conjunctivitis- (follicular hypertrophy + papillary hyperplasia + pannus + cicatrization)
  • Stages: Trachoma dubium, protrachoma, established trachoma I –IV
  • Inclusion body (HP- Halberstaedter - Prowazek) seen only in, established trachoma stages I-IV
 
LGV
  • MC Serotype-by L2>L1, L3
  • LGV serovars are more invasive than others
  • Incidence is falling, Male: female:3.4:1,
  • Clinical feature- Pain less ulcer + painful LN↑
  • Esthiomone (elephantiasis of vulva) rectal stricture, proctitis
  • Skin test → Frie test
 
Non Gonococcal Urethritis (NGU)
  • Chlamydia trachomatis (MC)
  • Mycoplasma genitalium & hominis
  • Ureoplasma urealyticum
  • Herpes Simplex virus
  • Cytomegalovirus
  • Trichomonas vaginalis
  • Candida albicans
 
MYCOPLASMA
 
General Properties
  • Smallest free living organism known
  • Filterable (Hence known as Eaton's agent)
  • PPLO- Pleuro pneumonia like organism
  • Flagella, pili –absent but some strains show gliding mobility
  • Lack rigid cell wall. Need cholesterol as a growth factor
  • Resistance to cell wall–active antimicrobial agents.
 
Cinical Feature of M.pneumoniae
  • Incubation period -is 2–4 weeks, person to person spread by respiratory droplets
  • 260MC manifestation-upper respiratory illness
  • MC cause of community-acquired atypical pneumonia in adults
  • Pneumonia is called as Primary atypical pneumonia (PAP) or “walking” pneumonia or Eaton agent pneumonia
  • Extrapulmonary Manifestations-neurologic, dermatologic, cardiac, rheumatologic, and hematologic.
 
Lab Diagnosis
  • Poorly gram negative Shows Pleomorphism, Resemble like L forms
  • Stain with Dienes stain- Dines Method is followed to demonstrate Mycoplasma colonies under microscope. Block of agar containing Mycoplasma colony added to meth- ylene blue is observed under microscope.
  • Produces Fried egg colonies
  • Culture medium -PPLO broth (bovine heart infusion broth + 20% horse serum + 10% fresh yeast extract)
  • Detection of antigen -Immunfluroscence
  • PCR- More sensitive while culture is more specific
Detection of Antibody:
  • Heterophile antibody:
    • Cold agglutination test- detects Mycoplasma antibody by using human ‘O’ RBC antigen
    • Streptococcus MG test- detects Mycoplasma antibody by using Streptococ- cus MG antigen
  • Specific antibody:
    • CFT (complement fixation test)
    • ELISA for IgM, IgG & IgA - Acute- and convalescent-phase serum samples are recommended.
  • The combination of PCR of respiratory tract secretions and serologic testing constitutes the most sensitive and rapid approach to the diagnosis of M. pneumoniae infection.
  • Ureoplasma urealyticum→ cause NGU, epididymitis, Vaginitis and cervicitis
 
Treatment
  • Mycoplasma pneumoniae & Ureoplasma urealyticum :-DOC- Azithromycin
  • M. hominis – DOC:-Doxycycline
Characters
Chlamydia
Rickettsia
Mycoplasma
Intracellular
Yes
Yes
No, free living
Make ATP
No ATP
Limited ATP
Normal ATP
Peptidoglycan layer in cell envelop
Modified Peptidoglycan
Normal Peptidoglycan
No Peptidoglycan
Growth on artificial media
No
No
Yes
261MULTIPLE CHOICE QUESTIONS
 
RICKETTSIA
1. Which is transmitted by louse: (PGI Nov2012)
  1. Endemic typhus
  2. Scrub typhus
  3. Trench fever
  4. Q fever
  5. Relapsing fever
2. Which of the following disease is less likely to cause epidemic soon following disaster? [AIMS Nov 2013]
  1. Acute respiratory infection
  2. Rickettsiosis
  3. Leptospirosis
  4. Visceral leishmaniasis
3. Well felix reaction for Scrub typhus shows positivity for: (NEET Pattern Based)
  1. OXTh
  2. OXTh & OXI9
  3. OX-2
  4. OX-19
4. All are true about scrub typhus except? (AI 2010)
  1. Mite is vector
  2. Adult mite feeds on vertebral host
  3. Caused by Orientia tsutsugamushi
  4. Tetracycline is DOC
5. Disease caused by both, Rickettsia and Orientia is transmitted by: (PGI June 2011)
  1. Rat flea
  2. Tick
  3. Louse
  4. Trombiculid mite
  5. Gamasid mite
6. Which of the following gives positive test with both Weil felix) OX 2and OX 19? (DNB Dec 2011)
  1. Spotted fever
  2. Scrub typhus
  3. Epidemic typhus
  4. None of the above
7. Neil mosser reaction or tunica reaction is useful to differentiate between: (DNB Dec 2010)
  1. R. prowazekii and R. typhi
  2. R. typhi and R. rickettsii
  3. R. prowazekii and R. rickettssii
  4. R. rickettsia and C. burnetti
8. Q fever is caused by: (DNB June 2010)
  1. Rickettsia typhi
  2. Coxiella burnettii
  3. Salmonella
  4. Escherichia coli
9. Mite is a vector for : (PGI June 2008)
  1. R. typhi
  2. R.prowazeki
  3. R. rickettsii
  4. R. tsutsugamushi
  5. R.conori
10. Scrub typhus transmitted by: (AIIMS Nov 2007), DNB 2006, RJ 2000)
  1. Reduviid bug
  2. Trombiculid mite
  3. Enteric pathogens
  4. Cyclops
11. Which of the following species belong(s) to the genus Rickettsia? (PGI Dec 2007)
  1. Rickettsia tsutsugamushi
  2. Rochalimaea quintana
  3. Rickettsia prowazekii
  4. Rickettsia Typhi
12. A patient complained of chills and fever following a louse bite 2 weeks before. He had rashes all over the body and was delirious at the time of presentation to the hospital and subsequently went into coma. A provisional diagnosis of vasculitis due to Rickettsial infection was made. Which one of the following can be causative agent: (AI 2006)
  1. Rickettsia typhi
  2. Rickettsia rickettsii
  3. Rickettsia prowazekii
  4. Rickettsia akari
13. It is true regarding endemic typhus that: (AIIMS 2006, AI 2003, AI 2004)
  1. Man is the only reservoir of infection
  2. Flea is a vector for the disease
  3. The rash developing into eschar is a characteristic presentation
  4. Culture of the etiological agent in tissue culture is a diagnostic
14. An army jawan posted in remote forest area following a tick bite had fever and headache. His fever was 104 F and pulse was 70 per min. He had an erythematous lesion of about 1 cm on the leg surrounded by small vesicles, along with generalized lymphadenopathy at the time of presentation to the referral hospital. His blood sample was collected to perform serology for the diagnosis of Rickettsial disease. Which one of the following results in Weil-felix reaction will be diagnostic in this clinical setting: (AI 2005)
  1. High OX -2
  2. High OX -19
  3. High OX – Th
  4. High OX – 19 and OX – 2
15. The following is the etiological agent of Rocky mountain spotted fever: (AIIMS May 2005)
  1. Rickettsia rickettsii
  2. Rochalimaea quintana
  3. Rickettsia tsutsugamushi
  4. Coxiella burnetii
16. About Rickettsia, all are true except: (SGPGI 2005)
  1. These are transmitted by arthropod vectors
  2. Eschar is not seen in RMSF
  3. 262Weil-Felix reaction may be diagnostic
  4. Cephalosporins are drugs of choice
17. Mode of transmission of Q fever: (AIIMS May 2004)
  1. Bite of infected louse
  2. Bite of infected tick
  3. Inhalation of aerosol
  4. Bite of infected mice
18. All of the following statement are true regarding Q fever except: (AI 2003, AIIMS May 2003, SGPGI 2005)
  1. It is zoonotic disease
  2. Human disease is characterized by an interstitial pneumonia
  3. No rash is seen
  4. Weil–felix reaction is very useful for diagnosis
19. A man with very high fever shows strongly positive agglutination test with OXTh antigen. The most likely diagnosis is: (AI 2000)
  1. Tsutsugamushi fever
  2. Trench fever
  3. Undulant fever
  4. Relapsing fever
20. A lady recently returned from USA, presented with fever and rashes, 1 week later of exposure to tick? (Recent Question 2013)
  1. Epidemic typhus
  2. Scrub typhus
  3. RMS
  4. Oraya fever
21. Not true about scrub typhus: (PGI Nov 2012)
  1. Chigger-borne
  2. Vector is mite
  3. Caused by Orientiatsutsugamushi
  4. Chlamydial disease
  5. Has 3 serotypes
 
BARTONELLOSIS
22. Bartonella henselae causes all except: (NEET Pattern Based)
  1. Oroya fever
  2. Cat scratch disease
  3. Bacillary angiomatosis
  4. SABE
23. All are true about B. quintana except: (NEET Pattern Based)
  1. Causes trench fever
  2. Not detected by Weil felix reaction
  3. Recurrence is common
  4. Tick is the vector
24. Disease(s) caused by cat licking or bite is/are: (PGI May 2012)
  1. Bartonella hansellae
  2. Bartonella bacilliformis
  3. Pasteurella multocida
  4. Bartonella vinsoni
  5. Yersina pestis
25. Bacillary Angiomatosis occur due to: (PGI May 2012)
  1. Mycoplasma
  2. Gardenella
  3. Bartonella bacilliformis
  4. Hemophilus influenza
  5. No relation with any above mentioned
26. Cat scratch disease is: (PGI 2000)
  1. Associated with positive Frie skin test
  2. Caused by a DNA virus
  3. Associated with a pathognomonic histological picture
  4. Associated with regional lymphadenopathy
  5. Associated with a positive Weil felix test
27. A patient has presented with a macule in hand followed by axillary lymphadenopathy. What will you ask in the history to the patient? (Recent Question 2013)
  1. History of exposure to rat
  2. History of exposure to cat
  3. History of similar complain in family members
28. Ehrlichia chaffeensis is causative agent of: (NEET Pattern Based)
  1. HME
  2. HE
  3. Glandular fever
  4. None
29. Verrugaperuana caused by: (PGI Nov 2014)
  1. Bartonellabacilliformis
  2. Bartonellaquintana
  3. Bartonellahensale
30. Oroya fever is caused by: (PGI Nov 2014)
  1. Bartonellahenselae
  2. Bartonellabaciliformis
  3. Bartonella Quintana
 
CHLAMYDIA
31. True about chlamydia are all except: (NEET Pattern Based)
  1. Obligate intracellular organism
  2. Gram positive
  3. Reticulate body is metabolically active
  4. Replicated by biniary fission
32. Inclusion body is seen in: (NEET Pattern Based)
  1. Rickettsiae
  2. Chlamydia
  3. Mycoplasma
  4. H. Pylori
33. In a patient with urethral syndrome, urine microscopy shows full of polymorph, but no bacteria. The most appropriate culture medium is? (AIIMS May 2012) (AI 2012, AIIMS Nov 2011, 2006)
  1. Mc'coy cell
  2. Thayer martin
  3. Cooked meat
  4. PPLO broth
34. Prophylaxis with Azithromycin is given in cases of trachoma when prevalence in a community is: (AIIMS Nov 2011)
  1. 4%
  2. 6%
  3. 8 %
  4. 10%
35. Chlamydia escape killing by? (DNB June 2011)
  1. Causes cell membrane perforation
  2. Produces factors that Camouflage it
  3. Molecular mimicry
  4. Inhibit phagolysosome fusion
36. 263Which of the following is true about Chlamydia trachomatis? (AI 2011)
  1. It is usually symptomatic
  2. It is routinely treated with penicillin
  3. It is diagnosed by culture of cervical purulent discharge
  4. Women using oral pills are more prone for infection
37. Rieters disease is associated with: (JIPMER 2010)
  1. Pseudomonas aeruginosa
  2. Staph aureus
  3. Strept pyogenes
  4. Chlamydia
38. All of the following statements about Chlamydia trachomatis are true except: (AI 2009)
  1. Genital chlamydial infections are often asymptomatic
  2. Can be cultured
  3. Inclusion conjunctivitis is caused by C. Trachomatis serotypes D-Th
  4. Penicillin is the treatment of choice
39. Chlaymydia in asymptomatic carriers, the most sensitive test is: (AIIMS Nov 2009, AI 2004)
  1. Tissue culture
  2. Nucleic acid amplification test
  3. Serology
  4. Serum electrophoresis
40. Chlamydia psittacosis all are true except: (AI 2007)
  1. Acquired from bird's droppings
  2. Causes urethritis
  3. Causes pneumonia
  4. Treatment is tetracycline
41. Triad of Reiter's syndrome: (PGI Dec 2007)
  1. Conjunctivitis
  2. Uveitis
  3. Mucosal lesions
  4. Glaucoma
42. Chlamydia trachomatis false is: (AIIMS Nov 2006, AI 2007)
  1. Elementary body is metabolically active
  2. It is biphasic
  3. Reticulate body divides by binary fission
  4. Inside the cell it evades phagolysozome fusion
43. The following statements are true regarding Chlamydia except: (AIIMS May 2005)
  1. Erythromycin is effective for therapy of Chlamydial infections
  2. Their cell wall lacks a Peptidoglycan layer
  3. They can grow in cell free culture media
  4. They are obligate intracellular bacteria
44. Which one of the following statements is true regarding Chlamydia pneumoniae: (AI 2005)
  1. Fifteen serovars have been identified as human pathogens
  2. Mode of transmission is by the airborne bird excreta
  3. The Cytoplasmic inclusions present in the sputum specimen are rich in glycogen
  4. The group specific antigen is responsible for the production of complement fixing antibodies
45. Chlamydia trachomatis is associated with the following except: (AI 2005)
  1. Endemic trachoma
  2. Inclusion conjunctivitis
  3. Lymphogranuloma venereum
  4. Community acquired pneumonia
46. The following is not a method of isolation of Chlamydia from clinical specimens: (AIIMS Nov 2005)
  1. Yolk sac inoculation
  2. Enzyme immunoassay
  3. Tissue culture using irradiated McCoy cells
  4. Tissue culture using irradiated BHTh cells
47. Which of the following is used for isolation of Chlamydia: (AI 2001)
  1. Microimmunofluorescence
  2. Yolk sac inoculation
  3. ELISA
  4. Immunfluroscence
48. Chlamydia grows in which of the following cell lines: (PGI Dec 2001)
  1. HeLa
  2. HeP2
  3. McCoy
  4. Human diploid fibroblast series
  5. Vero cells
49. Chlamydia causes: (PGI Dec 2000, PGI Dec 2005)
  1. Infertility
  2. Pneumothorax
  3. Pelvic inflammatory disease
  4. Congenital Malformation in fetus
50. A 32 year female is presented with vaginal discharge resembling chlamydial urethritis. Most sensitive test for diagnosing this condition is? (AIIMS Nov 2014)
  1. Nucleic acid amplification test
  2. Gram stain
  3. Culture on McCoy cell line
  4. Serum antibody detection by MIF
51. DOC of Chlamydia in pregnancy? (Recent Question 2013)
  1. Tetracycline
  2. Doxycycline
  3. Amoxycillin
  4. Metronidazole
52. Which of the Chlamydia species is having one serotype? (Recent Question 2013)
  1. Chlamydia psittaccii
  2. Chlamydia trachomatis
  3. Chlamydia pneumophilla
  4. All of the above
 
MYCOPLASMA
53. Diene's method is used for: (NEET Pattern Based)
  1. Mycoplasma
  2. Chlamydiae
  3. Plague
  4. Diphtheria
54. Ureaplasma is naturally resistant to: (NEET Pattern Based)
  1. Erythromycin
  2. Chloramphenicol
  3. Cephalosporins
  4. Tetracyclines
55. Fried Egg colonies are produced by: (DNB Dec 2010)
  1. Chlamydia
  2. Mycoplasma
  3. Niesseria
  4. Diphtheria
56. 264Which is Eaton agent? (JIPMER 2011)
  1. Mycoplasma
  2. H.influenzae
  3. Thlebsiella
  4. Chlamydia pneumoniae
57. The following statements are true with reference to Mycoplasma except: (AIIMS Nov 2005)
  1. They are all smallest prokaryotic organisms that can grow in cell free culture media
  2. They are obligate intracellular organisms
  3. They lack a cell wall
  4. They are resistant to Beta-lactam drugs
58. In reference to the Mycoplasma, the following are true except: (AIIMS May 2005)
  1. They are inhibited by penicillin
  2. They can reproduce in cell free media
  3. They have an affinity for mammalian cell membranes
  4. They can pass through filters of 450nm pore size
59. All are features of Ureoplasma urealyticum except: (AI 2001)
  1. Non gonococcal urethritis
  2. Salpingitis
  3. Epididymitis
  4. Bacterial vaginosis
265EXPLANATIONS
1. Ans: (c), (e) (trench fever, Relapsing fever) Ref: Apurba Sastry's Essentials of Medical Parasitology,p/315, Park 21st/ p708
Louse borne diseases are: Epidemic typhus, Epidemic Relapsing fever, Trench fever and Pediculosis.
2. Ans. (d) (Visceral leishmaniasis) Ref: Park 22/e p742
  • Among the options, Visceral leishmaniasis is the best answer because- “in post disaster period, vector borne diseases will not appear immediately, but may take several weeks to reach epidemic levels.”
Communicable diseases that occur in post disaster period:
  • Outbreaks of gastroenteritis (MC disease to occur due to overcrowding, poor sanitation, population displacement and contamination of water supply
  • Acute respiratory infection- occurs commonly due to population displacement
  • Increase risk of zoonoses due to displacement of domestic animals (dogs & cats may be reservoir of infections like leptospirosis & rickettsiosis) and wild animals (that may lead to rabies, equine encephalitis).
  • Vector borne diseases will not appear immediately, but may take several weeks to reach epidemic levels.
3. Ans. (a) (OXK) Ref: Ananthanarayan 9/e p408
  • In Scrub typhus, Well felix reaction shows a raise in OXK titre.
4. Ans. (b) (Adult mite feeds on vertebral host) Ref: Ananthanarayan 9/e p408
  • Mite is vector for scrub typhus
  • Larval form of mite- i.e chiggers feed on man, other forms of mite including adult form don't feed man.
  • Scrub typhus Caused by Orientia tsutsugamushi
  • Tetracycline is DOC
5. Ans. (a) (b) (c) (d) (e) (Rat flea, Tick, Louse, Trombiculid mite, Gamasid mite)
Ref: Ananthanarayan 9/e p406, 8/e p406, Harrison 18/e p 1408, 17/e p1060
  • The various groups of Rickettsia and Orientia both are transmitted by Rat flea, Tick, Louse, Trombiculid, mite Gamasid mite.
  • For detail- refer chapter review
6. Ans. (a) (Spotted fever) Ref: Ananthanarayan 9/e p410, 8/e p410
Weil felix reaction:
  • Antibodies to OX19- Epidemic and endemic typhus
  • Antibodies to OX 2 and OX19- Rocky mountain spotted fever
  • Antibodies to OX K- Scrub typhus
7. Ans. (a) (R. prowazekii and R. typhi) Ref: Ananthanarayan 9/e p409, 8/e p409
  • Neilmooser reaction is done to differentiate epidemic typhus (R. Prowazekii) which gives a negative tunica reaction and endemic typhus (R. typhi) which gives a positive tunica reaction.
  • For detail- refer chapter review
8. Ans.(b) (Coxiella burnetii) Ref: Ananthanarayan 9/e p411, 8/e p410
  • Q fever is caused by Coxiella burnetii.
9. Ans. (d) (R. tsutsugamushi) Ref: Harrison 17/e p1060 & 18/e p1408
  • Mite transmits- Rickettsial Pox and Scrub Typhus
  • Agent for Rickettsial Pox- R.akari
  • Agent for Scrub Typhus – Orientia tsutsugamushi (old name- R. tsutsugamushi)
26610. Ans. (b) (Trombiculid mite) Ref: Harrison 17/e p1060 & 18/e p1408
  • Rickettsial Pox - Vector is gamasid mite
  • Scrub Typhus-Vector is Trombiculid mite
11. Ans. (a), (c), (d) (Rickettsia tsutsugamushi, Rickettsia prowazekii, Rickettsia Typhi)
Ref: Harrison 17/e p1059, 60 & 18/e p1407, 08
  • Rickettsia are characterized by-
    • ✓ Obligate intracellular parasite
    • ✓ Cannot be cultivated in artificial cell free media
    • ✓ Transmitted by arthropods
  • Bartonella quintana (R.quintana) (can be cultured in blood agar) and Coxiella (No vector, but transmitted by droplet) are recently excluded in Rickettsia. ………………..Ananthanarayan 9/e p405,8/e p405
  • R.Quintana is the old name of Bartonella Quintana
12. Ans. (c) (R.prowazekii) Ref: Harrison 17/e p1060 & 18/e p1409
  • History states- Following louse bite, person develops fever, delirious and rashes all over the body.
  • Only Louse bite Rickettsial infection – Epidemic Typhus (R.prowazekii)
  • Epidemic Typhus- rashes are distributed all over except palm and sole
13. Ans. (b) (Flea is the vector) Ref: Harrison 17/e p1064 & 18/e p1412
  • All Rickettsial can be grown in egg yolk sac and tissue culture, however it is not that useful for primary isolation. (Ananthanarayan 8/e p405)
Endemic Typhus- Vector- Rat Flea (R.typhi/mooseri) and Cat flea (R.felis)
  • Agent- R.typhi/mooseri and R.felis
  • Reservoir- Rat and opossum
  • Rash- present
  • Eschar- absent
  • Lymphadenopathy- absent
14. Ans. (d) (High OX-19 and OX2) Ref: Harrison 17/e p1060, 64 & 18/e p1408, 12
  • History states- Tick bite followed by development of fever, headache, erythematous lesion surrounded by small vesicles and lymphadenopathy…………… Its cases of African tick Typhus
  • Vesicular rash seen in - Rickettsial Pox and African tick Typhus
  • Rickettsial Pox -Weil Felix reaction is –ve
  • In African tick Typhus - Weil Felix reaction shows High OX-19 and OX2
  • Weil Felix is negative for - Q fever, R.pox, Ehrlichia, Bartonella
15. Ans. (a) (Rickettsia rickettsii) Ref: Harrison 17/e p1060, 64 & 18/e p1408, 12
Already explained.
16. Ans. (d) (Cephalosporins are drugs of choice) Ref: Harrison 17/e p1064 & 18/e p1410, 11
“Drug of choice for Rickettsial infection – Doxycycline (followed by chloramphenicol)”
About other options
  • All Rickettsia are transmitted by arthropod vectors (Except Q fever)
  • Eschar is not seen in RMSF
  • Weil-Felix reaction is diagnostic for Rickettsial infection (except Q fever, R.pox, Ehrlichia, Bartonella)
17. Ans. (c) (Inhalation of aerosol) Ref: Harrison 17/e p1066 & 18/e p1412
Coxiella burnetii – the agent of Q fever transmitted by- respiratory mode (without arthropod)
18. Ans. (d) (Weil–felix reaction is very useful for diagnosis) Ref: Harrison 17/e p1060, 64 & 18/e p1408, 12
Weil-Felix reaction is not useful for-Q fever, R.pox, Ehrlichia, Bartonella.
19. Ans. (a) (Tsutsugamushi fever) Ref: Harrison 17/e p1064 & 18/e p1413
  • For details refer chapter review.
26720. Ans. (c) (RMS) Ref: Ananthanarayan 9/e p408
  • Among the options, Rocky Mountain spotted fever is tick borne.
21. Ans.(d), (Chlamydial disease) Ref: Ananthnarayan 9th/p408
  • Scrub typhus is rickettsial disease, caused by Orientiatsutsugamushiwhich has three major antigenic types: Karp, Gilliam and Kato.
  • Scrub typhus is transmitted by larval form of trombiculid mite called chiggers.
 
Bartonellosis
22. Ans. (a) (Oroya fever) Ref: Harrison 18/e p1314
  • B. henselae is associated with Cat-scratch disease, bacillary angiomatosis, bacillary peliosis, bacteremia, endocarditis
23. Ans. (d) (Tick is the vector) Ref: Harrison 18/e p1314
  • Vector for B.quintana- Louse
  • Trench Fever is often periodic and recurrent, caused by B.quintana
24. Ans. (a) (c) (e) (Bartonella henselae, Pasteurella multocida, Yerisinia pestis) Ref: Harrison 18/e p1306, 1314
  • Bartonella species transmitted from cat- B. henselae, B. koehlerae, B. clarridgeiae
  • Pasteurella is transmitted from animals (dogs, cats, cattle and sheep).
  • Yerisinia pestis (agent of plague) is transmitted by bite of rat flea followed by direct exposure to rodent. However it may be transmitted from dogs and cats.
Dogs and cats may bring plague-infected fleas into the home, and infected cats may transmit plague directly to humans by the respiratory route…… Harrison 18/e p1306
Other options:
  • Bartonella bacilliformis- transmitted by Sandflies (Lutzomyia verrucarum)
  • Bartonella vinsonii subsp. arupensis - Reservoir Host-Mice, Arthropod Vector- Tick
  • B. vinsonii subsp. berkhoffii - Reservoir Host-Domestic dogs, Arthropod Vector- Tick
25. Ans. (e), (No relation with any above mentioned) Ref: Harrison 18/e p1318
  • Bacillary angiomatosis is a disease of severely immunocompromised patients, is caused by B. henselae or B. quintana.
  • Characterized by neovascular proliferative lesions of skin and other organs.
26. Ans. (c), (d) (Associated with a pathognomonic histological picture, Associated with regional lymphadenopathy)
Ref: Harrison 17/e p 987, & 18/e p1314, 15 Frie skin test is positive for LGV
Warthin-Starry stains of biopsy/ aspirated LN tissue reveal typical clusters of pleomorphic gram-negative organisms within areas of necrosis, blood vessel walls, or erythrocytes
27. Ans. (b) (History of exposure to cat) Ref: Ananthanarayan 9/e p413
  • Patient with a macule in hand followed by axillary lymphadenopathy- More suggestive of Cat scratch disease, hence we should ask first about the history of exposure to cat.
28. Ans. (a) (HME) Ref: Ananthanarayan 9/e p409,8/e p409
Ehrlichiosis
E. sennetsu
E. chaffeensis
E.phagocytophila
Causes-
Human Lymphocytic Ehrlichiosis
Human Monocytic Ehrlichiosis
Human Granulocytic Ehrlichiosis
29. Ans. (a) (Bartonella baciliformis) Ref: Ananthnarayan 9th/p412
VerrugaPeruana or Peruvian Wart is caused by Bartonella baciliformis, characterized by cutaneous rashes produced by a proliferation of endothelial cells.
30. Ans. (b) (Bartonella baciliformis) Ref: Ananthnarayan 9th/p412
  • Bartonella baciliformis causes Oroya fever or Carrion's disease.
 
CHLAMYDIAE
31. Ans. (b) (Gram positive) Ref: Ananthanarayan 9/e p418, 8/e p418
  • Chlamydia is poorly gram negative.
26832. Ans. (b) (Chlamydia) Ref: Ananthanarayan 9/e p418, 8/e p418
  • Chlamydia forms inclusion body like viruses.
33. Ans. (a) (Mc'coy cell)
Ref: Bailey and Scott's Diagnostic Microbiology 12/e p846, Ananthanarayan 9/e p418, 8/e p418
  • Chlamydia trachomatis is by far the most common cause of Urethral syndrome. So 1st attempt should be made for the isolation of Chlamydia trachomatis.
  • McCoy and HeLa cell lines are used for the isolation of Chlamydia trachomatis.
Among the options:
  • McCoy cell – Cell line used for Chlamydia ………………………answer
  • Thayer martin – Culture media used for Gonococcus
  • Cooked meat- Culture media used for anaerobic culture
  • PPLO broth - Culture media used for Mycoplasma
Urethral syndrome- lower UTI symtpoms with concomitant pyuria (>8 pus cells/mm3) but in the absence of significant bacteriuria (< 105 /ml of urine)in young sexually active female.
  • Infective causes:
    • ✓ STD (Chlamydia, gonorrhea),
    • ✓ Viral causes – HSV and CMV
  • Non-infective causes:
    • ✓ Trauma, Allergies,
    • ✓ Anatomical -Post surgical scarring.
34. Ans. (d) (10%) Ref: Internet source
  • WHO recommends a community should receive mass treatment of azithromycin annually for 3 years if the prevalence of active trachoma among children 1–9 years exceed 10%.
35. Ans. (d) (prevent phagosome-lysosome fusion) Ref: Harrison 18/e p1422
  • The Elementary Body is the infective form which enters the cell by receptor-mediated endocytosis and resides in an inclusion, where the entire growth cycle is completed. The chlamydiae prevent phagosome-lysosome fusion by which it can survive in the intracellular environment.
36. Ans. (d) (Women using oral pills are more prone for infection)
Ref: Harrison 17/e p1070 & 18/e p1422
“Use of oral contraceptive pills and the presence of cervical ectopy also confer an increased risk of chlamydial infection.”
About Other Options
  • Chlamydial infections are usually asymptomatic.
  • “The proportion of infections that are asymptomatic appears to be higher for C. Trachomatis than for N. gonorrhoeae, and symptomatic C. Trachomatis infections are clinically less severe”.
  • It is routinely treated with Single dose Azithromycin
  • It is diagnosed by isolation in McCoy and HeLa cell line. But they vary in their infectivity to cell lines.
37. Ans. (d) (Chlamydia) Ref: Harrison 17/e p1072 & 18/e p1423
  • Reiter's disease –seen in Chlamydia
  • Consists of:
    • Conjunctivitis,
    • Urethritis (or, in female, cervicitis),
    • Arthritis, and
    • Characteristic mucocutaneous lesions
Also know:
Ritter's disease- seen S.aureus (scalded skin syndrome in new borne)
26938. Ans. (d) (Penicillin is the treatment of choice) Ref: Harrison 17/e p1075 & 18/e p1427
DOC for Chlamydia infection
About other Options
  • Genital chlamydial infections are often asymptomatic
  • Chlamydia Cannot be cultured in artificial media but they can be isolated from yolk sac and cell lines like McCoy and HeLa
  • Inclusion conjunctivitis is caused by C. Trachomatis serotypes D-K
39. Ans. (b) (Nucleic acid amplification test) Ref: Harrison 17/e p1075, & 18/e p1426, Table 169.1
Diagnosis of C. Trachomatis infection
Presumptive diagnosis of Chlamydia trachomatis infection is often made –
  • ✓ When gonococci are not found in presence of pus cells in gram stained smear.
Confirmatory Test of Choice-
  • ✓ Nucleic acid amplification test (NAAT) for all type of C.trachomatis infection
For LGV- In addition to NAAT,
  • ✓ Culture, CF titer > 1:64; Micro-IF titer > 1:512
For neonatal conjunctivitis-
  • ✓ FA-stained scraping of conjunctival material can be done
For Infant pneumonia- Culture, Micro-IF
40. Ans. (b) (Causes urethritis) Ref: Harrison 17/e p1077 & 18/e p1429
  • ✓ Urethritis doesn't occur in psittacosis
Clinical feature of Psittacosis:
  • Pulmonary symptoms (Atypical pneumonia)
  • Others- Myalgia, disorientation, Horder spot (macular spot),
  • GIT symptoms, Splenomegaly, hepatomegaly, endocarditis and myocarditis
41. Ans. (a), (c) (Conjunctivitis, Mucosal lesions) Ref: Harrison 17/e p1072 & 18/e p1423
  • Reiter's syndrome: Consists of conjunctivitis, urethritis (or, in female, cervicitis), arthritis, and characteristic Mucocutaneous lesions in HLA-B27 phenotype patients
  • C. trachomatis –MC case, Others-Salmonella, Shigella, or Campylobacter, for example
42. Ans. (a) (Elementary body is metabolically active) Ref: Harrison 17/e p1070, 71 & 18/e p1421 Refer chapter review
  • Chlamydia biphasic- elementary and reticulate body
  • Elementary body is extracelluar, infectious form
  • Reticulate body is the replicating form, divides by binary fission and it is metabolically active
  • Chlamydia evades phagolysozome fusion, hence can survive intracellualrly.
43. Ans. (c) (They can grow in cell free culture media)
Ref: Harrison 17/e p1070, 71 & 18/e p1421, 22, Jawetz 24/e p357 & 25/e p327
  • Chlamydia cannot grow in cell free culture media, but they can be isolated from yolk sac and cell lines like McCoy and HeLa
  • Azithromycin is DOC for Chlamydia infections. Alternate- Doxycycline and Erythromycin
  • Chlamydia does not contain a typical bacterial Peptidoglycan. N-acetylmuramic acid appears to be absent. However PBP are present, so cell wall acting drugs can work
  • Chlamydia are obligate intracellular bacteria
27044. Ans. (d) (The group specific antigen is responsible for the production of complement fixing antibodies)
Ref: Harrison 17/e p1078 & 18/e p1426, Ananthanarayan 9/e p417, 8/e p415, Jawetz 24/e p358 & 25/e p332
Group (genus)-specific antigens of Chlamydia-
  • ✓ Heat-stable lipopolysaccharide- antibody against LPS is detected by CFT
Species-specific or serovar-specific antigens of Chlamydia-
  • ✓ Outer membrane proteins- antibody against OMP is detected by Micro-IF
Serovars of Chlamydia-
  • C. trachomatis- 15 serovars
  • C.psittaci- several serovars
  • C.pneumoniae- 1 serovar
Mode of transmission-
  • C. Trachomatis- Sexual, neonates through birth canal, ocular
  • C.psittaci and pneumonia – Inhalation from bird excreta
  • C. pneumoniae- Inhalation from man to man by coughing
Cytoplasmic inclusions -
  • ✓ Cytoplasmic compact inclusions with glycogen matrix- C. Trachomatis
  • ✓ Diffuse vacuolated inclusions without glycogen matrix C.psittaci and pneumoniae
45. Ans. (d) (Community acquired pneumonia) Ref: Ananthanarayan 9/e p421, 8/e p417 Refer text for explanation.
46. Ans. (b) (Enzyme immunoassay) Ref: Ananthanarayan 9/e p417, 18, 8/e p417, 18 Enzyme immunoassay detects either antigen or antibody (Doesn't help in isolation)
Cultivation of Chlamydia:
  • Not cultivable in artificial media
  • Mice inoculation (infective by only C.Psittaci and LGV)
  • Yolk sac, cell culture (McCoy Cells and HeLa) - useful for LGV and C.Psittaci
  • Pretreatment with irradiation, cycloheximide, DEAE dextran, centrifugation – enhance detection
  • Hep2 for C.Pneumoniae
47. Ans. (b) (Yolk sac inoculation) Ref: Ananthanarayan 9/e p418, 8/e p417
Refer Text.
48. Ans. (a), (b), (c) (HeLa, HeP2, McCoy) Ref: Ananthanarayan 9/e p418, 8/e p418
Refer Text.
49. Ans. (a), (c) (Infertility, Pelvic inflammatory disease)
Ref: Ananthanarayan 9/e p420, 8/e p419, Harrison 17/e p1072, 76 & 18/e p1421, 26
Refer Text.
50. Ans. (a) (Nucleic acid amplification test) Ref: Harrison 18th/p1427–29
Nucleic Acid Amplification Test (NAAT) has revolutionised the diagnosis of chlamydial infections.
  • Advantage- It is highly sensitive and specific, takes less time, and detects even few copies of DNA from the sample. It can also differentiate the species and serovars.
  • NAATs are currently the diagnostic assays of choice for chlamydial infection as recommended by the CDC, replacing the so called gold standard culture methods.
51. Ans. (c) (Amoxycillin) Ref: Harrison 18/e p1426
  • Treatment of genital Chlamydiasis- Single-dose azithromycin regimen is DOC.
  • Treatment of Chlamydia in pregnancy-
    • ✓ Although not approved by the FDA for use in pregnancy, single-dose regimen of azithromycin regimen appears to be safe and effective for this purpose.
      ✓ However, amoxycillin (500 mg three times daily for 7 days) can also be given to pregnant women.
      ✓ The fluoroquinolones are contraindicated in pregnancy.
52. Ans. (c) (Chlamydia pneumophilla) Ref: Ananthanarayan 9/e p416
  • Chlamydia pneumoniae has one serotype, C.trachomatis has 15 serotype and C.psittaccii has many serotypes.271
 
MYCOPLASMA
53. Ans. (a) (Mycoplasma) Ref: Ananthanarayan 9/e p387, 8/e p388
Dienes method is used to stain mycoplasma.
54. Ans. (c) (Cephalosporins) Ref: Ananthanarayan 9/e p389, 8/e p389
  • As there is lack of peptidoglycan layer, hence Ureoplasma is naturally resistant to all beta lactams.
55. Ans. (b) (Mycoplasma) Ref: Ananthanarayan 9/e p388, 8/e p389
  • Fried Egg colonies are produced – Mycoplasma and Malassezia furfur
56. Ans. (a) (Mycoplasma pneumoniae) Ref: Ananthanarayan 9/e p386,8/e p389
  • Eaton had isolated Mycoplasma pneumoniae from hamsters and cotton rats. As it was filterable, so it was 1st considered as virus as “Eaton agent”.
57. Ans. (b) (They are obligate intracellular organisms) Ref: Harrison 17/e p1067 & 18/e p1417
  • Mycoplasmas are smallest free living organism known.
  • Mycoplasma are smallest prokaryotic organisms that can grow in cell free culture media
  • Mycoplasma lack a true cell wall- peptidoglycan layer is absent. Contains sterols.
  • Their lack of a cell wall is associated with cellular pleomorphism and resistance to cell wall–active antimicrobial agents.
    General Character of Mycoplasma -Ananthanarayan 9/e p386,8/e p387
  • Refer text for explanation.
58. Ans. (a) (They are inhibited by penicillin)
Ref: Harrison 17/e p1067 & 18/e p1417, Ananthanarayan 9/e p386, 87, 8/e p387
About Other Options
  • Mycoplasma is completely resistant to penicillin because they lack the cell wall structures at which penicillin acts, but they are inhibited by tetracycline or erythromycin.
  • Mycoplasma have an affinity for mammalian cell membranes
  • Filterable, can pass through filters of 450 nm pore size
  • Mycoplasma can reproduce in cell-free media; on agar, the center of the whole colony is characteristically embedded beneath the surface.
59. Ans. None or (d) (Bacterial vaginosis) Ref: Harrison 17/e p1069 & 18/e p1419, Ananthanarayan 9/e p389, 8/e p389
“Given the close association of ureaplasmas with bacterial vaginosis, it is difficult to define an independent role for ureaplasmas in these conditions.”
Ureaplasma infections:
  • Nongonococcal Urethritis (NGU)
  • Epididymitis and Prostatitis, balanoposthitis
  • Infertility in both men and women
  • PID, cervicitis, vaginitis
  • Pneumonia In infants of very low birth weight
  • Reactive arthritis and Reiter's syndrome may be triggered by ureaplasmas
  • Salpingitis…Ananthanarayan 9/e p389,8/e p389, Alcamo's Fundamentals of Microbiology/p410
Ureaplasma is implicated in all the conditions given in the options, so I personally feel none of the options are correct.

Miscellaneous BacteriaCHAPTER 3.12

 
LISTERIA MONOCYTOGENES
  • Gram +ve coccobacilli, catalase +ve
  • Shows Tumbling type of motility
  • Motile at 25°C but nonmotile at 37°C (Differential motility)
  • Growth improves if cultured in thioglycollate broth at 4°C (cold enrichment)
  • It can grow in refrigerated food and can tolerate preserving agents
  • Mode of transmission- Raw milk intake
  • Pathogenicity-Intracellular, migrate from one host cell to another by host cell actin polymerization
  • Infections of Listeriosis:
    • Infection in Pregnancy – Before 20 weeks is rare, May lead to abortion, still birth.
    • Early onset neonatal disease- Septicemia, meningitis granulomatosis infantiseptica (Appear within 2 days of birth, in-utero transmission)
    • Late onset neonatal disease –Mainly meningitis, due to hospital cross infection, Appear after 5 days of birth
    • Adult –
      • ✓ Associated with steroid therapy, HIV, DM, malignancy (fludarabine treated)
      • ✓ Bacteremia> meningitis
      • ✓ MC cause of meningitis in kidney transplanted patient after 1 month
      • ✓ Also causes Gastroenteritis : following contaminated milk, meat and salad
  • Lab diagnosis:
    • Media- Blood agar, chocolate agar, PALCAM agar
    • Anton test – instillation to rabbit eye causes conjuntivitis
  • Treatment:
    • DOC- Ampicillin (also penicillin)
    • Alternate- Cotrimoxazole (if allergic to penicillin)
    • Cephalosporin- not affective
 
ERYSIPELOTHRIX RHUSIOPATHIAE
  • Gram+ve bacilli,
  • Catalase negative, H2S +ve
  • Causes erysipeloid skin lesion
 
ACTINOMYCETES
  • Gram +ve branching filamentous bacteria
  • Human pathogenic actinomycetes belong to four genera (1) Actinomyces (2) Streptomyces (3) Nocardia (4) Actinomadura
  • Actinomyces is non acid fast and anaerobic
  • Nocardia – Aerobe and acid fast (1% sulfuric acid)
  • Streptomyces and Actinomadura are aerobes and non acid fast – Can cause actinomycetoma273
Actinomyces
Nocardia
Non Acid fast
Partially acid fast
Anaerobe
Obligate aerobe
Found as Endogenous flora- mouth
Usual habitat- Soil, infections are exogenous
Disease occur in immunocompetent host also
Usually affects people with low immunity
Disease- Cevicofacial, thoracic, abdominal
Disease- pulmonary, CNS, Actinomycetoma
Sulfur granules-
  • Granules consist of gram positive bacilli and surrounded by clubs (sunray appearance)
  • Clubs are due to Antigen-Antibody reaction.
Culture- Spidery molar teeth colony in solid media and Fluffy ball at bottom of liquid medium
Paraffin bait technique- for isolation from soil and clinical specimen
DOC- Penicillin
DOC- Sulfonamide > Cotrimoxazole
 
CAMPYLOBACTER JEJUNI
  • Curved Gram Negative bacilli (gullwing shaped)
  • Single polar flagellum (darting motility),
  • Microaerophilic – (5% O2 + 85% N2 + 10% CO2)
  • Thermophilic (42°C)
  • Zoonotic (poultry, cattle, sheep, and swine)
  • Mode- ingestion of contaminated food and water or raw milk intake
  • Mechanism of diarrhea –Produces LT resembling Cholera Toxin + Invasion seen like Shigella+ Cytotoxin
  • Clinical feature-Diarrhea or dysentery, abdominal pain, and fever
  • Campylobacter fetus :Bacteremia, meningitis, vascular infections and diarrhea
  • Culture:
    • Transport media : Cary Blair medium
    • Selective media: Skirrow's, Campy BAP, Butzler, Charcoal based media
  • Treatment-Erythromycin for 5–7days- is the regimen of choice.
 
HELICOBACTER PYLORI
  • Curved rod, motile unipolar tuft of lophotrichous flagella
  • Microaerophilic, abundant urease production
  • Prevalence:
    • 30% (Developed Country),
    • 80% (Developing),
    • 50% (World)
  • Virulence:
    • Cytotoxin associated gene,
    • Vacuolated cytotoxin gene,
    • Urease
  • Risk factor- MC in childhood (but immunity doesn't develop), Low socioeconomic status, Crowding
Disease:
  • Acute gastritis (Antrum MC, spares cardiac end)
    • Antral gastritis- predisposes to duodenal ulcers
    • Pan gastritis- predisposes to Adenocarcinoma of stomach
  • Peptic ulcer disease (80% of duodenal ulcer and 60% of gastric ulcer)- By↓ in soma- tostatin-producing D cells.
  • Chronic atrophic gastritis and Autoimmune gastritis
  • Adenocarcinoma of stomach (non cardiac) and non-Hodgkin's gastric lymphoma
    • Diets high in salt and preserved foods & smoking increase risk
  • 274Protective role for H. pylori against
    • Gastro esophageal reflux disease (GERD),
    • Barrett's esophagus
    • Adenocarcinoma of the esophagus and gastric cardia
Diagnosis:
Invasive test:
  • Endoscopy guided biopsy/aspiration followed by-
    • Histopathology with Warthin starry silver staining
    • Culture – campylobacter media and chocolate agar – most specific, but not Sensitive
    • Biopsy urease test (at least 3 biopsy) /rapid urease test
Noninvasive test:
  • Urea breath test:
    • Using C14 labeled urea solution drinking followed by detection of lebelled CO2 in breath
    • Most consistent and accurate test
    • Most sensitive, quick, simple
    • Monitoring of treatment (become-ve after treatment)
  • Stool antigen (coproantigen) assay- Newer test, used for monitoring, useful for pediatric patient.
  • Serology : ELISA- Used for screening before endoscopy and for sero-epidemiolog- ical study
Treatment for H.pylori infection
1st line- (Triple drug therapy)-Omeprazole + Clarithromycin + Metronidazole or Amoxicillin (7–14 days)
Urea Breath test done after 1 month gap
If 1st line regimen fails (Urea Breath test +ve)-
2nd line-(Quadruple drug therapy)- Omeprazole + Metronidazole +Bismuth subsalicylate + Tetracycline (14 days)
If 2nd line Quadruple drug therapy fails then
Culture of Endoscopic guided biopsy is done and treatment based on antibiotic sensitivity
 
LEGIONELLA
  • Pleomorphic rods, Poorly gram – negative
  • Comprises of 50 species with more than 70 serogroups
  • MC to infect humans- L.pneumophila serogroup 1 (SG1)
  • L.micdadei- acid-fast (Pittsburgh pneumonia agent)
Epidemiology:
  • Reservoir:
    • Natural water source-Rivers/streams/amoebae
    • Artificial Aquatic source- AC, water coolers
  • Transmission – multiple routes
    • Aspiration (predominant mode)
    • Aerosols from contaminated AC,
    • Direct instillation into the lungs during respiratory tract manipulations.
  • No human – to human transmission
  • No carrier stage, no animal reservoir
  • Predisposing Factors:
    • Smokers, Alcohol,; advanced age
    • Immunosuppression,
    • Hairy cell leukemia, Chronic lung disease;
    • Prior hospitalization
275Pathogenesis:
  • Attachment to host cells -By bacterial type IV pili, heat-shock proteins, major OMP and complement.
  • Entry by- attachment of the OMP of bacteria to Fc receptors and complement receptors of alveolar macrophage (Coiling phagocytosis)
  • Inside the phagosome, it evades intracellular killing by inhibiting phagosome- lysosome fusion
  • Early neutrophil recruitment to the site of infection
Pontiac Fever: Acute, self-limiting, mild flu like
Legionnaires’ disease (“Atypical Pneumonia”)
  • Community acquired Pneumonia (h/o hospital discharge within 10 days)
  • MC extrapulmonary site – heart
Diagnosis:
  • Gram staining- Numerous PMN, but no organism in sputum
  • DFA (Direct Fluorescent Antibody) on biopsy and Silver stain
  • Culture- highly sensitive and specific – Diagnosis of choice
    • Sample- Transtracheal aspirate > Sputum,
    • Fastidious requiring iron and cysteine (BCYE medium, buffered charcoal Yeast Extract)
  • Antigen detection in urine
  • Fourfold increase in antibody -use primarily in epidemiologic studies
Treatment:
  • DOC- Macrolide(Azithromycin)> Quinolone
  • β lactams and Aminoglycoside- not affective
 
CALYMMATOBACTERIUM GRANULOMATIS
  • Present name Klebsiella granulomatis
  • Produce granuloma inguinale, Granuloma venereum or Donovanosis
  • Shows Bipolar staining, safety pin appearance
  • Clinical feature→ painless ulcer without Lymphadenopathy
  • Description of ulcer:
    • Classic ulcerogranulomatous beefy red ulcer that bleeds readily when touched
      • ✓ Other types: Hypertrophic or verrucous ulcer, Necrotic, offensive-smelling ulcer, Sclerotic or cicatricial ulcer with fibrous and scar tissue
  • Donovan bodies: body of the bacillus blue and capsule pink, contained within the cytoplasmic vacuoles of large macrophages (stained by Wrights, Giemsa)
  • Culture: Embryonated hens egg
  • Treatment à DOC-Azithromycin, Alternate-Cotrimoxazole, Erythromycin Tetra- cycline,
 
RAT BITE FEVER
  • Streptobacillary moniliformis and Spirillum minus
Streptobacillary moniliformis:
  • Gram negative, highly Pleomorphic bacillus
  • String of bead appearance, non motile, fastidious
  • Exists as L form also
  • Haverhill fever – Erythema arthriticum epidemicum
  • Lab diagnosis → Grow on culture media containing blood serum or ascitic fluid
276Spirillum minus:
  • Motile, Not cultivated
  • Causes Sudoku,
 
GARDNERELLA VAGINALIS
  • Causes bacterial vaginosis,
  • Gram variable (mostly-Gram negative coccobacilli), possess metachromatic granules
  • Other organisms implicated in BV-
    • Mycoplasma,
    • Mobilincus,
    • Prevotella,
    • Peptostreptococcus
  • Bacterial vaginosis is characterized by:
    • Thin profuse vaginal discharge:
    • pH > 4.5
    • Fishy odor (Whiff test- addition of 10% KOH enhances the odor)
    • Clue cells – epithelial cell studded with organism
  • Nugent scoring used for diagnosis
  • Treatment- DOC-Metronidazole
277MULTIPLE CHOICE QUESTIONS
 
LISTERIA
1. Gram positive short rods are seen in the CSF sample of a new borne child suffering from lethargy, fever and seizure. Organism responsible is: (AIIMS Nov 2014)
  1. Group B Streptococcus
  2. Listeria
  3. Clostridium tetani
  4. Bacillus anthracis
2. Most common serotype of Listeria causing infections: (NEET Pattern Based)
  1. 1/2a
  2. 1/3a
  3. 1/3b
  4. 1/2c
3. Tumbling motility is shown by: (NEET Pattern Based)
  1. Listeria monocytogenes
  2. Proteus vulgaris
  3. Borrelia
  4. Clostridia
4. Drug of choice for Listeria monocytogenes is: (DNB Dec 2009)
  1. Ampicillin
  2. Amoxycillin
  3. Vancomyucin
  4. Amikacin
5. A 39 year old female comes with high grade fever and signs suggesting meningitis. On CSF examination a gram positive bacilli was isolated. The organism involved is: (AI 2011)
  1. Streptococcus pneumoniae
  2. Listeria monocytogenes
  3. H. influenzae
  4. Staph aureus
6. Culture Media for Listeria is: (AIIMS May 2009)
  1. Baker's
  2. Thortoff
  3. Tinsdale
  4. Blood agar
7. Which is not true about bacterial transmission: (AI 2009)
  1. Legionella – through water aerosol
  2. Listeria- Refrigerated food
  3. Leptospira- Urine
  4. Tetanus- droplet/dust
8. A 3 – week old child presented to the pediatrician with meningitis. A presumptive diagnosis of late onset of a perinatal infection was made. The CSF culture was positive for gram positive bacilli. Which of the following characteristic of this bacteria would be helpful in differentiating it from other bacteria agents: (AIIMS May 2005)
  1. Ability to grow on blood agar
  2. Ability to produce catalase
  3. Fermentative attack on sugars
  4. Motility at 25 C
9. A major step in the pathogenesis of listeriosis is? (AIIMS Nov 2005)
  1. The formation of antigen –antibody complexes with resultant complement activation and tissue damage
  2. The release of hyaluronidase by L. monocytogenes, which contributes to its dissemination from local sites
  3. The antiphagocytic activity of the L. monocytogenes capsule
  4. The survival and multiplication of L. monocytogenes within mononuclear phagocytes and host epithelial cells
10. A 30 year old woman with a bad obstetric history presents with fever. The blood culture from the patient grows gram- positive small to medium coccobacilli that are pleomorphic, occurring in short chains. Direct wet from the culture shows tumbling motility. The most likely organism is: (AI 2004)
  1. Listeria monocytogenes
  2. Corynebacterium sp.
  3. Enterococcus spp
  4. Erysipelothrix rhusiopathiae
11. All of the following are true about Listeria except: (AI 2004)
  1. Transmitted by contaminated milk
  2. Gram negative bacteria
  3. Cause abortion in pregnancy
  4. Causes meningitis in neonates
12. Seal finger and whale finger are associated with: (NIMHANS 2006)
  1. Listeria
  2. Erysipelothrix
  3. Corynebacterium
  4. Treponema
13. In patient with Listeria meningitis who is allergic to penicillin the treatment of choice is: (AIIMS 2004)
  1. Vancomycin
  2. Gentamycin
  3. Trimethoprim – sulphamethoxazole
  4. Ceftriaxone
 
ACTINOMYCES
14. Actinomycetes, most common site is: (NEET Pattern Based)
  1. Cervicofacial
  2. Thoracic
  3. Abdomen
  4. Brain
15. Color of granule of Actinomycetes: (NEET Pattern Based)
  1. Black
  2. Yellow
  3. Red
  4. Brown
16. Which of the following is the most predominant constituent of sulfur granules of Actinomycosis is? (AIIMS May 2004, 2002)
  1. Organisms
  2. Neutrophils and monocytes
  3. Monocytes and lymphocytes
  4. Eosinophils
 
278NOCARDIA
17. Nocardia infection is most commonly associated with: (AI 2012)
  1. Liver abscesses
  2. Thidney abscesses
  3. Brain abscesses
  4. Subcutaneous
18. True about Nocardia are all EXCEPT: (DNB June 2009)
  1. Penicillin is drug of choice
  2. Most common disease caused is pneumonia
  3. Aerobic
  4. Acid fast
19. A patient comes with history of unresponsive fever and cough. On examination of sputum it is negative for mycobacterium. Chest x-ray shows pneumonia. BAL shows gram positive branching filaments. Organism was partially AFB positive. Causative organism is: (AI 2011)
  1. Actinomycosis
  2. Nocardiosis
  3. Aspergillus
  4. Penicillium
20. Nocardia is stained by: (AIIMS May 2008)
  1. Acid fast stain
  2. Thiram's stain
  3. Alcian blue stain
  4. Mucin stain
21. A clinical specimen was obtained from the wound of a patient diagnosed as Nocardiosis. For the selective isolation of Nocardia sp. which one of the following would be the best method: (AIIMS May 2004)
  1. Paraffin bait technique
  2. Castaneda's culture method
  3. Craige's tube method
  4. Hair bait technique
22. Nocardia is differentiated from Actinomyces by: (PGI June 2002)
  1. Gram stain
  2. ZN stain
  3. Nocardia causes Mycetoma, Actinomyces does not
  4. Nocardia is facultative anaerobe
23. What are the features that differentiate Nocardia from actinomycetes: (PGI 2002)
  1. Gram's stain
  2. Modified acid fast stain
  3. Disease characteristics
  4. Growth on Sabouraud's agar
  5. Animal pathogenicity
 
CAMPYLOBACTER
24. The following statements are true for Campylobacter except: (AI 2011, AI Nov 09)
  1. Campylobactereriosis in humans is induced mainly by campylobacter jejuni.
  2. Poultry is a major source of infection
  3. Humans are the reservoir of infection
  4. Gullian-Barre syndrome is a sequelae of campylobacter infection
25. Campylobacter culture media are: (PGI Dec 2008)
  1. Schaedler's agar
  2. CVA medium
  3. Regan-Lowe medium
  4. Skirrow medium
  5. Campylobacter blood agar
26. A 35 year old patient complains of abdominal cramps along with profuse diarrhea. The treating physician wants to process the stool specimen for isolation of Campylobacter jejuni. Which of the following is the method of choice for the culture of stool: (AIIMS Nov 2004, AI 2005)
  1. Culture on TCBS medium incubated at 37 C under aerobic conditions
  2. Culture on Skirrow's medium incubated at 42 C under micro-aerophilic conditions.
  3. Culture on MacConkey medium incubated at 42 C under anaerobic conditions
  4. Culture on Wilson and Blair's medium incubated at 37 C under micro-aerophilic conditions
 
HELICOBACTER
27. H. pylori is found in: (DNB 2012, NEET Pattern Based)
  1. Epithelial lining
  2. Mucous layer
  3. Lamina propria
  4. Sub mucosa
28. Incidence of H.pylori in Gastric ulcer is: (JIPMER 2011)
  1. 5%
  2. 20%
  3. 60%
  4. 80%
29. Non invasive test for H.pylori: (PGI June 2008)
  1. Rapid urease test
  2. Urease breath test
  3. Stool antigen assay
  4. Stomach aspiration culture
  5. Biopsy
30. What is true about H.pylori? (PGI Dec 2005)
  1. Gram –ve Cocci curved
  2. Gram +–ve Cocci straight
  3. Gram +ve bacilli curved
  4. Gram –ve bacilli straight
  5. Gram –ve bacilli, curved
31. True about H.pylori: (PGI June 2004, AIIMS Nov 2003)
  1. It is flagellated
  2. Involved in causation of peptic ulcer disease
  3. Hypergastrinomia caused by it
  4. Eradication leads to improved life style
  5. It is a gram –ve organism
32. H.pylori true about: (PGI June 2004, PGI June 2002)
  1. Gram +ve spiral organism
  2. It is a protozoa
  3. Causes chronic gastritis in adults due to re-infection
  4. Treatment causes regression of gastric lymphoma
  5. Duodenal mucosa normal
33. True about H.pylori: (PGI Dec 2000)
  1. Seen in 85 to 90% cases of gastric ulcer
  2. Seen in 20 to 25% cases of duodenal ulcer
  3. 279Transmitted from man to man, feco-orally and by orogastric route
  4. Common in adults of developing countries
34. Maximum urease +ve is produced by: (PGI 2000)
  1. H.pylori
  2. P.mirabilis
  3. Th.rhinoscleromatis
  4. Ureoplasma
35. Which of the following is false regarding H.pylori: (AIIMS 2000)
  1. With chronic infection urease breath test become negative
  2. H.pylori infection remains lifelong if untreated
  3. Endoscopy is diagnostic
  4. Toxigenic strains usually cause ulcer
36. True about H.pylori is a all except: (PGI June 2000)
  1. It splits urea and produces ammonia to survive
  2. Produces gastric carcinoma
  3. Gram –ve curved rod
  4. Cag –A gene is not associated with risk of duodenal ulcer
37. Endoscopic biopsy is positive for urease test and organism grows on Skirrows media. Identify the organism. (Recent Question 2013)
  1. Campylobacter
  2. Helicobacter pylori
  3. Proteus
38. True about H. pylori: (PGI Nov 2014)
  1. Spiral
  2. Microaerophilic
  3. Gram negative bacillus
  4. Survives in acidic environment by producing urease
39. True about H pylori: (PGI Nov 2012)
  1. Move by tuft of flagella
  2. Produce urease enzyme
  3. More common in developed countries
  4. Gram positive bacteria
 
LEGIONELLA
40. Legionnaire disease is caused by: (NEET Pattern Based)
  1. Motile gram positive
  2. Motile gram negative
  3. Non-motile gram positive
  4. Non-motile gram negative
41. Legionella pneumophila spreads by: (AIIMS Nov 2012, AIIMS May 2013)
  1. Person to person
  2. A.C Aerosol
  3. Infected Meat
  4. Contaminated air
42. An elderly man presented with fever and cough. Sputum examination revealed gram negative organisms that were grown on Buffered charcoal yeast extract agar. The organism involved is? (AIIMS Nov 2011, AI 2007, AIIMS Nov 2006)
  1. H. influenzae
  2. Legionella pneumophila
  3. Burkholderia cepacia
  4. Brucella
43. Legionella pathogenicity is due to? (DNB June 2010)
  1. Capsule
  2. Toxin
  3. Bacteriophage
  4. Failure of oxidative burst of neutrophils
44. Pontiac fever is caused by: (PGI Dec 2007)
  1. Legionella
  2. Listeria
  3. Scrub typhus
  4. Leptospira
  5. Rickettsia
45. True about Legionella: (PGI Dec 2006)
  1. Epidemics seen
  2. Splenomegaly
  3. Easily seen on sputum
  4. Scanty neutrophils with fed organisms
  5. Purulent sputum common
46. All of the following are correct regarding Legionella except: (AIIMS Nov 2005)
  1. Legionella can be grown on complex media
  2. Legionella pneumophila serogroup is the most common serogroup isolated from humans
  3. Legionella are communicable from infected patients to others
  4. Legionella pneumophila is not effectively killed by polymorphonuclear leukocytes
47. AYear old man isdiagnosed to besuffering from Legionnaires’ disease after the returns home from attending a convention. He could have acquired it: (AI 2003)
  1. From a person suffering from the infection while travelling in the aeroplane
  2. From a chronic carrier in the convention center
  3. From inhalation of the aerosol in the air conditioned room at convention center
  4. By sharing an infected towel with a fellow delegate at the convention
48. Which of the following is a good media to use for diagnosis of Legionnaires disease? (AIIMS Nov 2001, PGI 1999)
  1. Thayer Martin Media
  2. BCYE agar
  3. Bordet Gengou media
  4. Chocolate agar
49. Devi, a 28 year female, has diarrhea, confusion high grade fever with bilateral pneumonitis. The diagnosis is: (AI 2000)
  1. Legionella
  2. Neisseria meningitidis
  3. Streptococcus pneumoniae
  4. H. influenzae
 
MISCELLANEOUS
50. In donovanosis: (NEET Pattern Based)
  1. Pseudolymphadenopathy
  2. Penicillin is used for treatment
  3. Painful ulcer
  4. Suppurative lymphadenopathy
51. 280Clue cell is seen in: (AI 2011, JIPMER 2011, 2010, APPG 2012)
  1. Trichomonas vaginalis
  2. Bacterial Vaginosis
  3. Candidiasis
  4. Herpes
52. Beefy red bleeding ulcer: (JIPMER 2010)
  1. Chancroid
  2. syphilis
  3. donovanosis
  4. LG
53. Bacterial vaginosis- Which is true? (Recent Question 2013)
  1. pH<4.5
  2. Clue cells seen
  3. Odourless discharge
  4. Bacilli with School of fish appearance seen
54. Drug of choice for rat bite fever: (AIIMS Nov 2014)
  1. Penicillin G
  2. Cephalosporin
  3. Amikacin
  4. Tetracycline
281EXPLANATIONS
1. Ans. (b) (Listeria) Ref: Ananthnarayan 9th/p399, Harrison 18th/p1194–97
Common cause of meningitis in new borne – Group B Streptococcus (Gram positive cocci), Listeria (Gram positive short bacilli) and E.coli (Gram negative bacilli).
2. Ans. (a) (1/2a) Ref: Ananthanarayan 9/e p396, 8/e p395
  • Most common serovar to cause human infection- 1/2a or 1/2b and 4b
3. Ans. (a) (Listeria monocytogenes) Ref: Ananthanarayan 9/e p396, 8/e p395
Listeria shows characteristic tumbling motility at 25°c however, it is non motile at 37°c.
4. Ans. (a) (Ampicillin) Ref: Harrison 18/e p1196, 17/e p895, 97 Treatment for Listeriosis:
  • Ampicillin is the drug of choice, high doses (2 g IV every 4 h),
  • Some recommend-Addition of gentamicin for synergy
  • Alternate-Cotrimoxazole, for the penicillin-allergic patient
  • Cephalosporins are not effective and should not be used.
  • The duration of therapy:
    • ✓ 2 weeks for bacteremia,
    • ✓ 3 weeks for meningitis,
    • ✓ 6–8 weeks for encephalitis
    • ✓ 4–6 weeks for endocarditis in both neonates and adults.
    • ✓ Early-onset neonatal disease - for >2 weeks
5. Ans. (b) (Listeria monocytogenes) Ref: Harrison 17/e p895, 97 & 18/e p1194, 96 Points in favor:
  • Meningitis in newborn
  • Gram positive bacilli isolated from CSF.
  • Among the options, the only gram positive bacilli - L.monocytogenes.
6. Ans. (d) (blood agar) Ref: Ananthanarayan 9/e p396, 8/e p395, Gillespie Bacteriology 2/e p129
  • Listeria can grow on blood agar producing hemolytic colony.
  • Selective media – PALCAM agar
7. Ans. (d) (Tetanus-droplet/dust)
Ref: Ananthanarayan 9/e p61, 40 18/e p259, 400, Harrison 17/e p896, 1049 & 18/e p1194, 1393 “Tetanus is transmitted through injury (preferably punctured), surgery done aseptically, following otitis media (otogenic tetanus), by unhygienic practice like circumcision, ear boring, cow dung on umbilical stump.” ……………..Ananthanarayan 8/e p259 I have not got any reference for Droplet spread of tetanus.
About Other Options: Option a:
  • ✓ Legionella are widely distributed in natural water like hot spring, stagnant water and artificial water sources like AC which serves as amplifier.
  • ✓ Human infection is typically by aerosol inhalation produced by cooling tower, AC and shower heads. …………………Ananthanarayan 9/e p401, 8/e p400
282Option b:
  • ✓ Due to survival and multiplication at refrigeration temperatures, L. monocytogenes is commonly found in processed and unprocessed foods of animal and plant origin, especially soft cheeses, delicatessen meats, hot dogs, milk, and cold salads. Human-to-human transmission is rare. ………………Harrison 17/e p896, 18/e p1194,1393
Option c:
  • ✓ Transmission of Leptospira to humans may follow direct contact with urine, blood, or tissue from an infected animal or exposure to a contaminated environment; human-to-human transmission is rare. ………………Harrison 17/e p1049, 18/e p1194, 1393
8. Ans. (d) (Motility at 25°C) Ref: Harrison 17/e p895, 97& 18/e p1194, 95, Gillespie's Bacteriology 2/e p129
  • Agents of Perinatal meningitis – E.coli, Strept. agalactiae, Listeria monocytogenes
  • Perinatal meningitis showing gram positive bacilli in CSF – Suggestive of Listeria
Next step is:
The characteristic feature of Listeria which differentiate from other organism causing neonatal meningitis is - Differential Tumbling motility at 25°C but non motile at 37°C. (Gillespie Bacteriology 2nd /p129)
Listeria has >6 species and all show Differential Tumbling motility.
However Listeria monocytogenes is the main pathogen where as others rarely cause human infection. Test to differentiate L.monocytogenes from other Listeria spp –
  • Beta hemolysis
  • Fermentative pattern of sugars
  • I think, the intention of the question is how to differentiate Listeria from other organism causing neonatal meningitis. So I suggest you to go with Option‘d’.
9. Ans. (d) (The survival and multiplication of L. monocytogenes within mononuclear phagocytes and host epithelial cells)
Ref: Harrison 17/e p895& 18/e p1194
Pathogenicity
Enteric route→ bacterial internalin attaches to epithelial cell cadherin receptor → phagocytosed by epithelial cell → phagolysozome lysed by listeriolysin→ Listeria escapes to cytoplasm induces host cell actin polymerization→ Listeria propels to cell membrane → produces listeriopods in cell membrane through which it migrates to adjacent epithelial cell / macrophages (cell to cell spread)
10. Ans. (a) (Listeria monocytogenes) Ref: Harrison 17/e p895, 897 & 18/e p1194, 95
  • Listeria in pregnancy can lead to abortion.
  • Febrile woman with a bad obstetric history presents with gram-positive coccobacilli and tumbling motility is suggestive of Listeria monocytogenes.
11. Ans. (b) (Gram negative bacteria) Ref: Harrison 17/e p895–97& 18/e p1194, 95
  • Option b: Listeria is – Gram +ve coccobacilli
  • Option a: Transmitted by contaminated milk
  • Option c: Listeria infection in pregnancy – occurs before 20 weeks is rare. May lead to abortion, still birth.
  • Option d: Listeria infection in neonates – meningitis septicemic, granulomatosis infantiseptica.
12. Ans. (b) (Erysipelothrix) Ref: Jawetz 24/e p219 & 25/e p181
“The most common E rhusiopathiae infection in humans is called erysipeloid it usually occurs on the fingers by direct inoculation at the site of a cut or abrasion and has been called seal finger and whale finger.
13. Ans. (c) (Trimethoprim – sulphamethoxazole) Ref: Harrison 17/e p897& 18/e p1196
  • Treatment for Listeria- Explanation of Refer Q.No.1283
 
ACTINOMYCES
14. Ans. (a) (Cervicofacial) Ref: Ananthanarayan 9/e p391, 8/e p392
  • Actinomycosis occurs in four clinical forms- Cervicofacial (most common form), thoracic, abdominal and pelvic.
15. Ans. (b) (yellow) Ref: Ananthanarayan 9/e p391, 8/e p392
  • Sulfur granules in Actinomyces exhibit white to yellow color.
16. Ans. (a) (Organisms) Ref: Jawetz 24/e p221& 25/e p181, 82
  • Microscopy of Actinomyces lesion reveals→ sulphur granules
  • Sulphur Granules consist of gram positive bacilli and surrounded by clubs (sunray appearance)
  • Clubs are due to Antigen antibody reaction.
Jawetz 24/e p221 states - “The sulfur granules found in tissue are yellowish in appearance, up to 1 mm in size, and are composed of macrophages, other tissue cells, fibrin, and the bacteria.”
 
NOCARDIA
17. Ans. (c) (Brain abscesses) Ref: Ananthanarayan 9/e p393, 8/e p392, Harrison 18/e p162–1323
  • MC form of nocardiosis -Pulmonary nocardiosis,
  • The most common site of extrapulmonary dissemination- is the brain.
  • Other common sites include the skin and supporting structures, kidneys, bone, and muscle, but almost any organ can be involved.
  • The typical manifestation of extrapulmonary dissemination is a subacute brain abscess.
  • Brain abscesses are usually supratentorial, are often multiloculated, and may be single or multiple.
  • Brain abscesses tend to burrow into the ventricles or extend out into the subarachnoid space.
18. Ans. (a) (Penicillin is drug of choice) Ref: Harrison 18/chapter-162, p1325
  • MC form of nocardiosis -Pulmonary nocardiosis
  • Nocardia is – Obligate Aerobe and partially Acid fast
  • DOC- Sulfonamides are the drugs of choice. Cotrimoxazole may work even better but pose toxicity. Duration 6–12 months depending on site of involvement.
19. Ans. (b) (Nocardiosis) Ref: Ananthanarayan 9/e p393, 8/p392
  • Gram positive branching filaments – Actinomyces or Nocardia
  • Partially AFB positive- Suggestive of Nocardia
20. Ans. (a) (Acid fast stain) Ref: Harrison 17/e p994 & 18/e p1325 Stains used for Nocardia:
  • Modified Kinyoun,
  • Modified Ziehl-Neelsen,
  • Fite-Faraco methods
  • Silver stains- GMS
21. Ans. (a) (Paraffin bait technique) Ref: Harrison 17/e p995 & 18/e p1325
“Since Nocardia are among the few aerobic microorganisms that use paraffin as a carbon source, paraffin baiting can be used to isolate the organisms from mixed cultures
22. Ans. (b) (ZN stain) Ref: Jawetz 24/e p 219, 20 & 25/e p181, 82, Ananthanarayan 9/e p393, 8/e p393
  • Nocardia are partially acid fast while Actionomyces are non acidfast.
About other options Option c:
  • The most common causes of Actinomycetoma are Nocardia asteroides, Nocardia brasiliensis, Streptomyces somaliensis, and Actinomadura madurae. Jawetz 24/e p220
  • Actinomyces israelii and A.bovis also can cause Mycetoma Ananthanarayan 9/e p393
284Option a, d:
  • ✓ Both are gram +ve filamentous bacteria
  • ✓ Nocardia are obligate aerobe (Source- Wikipedia)
23. Ans. (b), (c), (d) (Modified acid fast stain, Disease characteristics, Growth on Sabouraud's agar)
Ref: Jawetz 24/e p219, 20 & 25/e p181, 82, Ananthanarayan 9/e p393 & 8/e p393
  • Option b: Nocardia are partially acid fast while Actinomyces are non acid fast.
  • Option d: Nocardia Grows on Sabouraud's agar while Actinomyces cannot (Internet source)
  • Option a: Both are gram +ve filamentous bacteria
  • Option c: Refer the table of the previous explanation.
 
CAMPYLOBACTER
24. Ans. (c) (Humans are the reservoir of infection) Ref: Harrison 17/e p966 & 18/e p1286
  • Humans are not the only reservoir of infection. Campylobacter are found in the gastrointestinal tract of many animals used for food (including poultry, cattle, sheep, and swine) and many household pets.
About Other Options
  • ‘The principal diarrheal pathogen is C. jejuni, which accounts for 80–90% of all cases of recognized illness due to campylobacters and related genera’
  • ‘Ingestion of contaminated In cooked poultry is the most common mode of acquisition (30–70% of cases). Other modes include ingestion of raw (unpasteurized) milk or untreated water, contact with infected household pets, travel to developing countries’
  • ‘Campylobacter infections, because of their high incidence, may trigger 20–40% of all cases of Guillain-Barré syndrome’
25. Ans. (b), (d), (e) (CVA medium, Skirrow medium, Campylobacter blood agar)
Ref: Harrsion 17/e p 967, & 18/e p1288, Ananthanarayan 9/e p 399, 8/e p398
Culture media for Campylobacter
  • Transport: Cary Blair medium
  • Selective:
    • Skirrow's- blood agar with Vancomycin
    • Preston's- blood agar with rifampicin
    • Charcoal based media- Cefoperazone charcoal deoxycholate agar
    • Campy CVA Agar- Blood agar, Cefoperazone, Vancomycin, Amphotericin
    • Campy BAP,
    • Butzler media
  • Incubation in Microaerophilic condition (5% O2) at 42°C.
  • C.jejuni, C.lari and C.coli - Thermophilic and can grow at 42c but other cannot grow.
  • Hippurate hydrolysis-C jejuni (+ve) and C.coli (-ve)
  • Nalidixic acid- C.jejuni (sensitive), C.lari (resistant)
About Other Options
  • Schaedler Agar with Vitamin K- used for anaerobic microorganisms.
  • Regan-Lowe medium –used for Bordetella
26. Ans. (b) (Culture on Skirrow's medium incubated at 42°C under micro-aerophilic conditions)
Ref: Ananthanarayan 9/e p399, 8/e p398
Explained already
 
HELICOBACTER
27. Ans. (b) (mucous layer) Ref: Harrison 17/e p947 & 18/e p1261
‘H.pylori lives in gastric mucus, with a small proportion gets adherent to the mucosa and possibly a very small number of the organisms entering cells or penetrating the mucosa; its distribution is never systemic.”
28. Ans. (c) (60%) Ref: Harrison 17/e p 947 & 18/e p1262
  • “Worldwide, >80% of duodenal ulcers and >60% of gastric ulcers are related to H. pylori colonization”
28529. Ans. (b), (c) (Urease breath test, Stool antigen assay)
Ref: Ananthanarayan 9/e p401, 8/e p400
Diagnosis of H.pylori infection: Refer text for explanation.
30. Ans. (e) (Gram –ve bacilli, curved) Ref: Jawetz 24/e p275 & 25/e p240
‘Helicobacter pylori is a spiral-shaped gram-negative rod’
31. Ans. (a), (b), (c), (e) (Flagellated, Associated peptic ulcer disease, Hypergastrinomia caused by it, It is a gram –ve organism)
Ref: Ananthanarayan 9/e p400, 01, 8/e p399, Harrison 17/e p946, 49 & 18/e p1261, 63
  • Improved life style leads to Eradication of H.pylori i.e. why the colonization of H. pylori is low (30%) in developed country compared to 80% in developing country.
  • But Eradication of H.pylori cannot cause Improved life style Harrison 17/e p949 also states that
  • H.pylori eradication is found to increase the risk of
  • Emerging diseases such as asthma, obesity, and type 2 diabetes mellitus reflecting aspects of the current Western lifestyle
  • Also increase risk to Diarrheal diseases and GERD and its complications, including esophageal Adenocarcinoma.
About Other Options
  • H.pylori is a gram –ve spirally coiled curved organism
  • H.pylori has Unipolar tuft of lophotrichous flagella
  • Definite Association exists with peptic ulcer disease
  • H. pylori–induced gastritis diminishes the number of somatostatin-producing D cells. Since somatostatin inhibits gastrin release, gastrin levels become higher.
32. Ans. (d) (Treatment causes regression of gastric lymphoma) Ref: Harrison 17/e p946–49 & 18/e p1261–64
“The most clear-cut indications for treatment are H. pylori–related duodenal or gastric ulceration or low grade gastric B cell lymphoma” ……………Harrison 17/e p948
“Low-grade gastric MALT lymphoma can be histologically cured with eradication therapy for H. pylori”. …………..Oxford Journal- Annals of Oncology 1997/Vol8/Iss 2
About Other Options:
  • Spiral organism but gram -ve
  • It is a bacteria, not a protozoa
  • “H.pylori causes chronic gastritis in adults due to its primary colonization and persistence in stomach, not due to re- infection.”……………….Harrison 17th/ p946
  • “Increased acid secretion induces protective gastric metaplasia in the duodenum; the duodenum can then become colonized by H. pylori, inflamed, and ulcerated.”……….Harrison 17th /946–47
33. Ans. (d) (Common in adults of developing countries) Ref: Jawetz 24/e p277& 25/e p241
  • In developing countries, the prevalence of infection may be 80% or higher in adults…Jawetz 24/e p277
  • “Whether transmission takes place by the fecal-oral or the oral-oral route is unknown
    …....Harrrison17/e p946 & 18/e p1261
  • “Worldwide, >80% of duodenal ulcers and >60% of gastric ulcers are related to H. pylori colonization Diagnosis of H.pylori infection” ……...Harrrison 17/e p947& 18/e p1262
34. Ans. (a) (H.pylori) Ref: Bailey and Scott Diagnostic Microbiology 12th
Urease production:
  • Maximum- H,pylori followed by Brucella canis, Proteus
  • Moderate- Klebsiella
Urease producing Fungus:
  • Cryptococcus neoformans
  • Trichophyton mentagrophytes
  • Trichosporon beigelii
  • Malasseiza furfur
35. Ans. (a) (With chronic infection urease breath test become negative) Ref: Jawetz 24/e p276 & 25/e p241 After treatment, urease breath test becomes negative (not chronic infection)
286About Other Options
  • Once colonized, the H pylori infection persists for years and even for a lifetime
  • Endoscopic guided biopsy usually done for the diagnosis.
  • H.pylori infection has been associated with two important virulent gene- Cytotoxin associated gene (Cag) and Vacuolating Cytotoxin gene (Vac)
  • Toxins and lipopolysaccharide may damage the mucosal cells, and the ammonia produced by the urease activity may directly damage the cells (Jawetz 24/e p276)
36. Ans. (d) (Cag –A gene is not associated with risk of duodenal ulcer) Ref: Harrison 17/e p947 & 18/e p1261, 62
  • H. pylori colonization increases the lifetime risk of:
    • ✓ Peptic ulcer disease,
    • ✓ Non-cardia gastric cancer,
    • ✓ B cell non-Hodgkin's gastric lymphoma
  • Cytotoxin associated gene (Cag), Vacuolating Cytotoxin gene (Vac) and urease enzyme are the principle virulence factors.
  • However, CagA+ H.pylori strains protect against:
    • ✓ Adenocarcinoma of the esophagus,
    • ✓ Adenocarcinoma of gastric cardia and
    • ✓ Premalignant lesions such as Barrett's esophagus
37. Ans. (b) (Helicobacter pylori) Ref: Ananthanarayan 8/e p399 & 9/e p400, 401.
  • Gastric biopsy showing urease positive and growth in Skirrow's media- Suggestive of Helicobacter pylori.
38. Ans. (b) (c) (d)(Microaerophilic, Gram negative bacilli, Survives in acidic environment by producing urease)
Ref: Ananthnarayan 9th/p400
39. Ans. (a) (b) (Move by tuft of flagella, Produce urease enzyme) Ref: Ananthnarayan 9/e p400
  • Motility in H. pylori (conferred by four to eight unipolar flagella) allows it to remain highly motile in the viscous environment of the mucus layer overlying the gastric mucosa
  • H pylori produces maximum urease among all bacteria
  • H.pylori colonizes the stomach of 50% of the world's human population (vary from 30% in developed country to nearly 80% in developing country).
  • H pylori is curved Gram negative bacteria
 
LEGIONELLA
40. Ans- (d) (Non-motile gram negative) Ref: Ananthanarayan 8/e p400
  • Legionella is non motile, poorly gram negative coccobacilli.
41. Ans- (b), (A.C Aerosol), Ref: Harrison 18/e p1236, Ananthanarayan 8/e p400, 401
  • Multiple modes of transmission of Legionella to humans exist, including:
    • ✓ Aerosolization,
    • ✓ Aspiration,
    • ✓ Direct instillation into the lungs during respiratory tract manipulations.
  • Aspiration is now known to be the predominant mode of transmission
  • Aerosolization of Legionella by devices filled with tap water, including whirlpools, nebulizers, and humidifiers, cooling tower, air-conditioners, has been implicated.
42. Ans. (b) (Legionella) Ref: Harrison 17/e p930 & 18/e p1237
Fever, chest pain and dry cough is suggestive of atypical pneumonia.
Among the agents of atypical pneumonia, Legionella grows on charcoal yeast medium.
Causes of atypical pneumonia:
  • Chlamydia pneumoniae,
  • Chlamydia psittaci,
  • Mycoplasma pneumoniae (MC cause)
  • Coxiella burnetii,
  • Virus
28743. Ans. (d) (Failure of oxidative burst of neutrophils) Ref: Harrison 17/e p930 & 18/e p1237
  • The role of neutrophils in immunity appears to be minimal: neutropenic patients are not predisposed to Legionnaires’ disease.
  • Although L. pneumophila is susceptible to oxygen-dependent microbiologic systems in vitro, it resists killing by neutrophils
44. Ans. (a) (Legionella) Ref: Harrison 17/e p930 & 18/e p1237
  • Pontiac fever is an acute, self-limiting, flu-like illness with an incubation period of 24–48 hour.
  • It is the milder form of Leginella infection.
  • Severe for is known as Legionnaire's disease
45. Ans. (a) (Epidemics seen) Ref: Harrison 17/e p930–31 & 18/e p1237–38
  • Pontiac fever (milder form of Legionella infection) occurs in epidemics.
  • In Sputum numerous neutrophils but no organisms revealed by Gram's staining of respiratory secretions
  • Legionella has been identified in lymph nodes, spleen, liver, or kidneys in autopsied cases but splenomegaly is uncommon. (Textbook of pediatric infectious diseases, V-1,Ralph D. Feigin)
  • The clinical picture includes a relatively nonproductive cough and a low incidence of grossly purulent sputum. ……..Harrison 17/e p930
46. Ans. (c) (Legionella are communicable from infected patients to others) Ref: Ananthanarayan 9/e p401,8/e p400
  • Man to man transmission doesn't occur. ……..Ananthanarayan 9/e p401,8/e p400
About Other Options
  • “The role of neutrophils in immunity appears to be minimal: neutropenic patients are not predisposed to Legionnaires’ disease. Although L. pneumophila is susceptible to oxygen-dependent microbiologic systems in vitro, it resists killing by neutrophils.” …..Harrison 17/e p930
  • ✓ Legionella are fastidious and can be grown on complex media containing cysteine (BCYE media- buffered charcoal yeast extract with cysteine and antibiotics) ……..Ananthanarayan 9/e p401,8/e p400
  • Out of 49 species, L. pneumophila causes 80–90% of human infections and which can be further grouped to at least 16 serogroups.
  • Serogroup 1 followed by 4 and 6 are most commonly implicated in human infections….Harrison 17/e p930
47. Ans. (c) (From inhalation of the aerosol in the air conditioned room at convention center)
Ref: Ananthanarayan 9/e p401, 8/e p400
  • Human infection occurs by inhalation of aerosol produced by cooling tower, AC, shower heads which acts as disseminators.
  • Natural source- Legionella is widely distributed in natural water (stagnant water, mud and hot spring) where the growth requirement is provided by algae and protozoa like amoeba.
  • No carrier state, No animal reservoir and No man to man transmission
48. Ans. (b) (BCYE agar) Ref: Ananthanarayan 9/e p401, 8/e p400
BCYE media- buffered charcoal yeast extract with cysteine and antibiotics
49. Ans. (a) (Legionella) Ref: Harrison 17/e p930–32 & 18/e p1237, 38
  • Atypical Pneumonia– 4th MC cause of community-acquired pneumonia (CAP)
  • 1st three cause of CAP are
    • Streptococcus pneumoniae, Haemophilus influenzae, Chlamydia pneumonia
Clinical Clues Suggestive of Legionnaires’ Disease:
  • Diarrhea
  • High fever (>40°C; >104°F)
  • Numerous neutrophils but no organisms
  • Hyponatremia
  • Failure to respond to β-lactam drugs
  • Contaminated potable water supply
  • Onset within 10 days after hospital discharge
28850. Ans. (a) (Pseudolymphadenopathy)
Ref: Harrison 17/e p991 & 18/e p1320, Ananthanarayan 9/e p397, 8/e p396, 97
  • Donovaniasis is characterized by pseudo bubo i.e subcutaneous swelling without lymphadenopathy.
51. Ans. (b) (Bacterial Vaginosis)
Ref: Ananthanarayan 9/e p403, 8/e p401
Bacterial vaginosis: Refer text for explanation.
52. Ans. (c) (donovanosis)
Ref: Harrison 17/e p991 & 18/e p1320, Ananthanarayan 9/e p397, 8/e p396, 97
Donovanosis presents as- Beefy lesions, which bleed readily on contact. Donovanosis: Refer text for explanation.
53. Ans. (b) (Clue cells seen) Ref: Ananthanarayan 8/e p401 & 9/e p403
  • Bacterial vaginosis- pH >4.5
  • Clue cells (vaginal epithelial cells studded with numerous gram negative coccobacilli) are characteristic feature of Bacterial vaginosis.
  • Vaginal discharge in Bacterial vaginosis has a fishy odour.
  • Bacilli with School of fish appearance seen in Chancroid (H.ducreyi)
53. Ans. (a) (Penicillin G) Ref: Ananthnarayan 9/e p398
  • Penicillin is considered to be the treatment of choice for both forms of rat bite fever i.e. Streptobacillus moniliformisand Spirillum minus
  • Alternative drug-Streptomycin, tetracycline, doxycycline, cephalosporin
  • Penicillin resistance, though rare, but has been reported for S. moniliformis.
289Virology
  • 4.1 General Virology
  • 4.2 Herpes Viruses and Other DNA Viruses
  • 4.3 Myxoviruses
  • 4.4 Arboviruses, Picorna Viruses and Rabies
  • 4.5 Hepatitis Viruses
  • 4.6 HIV
  • 4.7 Miscellaneous Viruses

General VirologyCHAPTER 4.1

291
 
VIRUS VS BACTERIA
Property
Virus
Bacteria
Nucleic acid
DNA or RNA
Both
Binary fission
No
Yes
Cellular organelle
absent
Ribosome present, others- absent
Cellular organization
No
Present
Location
Intracellular
Intra / Extra cellular
Susceptibility to Antibiotics
Resistant (exception- Rifampicin to Pox)
Sensitive
Culture in artificial media
No
Can be grown (except Rickettsia & Chlamydia)
 
DNA Virus
  • Pox – largest virus
  • Papova
  • Parvo –smallest virus, possess ss-DNA,
  • Herpes
  • Hepatitis B
  • Adenovirus
  • Bacteriophage
 
RNA VIRUS - Family
Picorna viridae – Polio, Coxsackie
Rhabdo – Rabies
Orthomyxo – Influenza
Reo – e.g. Rotavirus (ds)
Paramyxo – Parainfluenza
Measles, RSV, Mumps
Corona – SARS
Retro – HIV
Arbovirus-Toga, Flavi, Bunya
Calci
Arena – LCM, Lassa
Filo – Marbug & Ebola
Segmented RNA Viruses (BIRA)
  • Bunya,
  • Influenza,
  • Rota,
  • Arena(LCM i.e. lymphocytic chorio-meningitis virus)
 
Size of Viruses
  • Size – determined by
    • Ultra filtration in gradcol membrane of graded porosity
    • Ultracentrifugation
    • Electron microscopy
  • Largest – Pox (300nm) – Possess Elementary body
  • Smallest – Parvovirus (20nm)
 
Shape of Viruses
  • Structure – consists of Nucleocapsid (Nucleic acid and protein layer capsid)
  • Capsid is made up Capsomer units
  • 292Most of the viruses are roughly spherical except:
    • Rabies –Bullet
    • Pox virus- Brick
    • Ebola virus- Filamentous
    • Tobaco mosaic virus– Rod
    • Adenovirus - Space vehicle
 
Nucleic Acid
  • Made up either DNA or RNA
  • All the RNA viruses are single stranded except Reo viruses (double stranded)
  • All the DNA viruses are double stranded except Parvoviruses (single stranded)
 
Symmetry
  • Icosahedron symmetry – E.g. All DNA viruses (except Pox) and most of the RNA viruses
  • Helical symmetry – Few RNA viruses (Myxo, Rhabdo, Filoviridae, Bunya) (MRF- Bat)
  • Pox – complex symmetry
 
Envelop
Enveloped Virus
  • Made up lipoprotein subunits called peplomere
  • Lipid part is host cell membrane derived and protein part is virus derived,
  • Envelop provides chemical, physical and biological properties to cell.
  • Ether sensitive, heat labile, pleomorphic
  • Example – All, other than non enveloped virus are enveloped virus (See below)
Non-Enveloped Virus
  • Ether resistant, heat stable and non-pleomorphic
  • DNA- Parvo, Adeno, Papova (PAP)
  • RNA- Picorna, Astrovirus, Reovirus, Calcivirus, (PARC)
 
Viral Replication
  • Adsorption
    • Most specific step requires respective receptors
    • If bypassed, then any virus can attack any cell.
  • Penetration
  • Uncoating (Bacteriophage doesn't have this step)
  • Biosynthesis
    • DNA Viruses –Replicates in nucleus (except-Pox)
    • RNA Virus -Replicates in Cytoplasm (except Orthomyxo and Retro virus)
    • Viral protein is synthesized only in cytoplasm (for both DNA and RNA viruses)
    • +ve sense RNA virus:
      • ✓ Viral RNA itself acts as mRNA
      • ✓ Infectious and translated directly to protein
      • ✓ E.g. –Most RNA viruses are +ve sense- E.g.Picorna, Togaviruses
    • -ve Sense RNA virus:
      • ✓ Have polarity opposite to mRNA
      • ✓ Non infectious and possess their own RNA polymerase for transcription to form mRNA.
      • ✓ E.g. -Myxo, Rabies
    • Retroviruses:
      • ✓ Viral reverse transcriptase converts viral ssRNA to dsDNA
      • ✓ Then dsDNA integrates with host DNA.
  • Maturation and Assembly
  • Release
    • Bacteriophage- by host cell lysis
    • Animal virus- usually without lysis (E.g.Myxo – by budding)
    • Exception- Picorna- by host cell lysis, Varicella- cell to cell transmission
  • 293Eclipse phase: From the stage of penetration till appearance of mature daughter virions- The virus cannot be demonstrable. For animal viruses-15-30hr, for bacteriophage-15-30min
  • Viropexis: The process of engulfment (phagocytosis) of virus particles by the animal cells
  • Von Magnus phenomena: A proportion of daughter viruses may not be infective (incomplete viruses) due to defective assembly. Usually seen when high dose of Influenza virus infecting a cell. The virus yield has high hemagglutinin titer but low infectivity.
  • Viruses can be classified to:
    • Viroin: True viral particles (contain both protein and nucleic acid)
    • Prion: Protein without nucleic cid
    • Viroid: Nucleic acid without protein
 
VIRAL CULTIVATION
 
Animal Inoculation
  • Coxsackie – intra cerebral inoculation to Suckling Mice-
    • Coxsackie A-flaccid paralysis,
    • Coxsackie B- spastic paralysis
  • Arboviruses
 
Egg Inoculation
  • Chorioallantoic membrane – Produce pocks. E.g. Vaccinia, Variola, HSV 1 and 2
  • Yolk sac- Arbovirus, Chlamydia, Rickettsia
  • Amniotic membrane- Influenza culture (for diagnosis)
  • Allantoic cavity- vaccine preparation for Influenza, Yellow (17D), Rabies (Flury)
 
Tissue Culture
  • Organ culture- tracheal ring (corona)
  • Explant culture- adenoid (Adeno)
  • Cell Line
Primary cell line
Undergo limited division (5-10), diploid karyosome
E.g.-Rhesus Kidney cell line, Human amniotic cell line, Chick embryo fibroblast
Secondary cell line
Undergo moderate cell division(10-50), diploid karyosome
Human fibroblast used for CMV
WI-38
Human embryonic lung cell strain
HL-8
Rhesus embryo cell strain
Continuous cell line
Indefinite divisions, haploid karyosome
HeLa
Human Ca cervix cell line
HEp-2
Human epithelioma of layrnx cell line
KB
Human Ca nasopharynx cell line
McCoy
Human synovial carcinoma cell line
Vero
Vervet monkey kidney cell line
BHK
Baby hamster kidney cell line
Detroid-6
Sterna marrow cell line
 
Detection of Viral Growth in Cell Culture
  • Cytopathic effect: Morphological changes produced by virus in the cell line observed under microscope294
Cytopathic Effect
Virus
Crenation of cells and degeneration of entire cell sheet
Enterovirus
Syncytium formation
Measles virus, RSV, Herpes, Parainfluenza
Large granular clumps like grapes
Adenovirus
Cytoplasmic vacuolation
SV40
Diffuse roundening of cells
Herpes
  • Other methods for Detection of viral growth in cell culture:
    • Viral gene detection by PCR
    • Viral antigen detection by Immunofluorescence
    • Viral interference
      • ✓ When two viruses infect a cell, one inhibits the multiplication of the other virus.
      • ✓ Seen in Rubella, Polio
    • Viral hemagglutnation
      • ✓ Myxovirus, Rabies, Arbo, Pox
      • ✓ Elution (d/t neuraminidase)– only in Myxovirus (except RSV and Measles)
    • Hemadsorption- RBC adsorbed onto virus infected cell surface
      • ✓ Mumps, Influenza, Parainfluenzae
    • Metabolic inhibition
Inclusion Bodies Morphological changes produced by virus inside the tissue
Intracytoplasmic Inclusion Bodies
  • Negri body - Rabies
  • Guarnier body – Vaccinia,
  • Paschen body - Variola
  • Bollinger body- Fowl pox
  • Molluscum body – Molluscum contagiosum virus
Intranuclear Inclusion Bodies
  • Cowdry A (HAY) -Herpes (Lipschultz body), Yellow fever (Torres body)
  • Cowdry B (BAP) - Adeno, Polio
Both Intracytoplasmic & Intranuclear Inclusion Bodies-Seen in Measles and CMV (Owl's eye appearance)
 
Assay of Infectivity of Viruses
  • Quantal assay
    • End point biologic assays depend on the measurement of animal death, animal infection, or cytopathic effects in tissue culture at a series of dilutions of the virus being tested.
  • Quantitative infectivity assay
    • Plaque assay :
      • ✓ The most widely used assay for infectious virus is the plaque assay.
      • ✓ Mono layer of host cells are inoculated with suitable dilutions of virus. Each infectious viral particle gives rise to a localized focus of infected cells known as plaques.
    • Pock assay on chick embryo chorioallantoic membrane (CAM)- done for Pox, HSV
 
Latent Virus
  • Herpes
    • HSV I,II, VZV - nerve
    • CMV – kidney, secretory gland
    • EBV –lymphoid
    • HHV6,8
  • HIV – CD4 T cell
  • HPV- Squamous epithelial cell
  • Measles
  • Rubella
  • Adenovirus
  • 295Molluscum contagiosum (Persistent but not latent)
  • Slow virus – neuron
 
Teratogenic Virus
  • CMV
  • Rubella
  • Herpes
  • Varicella & Parvo B19
 
Viruses Transfer Through Placenta
  • Coxsackie B
  • Hepatitis B, C
  • HIV
  • Measles, Mumps
 
VIRAL VACCINE
 
Inactivated Vaccine
  • Salk polio -formalinized MKD
  • Influenza (killed)
  • Japanese B –
    • Nakayama strain, formalinized mouse brain
    • Beijing strain
  • Hepatitis B (subunit –HBsAg cloned in yeast)
  • Rabies:
    • Neural (BPL, Semple and infant mouse brain)
    • Non neural vaccine-
      • ✓ Purified chick embryo cell (PCEC)vaccine
      • ✓ Purified Vero cell (PVC) vaccine
      • ✓ Human diploid cell (HDC) vaccine
 
Live Vaccine
  • Sabin Polio – Avirulent prepared in MKC
  • Influenza – live, prepared in egg
  • Japanese B- (14-14-2 strain)
  • Yellow (17D) – prepared in chick embryo
  • Small Pox Vaccine (Vaccinia)
  • Varicella- Oka Strain
  • CMV -Towne and AD
  • Mumps- Jerryl Lynn
  • Measles -Edmonston – Zagreb
  • Rubella -RA 27/3
  • MMR
296MULTIPLE CHOICE QUESTIONS
1. Which of the following is an e.g. of RNA oncogenic virus? (Recent MCQ 2013)
  1. Retroviruses
  2. Reoviruses
  3. Coronaviruses
  4. Roboviruses
2. Virus quantification is done by: (NEET Pattern Based)
  1. Egg inoculation
  2. Hemadsorption
  3. Plaque assay
  4. Electron microscopy
3. Brick-shaped virus: (NEET Pattern Based)
  1. Chicken pox
  2. Small pox
  3. CMV
  4. EBV
4. Non-enveloped ss-RNA virus is: (NEET Pattern Based)
  1. Picornavirus
  2. Poxvirus
  3. Retrovirus
  4. Bunyavirus.
5. Suckling mice is used for isolation of: (NEET Pattern Based)
  1. Coxsackie virus
  2. Herpes
  3. Pox
  4. Adenovirus
6. Human fibroblast cell line is used for cultivation of: (NEET Pattern Based)
  1. Adenovirus
  2. Poliovirus
  3. CMV
  4. Measles
7. Both intranuclear and cytoplasmic inclusion is seen in: (NEET Pattern Based)
  1. Poxvirus
  2. Herpesvirus
  3. Measles virus
  4. Mumps virus
8. True about interferon is: (NEET Pattern Based)
  1. Host protein
  2. Viral protein
  3. Inactivated by nucleases
  4. Virus specific
9. Which of the following is primary cell line? (NEET Pattern Based)
  1. Chick embryo fibroblast
  2. HeLa cells
  3. Vero cells
  4. WI-38
10. Live attenuated vaccines are A/E: (PGI June 2011)
  1. Measles
  2. Rabies
  3. Oral polio
  4. Yellow fever
  5. Influenza
11. Continuous cell line for viruses not present for: (DNB Dec 2011)
  1. Vero
  2. Hep2
  3. WI-38
  4. Hela
12. Viral inclusion bodies are all except:(DNB June 2010)
  1. Psamomma bodies
  2. Molluscum
  3. Negri
  4. Bollinger
13. Viral infection that is least transmitted transplacentally is: (AI 2011)
  1. Hepatitis B.
  2. Rubella.
  3. Herpes simplex.
  4. HIV
14. Latency seen in viral infections: (PGI Dec 2008, June 2005)
  1. HSV – II
  2. CMV
  3. Rotavirus
  4. HIV
  5. EBV
15. Which of the following is not an RNA virus: (AIIMS Nov 2008)
  1. Ebola
  2. Simian 40
  3. Rabies
  4. Vesicular stomatitis virus
16. Viruses can be isolated from clinical samples by cultivation in the following except: (AI 2005)
  1. Tissue culture
  2. Embryonated eggs
  3. Animals
  4. Chemically defined media
17. Laboratory diagnosis of viral respiratory tract infection can be established by all of the following tests except: (AI 2004)
  1. Detection of virus specific IgM antibodies in single serum specimen
  2. Demonstrationofviralantigensbyindirect immunofluorescene assay in nasopharyngeal washing
  3. Isolation of viruses using centrifugation enhanced culture
  4. Detection of viral hemagglutination inhibiting (HAI) antibodies in a single serum specimen
18. Choose the correct matches: (PGI June 2002)
  1. Mumps – RA 27/3 strain
  2. Rubella – Jeryl – Lynn strain
  3. Measles – Edmonston – Zagreb stain
  4. BCG – Danish 1331 strain
19. 297Vaccine preparation requires which virus as vector: (PGI June 2001)
  1. Rhinovirus
  2. Vaccinia
  3. Adenovirus
  4. Ebola
  5. Hepatitis B
20. Vaccine prepared by Embryonated Hen's egg are: (PGI June 2004)
  1. Measles
  2. Rabies
  3. Rubella
  4. Varicella
21. Not true about viral antibodies? (AIIMS Nov 2014)
  1. Formed before interferons
  2. Formed against viral surface antigens
  3. Formed against nucleoproteins
22. Both DNA and RNA are found in: (AIIMS May 2014)
  1. All Bacteria
  2. Prion
  3. Viroid
  4. Plasmid
23. All the following vaccines are developed from embryonated eggs except: (PGI May 2013)
  1. Influenza
  2. Hepatitis-A
  3. Yellow fever
  4. Rabies
  5. CMV298
EXPLANATIONS
1. Ans. (a) (Retroviruses) Ref: Ananthanarayan 9/e p565
  • Oncogenic RNA viruses- HTLV1 (Retroviruses) and HCV.
2. Ans. (c) (Plaque assay) Ref: Ananthanarayan 9/e p437, 8/e p436
Assay of infectivity of viruses:
  • Quantal assay- End point biologic assays
  • Quantitative infectivity assay- Plaque assay and Pock assay
3. Ans. (b) (Small pox) Ref: Ananthanarayan 9/e p429, 8/e p427
  • Most of the viruses are roughly spherical except
    • ✓ Rabies –Bullet shaped, Pox virus- Brick shaped, Tobaco mosaic virus– Rod shaped
    • ✓ Adenovirus - Space vehicle shaped, Ebola virus- Filamentous shaped
4. Ans. (a) (Picornavirus) Ref: Ananthanarayan 9/e p429, 8/e p426
  • Picornaviruses are Non-enveloped and ss-RNA viruses
5. Ans. (a) (Coxsackie virus) Ref: Ananthanarayan 9/e p452, 8/e p451
  • Suckling mice is used for isolation of Coxsackie and Arbovirus
6. Ans. (c) (CMV) Ref: Ananthanarayan 9/e p434, 8/e p433
  • Human fibroblast cell line is used for cultivation of CMV
7. Ans. (c) (Measles virus) Ref: Ananthanarayan 9/e p452, 8/e p448
  • Both intranuclear and cytoplasmic inclusion is seen in:- Measles and CMV
8. Ans. (a) (Host protein) Ref: Ananthanarayan 9/e p452, 8/e p451
  • Interferons are Host specific protein, not virus specific.
  • Interferons are inactivated by proteolytic enzymes but resistant to nucleases & lipases.
9. Ans. (a) (Chick embryo fibroblast) Ref: Ananthanarayan 9/e p436, 8/e p434
  • Examples of primary cell lines- Rhesus Kidney cell line, Human amniotic cell line, Chic